Performed · Web viewAAA, kidney stones, groin hernia, ileus, kidney/bladder CA, testicular...

32
FAMILY MEDICINE I MODULES Which of the following is true regarding Fryette’s Principles? a. Fryette’s Principle 2 describes mechanics of group curves b. The lower cervical spine follows Fryette’s Principle 1 c. In Fryette’s Principle 2, sidebending and rotation occur to the same side d. Fryette’s Principle 2 describes motion in a neutral position e. Fryette’s Principle 1 only applies to the lumbar spine 2. Which of the following is true regarding thoracic diagnosis? a. Rotation is named for the side of the deeper transverse process b. If rotation only becomes more symmetrical in flexion, the dysfunction is an extension dysfunction c. If rotation becomes symmetrical in both flexion and extension, sidebending occurs to the same side as rotation d. If rotation only becomes more symmetrical in flexion, the dysfunction follows Fryette’s Principle 2 e. Rotation and sidebending always occur to the same side regardless of the change in flexion and extension 3. You examine L3 and find the right transverse process to be deeper than the left. The findings are unchanged in flexion and become

Transcript of Performed · Web viewAAA, kidney stones, groin hernia, ileus, kidney/bladder CA, testicular...

Page 1: Performed · Web viewAAA, kidney stones, groin hernia, ileus, kidney/bladder CA, testicular torsion, kidney/bladder infection, appendicitis, C D Osteopathic Diagnosis: assess where

FAMILY MEDICINE I MODULES

Which of the following is true regarding Fryettersquos Principles

a Fryettersquos Principle 2 describes mechanics of group curves

b The lower cervical spine follows Fryettersquos Principle 1

c In Fryettersquos Principle 2 sidebending and rotation occur to the same side

d Fryettersquos Principle 2 describes motion in a neutral position

e Fryettersquos Principle 1 only applies to the lumbar spine

2 Which of the following is true regarding thoracic diagnosis

a Rotation is named for the side of the deeper transverse process

b If rotation only becomes more symmetrical in flexion the dysfunction is an

extension dysfunction

c If rotation becomes symmetrical in both flexion and extension sidebending

occurs to the same side as rotation

d If rotation only becomes more symmetrical in flexion the dysfunction follows

Fryettersquos Principle 2

e Rotation and sidebending always occur to the same side regardless of the

change in flexion and extension

3 You examine L3 and find the right transverse process to be deeper than the left The findings are unchanged in flexion and become symmetrical in extension Which of the following is the correct diagnosis

a L3 flexed sidebent left rotated left

b L3 flexed sidebent right rotated right

c L3 extended sidebent left rotated left

d L3 extended sidebent right rotated left

e L3 flexed sidebent left rotated right

4 You assess OA translation and find translation to be easier to the right The findings persist in both flexion and extension Which of the following is the correct diagnosis

a Flexed rotated right sidebent left

b Neutral rotated left sidebent right

c Neutral rotated right sidebent left

d Neutral rotated left sidebent left

e Extended rotated right sidebent right

5 You assess C4 translation and find translation to be easier to the left The findings become symmetrical in flexion but not in extension Which of the following is the correct diagnosis

a Flexed rotated right sidebent left

b Neutral rotated left sidebent right

c Neutral rotated right sidebent left

d Flexed rotated right sidebent right

e Extended rotated right sidebent right

6 Which of the following is true regarding rib motion

a Rib 7 is primarily caliper motion

b Motion occurs on a vertical axis

c Rib 11 is predominantly bucket handle motion

d Rib 1 is predominantly anteriorposterior motion

e Rib 3 is both pump handle and bucket handle motion X

7 Which of the following is a finding of a rib inhalation somatic dysfunction

a Lives in exhalation

b Prominence on anterior chest

c Limitation to inhalation

d Prominence on posterior chest

e Recession on anterior chest

8 Which of the following is true regarding rib diagnosis

a Ribs that only move during inhalation are exhalation dysfunctions

b Palpation of Rib 1 is performed inferior to the clavicle

c Bucket handle motion is assessed in the mid-axillary line

d Anterior rib prominence is a finding of an exhalation dysfunction

e An inhalation dysfunction will have both anterior and posterior prominence

9 Which of the following is a test of laterality for assessment of innominate dysfunctions

a Respiratory motion test

b ASIS compression test

c ASIS to midline distance

d Seated flexion test

e Relative static PSIS heights

10 Which of the following findings would be expected in a right innominate anterior somatic Dysfunction

a Positive seated flexion test on the right

b Positive ASIS compression test on the left

c Right ASIS inferior compared to left

d Left PSIS superior compared to right

e Right ASIS closer to midline than right

11 You assess the innominates and find the following

Positive standing flexion test on the leftRight ASIS inferior compared to leftRight PSIS inferior compared to leftASIS to midline distance equal bilaterallyWhich of the following is the correct diagnosis

a Right innominate anterior

b Left superior innominate shear

c Left innominate posterior

d Left innominate inflare

e Right innominate posterior

12 Which of the following is true regarding the seated flexion test

a If both sides rise the same amount it indicates that no dysfunction is present

b The test is positive opposite the side of greatest PSIS excursion

c The axis of a sacral torsion is located on the same side of greatest excursion

d A unilateral dysfunction will be on the same side of the greater excursion

e A false negative occurs with unilateral dysfunctions

13 Which of the following is true regarding sacral torsions

a The right base moves forward in a left on right torsion

b The axis passes through the right base in a left on left torsion

c A right on right torsion is termed a backward sacral torsion

d The right sulcus is deeper in a right on right torsion

e Points along the axis do not move in a sacral torsion

14 Which of the following is true regarding a bilateral sacral extension

a The sacral sulci are deep bilaterally

b The seated flexion test will be positive

c The base will not move posteriorly during inhalation

d The base will not move posteriorly during exhalation

e The sacral sulci are shallow bilaterally

15 You assess the sacrum and find the following

Positive seated flexion test on the leftLeft sulcus shallowLeft ILA markedly superiorWhich of the following is the correct diagnosis

a Right on left sacral torsion

b Right unilateral sacral flexion

c Left on right sacral torsion

d Left unilateral sacral extension

e Left on left sacral torsion

16 You examine T7 and find the left transverse process to be deeper than the right The findings become symmetrical in flexion and are unchanged in extension Which of the following is the correct diagnosis

a T7 flexed sidebent left rotated left

b T7 flexed sidebent right rotated right

c T7 extended sidebent left rotated left

d T7 extended sidebent right rotated left

e T7 flexed sidebent left rotated right

17 Which of the following is true regarding lumbar diagnosis

a Rotation and sidebending occur to opposite sides in flexion dysfunctions

b If the right transverse process is deeper the vertebra is rotated to the left

c Rotation and sidebending occur to the same side in neutral dysfunctions

d A lumbar dysfunction that becomes symmetrical in both flexion and extension

is an extension dysfunction

e Only Fryettersquos Principle 1 applies to the lumbar spine

18 Which of the following is true regarding cervical mechanicsdiagnosis

a Motion testing of the AA is performed by translation

b C2-C7 only follow Fryettersquos Principle 1

c The OA is the only cervical segment that must be tested in flexion extension and neutral

d Right translation is equal to left sidebending at C4

e Left translation is equal to left rotation at C6

19 Which of the following is true regarding sacral dysfunctions

a A right unilateral sacral flexion will have a positive seated flexion test on the left

b A left on left sacral torsion will have a deep right sulcus

c A right on left sacral torsion will have a deep right sacral sulcus

d A left unilateral seated flexion will have a significantly higher ILA on the left

e A bilateral sacral flexion will have bilaterally shallow sulci

20 Which of the following is true regarding innominate dysfunctions

a Innominate flares are named for the side on which the ASIS is closer to midline

b The standing flexion test is negative with an innominate shear

c The ASIS compression test will be positive on the side opposite the positive standing flexion test

d A positive seated flexion test is expected on the right with a right innominate posterior

e The right ASIS is inferior with a right innominate anterior

21 You evaluate the 3rd ribs bilaterally and find the following

Right 3rd rib more prominent anteriorlyBoth sides rise during inhalationOnly the left side falls during exhalationWhich of the following is the correct diagnosis

a Right 3rd rib inhalation dysfunction

b Left 3rd rib inhalation dysfunction

c Right 3rd rib inferior dysfunction

d Left 3rd rib exhalation dysfunction

e Left 3rd rib superior dysfunction

22 Which of the following is paired correctly

a Fryettersquos Principle 1 ndash Typically involves a single segment

b AA dysfunctions ndash Assessed with rotation

c Ribs 11 amp 12 ndash Predominantly superiorinferior motion

d Left oblique sacral axis ndash Positive left seated flexion test

e Right innominate posterior ndash Positive left standing flexion test

23 Which of the following dysfunctions will have a left positive standing flexion test and

a negative AP compression test

a Left innominate posterior

b Right superior innominate shear

c Left innominate outflare

d Right innominate inflare

e Left inferior innominate shear

24 You examine L1 and find the right transverse process to be more shallow than the left The findings are unchanged in both flexion and extension Which of the following is the correct diagnosis

a L1 flexed sidebent left rotated left

b L1 neutral sidebent right rotated right

c L1 neutral sidebent left rotated left

d L1 extended sidebent right rotated left

e L1 neutral sidebent left rotated right

25 Which of the following is true regarding a right superior innominate shear

a Negative standing flexion test

b Negative AP compression test

c ASIS and PSIS both superior on the right

d Right ASIS closer to midline

e Positive standing flexion test on the left

MODULE 2

1 Which of the following is true regarding muscle energy treatment

a Contraindicated in patients with osteopenia

b Primary treatment for patients in the ICU

c Utilizes patient force with equal physician counterforce X

d Patient effort is held for 90 seconds

e A-P curve must be flattened out during treatment

2 A patient is diagnosed with a right anterior innominate dysfunction Which of the

following is the correct description for muscle energy treatment of this dysfunction

a Right hip extended ndash patient flexes hip against resistance

b Right hip flexed ndash patient extends hip against resistance X

c Right hip extended ndash patient extends hip against resistance

d Right hip externally rotated ndash patient internally rotates hip against resistance

e Right hip abducted ndash patient adducts hip against resistance

3 Which of the following is true regarding HVLA thrust technique

a Indirect technique

b Useful in patients with joint hypermobility

c Thrust applied after backing away from barrier for 1-2 seconds

d Lumbar HVLA is contraindicated in patients with Down Syndrome

e Restrictive barrier must be engaged in all planes of motion X

4 Which of the following is true regarding Counterstrain treatment

a Restrictive barrier must be engaged in all planes of motion

b Treatment position should decrease tenderness by at least 70 X

c Treatment position is held for 3-4 seconds then released

d Patient assists in positioning for treatment

e Patient must slowly bring himself to neutral after treatment

5 Which of the following is correctly matched

a Counterstrain treatment of hip flexor ndash patientrsquos hip passively flexed X

b FPR treatment of shoulder external rotator ndash patientrsquos shoulder internally rotated

c HVLA treatment of C3 ERRSR ndash C3 placed into extension right rotation and right sidebending

d Muscle energy treatment of a bilateral sacral flexion ndash pressure on sacral base

e HVLA treatment of right unilateral sacral extension ndash superior thrust on right ILA

6 The heart receives sympathetic innervation from which of the following levels

a OA

b T1-T5 X

c T5-T9

d T10-L2

e S2-S4

7 Which of the following is true regarding FPR treatment

a Restrictive barrier must be engaged in all planes of motion

b Treatment position should decrease tenderness by at least 70

c Treatment position is held for 3-4 seconds then released X

d A-P curve is accentuated during treatment

e Facilitating force is an isometric contraction of the involved muscles

8 Which of the following is true about autonomic control of the lungs

a Sympathetic stimulation results in bronchoconstriction

b The lower lungs receive parasympathetic innervation from S2-S4

c Sympathetic innervation arises from the T5-T9 levels

d Increased thick secretions are a result of increased sympathetic activity X

e Sacral rocking will result in increased thin watery secretions

9 Which of the following techniques would be most useful in improving upper GI

motility in a patient with ileus

a Sacral rocking

b Rib raising for 30 seconds

c OA decompression

d Paraspinal inhibition for 120 seconds X

e Sacral inhibition

10 You assess C4 translation and find translation to be easier to the left The findings

become symmetrical in flexion but not in extension Which of the following is the

correct position of C4 for a HVLA thrust

a Flexed rotated right sidebent left

b Extended rotated left sidebent left X

c Neutral rotated right sidebent left

d Flexed rotated right sidebent right

e Extended rotated right sidebent right

11 You examine L3 and find the right transverse process to be deeper than the left The

findings are unchanged in flexion and become symmetrical in extension Which of

the following is the correct position of L3 for FPR treatment

a Flexed sidebent left rotated left

b Flexed sidebent right rotated right

c Extended sidebent left rotated left X

d Extended sidebent right rotated left

e Flexed sidebent left rotated right

12 Which of the following is true regarding autonomic innervation to the GU system

a Sympathetic stimulation increases ureteral peristalsis

b Sympathetic stimulation relaxes the bladder wall X

c Parasympathetic stimulation activates the bladder sphincter

d Parasympathetic stimulation activates the trigone

e Sympathetic stimulation increases GFR

13 Which of the following is a result of parasympathetic stimulation

a Peripheral vasodilation

b Ejaculation

c Increased salivation X

d Uterine contraction

e Ureterospasm

14 The gall bladder receives sympathetic innervation from which of the following

levels

a OA

b T1-T5

c T5-T9 X

d T10-L2

e S2-S4

15 You are attempting to treat a right 7th rib exhalation dysfunction with counterstrain

Which of the following is correct regarding the treatment

a Shaft of right 7th rib positioned closer to shaft of right 8th rib X

b Position should decrease tenderness by 50

c Position is held for 30 seconds

d Angle of right 7th rib tractioned inferiorly

e Positioning involves left sidebending and extension

16 Which of the following is correctly matched

a Muscle energy ndash indirect treatment

b HVLA ndash articulation positioned in free motion

c Counterstrain ndash starts with a pain scale X

d FPR ndash active direct treatment

e Muscle energy ndash useful for painful muscle groups

17 Which of the following would be most likely to inhibit uterine contractions in a

patient in premature labor

a Sacral rocking X

b OA decompression

c Paraspinal inhibition gt90 seconds at T5-T9

d Rib raising lt60 seconds at T10-L2

e Sacral inhibition

18 You assess the innominates and find the following

Positive standing flexion test on the left

Right ASIS inferior compared to left

Right PSIS inferior compared to left

ASIS to midline distance equal bilaterally

Which of the following correctly describes a treatment for this dysfunction

a Muscle energy ndash left hip flexed patient extends against resistance

b HVLA thrust ndash thrust directed inferiorly on left innominate X

c FPR ndash right innominate pushed inferiorly for 3-4 seconds

d Muscle energy ndash right hip pushed superiorly patient pushes inferiorly

e Counterstrain ndash left innominate held inferiorly for 90 seconds

19 The pancreas receives parasympathetic innervation from which of the following

levels

a OA X

b T1-T5

c T5-T9

d T10-L2

e S2-S4

20 Which of the following would most likely directly decrease gastric secretions

a Sacral rocking

b OA decompression

c Paraspinal inhibition gt90 seconds at T10-L2

d Rib raising lt60 seconds at T5-T9 X

e Sacral inhibition

21 Which of the following statements is true regarding osteopathic treatment techniques

a Counterstrain is contraindicated for painful muscle groups

b Osteoporosis is a contraindication for FPR

c Muscle energy should not be used on a person in the CCU X

d HVLA is useful in patients with active Rheumatoid Arthritis

e The restrictive barrier is engaged when using indirect techniques

22 You examine T7 and find the left transverse process to be deeper than the right The

findings become symmetrical in flexion and are unchanged in extension Which of

the following is the correct starting position for a muscle energy treatment

a T7 flexed sidebent left rotated left

b T7 flexed sidebent right rotated right

c T7 extended sidebent left rotated left X

d T7 extended sidebent right rotated left

e T7 flexed sidebent left rotated right

23 The sigmoid colon receives parasympathetic innervation from which of the following

levels

a OA

b T1-T5

c T5-T9

d T10-L2

e S2-S4 X

24 You assess OA translation and find translation to be easier to the right The findings

persist in both flexion and extension Which of the following is the correct position

for FPR treatment

a Flexed rotated right sidebent left

b Neutral rotated left sidebent right

c Neutral rotated right sidebent left X

d Neutral rotated left sidebent left

e Extended rotated right sidebent right

25 Which of the following would most likely directly increase lacrimation

a Sacral rocking

b OA decompression X

c Paraspinal inhibition gt90 seconds at T5-T9

d Rib raising lt60 seconds at T1-T4 X

e Sacral inhibition

MODULE 3 ndash Joanne Smith

1 Top 5 ddx peptic ulcer GERD costochondritis Pancreastitis MI angina pectoris2 E no contraindications3 A OA4 E strep throat

dysfunctions Performed Omitted

1 Examine the cervical region for somatic dysfunctionA OA ndash leftright translation1 neutral 10486351048635

2 flexion 104863510486353 extension 104863510486352 Examine the thoracic spine for dysfunction (T5-T9)A palpate transverse processes for rotation1 neutral 104863510486352 flexion 104863510486353 extension 10486351048635B palpate soft tissue for TART findings1 trapezius 104863510486352 rhomboids 104863510486353 levator scapulae 104863510486354 T5 ndash T9 Paraspinal muscles 10486351048635Osteopathic Treatment You must treat a minimum of 2 dysfunctions you found during your examination1 Assume OA NSLRR appropriate techniques to include HVLA muscle energy articulatory Still counterstrainindirect myofascial release facilitated positional release

10486351048635

2 Assume T7 FRLSL appropriate techniques to include HVLA muscle energy articulatory Still counterstrainindirect myofascial release facilitated positional release

10486351048635

3 Assume acute bilateral spasm of upper thoracic (T5-T9) paraspinal muscles appropriate techniques to include soft tissue counterstrainindirect facilitated positional release

10486351048635

4 Autonomic technique for altered vagal tone OA decompression 104863510486355 Autonomic technique for hypersympathetic tone in T5-T9 paraspinal muscles rib raising or paraspinal inhibition for gt 90 seconds

10486351048635

MODULE 4- Ernesto Rodriguez

PRETEST

1 Back pain from Colon cancer Lumbar strain Osteomyelitits Diverticulitis Back pain osteoarthritis Cauda equina syndrome

2 D3 E

Osteopathic Diagnosis assess where you would expect to find somatic dysfunctions

Performed Omitted

1 Examine the lumbar region (L1-L5) for somatic dysfunction

A palpate transverse processes for rotation

1 neutral _

2 flexion _

3 extension _

B palpate soft tissue for TART (Tenderness Asymmetry Restrictions Tissue texture change) findings

1 quadratus lumborum muscles _

2 Piriformis muscles _

3 Paraspinal muscles _

2 Examine the sacrum for somatic dysfunction (requires at least one motion test)

A depth at 4 corners of sacrum _

B ILA levelness _

C seated flexion test _

D motion at 4 corners of sacrum _

E motion in backward bending test _

F respiratory motion of sacrum assess at 4 corners _

3 Examine the pelvis for somatic dysfunction

A standing flexion test or AP compression test _

B ASIS heights _

C PSIS heights _

Osteopathic Treatment You must treat a minimum of 2 dysfunctions you found during your examination

1 Assume L5 FRLSL _

2 Assume left on left sacral torsion _

3 Assume left innominate anterior _

4 Assume spasmtender point in the left piriformis muscle appropriate techniques to include counterstrainindirect myofascial release

-

MODULE 5 ndash Connie Jones

Pretest

1 Bradycardia acute pulmonary edemaCHF pneumonia acute MI Acute pulmonary edema CHF2 A3 A4 A5 For the video this is what was done and not done

OA - nf NO E --gtNslrrno AA dx doneno cervical dx doneT1-T5 - nfe --gt t2 f sl rlparapsinals TART --gt L upper thoracic paraspinalno levator scapula no trap no rhomboids dx tx -

T2 HLVAOA decompr ndash for vagal tonerib raisin for sympth

MODULE 6 ndash Gretchen White

1 Atrial fibrillation hyperthyroidism and- thyrotoxicosis thyroid cancer hyperthryroid 2 a3 a4 a5

Osteopathic Diagnosis assess where you would expect to find somatic dysfunctions

Performed Omitted

1 Examine the cervical region for somatic dysfunction

A OA ndash leftright translation

1 neutral X

2 flexion X

6 3 extension X

7 B AA ndash rotation X

8 2 Examine the upper thoracic spine for dysfunction (T1-T5)

A palpate transverse processes for rotation

1 neutral X

2 flexion X

3 extension X

B palpate soft tissue for TART findings

1 trapezius X

2 rhomboids X

3 levator scapulae X

4 T1-T5 paraspinal muscles X

Osteopathic Treatment You must treat a minimum of 2 dysfunctions you found during your examination

1 Assume OA NSLRR appropriate techniques to include counterstrainindirect myofascial release facilitated positional release

X

2 Assume T2 FSLRL appropriate techniques to include counterstrainindirect myofascial release facilitated positional release

X

3 Assume acute spasm of left upper thoracic paraspinal muscles appropriate techniques to include counterstrainindirect myofascial release and facilitated positional release

X

4 Autonomic technique for altered vagal tone OA decompression X

5 Autonomic technique for hypersympathetic tone rib raising or paraspinal inhibition for gt 90 seconds

X

MODULE 7 ndash Emmitt Brown

1 AAA kidney stones groin hernia ileus kidneybladder CA testicular torsion kidneybladder infection appendicitis

2 C3 D

Osteopathic Diagnosis assess where you would expect to find somatic dysfunctions

Performed Omitted

1 Examine the cervical region for somatic dysfunctionA OA ndash leftright translation1 neutral X2 flexion X3 extension XB AA ndash rotation must flex at least 45deg X2 Examine the thoracic spine for dysfunction (T10-L2)A palpate transverse processes for rotation1 neutral X2 flexion X3 extension XB palpate soft tissue for TART findings4 T10 ndash L2 Paraspinal muscles X3 Examine the sacrum for somatic dysfunction (requires at least one motion test)A depth at 4 corners of sacrum XB ILA levelness XC seated flexion test XD motion at 4 corners of sacrum XE motion in backward bending test XF respiratory motion of sacrum assess at 4 corners X4 Examine the pelvis for somatic dysfunctionA standing flexion test or AP compression test XB ASIS heights XC PSIS heights XOsteopathic Treatment You must treat a minimum of 2 dysfunctions you found during your examination1 Assume OA NSLRR appropriate techniques to include HVLA muscle energy articulatory Still counterstrainindirect myofascial release facilitated positional release

X _

2 Assume T10 NRRSL appropriate techniques to include HVLA muscle energy articulatory Still counterstrainindirect myofascial release facilitated positional release

X

3 Assume acute bilateral spasm of thoracolumbar (T10-L2) paraspinal muscles RgtL appropriate techniques to include soft tissue counterstrainindirect facilitated positional release

X

4 Assume left on right sacral torsion appropriate techniques to include muscle energy counterstrainindirect myofascial release and facilitated positional release (No HVLA)

X

5 Autonomic technique for hypersympathetic tone in T10-L2 paraspinal muscles rib raising or paraspinal inhibition for gt 90 seconds

X

MODULE 8 ndash Peter Parker1 Kidney stone Psoas syndrome spondylolysis piriformis syndrome lumbar compression

fracture appendicitis pelvicabdominal CA2 C3 B

Osteopathic Diagnosis assess where you would expect to find somatic dysfunctions

Performed Omitted

1 Examine the thoracolumbar region (T10-L5) for somatic dysfunctionA palpate transverse processes for rotation1 neutral _2 flexion _3 extension _B palpate soft tissue for TART (Tenderness Asymmetry Restrictions Tissue texture change) findings1 Psoas muscles _2 Piriformis muscles _3 Paraspinal muscles _2 Examine the sacrum for somatic dysfunction (requires at least one motion test)A depth at 4 corners of sacrum _B ILA levelness _C seated flexion test _D motion at 4 corners of sacrum _E motion in backward bending test _F respiratory motion of sacrum assess at 4 corners _3 Examine the pelvis for somatic dysfunctionA standing flexion test or AP compression test _B ASIS heights _C PSIS heights _Osteopathic Treatment You must treat a minimum of 2 dysfunctions you found during your examination1 Assume L1 FRRSR appropriate techniques to include muscle energy counterstrain indirect myofascial release facilitated positional release (No HVLA)

_

2 Assume left on right sacral torsion appropriate techniques to include muscle energy counterstrainindirect myofascial release and facilitated positional release (No HVLA)

_

3 Assume spasmtender point in the right psoas muscle appropriate techniques to include counterstrainindirect myofascial release FPR ME

_

4 Assume spasmtender point in the left piriformis muscle appropriate techniques to include counterstrainindirect myofascial release FPR ME

_

MODULE 9 ndash Ingrid Bergman

1 Lumbar strain Dysmenorrhea PID lumbar strain ovarian cyst sacral somatic dysfunction2 D3 E

Osteopathic Diagnosis assess where you would expect to find somatic dysfunctions

Performed Omitted

1 Examine the lumbar region (L1-L5) for somatic dysfunctionA palpate transverse processes for rotation1 neutral _2 flexion _3 extension _B palpate soft tissue for TART (Tenderness Asymmetry Restrictions Tissue texture change) findings1 quadratus lumborum muscles _2 Piriformis muscles _3 Paraspinal muscles _2 Examine the sacrum for somatic dysfunction (requires at least one motion test)A depth at 4 corners of sacrum _B ILA levelness _C seated flexion test _D motion at 4 corners of sacrum _E motion in backward bending test _F respiratory motion of sacrum assess at 4 corners _3 Examine the pelvis for somatic dysfunctionA standing flexion test or AP compression test _B ASIS heights _C PSIS heights _Osteopathic Treatment You must treat a minimum of 2 dysfunctions you found during your examination1 Assume L3 FRLSL2 Assume left on right sacral torsion _3 Assume left superior innominate shear _

4 Autonomic technique for altered parasympathetic tone

Module 10 ndash Minnie Driver

Pretest

1 Pneumonia dissecting aortic aneurysm thoracic spine fracture MI pleurisy2 B3 E

Osteopathic Diagnosis assess where you would expect to find somatic dysfunctions

Performed Omitted

1 Examine the cervical region for somatic dysfunctionA OA ndash leftright translation1 neutral 104863510486352 flexion 104863510486353 extension 10486351048635B AA ndash rotation must flex at least 45deg 10486351048635C C2-C7 - translation1 neutral 104863510486352 flexion 104863510486353 extension 104863510486352 Examine the thoracic spine for dysfunction (T5-T9)A palpate transverse processes for rotation1 neutral 104863510486352 flexion 104863510486353 extension 10486351048635B palpate soft tissue for TART findings1 trapezius 104863510486352 rhomboids 104863510486353 levator scapulae 104863510486354 T5 ndash T9 Paraspinal muscles 104863510486353 Examine the ribs for somatic dysfunctionA Pump handle ribs (ribs 2-6) at or near sternal junction1 static 104863510486352 motion 10486351048635B Bucket handle ribs (ribs 6-10) between anterior and posterior axillary lines1 static 104863510486352 motion 10486351048635Osteopathic Treatment You must treat a minimum of 2 dysfunctions you found during your examination1 Assume T5 ESLRL 104863510486352 Assume acute bilateral spasm of upper thoracic (T4-T8) paraspinal 10486351048635

muscles3 Assume (R) Rib 2 pump handle inhalation dysfunction 104863510486354 Autonomic technique for altered vagal tone OA decompression 10486351048635

MODULE 11 ndash Jeff Weaver

1 Cervical radiculopathy thoracic outlet syndrome cubital tunnel syndrome carpal tunnel syndrome angina pectoris ulnar groove entrapment Guillian Barre

2 A3 B4 C

Osteopathic Diagnosis assess where you would expect to find somatic dysfunctions

Performed Omitted

1 Examine the cervical region for somatic dysfunctionA C2-C7 - translation1 neutral 104863510486352 flexion 104863510486353 extension 10486351048635B palpate soft tissue for TART (Tenderness Asymmetry Restrictions Tissue texture change) findings1 cervical paraspinal muscles 104863510486352 sternocleidomastoids 104863510486353 scalenes 104863510486352 Examine the thoracic spine for somatic dysfunctionA palpate transverse processes for rotation1 neutral 104863510486352 flexion 104863510486353 extension 10486351048635B palpate soft tissue for TART findings1 trapezius 104863510486352 pectoral musculature 104863510486353 T1-T4 paraspinal muscle 104863510486353 Examine the 1st rib for somatic dysfunction1 static 104863510486352 dynamic 10486351048635Osteopathic Treatment You must treat a minimum of 2 dysfunctions you found during your examination1 Assume hypertonic left pectoral muscle Appropriate techniques to include ME MFR counterstrain

10486351048635

2 Assume hypertonic left scalene muscle Appropriate techniques to include MFR counterstrain ME

10486351048635

3 Assume T4 NRLSR appropriate techniques to include HVLA muscle energy articulatory Still counterstrainindirect myofascial release facilitated positional release

10486351048635

4 Assume left sided thoracic paraspinal hypertonicity T1 ndash T4 appropriate techniques to include soft tissue deep pressure counterstrainindirect facilitated positional release

10486351048635

5 Assume left 1 st rib inhalation somatic dysfunction appropriate techniques to include HVLA ME Stillrsquos articulatory

10486351048635

MODULE 12 ndash Scarlett O-Hara

1 Osteoarthritis piriformis syndrome spinal stenosis lumbar radiculopathy DVT2 C3 D

Osteopathic Diagnosis assess where you would expect to find somatic dysfunctions

Performed Omitted

1 Examine the lumbar region (L1-L5) for somatic dysfunctionA palpate transverse processes for rotation1 neutral _2 flexion _3 extension _B palpate soft tissue for TART (Tenderness Asymmetry Restrictions Tissue texture change) findings1 Psoas muscles _2 Piriformis muscles _3 Paraspinal muscles _2 Examine the sacrum for somatic dysfunction (requires at least one motion test)A depth at 4 corners of sacrum _B ILA levelness _C seated flexion test _D motion at 4 corners of sacrum _E motion in backward bending test _F respiratory motion of sacrum assess at 4 corners _3 Examine the pelvis for somatic dysfunctionA standing flexion test or AP compression test _B ASIS heights _C PSIS heights _Osteopathic Treatment You must treat a minimum of 2 dysfunctions you found during your examination1 Assume left on right sacral torsion appropriate techniques to include muscle energy counterstrainindirect myofascial release and facilitated positional release (No HVLA)2 Assume left innominate anterior appropriate techniques to include muscle energy counterstrainindirect myofascial release and facilitated positional release (No HVLA)

_

3 Assume spasmtender point in the left piriformis muscle appropriate techniques to _

include counterstrainindirect myofascial release

MODULE 13 Clark Kent1 Asthma atypical pneumonia bronchitis upper respiratory infection pneumothorax2 B3 A

Osteopathic Diagnosis assess where you would expect to find somatic dysfunctions

Performed Omitted

1 Examine the cervical region for somatic dysfunctionA OA ndash leftright translation1 neutral _2 flexion _3 extension _B AA ndash rotation must flex at least 45deg _C C2-C7 - translation1 neutral 2 flexion _3 extension 2 Examine the thoracic spine for dysfunction (T1-T4)A palpate transverse processes for rotation1 neutral _2 flexion _3 extension _B palpate soft tissue for TART findings1 trapezius X2 rhomboids X3 levator scapulae X4 T1 ndash T4 Paraspinal muscles X3 Examine the ribs for somatic dysfunctionA Pump handle ribs (ribs 2-6) at or near sternal junction1 static X2 motion XB Bucket handle ribs (ribs 6-10) between anterior and posterior axillary lines1 static X2 motion XOsteopathic Treatment You must treat a minimum of 2 dysfunctions you found during your examination1 Assume OA NSLRR appropriate techniques to include HVLA muscle energy articulatory Still counterstrainindirect myofascial release facilitated positional release

_

2 Assume T2-4 NSLRR appropriate techniques to include HVLA muscle energy articulatory Still counterstrainindirect myofascial release facilitated positional release

_

3 Assume (L) ribs 2-4 exhalation pump handle appropriate techniques to include HVLA muscle energy articulatory Still counterstrainindirect myofascial release facilitated positional release

4 Autonomic technique for altered vagal tone OA decompression _5 Autonomic technique for hypersympathetic tone in T1-T5 paraspinal muscles rib raising or paraspinal inhibition for gt 90 seconds

X

MODULE 14 Bruce Wayne

1 Depression thyroid disorder migraine headache tension tension headache migraine2 C3 A

Osteopathic Diagnosis assess where you would expect to find somatic dysfunctions

Performed Omitted

1 Examine the cervical region for somatic dysfunctionA OA ndash leftright translation1 neutral 2 flexion 3 extension B AA ndash rotation must flex at least 45deg C C2-C7 - translation1 neutral 2 flexion 3 extension _2 Examine the thoracic spine for dysfunction (T1-T4)A palpate transverse processes for rotation1 neutral X2 flexion X3 extension XB palpate soft tissue for TART findings1 trapezius X2 T1 ndash T4 Paraspinal muscles XOsteopathic Treatment You must treat a minimum of 2 dysfunctions you found during your examination1 Assume OA NSRRL appropriate techniques to include HVLA muscle energy articulatory Still counterstrainindirect myofascial release facilitated positional release

2 Assume C4 FSLRL appropriate techniques to include HVLA muscle energy

articulatory Still counterstrainindirect myofascial release facilitated positional release2 Assume T2-4 NSLRR appropriate techniques to include HVLA muscle energy articulatory Still counterstrainindirect myofascial release facilitated positional release

_

4 Autonomic technique for altered vagal tone OA decompression _

  • Performed
  • Performed
  • Performed
  • Performed
  • Performed
  • Performed
Page 2: Performed · Web viewAAA, kidney stones, groin hernia, ileus, kidney/bladder CA, testicular torsion, kidney/bladder infection, appendicitis, C D Osteopathic Diagnosis: assess where

4 You assess OA translation and find translation to be easier to the right The findings persist in both flexion and extension Which of the following is the correct diagnosis

a Flexed rotated right sidebent left

b Neutral rotated left sidebent right

c Neutral rotated right sidebent left

d Neutral rotated left sidebent left

e Extended rotated right sidebent right

5 You assess C4 translation and find translation to be easier to the left The findings become symmetrical in flexion but not in extension Which of the following is the correct diagnosis

a Flexed rotated right sidebent left

b Neutral rotated left sidebent right

c Neutral rotated right sidebent left

d Flexed rotated right sidebent right

e Extended rotated right sidebent right

6 Which of the following is true regarding rib motion

a Rib 7 is primarily caliper motion

b Motion occurs on a vertical axis

c Rib 11 is predominantly bucket handle motion

d Rib 1 is predominantly anteriorposterior motion

e Rib 3 is both pump handle and bucket handle motion X

7 Which of the following is a finding of a rib inhalation somatic dysfunction

a Lives in exhalation

b Prominence on anterior chest

c Limitation to inhalation

d Prominence on posterior chest

e Recession on anterior chest

8 Which of the following is true regarding rib diagnosis

a Ribs that only move during inhalation are exhalation dysfunctions

b Palpation of Rib 1 is performed inferior to the clavicle

c Bucket handle motion is assessed in the mid-axillary line

d Anterior rib prominence is a finding of an exhalation dysfunction

e An inhalation dysfunction will have both anterior and posterior prominence

9 Which of the following is a test of laterality for assessment of innominate dysfunctions

a Respiratory motion test

b ASIS compression test

c ASIS to midline distance

d Seated flexion test

e Relative static PSIS heights

10 Which of the following findings would be expected in a right innominate anterior somatic Dysfunction

a Positive seated flexion test on the right

b Positive ASIS compression test on the left

c Right ASIS inferior compared to left

d Left PSIS superior compared to right

e Right ASIS closer to midline than right

11 You assess the innominates and find the following

Positive standing flexion test on the leftRight ASIS inferior compared to leftRight PSIS inferior compared to leftASIS to midline distance equal bilaterallyWhich of the following is the correct diagnosis

a Right innominate anterior

b Left superior innominate shear

c Left innominate posterior

d Left innominate inflare

e Right innominate posterior

12 Which of the following is true regarding the seated flexion test

a If both sides rise the same amount it indicates that no dysfunction is present

b The test is positive opposite the side of greatest PSIS excursion

c The axis of a sacral torsion is located on the same side of greatest excursion

d A unilateral dysfunction will be on the same side of the greater excursion

e A false negative occurs with unilateral dysfunctions

13 Which of the following is true regarding sacral torsions

a The right base moves forward in a left on right torsion

b The axis passes through the right base in a left on left torsion

c A right on right torsion is termed a backward sacral torsion

d The right sulcus is deeper in a right on right torsion

e Points along the axis do not move in a sacral torsion

14 Which of the following is true regarding a bilateral sacral extension

a The sacral sulci are deep bilaterally

b The seated flexion test will be positive

c The base will not move posteriorly during inhalation

d The base will not move posteriorly during exhalation

e The sacral sulci are shallow bilaterally

15 You assess the sacrum and find the following

Positive seated flexion test on the leftLeft sulcus shallowLeft ILA markedly superiorWhich of the following is the correct diagnosis

a Right on left sacral torsion

b Right unilateral sacral flexion

c Left on right sacral torsion

d Left unilateral sacral extension

e Left on left sacral torsion

16 You examine T7 and find the left transverse process to be deeper than the right The findings become symmetrical in flexion and are unchanged in extension Which of the following is the correct diagnosis

a T7 flexed sidebent left rotated left

b T7 flexed sidebent right rotated right

c T7 extended sidebent left rotated left

d T7 extended sidebent right rotated left

e T7 flexed sidebent left rotated right

17 Which of the following is true regarding lumbar diagnosis

a Rotation and sidebending occur to opposite sides in flexion dysfunctions

b If the right transverse process is deeper the vertebra is rotated to the left

c Rotation and sidebending occur to the same side in neutral dysfunctions

d A lumbar dysfunction that becomes symmetrical in both flexion and extension

is an extension dysfunction

e Only Fryettersquos Principle 1 applies to the lumbar spine

18 Which of the following is true regarding cervical mechanicsdiagnosis

a Motion testing of the AA is performed by translation

b C2-C7 only follow Fryettersquos Principle 1

c The OA is the only cervical segment that must be tested in flexion extension and neutral

d Right translation is equal to left sidebending at C4

e Left translation is equal to left rotation at C6

19 Which of the following is true regarding sacral dysfunctions

a A right unilateral sacral flexion will have a positive seated flexion test on the left

b A left on left sacral torsion will have a deep right sulcus

c A right on left sacral torsion will have a deep right sacral sulcus

d A left unilateral seated flexion will have a significantly higher ILA on the left

e A bilateral sacral flexion will have bilaterally shallow sulci

20 Which of the following is true regarding innominate dysfunctions

a Innominate flares are named for the side on which the ASIS is closer to midline

b The standing flexion test is negative with an innominate shear

c The ASIS compression test will be positive on the side opposite the positive standing flexion test

d A positive seated flexion test is expected on the right with a right innominate posterior

e The right ASIS is inferior with a right innominate anterior

21 You evaluate the 3rd ribs bilaterally and find the following

Right 3rd rib more prominent anteriorlyBoth sides rise during inhalationOnly the left side falls during exhalationWhich of the following is the correct diagnosis

a Right 3rd rib inhalation dysfunction

b Left 3rd rib inhalation dysfunction

c Right 3rd rib inferior dysfunction

d Left 3rd rib exhalation dysfunction

e Left 3rd rib superior dysfunction

22 Which of the following is paired correctly

a Fryettersquos Principle 1 ndash Typically involves a single segment

b AA dysfunctions ndash Assessed with rotation

c Ribs 11 amp 12 ndash Predominantly superiorinferior motion

d Left oblique sacral axis ndash Positive left seated flexion test

e Right innominate posterior ndash Positive left standing flexion test

23 Which of the following dysfunctions will have a left positive standing flexion test and

a negative AP compression test

a Left innominate posterior

b Right superior innominate shear

c Left innominate outflare

d Right innominate inflare

e Left inferior innominate shear

24 You examine L1 and find the right transverse process to be more shallow than the left The findings are unchanged in both flexion and extension Which of the following is the correct diagnosis

a L1 flexed sidebent left rotated left

b L1 neutral sidebent right rotated right

c L1 neutral sidebent left rotated left

d L1 extended sidebent right rotated left

e L1 neutral sidebent left rotated right

25 Which of the following is true regarding a right superior innominate shear

a Negative standing flexion test

b Negative AP compression test

c ASIS and PSIS both superior on the right

d Right ASIS closer to midline

e Positive standing flexion test on the left

MODULE 2

1 Which of the following is true regarding muscle energy treatment

a Contraindicated in patients with osteopenia

b Primary treatment for patients in the ICU

c Utilizes patient force with equal physician counterforce X

d Patient effort is held for 90 seconds

e A-P curve must be flattened out during treatment

2 A patient is diagnosed with a right anterior innominate dysfunction Which of the

following is the correct description for muscle energy treatment of this dysfunction

a Right hip extended ndash patient flexes hip against resistance

b Right hip flexed ndash patient extends hip against resistance X

c Right hip extended ndash patient extends hip against resistance

d Right hip externally rotated ndash patient internally rotates hip against resistance

e Right hip abducted ndash patient adducts hip against resistance

3 Which of the following is true regarding HVLA thrust technique

a Indirect technique

b Useful in patients with joint hypermobility

c Thrust applied after backing away from barrier for 1-2 seconds

d Lumbar HVLA is contraindicated in patients with Down Syndrome

e Restrictive barrier must be engaged in all planes of motion X

4 Which of the following is true regarding Counterstrain treatment

a Restrictive barrier must be engaged in all planes of motion

b Treatment position should decrease tenderness by at least 70 X

c Treatment position is held for 3-4 seconds then released

d Patient assists in positioning for treatment

e Patient must slowly bring himself to neutral after treatment

5 Which of the following is correctly matched

a Counterstrain treatment of hip flexor ndash patientrsquos hip passively flexed X

b FPR treatment of shoulder external rotator ndash patientrsquos shoulder internally rotated

c HVLA treatment of C3 ERRSR ndash C3 placed into extension right rotation and right sidebending

d Muscle energy treatment of a bilateral sacral flexion ndash pressure on sacral base

e HVLA treatment of right unilateral sacral extension ndash superior thrust on right ILA

6 The heart receives sympathetic innervation from which of the following levels

a OA

b T1-T5 X

c T5-T9

d T10-L2

e S2-S4

7 Which of the following is true regarding FPR treatment

a Restrictive barrier must be engaged in all planes of motion

b Treatment position should decrease tenderness by at least 70

c Treatment position is held for 3-4 seconds then released X

d A-P curve is accentuated during treatment

e Facilitating force is an isometric contraction of the involved muscles

8 Which of the following is true about autonomic control of the lungs

a Sympathetic stimulation results in bronchoconstriction

b The lower lungs receive parasympathetic innervation from S2-S4

c Sympathetic innervation arises from the T5-T9 levels

d Increased thick secretions are a result of increased sympathetic activity X

e Sacral rocking will result in increased thin watery secretions

9 Which of the following techniques would be most useful in improving upper GI

motility in a patient with ileus

a Sacral rocking

b Rib raising for 30 seconds

c OA decompression

d Paraspinal inhibition for 120 seconds X

e Sacral inhibition

10 You assess C4 translation and find translation to be easier to the left The findings

become symmetrical in flexion but not in extension Which of the following is the

correct position of C4 for a HVLA thrust

a Flexed rotated right sidebent left

b Extended rotated left sidebent left X

c Neutral rotated right sidebent left

d Flexed rotated right sidebent right

e Extended rotated right sidebent right

11 You examine L3 and find the right transverse process to be deeper than the left The

findings are unchanged in flexion and become symmetrical in extension Which of

the following is the correct position of L3 for FPR treatment

a Flexed sidebent left rotated left

b Flexed sidebent right rotated right

c Extended sidebent left rotated left X

d Extended sidebent right rotated left

e Flexed sidebent left rotated right

12 Which of the following is true regarding autonomic innervation to the GU system

a Sympathetic stimulation increases ureteral peristalsis

b Sympathetic stimulation relaxes the bladder wall X

c Parasympathetic stimulation activates the bladder sphincter

d Parasympathetic stimulation activates the trigone

e Sympathetic stimulation increases GFR

13 Which of the following is a result of parasympathetic stimulation

a Peripheral vasodilation

b Ejaculation

c Increased salivation X

d Uterine contraction

e Ureterospasm

14 The gall bladder receives sympathetic innervation from which of the following

levels

a OA

b T1-T5

c T5-T9 X

d T10-L2

e S2-S4

15 You are attempting to treat a right 7th rib exhalation dysfunction with counterstrain

Which of the following is correct regarding the treatment

a Shaft of right 7th rib positioned closer to shaft of right 8th rib X

b Position should decrease tenderness by 50

c Position is held for 30 seconds

d Angle of right 7th rib tractioned inferiorly

e Positioning involves left sidebending and extension

16 Which of the following is correctly matched

a Muscle energy ndash indirect treatment

b HVLA ndash articulation positioned in free motion

c Counterstrain ndash starts with a pain scale X

d FPR ndash active direct treatment

e Muscle energy ndash useful for painful muscle groups

17 Which of the following would be most likely to inhibit uterine contractions in a

patient in premature labor

a Sacral rocking X

b OA decompression

c Paraspinal inhibition gt90 seconds at T5-T9

d Rib raising lt60 seconds at T10-L2

e Sacral inhibition

18 You assess the innominates and find the following

Positive standing flexion test on the left

Right ASIS inferior compared to left

Right PSIS inferior compared to left

ASIS to midline distance equal bilaterally

Which of the following correctly describes a treatment for this dysfunction

a Muscle energy ndash left hip flexed patient extends against resistance

b HVLA thrust ndash thrust directed inferiorly on left innominate X

c FPR ndash right innominate pushed inferiorly for 3-4 seconds

d Muscle energy ndash right hip pushed superiorly patient pushes inferiorly

e Counterstrain ndash left innominate held inferiorly for 90 seconds

19 The pancreas receives parasympathetic innervation from which of the following

levels

a OA X

b T1-T5

c T5-T9

d T10-L2

e S2-S4

20 Which of the following would most likely directly decrease gastric secretions

a Sacral rocking

b OA decompression

c Paraspinal inhibition gt90 seconds at T10-L2

d Rib raising lt60 seconds at T5-T9 X

e Sacral inhibition

21 Which of the following statements is true regarding osteopathic treatment techniques

a Counterstrain is contraindicated for painful muscle groups

b Osteoporosis is a contraindication for FPR

c Muscle energy should not be used on a person in the CCU X

d HVLA is useful in patients with active Rheumatoid Arthritis

e The restrictive barrier is engaged when using indirect techniques

22 You examine T7 and find the left transverse process to be deeper than the right The

findings become symmetrical in flexion and are unchanged in extension Which of

the following is the correct starting position for a muscle energy treatment

a T7 flexed sidebent left rotated left

b T7 flexed sidebent right rotated right

c T7 extended sidebent left rotated left X

d T7 extended sidebent right rotated left

e T7 flexed sidebent left rotated right

23 The sigmoid colon receives parasympathetic innervation from which of the following

levels

a OA

b T1-T5

c T5-T9

d T10-L2

e S2-S4 X

24 You assess OA translation and find translation to be easier to the right The findings

persist in both flexion and extension Which of the following is the correct position

for FPR treatment

a Flexed rotated right sidebent left

b Neutral rotated left sidebent right

c Neutral rotated right sidebent left X

d Neutral rotated left sidebent left

e Extended rotated right sidebent right

25 Which of the following would most likely directly increase lacrimation

a Sacral rocking

b OA decompression X

c Paraspinal inhibition gt90 seconds at T5-T9

d Rib raising lt60 seconds at T1-T4 X

e Sacral inhibition

MODULE 3 ndash Joanne Smith

1 Top 5 ddx peptic ulcer GERD costochondritis Pancreastitis MI angina pectoris2 E no contraindications3 A OA4 E strep throat

dysfunctions Performed Omitted

1 Examine the cervical region for somatic dysfunctionA OA ndash leftright translation1 neutral 10486351048635

2 flexion 104863510486353 extension 104863510486352 Examine the thoracic spine for dysfunction (T5-T9)A palpate transverse processes for rotation1 neutral 104863510486352 flexion 104863510486353 extension 10486351048635B palpate soft tissue for TART findings1 trapezius 104863510486352 rhomboids 104863510486353 levator scapulae 104863510486354 T5 ndash T9 Paraspinal muscles 10486351048635Osteopathic Treatment You must treat a minimum of 2 dysfunctions you found during your examination1 Assume OA NSLRR appropriate techniques to include HVLA muscle energy articulatory Still counterstrainindirect myofascial release facilitated positional release

10486351048635

2 Assume T7 FRLSL appropriate techniques to include HVLA muscle energy articulatory Still counterstrainindirect myofascial release facilitated positional release

10486351048635

3 Assume acute bilateral spasm of upper thoracic (T5-T9) paraspinal muscles appropriate techniques to include soft tissue counterstrainindirect facilitated positional release

10486351048635

4 Autonomic technique for altered vagal tone OA decompression 104863510486355 Autonomic technique for hypersympathetic tone in T5-T9 paraspinal muscles rib raising or paraspinal inhibition for gt 90 seconds

10486351048635

MODULE 4- Ernesto Rodriguez

PRETEST

1 Back pain from Colon cancer Lumbar strain Osteomyelitits Diverticulitis Back pain osteoarthritis Cauda equina syndrome

2 D3 E

Osteopathic Diagnosis assess where you would expect to find somatic dysfunctions

Performed Omitted

1 Examine the lumbar region (L1-L5) for somatic dysfunction

A palpate transverse processes for rotation

1 neutral _

2 flexion _

3 extension _

B palpate soft tissue for TART (Tenderness Asymmetry Restrictions Tissue texture change) findings

1 quadratus lumborum muscles _

2 Piriformis muscles _

3 Paraspinal muscles _

2 Examine the sacrum for somatic dysfunction (requires at least one motion test)

A depth at 4 corners of sacrum _

B ILA levelness _

C seated flexion test _

D motion at 4 corners of sacrum _

E motion in backward bending test _

F respiratory motion of sacrum assess at 4 corners _

3 Examine the pelvis for somatic dysfunction

A standing flexion test or AP compression test _

B ASIS heights _

C PSIS heights _

Osteopathic Treatment You must treat a minimum of 2 dysfunctions you found during your examination

1 Assume L5 FRLSL _

2 Assume left on left sacral torsion _

3 Assume left innominate anterior _

4 Assume spasmtender point in the left piriformis muscle appropriate techniques to include counterstrainindirect myofascial release

-

MODULE 5 ndash Connie Jones

Pretest

1 Bradycardia acute pulmonary edemaCHF pneumonia acute MI Acute pulmonary edema CHF2 A3 A4 A5 For the video this is what was done and not done

OA - nf NO E --gtNslrrno AA dx doneno cervical dx doneT1-T5 - nfe --gt t2 f sl rlparapsinals TART --gt L upper thoracic paraspinalno levator scapula no trap no rhomboids dx tx -

T2 HLVAOA decompr ndash for vagal tonerib raisin for sympth

MODULE 6 ndash Gretchen White

1 Atrial fibrillation hyperthyroidism and- thyrotoxicosis thyroid cancer hyperthryroid 2 a3 a4 a5

Osteopathic Diagnosis assess where you would expect to find somatic dysfunctions

Performed Omitted

1 Examine the cervical region for somatic dysfunction

A OA ndash leftright translation

1 neutral X

2 flexion X

6 3 extension X

7 B AA ndash rotation X

8 2 Examine the upper thoracic spine for dysfunction (T1-T5)

A palpate transverse processes for rotation

1 neutral X

2 flexion X

3 extension X

B palpate soft tissue for TART findings

1 trapezius X

2 rhomboids X

3 levator scapulae X

4 T1-T5 paraspinal muscles X

Osteopathic Treatment You must treat a minimum of 2 dysfunctions you found during your examination

1 Assume OA NSLRR appropriate techniques to include counterstrainindirect myofascial release facilitated positional release

X

2 Assume T2 FSLRL appropriate techniques to include counterstrainindirect myofascial release facilitated positional release

X

3 Assume acute spasm of left upper thoracic paraspinal muscles appropriate techniques to include counterstrainindirect myofascial release and facilitated positional release

X

4 Autonomic technique for altered vagal tone OA decompression X

5 Autonomic technique for hypersympathetic tone rib raising or paraspinal inhibition for gt 90 seconds

X

MODULE 7 ndash Emmitt Brown

1 AAA kidney stones groin hernia ileus kidneybladder CA testicular torsion kidneybladder infection appendicitis

2 C3 D

Osteopathic Diagnosis assess where you would expect to find somatic dysfunctions

Performed Omitted

1 Examine the cervical region for somatic dysfunctionA OA ndash leftright translation1 neutral X2 flexion X3 extension XB AA ndash rotation must flex at least 45deg X2 Examine the thoracic spine for dysfunction (T10-L2)A palpate transverse processes for rotation1 neutral X2 flexion X3 extension XB palpate soft tissue for TART findings4 T10 ndash L2 Paraspinal muscles X3 Examine the sacrum for somatic dysfunction (requires at least one motion test)A depth at 4 corners of sacrum XB ILA levelness XC seated flexion test XD motion at 4 corners of sacrum XE motion in backward bending test XF respiratory motion of sacrum assess at 4 corners X4 Examine the pelvis for somatic dysfunctionA standing flexion test or AP compression test XB ASIS heights XC PSIS heights XOsteopathic Treatment You must treat a minimum of 2 dysfunctions you found during your examination1 Assume OA NSLRR appropriate techniques to include HVLA muscle energy articulatory Still counterstrainindirect myofascial release facilitated positional release

X _

2 Assume T10 NRRSL appropriate techniques to include HVLA muscle energy articulatory Still counterstrainindirect myofascial release facilitated positional release

X

3 Assume acute bilateral spasm of thoracolumbar (T10-L2) paraspinal muscles RgtL appropriate techniques to include soft tissue counterstrainindirect facilitated positional release

X

4 Assume left on right sacral torsion appropriate techniques to include muscle energy counterstrainindirect myofascial release and facilitated positional release (No HVLA)

X

5 Autonomic technique for hypersympathetic tone in T10-L2 paraspinal muscles rib raising or paraspinal inhibition for gt 90 seconds

X

MODULE 8 ndash Peter Parker1 Kidney stone Psoas syndrome spondylolysis piriformis syndrome lumbar compression

fracture appendicitis pelvicabdominal CA2 C3 B

Osteopathic Diagnosis assess where you would expect to find somatic dysfunctions

Performed Omitted

1 Examine the thoracolumbar region (T10-L5) for somatic dysfunctionA palpate transverse processes for rotation1 neutral _2 flexion _3 extension _B palpate soft tissue for TART (Tenderness Asymmetry Restrictions Tissue texture change) findings1 Psoas muscles _2 Piriformis muscles _3 Paraspinal muscles _2 Examine the sacrum for somatic dysfunction (requires at least one motion test)A depth at 4 corners of sacrum _B ILA levelness _C seated flexion test _D motion at 4 corners of sacrum _E motion in backward bending test _F respiratory motion of sacrum assess at 4 corners _3 Examine the pelvis for somatic dysfunctionA standing flexion test or AP compression test _B ASIS heights _C PSIS heights _Osteopathic Treatment You must treat a minimum of 2 dysfunctions you found during your examination1 Assume L1 FRRSR appropriate techniques to include muscle energy counterstrain indirect myofascial release facilitated positional release (No HVLA)

_

2 Assume left on right sacral torsion appropriate techniques to include muscle energy counterstrainindirect myofascial release and facilitated positional release (No HVLA)

_

3 Assume spasmtender point in the right psoas muscle appropriate techniques to include counterstrainindirect myofascial release FPR ME

_

4 Assume spasmtender point in the left piriformis muscle appropriate techniques to include counterstrainindirect myofascial release FPR ME

_

MODULE 9 ndash Ingrid Bergman

1 Lumbar strain Dysmenorrhea PID lumbar strain ovarian cyst sacral somatic dysfunction2 D3 E

Osteopathic Diagnosis assess where you would expect to find somatic dysfunctions

Performed Omitted

1 Examine the lumbar region (L1-L5) for somatic dysfunctionA palpate transverse processes for rotation1 neutral _2 flexion _3 extension _B palpate soft tissue for TART (Tenderness Asymmetry Restrictions Tissue texture change) findings1 quadratus lumborum muscles _2 Piriformis muscles _3 Paraspinal muscles _2 Examine the sacrum for somatic dysfunction (requires at least one motion test)A depth at 4 corners of sacrum _B ILA levelness _C seated flexion test _D motion at 4 corners of sacrum _E motion in backward bending test _F respiratory motion of sacrum assess at 4 corners _3 Examine the pelvis for somatic dysfunctionA standing flexion test or AP compression test _B ASIS heights _C PSIS heights _Osteopathic Treatment You must treat a minimum of 2 dysfunctions you found during your examination1 Assume L3 FRLSL2 Assume left on right sacral torsion _3 Assume left superior innominate shear _

4 Autonomic technique for altered parasympathetic tone

Module 10 ndash Minnie Driver

Pretest

1 Pneumonia dissecting aortic aneurysm thoracic spine fracture MI pleurisy2 B3 E

Osteopathic Diagnosis assess where you would expect to find somatic dysfunctions

Performed Omitted

1 Examine the cervical region for somatic dysfunctionA OA ndash leftright translation1 neutral 104863510486352 flexion 104863510486353 extension 10486351048635B AA ndash rotation must flex at least 45deg 10486351048635C C2-C7 - translation1 neutral 104863510486352 flexion 104863510486353 extension 104863510486352 Examine the thoracic spine for dysfunction (T5-T9)A palpate transverse processes for rotation1 neutral 104863510486352 flexion 104863510486353 extension 10486351048635B palpate soft tissue for TART findings1 trapezius 104863510486352 rhomboids 104863510486353 levator scapulae 104863510486354 T5 ndash T9 Paraspinal muscles 104863510486353 Examine the ribs for somatic dysfunctionA Pump handle ribs (ribs 2-6) at or near sternal junction1 static 104863510486352 motion 10486351048635B Bucket handle ribs (ribs 6-10) between anterior and posterior axillary lines1 static 104863510486352 motion 10486351048635Osteopathic Treatment You must treat a minimum of 2 dysfunctions you found during your examination1 Assume T5 ESLRL 104863510486352 Assume acute bilateral spasm of upper thoracic (T4-T8) paraspinal 10486351048635

muscles3 Assume (R) Rib 2 pump handle inhalation dysfunction 104863510486354 Autonomic technique for altered vagal tone OA decompression 10486351048635

MODULE 11 ndash Jeff Weaver

1 Cervical radiculopathy thoracic outlet syndrome cubital tunnel syndrome carpal tunnel syndrome angina pectoris ulnar groove entrapment Guillian Barre

2 A3 B4 C

Osteopathic Diagnosis assess where you would expect to find somatic dysfunctions

Performed Omitted

1 Examine the cervical region for somatic dysfunctionA C2-C7 - translation1 neutral 104863510486352 flexion 104863510486353 extension 10486351048635B palpate soft tissue for TART (Tenderness Asymmetry Restrictions Tissue texture change) findings1 cervical paraspinal muscles 104863510486352 sternocleidomastoids 104863510486353 scalenes 104863510486352 Examine the thoracic spine for somatic dysfunctionA palpate transverse processes for rotation1 neutral 104863510486352 flexion 104863510486353 extension 10486351048635B palpate soft tissue for TART findings1 trapezius 104863510486352 pectoral musculature 104863510486353 T1-T4 paraspinal muscle 104863510486353 Examine the 1st rib for somatic dysfunction1 static 104863510486352 dynamic 10486351048635Osteopathic Treatment You must treat a minimum of 2 dysfunctions you found during your examination1 Assume hypertonic left pectoral muscle Appropriate techniques to include ME MFR counterstrain

10486351048635

2 Assume hypertonic left scalene muscle Appropriate techniques to include MFR counterstrain ME

10486351048635

3 Assume T4 NRLSR appropriate techniques to include HVLA muscle energy articulatory Still counterstrainindirect myofascial release facilitated positional release

10486351048635

4 Assume left sided thoracic paraspinal hypertonicity T1 ndash T4 appropriate techniques to include soft tissue deep pressure counterstrainindirect facilitated positional release

10486351048635

5 Assume left 1 st rib inhalation somatic dysfunction appropriate techniques to include HVLA ME Stillrsquos articulatory

10486351048635

MODULE 12 ndash Scarlett O-Hara

1 Osteoarthritis piriformis syndrome spinal stenosis lumbar radiculopathy DVT2 C3 D

Osteopathic Diagnosis assess where you would expect to find somatic dysfunctions

Performed Omitted

1 Examine the lumbar region (L1-L5) for somatic dysfunctionA palpate transverse processes for rotation1 neutral _2 flexion _3 extension _B palpate soft tissue for TART (Tenderness Asymmetry Restrictions Tissue texture change) findings1 Psoas muscles _2 Piriformis muscles _3 Paraspinal muscles _2 Examine the sacrum for somatic dysfunction (requires at least one motion test)A depth at 4 corners of sacrum _B ILA levelness _C seated flexion test _D motion at 4 corners of sacrum _E motion in backward bending test _F respiratory motion of sacrum assess at 4 corners _3 Examine the pelvis for somatic dysfunctionA standing flexion test or AP compression test _B ASIS heights _C PSIS heights _Osteopathic Treatment You must treat a minimum of 2 dysfunctions you found during your examination1 Assume left on right sacral torsion appropriate techniques to include muscle energy counterstrainindirect myofascial release and facilitated positional release (No HVLA)2 Assume left innominate anterior appropriate techniques to include muscle energy counterstrainindirect myofascial release and facilitated positional release (No HVLA)

_

3 Assume spasmtender point in the left piriformis muscle appropriate techniques to _

include counterstrainindirect myofascial release

MODULE 13 Clark Kent1 Asthma atypical pneumonia bronchitis upper respiratory infection pneumothorax2 B3 A

Osteopathic Diagnosis assess where you would expect to find somatic dysfunctions

Performed Omitted

1 Examine the cervical region for somatic dysfunctionA OA ndash leftright translation1 neutral _2 flexion _3 extension _B AA ndash rotation must flex at least 45deg _C C2-C7 - translation1 neutral 2 flexion _3 extension 2 Examine the thoracic spine for dysfunction (T1-T4)A palpate transverse processes for rotation1 neutral _2 flexion _3 extension _B palpate soft tissue for TART findings1 trapezius X2 rhomboids X3 levator scapulae X4 T1 ndash T4 Paraspinal muscles X3 Examine the ribs for somatic dysfunctionA Pump handle ribs (ribs 2-6) at or near sternal junction1 static X2 motion XB Bucket handle ribs (ribs 6-10) between anterior and posterior axillary lines1 static X2 motion XOsteopathic Treatment You must treat a minimum of 2 dysfunctions you found during your examination1 Assume OA NSLRR appropriate techniques to include HVLA muscle energy articulatory Still counterstrainindirect myofascial release facilitated positional release

_

2 Assume T2-4 NSLRR appropriate techniques to include HVLA muscle energy articulatory Still counterstrainindirect myofascial release facilitated positional release

_

3 Assume (L) ribs 2-4 exhalation pump handle appropriate techniques to include HVLA muscle energy articulatory Still counterstrainindirect myofascial release facilitated positional release

4 Autonomic technique for altered vagal tone OA decompression _5 Autonomic technique for hypersympathetic tone in T1-T5 paraspinal muscles rib raising or paraspinal inhibition for gt 90 seconds

X

MODULE 14 Bruce Wayne

1 Depression thyroid disorder migraine headache tension tension headache migraine2 C3 A

Osteopathic Diagnosis assess where you would expect to find somatic dysfunctions

Performed Omitted

1 Examine the cervical region for somatic dysfunctionA OA ndash leftright translation1 neutral 2 flexion 3 extension B AA ndash rotation must flex at least 45deg C C2-C7 - translation1 neutral 2 flexion 3 extension _2 Examine the thoracic spine for dysfunction (T1-T4)A palpate transverse processes for rotation1 neutral X2 flexion X3 extension XB palpate soft tissue for TART findings1 trapezius X2 T1 ndash T4 Paraspinal muscles XOsteopathic Treatment You must treat a minimum of 2 dysfunctions you found during your examination1 Assume OA NSRRL appropriate techniques to include HVLA muscle energy articulatory Still counterstrainindirect myofascial release facilitated positional release

2 Assume C4 FSLRL appropriate techniques to include HVLA muscle energy

articulatory Still counterstrainindirect myofascial release facilitated positional release2 Assume T2-4 NSLRR appropriate techniques to include HVLA muscle energy articulatory Still counterstrainindirect myofascial release facilitated positional release

_

4 Autonomic technique for altered vagal tone OA decompression _

  • Performed
  • Performed
  • Performed
  • Performed
  • Performed
  • Performed
Page 3: Performed · Web viewAAA, kidney stones, groin hernia, ileus, kidney/bladder CA, testicular torsion, kidney/bladder infection, appendicitis, C D Osteopathic Diagnosis: assess where

8 Which of the following is true regarding rib diagnosis

a Ribs that only move during inhalation are exhalation dysfunctions

b Palpation of Rib 1 is performed inferior to the clavicle

c Bucket handle motion is assessed in the mid-axillary line

d Anterior rib prominence is a finding of an exhalation dysfunction

e An inhalation dysfunction will have both anterior and posterior prominence

9 Which of the following is a test of laterality for assessment of innominate dysfunctions

a Respiratory motion test

b ASIS compression test

c ASIS to midline distance

d Seated flexion test

e Relative static PSIS heights

10 Which of the following findings would be expected in a right innominate anterior somatic Dysfunction

a Positive seated flexion test on the right

b Positive ASIS compression test on the left

c Right ASIS inferior compared to left

d Left PSIS superior compared to right

e Right ASIS closer to midline than right

11 You assess the innominates and find the following

Positive standing flexion test on the leftRight ASIS inferior compared to leftRight PSIS inferior compared to leftASIS to midline distance equal bilaterallyWhich of the following is the correct diagnosis

a Right innominate anterior

b Left superior innominate shear

c Left innominate posterior

d Left innominate inflare

e Right innominate posterior

12 Which of the following is true regarding the seated flexion test

a If both sides rise the same amount it indicates that no dysfunction is present

b The test is positive opposite the side of greatest PSIS excursion

c The axis of a sacral torsion is located on the same side of greatest excursion

d A unilateral dysfunction will be on the same side of the greater excursion

e A false negative occurs with unilateral dysfunctions

13 Which of the following is true regarding sacral torsions

a The right base moves forward in a left on right torsion

b The axis passes through the right base in a left on left torsion

c A right on right torsion is termed a backward sacral torsion

d The right sulcus is deeper in a right on right torsion

e Points along the axis do not move in a sacral torsion

14 Which of the following is true regarding a bilateral sacral extension

a The sacral sulci are deep bilaterally

b The seated flexion test will be positive

c The base will not move posteriorly during inhalation

d The base will not move posteriorly during exhalation

e The sacral sulci are shallow bilaterally

15 You assess the sacrum and find the following

Positive seated flexion test on the leftLeft sulcus shallowLeft ILA markedly superiorWhich of the following is the correct diagnosis

a Right on left sacral torsion

b Right unilateral sacral flexion

c Left on right sacral torsion

d Left unilateral sacral extension

e Left on left sacral torsion

16 You examine T7 and find the left transverse process to be deeper than the right The findings become symmetrical in flexion and are unchanged in extension Which of the following is the correct diagnosis

a T7 flexed sidebent left rotated left

b T7 flexed sidebent right rotated right

c T7 extended sidebent left rotated left

d T7 extended sidebent right rotated left

e T7 flexed sidebent left rotated right

17 Which of the following is true regarding lumbar diagnosis

a Rotation and sidebending occur to opposite sides in flexion dysfunctions

b If the right transverse process is deeper the vertebra is rotated to the left

c Rotation and sidebending occur to the same side in neutral dysfunctions

d A lumbar dysfunction that becomes symmetrical in both flexion and extension

is an extension dysfunction

e Only Fryettersquos Principle 1 applies to the lumbar spine

18 Which of the following is true regarding cervical mechanicsdiagnosis

a Motion testing of the AA is performed by translation

b C2-C7 only follow Fryettersquos Principle 1

c The OA is the only cervical segment that must be tested in flexion extension and neutral

d Right translation is equal to left sidebending at C4

e Left translation is equal to left rotation at C6

19 Which of the following is true regarding sacral dysfunctions

a A right unilateral sacral flexion will have a positive seated flexion test on the left

b A left on left sacral torsion will have a deep right sulcus

c A right on left sacral torsion will have a deep right sacral sulcus

d A left unilateral seated flexion will have a significantly higher ILA on the left

e A bilateral sacral flexion will have bilaterally shallow sulci

20 Which of the following is true regarding innominate dysfunctions

a Innominate flares are named for the side on which the ASIS is closer to midline

b The standing flexion test is negative with an innominate shear

c The ASIS compression test will be positive on the side opposite the positive standing flexion test

d A positive seated flexion test is expected on the right with a right innominate posterior

e The right ASIS is inferior with a right innominate anterior

21 You evaluate the 3rd ribs bilaterally and find the following

Right 3rd rib more prominent anteriorlyBoth sides rise during inhalationOnly the left side falls during exhalationWhich of the following is the correct diagnosis

a Right 3rd rib inhalation dysfunction

b Left 3rd rib inhalation dysfunction

c Right 3rd rib inferior dysfunction

d Left 3rd rib exhalation dysfunction

e Left 3rd rib superior dysfunction

22 Which of the following is paired correctly

a Fryettersquos Principle 1 ndash Typically involves a single segment

b AA dysfunctions ndash Assessed with rotation

c Ribs 11 amp 12 ndash Predominantly superiorinferior motion

d Left oblique sacral axis ndash Positive left seated flexion test

e Right innominate posterior ndash Positive left standing flexion test

23 Which of the following dysfunctions will have a left positive standing flexion test and

a negative AP compression test

a Left innominate posterior

b Right superior innominate shear

c Left innominate outflare

d Right innominate inflare

e Left inferior innominate shear

24 You examine L1 and find the right transverse process to be more shallow than the left The findings are unchanged in both flexion and extension Which of the following is the correct diagnosis

a L1 flexed sidebent left rotated left

b L1 neutral sidebent right rotated right

c L1 neutral sidebent left rotated left

d L1 extended sidebent right rotated left

e L1 neutral sidebent left rotated right

25 Which of the following is true regarding a right superior innominate shear

a Negative standing flexion test

b Negative AP compression test

c ASIS and PSIS both superior on the right

d Right ASIS closer to midline

e Positive standing flexion test on the left

MODULE 2

1 Which of the following is true regarding muscle energy treatment

a Contraindicated in patients with osteopenia

b Primary treatment for patients in the ICU

c Utilizes patient force with equal physician counterforce X

d Patient effort is held for 90 seconds

e A-P curve must be flattened out during treatment

2 A patient is diagnosed with a right anterior innominate dysfunction Which of the

following is the correct description for muscle energy treatment of this dysfunction

a Right hip extended ndash patient flexes hip against resistance

b Right hip flexed ndash patient extends hip against resistance X

c Right hip extended ndash patient extends hip against resistance

d Right hip externally rotated ndash patient internally rotates hip against resistance

e Right hip abducted ndash patient adducts hip against resistance

3 Which of the following is true regarding HVLA thrust technique

a Indirect technique

b Useful in patients with joint hypermobility

c Thrust applied after backing away from barrier for 1-2 seconds

d Lumbar HVLA is contraindicated in patients with Down Syndrome

e Restrictive barrier must be engaged in all planes of motion X

4 Which of the following is true regarding Counterstrain treatment

a Restrictive barrier must be engaged in all planes of motion

b Treatment position should decrease tenderness by at least 70 X

c Treatment position is held for 3-4 seconds then released

d Patient assists in positioning for treatment

e Patient must slowly bring himself to neutral after treatment

5 Which of the following is correctly matched

a Counterstrain treatment of hip flexor ndash patientrsquos hip passively flexed X

b FPR treatment of shoulder external rotator ndash patientrsquos shoulder internally rotated

c HVLA treatment of C3 ERRSR ndash C3 placed into extension right rotation and right sidebending

d Muscle energy treatment of a bilateral sacral flexion ndash pressure on sacral base

e HVLA treatment of right unilateral sacral extension ndash superior thrust on right ILA

6 The heart receives sympathetic innervation from which of the following levels

a OA

b T1-T5 X

c T5-T9

d T10-L2

e S2-S4

7 Which of the following is true regarding FPR treatment

a Restrictive barrier must be engaged in all planes of motion

b Treatment position should decrease tenderness by at least 70

c Treatment position is held for 3-4 seconds then released X

d A-P curve is accentuated during treatment

e Facilitating force is an isometric contraction of the involved muscles

8 Which of the following is true about autonomic control of the lungs

a Sympathetic stimulation results in bronchoconstriction

b The lower lungs receive parasympathetic innervation from S2-S4

c Sympathetic innervation arises from the T5-T9 levels

d Increased thick secretions are a result of increased sympathetic activity X

e Sacral rocking will result in increased thin watery secretions

9 Which of the following techniques would be most useful in improving upper GI

motility in a patient with ileus

a Sacral rocking

b Rib raising for 30 seconds

c OA decompression

d Paraspinal inhibition for 120 seconds X

e Sacral inhibition

10 You assess C4 translation and find translation to be easier to the left The findings

become symmetrical in flexion but not in extension Which of the following is the

correct position of C4 for a HVLA thrust

a Flexed rotated right sidebent left

b Extended rotated left sidebent left X

c Neutral rotated right sidebent left

d Flexed rotated right sidebent right

e Extended rotated right sidebent right

11 You examine L3 and find the right transverse process to be deeper than the left The

findings are unchanged in flexion and become symmetrical in extension Which of

the following is the correct position of L3 for FPR treatment

a Flexed sidebent left rotated left

b Flexed sidebent right rotated right

c Extended sidebent left rotated left X

d Extended sidebent right rotated left

e Flexed sidebent left rotated right

12 Which of the following is true regarding autonomic innervation to the GU system

a Sympathetic stimulation increases ureteral peristalsis

b Sympathetic stimulation relaxes the bladder wall X

c Parasympathetic stimulation activates the bladder sphincter

d Parasympathetic stimulation activates the trigone

e Sympathetic stimulation increases GFR

13 Which of the following is a result of parasympathetic stimulation

a Peripheral vasodilation

b Ejaculation

c Increased salivation X

d Uterine contraction

e Ureterospasm

14 The gall bladder receives sympathetic innervation from which of the following

levels

a OA

b T1-T5

c T5-T9 X

d T10-L2

e S2-S4

15 You are attempting to treat a right 7th rib exhalation dysfunction with counterstrain

Which of the following is correct regarding the treatment

a Shaft of right 7th rib positioned closer to shaft of right 8th rib X

b Position should decrease tenderness by 50

c Position is held for 30 seconds

d Angle of right 7th rib tractioned inferiorly

e Positioning involves left sidebending and extension

16 Which of the following is correctly matched

a Muscle energy ndash indirect treatment

b HVLA ndash articulation positioned in free motion

c Counterstrain ndash starts with a pain scale X

d FPR ndash active direct treatment

e Muscle energy ndash useful for painful muscle groups

17 Which of the following would be most likely to inhibit uterine contractions in a

patient in premature labor

a Sacral rocking X

b OA decompression

c Paraspinal inhibition gt90 seconds at T5-T9

d Rib raising lt60 seconds at T10-L2

e Sacral inhibition

18 You assess the innominates and find the following

Positive standing flexion test on the left

Right ASIS inferior compared to left

Right PSIS inferior compared to left

ASIS to midline distance equal bilaterally

Which of the following correctly describes a treatment for this dysfunction

a Muscle energy ndash left hip flexed patient extends against resistance

b HVLA thrust ndash thrust directed inferiorly on left innominate X

c FPR ndash right innominate pushed inferiorly for 3-4 seconds

d Muscle energy ndash right hip pushed superiorly patient pushes inferiorly

e Counterstrain ndash left innominate held inferiorly for 90 seconds

19 The pancreas receives parasympathetic innervation from which of the following

levels

a OA X

b T1-T5

c T5-T9

d T10-L2

e S2-S4

20 Which of the following would most likely directly decrease gastric secretions

a Sacral rocking

b OA decompression

c Paraspinal inhibition gt90 seconds at T10-L2

d Rib raising lt60 seconds at T5-T9 X

e Sacral inhibition

21 Which of the following statements is true regarding osteopathic treatment techniques

a Counterstrain is contraindicated for painful muscle groups

b Osteoporosis is a contraindication for FPR

c Muscle energy should not be used on a person in the CCU X

d HVLA is useful in patients with active Rheumatoid Arthritis

e The restrictive barrier is engaged when using indirect techniques

22 You examine T7 and find the left transverse process to be deeper than the right The

findings become symmetrical in flexion and are unchanged in extension Which of

the following is the correct starting position for a muscle energy treatment

a T7 flexed sidebent left rotated left

b T7 flexed sidebent right rotated right

c T7 extended sidebent left rotated left X

d T7 extended sidebent right rotated left

e T7 flexed sidebent left rotated right

23 The sigmoid colon receives parasympathetic innervation from which of the following

levels

a OA

b T1-T5

c T5-T9

d T10-L2

e S2-S4 X

24 You assess OA translation and find translation to be easier to the right The findings

persist in both flexion and extension Which of the following is the correct position

for FPR treatment

a Flexed rotated right sidebent left

b Neutral rotated left sidebent right

c Neutral rotated right sidebent left X

d Neutral rotated left sidebent left

e Extended rotated right sidebent right

25 Which of the following would most likely directly increase lacrimation

a Sacral rocking

b OA decompression X

c Paraspinal inhibition gt90 seconds at T5-T9

d Rib raising lt60 seconds at T1-T4 X

e Sacral inhibition

MODULE 3 ndash Joanne Smith

1 Top 5 ddx peptic ulcer GERD costochondritis Pancreastitis MI angina pectoris2 E no contraindications3 A OA4 E strep throat

dysfunctions Performed Omitted

1 Examine the cervical region for somatic dysfunctionA OA ndash leftright translation1 neutral 10486351048635

2 flexion 104863510486353 extension 104863510486352 Examine the thoracic spine for dysfunction (T5-T9)A palpate transverse processes for rotation1 neutral 104863510486352 flexion 104863510486353 extension 10486351048635B palpate soft tissue for TART findings1 trapezius 104863510486352 rhomboids 104863510486353 levator scapulae 104863510486354 T5 ndash T9 Paraspinal muscles 10486351048635Osteopathic Treatment You must treat a minimum of 2 dysfunctions you found during your examination1 Assume OA NSLRR appropriate techniques to include HVLA muscle energy articulatory Still counterstrainindirect myofascial release facilitated positional release

10486351048635

2 Assume T7 FRLSL appropriate techniques to include HVLA muscle energy articulatory Still counterstrainindirect myofascial release facilitated positional release

10486351048635

3 Assume acute bilateral spasm of upper thoracic (T5-T9) paraspinal muscles appropriate techniques to include soft tissue counterstrainindirect facilitated positional release

10486351048635

4 Autonomic technique for altered vagal tone OA decompression 104863510486355 Autonomic technique for hypersympathetic tone in T5-T9 paraspinal muscles rib raising or paraspinal inhibition for gt 90 seconds

10486351048635

MODULE 4- Ernesto Rodriguez

PRETEST

1 Back pain from Colon cancer Lumbar strain Osteomyelitits Diverticulitis Back pain osteoarthritis Cauda equina syndrome

2 D3 E

Osteopathic Diagnosis assess where you would expect to find somatic dysfunctions

Performed Omitted

1 Examine the lumbar region (L1-L5) for somatic dysfunction

A palpate transverse processes for rotation

1 neutral _

2 flexion _

3 extension _

B palpate soft tissue for TART (Tenderness Asymmetry Restrictions Tissue texture change) findings

1 quadratus lumborum muscles _

2 Piriformis muscles _

3 Paraspinal muscles _

2 Examine the sacrum for somatic dysfunction (requires at least one motion test)

A depth at 4 corners of sacrum _

B ILA levelness _

C seated flexion test _

D motion at 4 corners of sacrum _

E motion in backward bending test _

F respiratory motion of sacrum assess at 4 corners _

3 Examine the pelvis for somatic dysfunction

A standing flexion test or AP compression test _

B ASIS heights _

C PSIS heights _

Osteopathic Treatment You must treat a minimum of 2 dysfunctions you found during your examination

1 Assume L5 FRLSL _

2 Assume left on left sacral torsion _

3 Assume left innominate anterior _

4 Assume spasmtender point in the left piriformis muscle appropriate techniques to include counterstrainindirect myofascial release

-

MODULE 5 ndash Connie Jones

Pretest

1 Bradycardia acute pulmonary edemaCHF pneumonia acute MI Acute pulmonary edema CHF2 A3 A4 A5 For the video this is what was done and not done

OA - nf NO E --gtNslrrno AA dx doneno cervical dx doneT1-T5 - nfe --gt t2 f sl rlparapsinals TART --gt L upper thoracic paraspinalno levator scapula no trap no rhomboids dx tx -

T2 HLVAOA decompr ndash for vagal tonerib raisin for sympth

MODULE 6 ndash Gretchen White

1 Atrial fibrillation hyperthyroidism and- thyrotoxicosis thyroid cancer hyperthryroid 2 a3 a4 a5

Osteopathic Diagnosis assess where you would expect to find somatic dysfunctions

Performed Omitted

1 Examine the cervical region for somatic dysfunction

A OA ndash leftright translation

1 neutral X

2 flexion X

6 3 extension X

7 B AA ndash rotation X

8 2 Examine the upper thoracic spine for dysfunction (T1-T5)

A palpate transverse processes for rotation

1 neutral X

2 flexion X

3 extension X

B palpate soft tissue for TART findings

1 trapezius X

2 rhomboids X

3 levator scapulae X

4 T1-T5 paraspinal muscles X

Osteopathic Treatment You must treat a minimum of 2 dysfunctions you found during your examination

1 Assume OA NSLRR appropriate techniques to include counterstrainindirect myofascial release facilitated positional release

X

2 Assume T2 FSLRL appropriate techniques to include counterstrainindirect myofascial release facilitated positional release

X

3 Assume acute spasm of left upper thoracic paraspinal muscles appropriate techniques to include counterstrainindirect myofascial release and facilitated positional release

X

4 Autonomic technique for altered vagal tone OA decompression X

5 Autonomic technique for hypersympathetic tone rib raising or paraspinal inhibition for gt 90 seconds

X

MODULE 7 ndash Emmitt Brown

1 AAA kidney stones groin hernia ileus kidneybladder CA testicular torsion kidneybladder infection appendicitis

2 C3 D

Osteopathic Diagnosis assess where you would expect to find somatic dysfunctions

Performed Omitted

1 Examine the cervical region for somatic dysfunctionA OA ndash leftright translation1 neutral X2 flexion X3 extension XB AA ndash rotation must flex at least 45deg X2 Examine the thoracic spine for dysfunction (T10-L2)A palpate transverse processes for rotation1 neutral X2 flexion X3 extension XB palpate soft tissue for TART findings4 T10 ndash L2 Paraspinal muscles X3 Examine the sacrum for somatic dysfunction (requires at least one motion test)A depth at 4 corners of sacrum XB ILA levelness XC seated flexion test XD motion at 4 corners of sacrum XE motion in backward bending test XF respiratory motion of sacrum assess at 4 corners X4 Examine the pelvis for somatic dysfunctionA standing flexion test or AP compression test XB ASIS heights XC PSIS heights XOsteopathic Treatment You must treat a minimum of 2 dysfunctions you found during your examination1 Assume OA NSLRR appropriate techniques to include HVLA muscle energy articulatory Still counterstrainindirect myofascial release facilitated positional release

X _

2 Assume T10 NRRSL appropriate techniques to include HVLA muscle energy articulatory Still counterstrainindirect myofascial release facilitated positional release

X

3 Assume acute bilateral spasm of thoracolumbar (T10-L2) paraspinal muscles RgtL appropriate techniques to include soft tissue counterstrainindirect facilitated positional release

X

4 Assume left on right sacral torsion appropriate techniques to include muscle energy counterstrainindirect myofascial release and facilitated positional release (No HVLA)

X

5 Autonomic technique for hypersympathetic tone in T10-L2 paraspinal muscles rib raising or paraspinal inhibition for gt 90 seconds

X

MODULE 8 ndash Peter Parker1 Kidney stone Psoas syndrome spondylolysis piriformis syndrome lumbar compression

fracture appendicitis pelvicabdominal CA2 C3 B

Osteopathic Diagnosis assess where you would expect to find somatic dysfunctions

Performed Omitted

1 Examine the thoracolumbar region (T10-L5) for somatic dysfunctionA palpate transverse processes for rotation1 neutral _2 flexion _3 extension _B palpate soft tissue for TART (Tenderness Asymmetry Restrictions Tissue texture change) findings1 Psoas muscles _2 Piriformis muscles _3 Paraspinal muscles _2 Examine the sacrum for somatic dysfunction (requires at least one motion test)A depth at 4 corners of sacrum _B ILA levelness _C seated flexion test _D motion at 4 corners of sacrum _E motion in backward bending test _F respiratory motion of sacrum assess at 4 corners _3 Examine the pelvis for somatic dysfunctionA standing flexion test or AP compression test _B ASIS heights _C PSIS heights _Osteopathic Treatment You must treat a minimum of 2 dysfunctions you found during your examination1 Assume L1 FRRSR appropriate techniques to include muscle energy counterstrain indirect myofascial release facilitated positional release (No HVLA)

_

2 Assume left on right sacral torsion appropriate techniques to include muscle energy counterstrainindirect myofascial release and facilitated positional release (No HVLA)

_

3 Assume spasmtender point in the right psoas muscle appropriate techniques to include counterstrainindirect myofascial release FPR ME

_

4 Assume spasmtender point in the left piriformis muscle appropriate techniques to include counterstrainindirect myofascial release FPR ME

_

MODULE 9 ndash Ingrid Bergman

1 Lumbar strain Dysmenorrhea PID lumbar strain ovarian cyst sacral somatic dysfunction2 D3 E

Osteopathic Diagnosis assess where you would expect to find somatic dysfunctions

Performed Omitted

1 Examine the lumbar region (L1-L5) for somatic dysfunctionA palpate transverse processes for rotation1 neutral _2 flexion _3 extension _B palpate soft tissue for TART (Tenderness Asymmetry Restrictions Tissue texture change) findings1 quadratus lumborum muscles _2 Piriformis muscles _3 Paraspinal muscles _2 Examine the sacrum for somatic dysfunction (requires at least one motion test)A depth at 4 corners of sacrum _B ILA levelness _C seated flexion test _D motion at 4 corners of sacrum _E motion in backward bending test _F respiratory motion of sacrum assess at 4 corners _3 Examine the pelvis for somatic dysfunctionA standing flexion test or AP compression test _B ASIS heights _C PSIS heights _Osteopathic Treatment You must treat a minimum of 2 dysfunctions you found during your examination1 Assume L3 FRLSL2 Assume left on right sacral torsion _3 Assume left superior innominate shear _

4 Autonomic technique for altered parasympathetic tone

Module 10 ndash Minnie Driver

Pretest

1 Pneumonia dissecting aortic aneurysm thoracic spine fracture MI pleurisy2 B3 E

Osteopathic Diagnosis assess where you would expect to find somatic dysfunctions

Performed Omitted

1 Examine the cervical region for somatic dysfunctionA OA ndash leftright translation1 neutral 104863510486352 flexion 104863510486353 extension 10486351048635B AA ndash rotation must flex at least 45deg 10486351048635C C2-C7 - translation1 neutral 104863510486352 flexion 104863510486353 extension 104863510486352 Examine the thoracic spine for dysfunction (T5-T9)A palpate transverse processes for rotation1 neutral 104863510486352 flexion 104863510486353 extension 10486351048635B palpate soft tissue for TART findings1 trapezius 104863510486352 rhomboids 104863510486353 levator scapulae 104863510486354 T5 ndash T9 Paraspinal muscles 104863510486353 Examine the ribs for somatic dysfunctionA Pump handle ribs (ribs 2-6) at or near sternal junction1 static 104863510486352 motion 10486351048635B Bucket handle ribs (ribs 6-10) between anterior and posterior axillary lines1 static 104863510486352 motion 10486351048635Osteopathic Treatment You must treat a minimum of 2 dysfunctions you found during your examination1 Assume T5 ESLRL 104863510486352 Assume acute bilateral spasm of upper thoracic (T4-T8) paraspinal 10486351048635

muscles3 Assume (R) Rib 2 pump handle inhalation dysfunction 104863510486354 Autonomic technique for altered vagal tone OA decompression 10486351048635

MODULE 11 ndash Jeff Weaver

1 Cervical radiculopathy thoracic outlet syndrome cubital tunnel syndrome carpal tunnel syndrome angina pectoris ulnar groove entrapment Guillian Barre

2 A3 B4 C

Osteopathic Diagnosis assess where you would expect to find somatic dysfunctions

Performed Omitted

1 Examine the cervical region for somatic dysfunctionA C2-C7 - translation1 neutral 104863510486352 flexion 104863510486353 extension 10486351048635B palpate soft tissue for TART (Tenderness Asymmetry Restrictions Tissue texture change) findings1 cervical paraspinal muscles 104863510486352 sternocleidomastoids 104863510486353 scalenes 104863510486352 Examine the thoracic spine for somatic dysfunctionA palpate transverse processes for rotation1 neutral 104863510486352 flexion 104863510486353 extension 10486351048635B palpate soft tissue for TART findings1 trapezius 104863510486352 pectoral musculature 104863510486353 T1-T4 paraspinal muscle 104863510486353 Examine the 1st rib for somatic dysfunction1 static 104863510486352 dynamic 10486351048635Osteopathic Treatment You must treat a minimum of 2 dysfunctions you found during your examination1 Assume hypertonic left pectoral muscle Appropriate techniques to include ME MFR counterstrain

10486351048635

2 Assume hypertonic left scalene muscle Appropriate techniques to include MFR counterstrain ME

10486351048635

3 Assume T4 NRLSR appropriate techniques to include HVLA muscle energy articulatory Still counterstrainindirect myofascial release facilitated positional release

10486351048635

4 Assume left sided thoracic paraspinal hypertonicity T1 ndash T4 appropriate techniques to include soft tissue deep pressure counterstrainindirect facilitated positional release

10486351048635

5 Assume left 1 st rib inhalation somatic dysfunction appropriate techniques to include HVLA ME Stillrsquos articulatory

10486351048635

MODULE 12 ndash Scarlett O-Hara

1 Osteoarthritis piriformis syndrome spinal stenosis lumbar radiculopathy DVT2 C3 D

Osteopathic Diagnosis assess where you would expect to find somatic dysfunctions

Performed Omitted

1 Examine the lumbar region (L1-L5) for somatic dysfunctionA palpate transverse processes for rotation1 neutral _2 flexion _3 extension _B palpate soft tissue for TART (Tenderness Asymmetry Restrictions Tissue texture change) findings1 Psoas muscles _2 Piriformis muscles _3 Paraspinal muscles _2 Examine the sacrum for somatic dysfunction (requires at least one motion test)A depth at 4 corners of sacrum _B ILA levelness _C seated flexion test _D motion at 4 corners of sacrum _E motion in backward bending test _F respiratory motion of sacrum assess at 4 corners _3 Examine the pelvis for somatic dysfunctionA standing flexion test or AP compression test _B ASIS heights _C PSIS heights _Osteopathic Treatment You must treat a minimum of 2 dysfunctions you found during your examination1 Assume left on right sacral torsion appropriate techniques to include muscle energy counterstrainindirect myofascial release and facilitated positional release (No HVLA)2 Assume left innominate anterior appropriate techniques to include muscle energy counterstrainindirect myofascial release and facilitated positional release (No HVLA)

_

3 Assume spasmtender point in the left piriformis muscle appropriate techniques to _

include counterstrainindirect myofascial release

MODULE 13 Clark Kent1 Asthma atypical pneumonia bronchitis upper respiratory infection pneumothorax2 B3 A

Osteopathic Diagnosis assess where you would expect to find somatic dysfunctions

Performed Omitted

1 Examine the cervical region for somatic dysfunctionA OA ndash leftright translation1 neutral _2 flexion _3 extension _B AA ndash rotation must flex at least 45deg _C C2-C7 - translation1 neutral 2 flexion _3 extension 2 Examine the thoracic spine for dysfunction (T1-T4)A palpate transverse processes for rotation1 neutral _2 flexion _3 extension _B palpate soft tissue for TART findings1 trapezius X2 rhomboids X3 levator scapulae X4 T1 ndash T4 Paraspinal muscles X3 Examine the ribs for somatic dysfunctionA Pump handle ribs (ribs 2-6) at or near sternal junction1 static X2 motion XB Bucket handle ribs (ribs 6-10) between anterior and posterior axillary lines1 static X2 motion XOsteopathic Treatment You must treat a minimum of 2 dysfunctions you found during your examination1 Assume OA NSLRR appropriate techniques to include HVLA muscle energy articulatory Still counterstrainindirect myofascial release facilitated positional release

_

2 Assume T2-4 NSLRR appropriate techniques to include HVLA muscle energy articulatory Still counterstrainindirect myofascial release facilitated positional release

_

3 Assume (L) ribs 2-4 exhalation pump handle appropriate techniques to include HVLA muscle energy articulatory Still counterstrainindirect myofascial release facilitated positional release

4 Autonomic technique for altered vagal tone OA decompression _5 Autonomic technique for hypersympathetic tone in T1-T5 paraspinal muscles rib raising or paraspinal inhibition for gt 90 seconds

X

MODULE 14 Bruce Wayne

1 Depression thyroid disorder migraine headache tension tension headache migraine2 C3 A

Osteopathic Diagnosis assess where you would expect to find somatic dysfunctions

Performed Omitted

1 Examine the cervical region for somatic dysfunctionA OA ndash leftright translation1 neutral 2 flexion 3 extension B AA ndash rotation must flex at least 45deg C C2-C7 - translation1 neutral 2 flexion 3 extension _2 Examine the thoracic spine for dysfunction (T1-T4)A palpate transverse processes for rotation1 neutral X2 flexion X3 extension XB palpate soft tissue for TART findings1 trapezius X2 T1 ndash T4 Paraspinal muscles XOsteopathic Treatment You must treat a minimum of 2 dysfunctions you found during your examination1 Assume OA NSRRL appropriate techniques to include HVLA muscle energy articulatory Still counterstrainindirect myofascial release facilitated positional release

2 Assume C4 FSLRL appropriate techniques to include HVLA muscle energy

articulatory Still counterstrainindirect myofascial release facilitated positional release2 Assume T2-4 NSLRR appropriate techniques to include HVLA muscle energy articulatory Still counterstrainindirect myofascial release facilitated positional release

_

4 Autonomic technique for altered vagal tone OA decompression _

  • Performed
  • Performed
  • Performed
  • Performed
  • Performed
  • Performed
Page 4: Performed · Web viewAAA, kidney stones, groin hernia, ileus, kidney/bladder CA, testicular torsion, kidney/bladder infection, appendicitis, C D Osteopathic Diagnosis: assess where

c Left innominate posterior

d Left innominate inflare

e Right innominate posterior

12 Which of the following is true regarding the seated flexion test

a If both sides rise the same amount it indicates that no dysfunction is present

b The test is positive opposite the side of greatest PSIS excursion

c The axis of a sacral torsion is located on the same side of greatest excursion

d A unilateral dysfunction will be on the same side of the greater excursion

e A false negative occurs with unilateral dysfunctions

13 Which of the following is true regarding sacral torsions

a The right base moves forward in a left on right torsion

b The axis passes through the right base in a left on left torsion

c A right on right torsion is termed a backward sacral torsion

d The right sulcus is deeper in a right on right torsion

e Points along the axis do not move in a sacral torsion

14 Which of the following is true regarding a bilateral sacral extension

a The sacral sulci are deep bilaterally

b The seated flexion test will be positive

c The base will not move posteriorly during inhalation

d The base will not move posteriorly during exhalation

e The sacral sulci are shallow bilaterally

15 You assess the sacrum and find the following

Positive seated flexion test on the leftLeft sulcus shallowLeft ILA markedly superiorWhich of the following is the correct diagnosis

a Right on left sacral torsion

b Right unilateral sacral flexion

c Left on right sacral torsion

d Left unilateral sacral extension

e Left on left sacral torsion

16 You examine T7 and find the left transverse process to be deeper than the right The findings become symmetrical in flexion and are unchanged in extension Which of the following is the correct diagnosis

a T7 flexed sidebent left rotated left

b T7 flexed sidebent right rotated right

c T7 extended sidebent left rotated left

d T7 extended sidebent right rotated left

e T7 flexed sidebent left rotated right

17 Which of the following is true regarding lumbar diagnosis

a Rotation and sidebending occur to opposite sides in flexion dysfunctions

b If the right transverse process is deeper the vertebra is rotated to the left

c Rotation and sidebending occur to the same side in neutral dysfunctions

d A lumbar dysfunction that becomes symmetrical in both flexion and extension

is an extension dysfunction

e Only Fryettersquos Principle 1 applies to the lumbar spine

18 Which of the following is true regarding cervical mechanicsdiagnosis

a Motion testing of the AA is performed by translation

b C2-C7 only follow Fryettersquos Principle 1

c The OA is the only cervical segment that must be tested in flexion extension and neutral

d Right translation is equal to left sidebending at C4

e Left translation is equal to left rotation at C6

19 Which of the following is true regarding sacral dysfunctions

a A right unilateral sacral flexion will have a positive seated flexion test on the left

b A left on left sacral torsion will have a deep right sulcus

c A right on left sacral torsion will have a deep right sacral sulcus

d A left unilateral seated flexion will have a significantly higher ILA on the left

e A bilateral sacral flexion will have bilaterally shallow sulci

20 Which of the following is true regarding innominate dysfunctions

a Innominate flares are named for the side on which the ASIS is closer to midline

b The standing flexion test is negative with an innominate shear

c The ASIS compression test will be positive on the side opposite the positive standing flexion test

d A positive seated flexion test is expected on the right with a right innominate posterior

e The right ASIS is inferior with a right innominate anterior

21 You evaluate the 3rd ribs bilaterally and find the following

Right 3rd rib more prominent anteriorlyBoth sides rise during inhalationOnly the left side falls during exhalationWhich of the following is the correct diagnosis

a Right 3rd rib inhalation dysfunction

b Left 3rd rib inhalation dysfunction

c Right 3rd rib inferior dysfunction

d Left 3rd rib exhalation dysfunction

e Left 3rd rib superior dysfunction

22 Which of the following is paired correctly

a Fryettersquos Principle 1 ndash Typically involves a single segment

b AA dysfunctions ndash Assessed with rotation

c Ribs 11 amp 12 ndash Predominantly superiorinferior motion

d Left oblique sacral axis ndash Positive left seated flexion test

e Right innominate posterior ndash Positive left standing flexion test

23 Which of the following dysfunctions will have a left positive standing flexion test and

a negative AP compression test

a Left innominate posterior

b Right superior innominate shear

c Left innominate outflare

d Right innominate inflare

e Left inferior innominate shear

24 You examine L1 and find the right transverse process to be more shallow than the left The findings are unchanged in both flexion and extension Which of the following is the correct diagnosis

a L1 flexed sidebent left rotated left

b L1 neutral sidebent right rotated right

c L1 neutral sidebent left rotated left

d L1 extended sidebent right rotated left

e L1 neutral sidebent left rotated right

25 Which of the following is true regarding a right superior innominate shear

a Negative standing flexion test

b Negative AP compression test

c ASIS and PSIS both superior on the right

d Right ASIS closer to midline

e Positive standing flexion test on the left

MODULE 2

1 Which of the following is true regarding muscle energy treatment

a Contraindicated in patients with osteopenia

b Primary treatment for patients in the ICU

c Utilizes patient force with equal physician counterforce X

d Patient effort is held for 90 seconds

e A-P curve must be flattened out during treatment

2 A patient is diagnosed with a right anterior innominate dysfunction Which of the

following is the correct description for muscle energy treatment of this dysfunction

a Right hip extended ndash patient flexes hip against resistance

b Right hip flexed ndash patient extends hip against resistance X

c Right hip extended ndash patient extends hip against resistance

d Right hip externally rotated ndash patient internally rotates hip against resistance

e Right hip abducted ndash patient adducts hip against resistance

3 Which of the following is true regarding HVLA thrust technique

a Indirect technique

b Useful in patients with joint hypermobility

c Thrust applied after backing away from barrier for 1-2 seconds

d Lumbar HVLA is contraindicated in patients with Down Syndrome

e Restrictive barrier must be engaged in all planes of motion X

4 Which of the following is true regarding Counterstrain treatment

a Restrictive barrier must be engaged in all planes of motion

b Treatment position should decrease tenderness by at least 70 X

c Treatment position is held for 3-4 seconds then released

d Patient assists in positioning for treatment

e Patient must slowly bring himself to neutral after treatment

5 Which of the following is correctly matched

a Counterstrain treatment of hip flexor ndash patientrsquos hip passively flexed X

b FPR treatment of shoulder external rotator ndash patientrsquos shoulder internally rotated

c HVLA treatment of C3 ERRSR ndash C3 placed into extension right rotation and right sidebending

d Muscle energy treatment of a bilateral sacral flexion ndash pressure on sacral base

e HVLA treatment of right unilateral sacral extension ndash superior thrust on right ILA

6 The heart receives sympathetic innervation from which of the following levels

a OA

b T1-T5 X

c T5-T9

d T10-L2

e S2-S4

7 Which of the following is true regarding FPR treatment

a Restrictive barrier must be engaged in all planes of motion

b Treatment position should decrease tenderness by at least 70

c Treatment position is held for 3-4 seconds then released X

d A-P curve is accentuated during treatment

e Facilitating force is an isometric contraction of the involved muscles

8 Which of the following is true about autonomic control of the lungs

a Sympathetic stimulation results in bronchoconstriction

b The lower lungs receive parasympathetic innervation from S2-S4

c Sympathetic innervation arises from the T5-T9 levels

d Increased thick secretions are a result of increased sympathetic activity X

e Sacral rocking will result in increased thin watery secretions

9 Which of the following techniques would be most useful in improving upper GI

motility in a patient with ileus

a Sacral rocking

b Rib raising for 30 seconds

c OA decompression

d Paraspinal inhibition for 120 seconds X

e Sacral inhibition

10 You assess C4 translation and find translation to be easier to the left The findings

become symmetrical in flexion but not in extension Which of the following is the

correct position of C4 for a HVLA thrust

a Flexed rotated right sidebent left

b Extended rotated left sidebent left X

c Neutral rotated right sidebent left

d Flexed rotated right sidebent right

e Extended rotated right sidebent right

11 You examine L3 and find the right transverse process to be deeper than the left The

findings are unchanged in flexion and become symmetrical in extension Which of

the following is the correct position of L3 for FPR treatment

a Flexed sidebent left rotated left

b Flexed sidebent right rotated right

c Extended sidebent left rotated left X

d Extended sidebent right rotated left

e Flexed sidebent left rotated right

12 Which of the following is true regarding autonomic innervation to the GU system

a Sympathetic stimulation increases ureteral peristalsis

b Sympathetic stimulation relaxes the bladder wall X

c Parasympathetic stimulation activates the bladder sphincter

d Parasympathetic stimulation activates the trigone

e Sympathetic stimulation increases GFR

13 Which of the following is a result of parasympathetic stimulation

a Peripheral vasodilation

b Ejaculation

c Increased salivation X

d Uterine contraction

e Ureterospasm

14 The gall bladder receives sympathetic innervation from which of the following

levels

a OA

b T1-T5

c T5-T9 X

d T10-L2

e S2-S4

15 You are attempting to treat a right 7th rib exhalation dysfunction with counterstrain

Which of the following is correct regarding the treatment

a Shaft of right 7th rib positioned closer to shaft of right 8th rib X

b Position should decrease tenderness by 50

c Position is held for 30 seconds

d Angle of right 7th rib tractioned inferiorly

e Positioning involves left sidebending and extension

16 Which of the following is correctly matched

a Muscle energy ndash indirect treatment

b HVLA ndash articulation positioned in free motion

c Counterstrain ndash starts with a pain scale X

d FPR ndash active direct treatment

e Muscle energy ndash useful for painful muscle groups

17 Which of the following would be most likely to inhibit uterine contractions in a

patient in premature labor

a Sacral rocking X

b OA decompression

c Paraspinal inhibition gt90 seconds at T5-T9

d Rib raising lt60 seconds at T10-L2

e Sacral inhibition

18 You assess the innominates and find the following

Positive standing flexion test on the left

Right ASIS inferior compared to left

Right PSIS inferior compared to left

ASIS to midline distance equal bilaterally

Which of the following correctly describes a treatment for this dysfunction

a Muscle energy ndash left hip flexed patient extends against resistance

b HVLA thrust ndash thrust directed inferiorly on left innominate X

c FPR ndash right innominate pushed inferiorly for 3-4 seconds

d Muscle energy ndash right hip pushed superiorly patient pushes inferiorly

e Counterstrain ndash left innominate held inferiorly for 90 seconds

19 The pancreas receives parasympathetic innervation from which of the following

levels

a OA X

b T1-T5

c T5-T9

d T10-L2

e S2-S4

20 Which of the following would most likely directly decrease gastric secretions

a Sacral rocking

b OA decompression

c Paraspinal inhibition gt90 seconds at T10-L2

d Rib raising lt60 seconds at T5-T9 X

e Sacral inhibition

21 Which of the following statements is true regarding osteopathic treatment techniques

a Counterstrain is contraindicated for painful muscle groups

b Osteoporosis is a contraindication for FPR

c Muscle energy should not be used on a person in the CCU X

d HVLA is useful in patients with active Rheumatoid Arthritis

e The restrictive barrier is engaged when using indirect techniques

22 You examine T7 and find the left transverse process to be deeper than the right The

findings become symmetrical in flexion and are unchanged in extension Which of

the following is the correct starting position for a muscle energy treatment

a T7 flexed sidebent left rotated left

b T7 flexed sidebent right rotated right

c T7 extended sidebent left rotated left X

d T7 extended sidebent right rotated left

e T7 flexed sidebent left rotated right

23 The sigmoid colon receives parasympathetic innervation from which of the following

levels

a OA

b T1-T5

c T5-T9

d T10-L2

e S2-S4 X

24 You assess OA translation and find translation to be easier to the right The findings

persist in both flexion and extension Which of the following is the correct position

for FPR treatment

a Flexed rotated right sidebent left

b Neutral rotated left sidebent right

c Neutral rotated right sidebent left X

d Neutral rotated left sidebent left

e Extended rotated right sidebent right

25 Which of the following would most likely directly increase lacrimation

a Sacral rocking

b OA decompression X

c Paraspinal inhibition gt90 seconds at T5-T9

d Rib raising lt60 seconds at T1-T4 X

e Sacral inhibition

MODULE 3 ndash Joanne Smith

1 Top 5 ddx peptic ulcer GERD costochondritis Pancreastitis MI angina pectoris2 E no contraindications3 A OA4 E strep throat

dysfunctions Performed Omitted

1 Examine the cervical region for somatic dysfunctionA OA ndash leftright translation1 neutral 10486351048635

2 flexion 104863510486353 extension 104863510486352 Examine the thoracic spine for dysfunction (T5-T9)A palpate transverse processes for rotation1 neutral 104863510486352 flexion 104863510486353 extension 10486351048635B palpate soft tissue for TART findings1 trapezius 104863510486352 rhomboids 104863510486353 levator scapulae 104863510486354 T5 ndash T9 Paraspinal muscles 10486351048635Osteopathic Treatment You must treat a minimum of 2 dysfunctions you found during your examination1 Assume OA NSLRR appropriate techniques to include HVLA muscle energy articulatory Still counterstrainindirect myofascial release facilitated positional release

10486351048635

2 Assume T7 FRLSL appropriate techniques to include HVLA muscle energy articulatory Still counterstrainindirect myofascial release facilitated positional release

10486351048635

3 Assume acute bilateral spasm of upper thoracic (T5-T9) paraspinal muscles appropriate techniques to include soft tissue counterstrainindirect facilitated positional release

10486351048635

4 Autonomic technique for altered vagal tone OA decompression 104863510486355 Autonomic technique for hypersympathetic tone in T5-T9 paraspinal muscles rib raising or paraspinal inhibition for gt 90 seconds

10486351048635

MODULE 4- Ernesto Rodriguez

PRETEST

1 Back pain from Colon cancer Lumbar strain Osteomyelitits Diverticulitis Back pain osteoarthritis Cauda equina syndrome

2 D3 E

Osteopathic Diagnosis assess where you would expect to find somatic dysfunctions

Performed Omitted

1 Examine the lumbar region (L1-L5) for somatic dysfunction

A palpate transverse processes for rotation

1 neutral _

2 flexion _

3 extension _

B palpate soft tissue for TART (Tenderness Asymmetry Restrictions Tissue texture change) findings

1 quadratus lumborum muscles _

2 Piriformis muscles _

3 Paraspinal muscles _

2 Examine the sacrum for somatic dysfunction (requires at least one motion test)

A depth at 4 corners of sacrum _

B ILA levelness _

C seated flexion test _

D motion at 4 corners of sacrum _

E motion in backward bending test _

F respiratory motion of sacrum assess at 4 corners _

3 Examine the pelvis for somatic dysfunction

A standing flexion test or AP compression test _

B ASIS heights _

C PSIS heights _

Osteopathic Treatment You must treat a minimum of 2 dysfunctions you found during your examination

1 Assume L5 FRLSL _

2 Assume left on left sacral torsion _

3 Assume left innominate anterior _

4 Assume spasmtender point in the left piriformis muscle appropriate techniques to include counterstrainindirect myofascial release

-

MODULE 5 ndash Connie Jones

Pretest

1 Bradycardia acute pulmonary edemaCHF pneumonia acute MI Acute pulmonary edema CHF2 A3 A4 A5 For the video this is what was done and not done

OA - nf NO E --gtNslrrno AA dx doneno cervical dx doneT1-T5 - nfe --gt t2 f sl rlparapsinals TART --gt L upper thoracic paraspinalno levator scapula no trap no rhomboids dx tx -

T2 HLVAOA decompr ndash for vagal tonerib raisin for sympth

MODULE 6 ndash Gretchen White

1 Atrial fibrillation hyperthyroidism and- thyrotoxicosis thyroid cancer hyperthryroid 2 a3 a4 a5

Osteopathic Diagnosis assess where you would expect to find somatic dysfunctions

Performed Omitted

1 Examine the cervical region for somatic dysfunction

A OA ndash leftright translation

1 neutral X

2 flexion X

6 3 extension X

7 B AA ndash rotation X

8 2 Examine the upper thoracic spine for dysfunction (T1-T5)

A palpate transverse processes for rotation

1 neutral X

2 flexion X

3 extension X

B palpate soft tissue for TART findings

1 trapezius X

2 rhomboids X

3 levator scapulae X

4 T1-T5 paraspinal muscles X

Osteopathic Treatment You must treat a minimum of 2 dysfunctions you found during your examination

1 Assume OA NSLRR appropriate techniques to include counterstrainindirect myofascial release facilitated positional release

X

2 Assume T2 FSLRL appropriate techniques to include counterstrainindirect myofascial release facilitated positional release

X

3 Assume acute spasm of left upper thoracic paraspinal muscles appropriate techniques to include counterstrainindirect myofascial release and facilitated positional release

X

4 Autonomic technique for altered vagal tone OA decompression X

5 Autonomic technique for hypersympathetic tone rib raising or paraspinal inhibition for gt 90 seconds

X

MODULE 7 ndash Emmitt Brown

1 AAA kidney stones groin hernia ileus kidneybladder CA testicular torsion kidneybladder infection appendicitis

2 C3 D

Osteopathic Diagnosis assess where you would expect to find somatic dysfunctions

Performed Omitted

1 Examine the cervical region for somatic dysfunctionA OA ndash leftright translation1 neutral X2 flexion X3 extension XB AA ndash rotation must flex at least 45deg X2 Examine the thoracic spine for dysfunction (T10-L2)A palpate transverse processes for rotation1 neutral X2 flexion X3 extension XB palpate soft tissue for TART findings4 T10 ndash L2 Paraspinal muscles X3 Examine the sacrum for somatic dysfunction (requires at least one motion test)A depth at 4 corners of sacrum XB ILA levelness XC seated flexion test XD motion at 4 corners of sacrum XE motion in backward bending test XF respiratory motion of sacrum assess at 4 corners X4 Examine the pelvis for somatic dysfunctionA standing flexion test or AP compression test XB ASIS heights XC PSIS heights XOsteopathic Treatment You must treat a minimum of 2 dysfunctions you found during your examination1 Assume OA NSLRR appropriate techniques to include HVLA muscle energy articulatory Still counterstrainindirect myofascial release facilitated positional release

X _

2 Assume T10 NRRSL appropriate techniques to include HVLA muscle energy articulatory Still counterstrainindirect myofascial release facilitated positional release

X

3 Assume acute bilateral spasm of thoracolumbar (T10-L2) paraspinal muscles RgtL appropriate techniques to include soft tissue counterstrainindirect facilitated positional release

X

4 Assume left on right sacral torsion appropriate techniques to include muscle energy counterstrainindirect myofascial release and facilitated positional release (No HVLA)

X

5 Autonomic technique for hypersympathetic tone in T10-L2 paraspinal muscles rib raising or paraspinal inhibition for gt 90 seconds

X

MODULE 8 ndash Peter Parker1 Kidney stone Psoas syndrome spondylolysis piriformis syndrome lumbar compression

fracture appendicitis pelvicabdominal CA2 C3 B

Osteopathic Diagnosis assess where you would expect to find somatic dysfunctions

Performed Omitted

1 Examine the thoracolumbar region (T10-L5) for somatic dysfunctionA palpate transverse processes for rotation1 neutral _2 flexion _3 extension _B palpate soft tissue for TART (Tenderness Asymmetry Restrictions Tissue texture change) findings1 Psoas muscles _2 Piriformis muscles _3 Paraspinal muscles _2 Examine the sacrum for somatic dysfunction (requires at least one motion test)A depth at 4 corners of sacrum _B ILA levelness _C seated flexion test _D motion at 4 corners of sacrum _E motion in backward bending test _F respiratory motion of sacrum assess at 4 corners _3 Examine the pelvis for somatic dysfunctionA standing flexion test or AP compression test _B ASIS heights _C PSIS heights _Osteopathic Treatment You must treat a minimum of 2 dysfunctions you found during your examination1 Assume L1 FRRSR appropriate techniques to include muscle energy counterstrain indirect myofascial release facilitated positional release (No HVLA)

_

2 Assume left on right sacral torsion appropriate techniques to include muscle energy counterstrainindirect myofascial release and facilitated positional release (No HVLA)

_

3 Assume spasmtender point in the right psoas muscle appropriate techniques to include counterstrainindirect myofascial release FPR ME

_

4 Assume spasmtender point in the left piriformis muscle appropriate techniques to include counterstrainindirect myofascial release FPR ME

_

MODULE 9 ndash Ingrid Bergman

1 Lumbar strain Dysmenorrhea PID lumbar strain ovarian cyst sacral somatic dysfunction2 D3 E

Osteopathic Diagnosis assess where you would expect to find somatic dysfunctions

Performed Omitted

1 Examine the lumbar region (L1-L5) for somatic dysfunctionA palpate transverse processes for rotation1 neutral _2 flexion _3 extension _B palpate soft tissue for TART (Tenderness Asymmetry Restrictions Tissue texture change) findings1 quadratus lumborum muscles _2 Piriformis muscles _3 Paraspinal muscles _2 Examine the sacrum for somatic dysfunction (requires at least one motion test)A depth at 4 corners of sacrum _B ILA levelness _C seated flexion test _D motion at 4 corners of sacrum _E motion in backward bending test _F respiratory motion of sacrum assess at 4 corners _3 Examine the pelvis for somatic dysfunctionA standing flexion test or AP compression test _B ASIS heights _C PSIS heights _Osteopathic Treatment You must treat a minimum of 2 dysfunctions you found during your examination1 Assume L3 FRLSL2 Assume left on right sacral torsion _3 Assume left superior innominate shear _

4 Autonomic technique for altered parasympathetic tone

Module 10 ndash Minnie Driver

Pretest

1 Pneumonia dissecting aortic aneurysm thoracic spine fracture MI pleurisy2 B3 E

Osteopathic Diagnosis assess where you would expect to find somatic dysfunctions

Performed Omitted

1 Examine the cervical region for somatic dysfunctionA OA ndash leftright translation1 neutral 104863510486352 flexion 104863510486353 extension 10486351048635B AA ndash rotation must flex at least 45deg 10486351048635C C2-C7 - translation1 neutral 104863510486352 flexion 104863510486353 extension 104863510486352 Examine the thoracic spine for dysfunction (T5-T9)A palpate transverse processes for rotation1 neutral 104863510486352 flexion 104863510486353 extension 10486351048635B palpate soft tissue for TART findings1 trapezius 104863510486352 rhomboids 104863510486353 levator scapulae 104863510486354 T5 ndash T9 Paraspinal muscles 104863510486353 Examine the ribs for somatic dysfunctionA Pump handle ribs (ribs 2-6) at or near sternal junction1 static 104863510486352 motion 10486351048635B Bucket handle ribs (ribs 6-10) between anterior and posterior axillary lines1 static 104863510486352 motion 10486351048635Osteopathic Treatment You must treat a minimum of 2 dysfunctions you found during your examination1 Assume T5 ESLRL 104863510486352 Assume acute bilateral spasm of upper thoracic (T4-T8) paraspinal 10486351048635

muscles3 Assume (R) Rib 2 pump handle inhalation dysfunction 104863510486354 Autonomic technique for altered vagal tone OA decompression 10486351048635

MODULE 11 ndash Jeff Weaver

1 Cervical radiculopathy thoracic outlet syndrome cubital tunnel syndrome carpal tunnel syndrome angina pectoris ulnar groove entrapment Guillian Barre

2 A3 B4 C

Osteopathic Diagnosis assess where you would expect to find somatic dysfunctions

Performed Omitted

1 Examine the cervical region for somatic dysfunctionA C2-C7 - translation1 neutral 104863510486352 flexion 104863510486353 extension 10486351048635B palpate soft tissue for TART (Tenderness Asymmetry Restrictions Tissue texture change) findings1 cervical paraspinal muscles 104863510486352 sternocleidomastoids 104863510486353 scalenes 104863510486352 Examine the thoracic spine for somatic dysfunctionA palpate transverse processes for rotation1 neutral 104863510486352 flexion 104863510486353 extension 10486351048635B palpate soft tissue for TART findings1 trapezius 104863510486352 pectoral musculature 104863510486353 T1-T4 paraspinal muscle 104863510486353 Examine the 1st rib for somatic dysfunction1 static 104863510486352 dynamic 10486351048635Osteopathic Treatment You must treat a minimum of 2 dysfunctions you found during your examination1 Assume hypertonic left pectoral muscle Appropriate techniques to include ME MFR counterstrain

10486351048635

2 Assume hypertonic left scalene muscle Appropriate techniques to include MFR counterstrain ME

10486351048635

3 Assume T4 NRLSR appropriate techniques to include HVLA muscle energy articulatory Still counterstrainindirect myofascial release facilitated positional release

10486351048635

4 Assume left sided thoracic paraspinal hypertonicity T1 ndash T4 appropriate techniques to include soft tissue deep pressure counterstrainindirect facilitated positional release

10486351048635

5 Assume left 1 st rib inhalation somatic dysfunction appropriate techniques to include HVLA ME Stillrsquos articulatory

10486351048635

MODULE 12 ndash Scarlett O-Hara

1 Osteoarthritis piriformis syndrome spinal stenosis lumbar radiculopathy DVT2 C3 D

Osteopathic Diagnosis assess where you would expect to find somatic dysfunctions

Performed Omitted

1 Examine the lumbar region (L1-L5) for somatic dysfunctionA palpate transverse processes for rotation1 neutral _2 flexion _3 extension _B palpate soft tissue for TART (Tenderness Asymmetry Restrictions Tissue texture change) findings1 Psoas muscles _2 Piriformis muscles _3 Paraspinal muscles _2 Examine the sacrum for somatic dysfunction (requires at least one motion test)A depth at 4 corners of sacrum _B ILA levelness _C seated flexion test _D motion at 4 corners of sacrum _E motion in backward bending test _F respiratory motion of sacrum assess at 4 corners _3 Examine the pelvis for somatic dysfunctionA standing flexion test or AP compression test _B ASIS heights _C PSIS heights _Osteopathic Treatment You must treat a minimum of 2 dysfunctions you found during your examination1 Assume left on right sacral torsion appropriate techniques to include muscle energy counterstrainindirect myofascial release and facilitated positional release (No HVLA)2 Assume left innominate anterior appropriate techniques to include muscle energy counterstrainindirect myofascial release and facilitated positional release (No HVLA)

_

3 Assume spasmtender point in the left piriformis muscle appropriate techniques to _

include counterstrainindirect myofascial release

MODULE 13 Clark Kent1 Asthma atypical pneumonia bronchitis upper respiratory infection pneumothorax2 B3 A

Osteopathic Diagnosis assess where you would expect to find somatic dysfunctions

Performed Omitted

1 Examine the cervical region for somatic dysfunctionA OA ndash leftright translation1 neutral _2 flexion _3 extension _B AA ndash rotation must flex at least 45deg _C C2-C7 - translation1 neutral 2 flexion _3 extension 2 Examine the thoracic spine for dysfunction (T1-T4)A palpate transverse processes for rotation1 neutral _2 flexion _3 extension _B palpate soft tissue for TART findings1 trapezius X2 rhomboids X3 levator scapulae X4 T1 ndash T4 Paraspinal muscles X3 Examine the ribs for somatic dysfunctionA Pump handle ribs (ribs 2-6) at or near sternal junction1 static X2 motion XB Bucket handle ribs (ribs 6-10) between anterior and posterior axillary lines1 static X2 motion XOsteopathic Treatment You must treat a minimum of 2 dysfunctions you found during your examination1 Assume OA NSLRR appropriate techniques to include HVLA muscle energy articulatory Still counterstrainindirect myofascial release facilitated positional release

_

2 Assume T2-4 NSLRR appropriate techniques to include HVLA muscle energy articulatory Still counterstrainindirect myofascial release facilitated positional release

_

3 Assume (L) ribs 2-4 exhalation pump handle appropriate techniques to include HVLA muscle energy articulatory Still counterstrainindirect myofascial release facilitated positional release

4 Autonomic technique for altered vagal tone OA decompression _5 Autonomic technique for hypersympathetic tone in T1-T5 paraspinal muscles rib raising or paraspinal inhibition for gt 90 seconds

X

MODULE 14 Bruce Wayne

1 Depression thyroid disorder migraine headache tension tension headache migraine2 C3 A

Osteopathic Diagnosis assess where you would expect to find somatic dysfunctions

Performed Omitted

1 Examine the cervical region for somatic dysfunctionA OA ndash leftright translation1 neutral 2 flexion 3 extension B AA ndash rotation must flex at least 45deg C C2-C7 - translation1 neutral 2 flexion 3 extension _2 Examine the thoracic spine for dysfunction (T1-T4)A palpate transverse processes for rotation1 neutral X2 flexion X3 extension XB palpate soft tissue for TART findings1 trapezius X2 T1 ndash T4 Paraspinal muscles XOsteopathic Treatment You must treat a minimum of 2 dysfunctions you found during your examination1 Assume OA NSRRL appropriate techniques to include HVLA muscle energy articulatory Still counterstrainindirect myofascial release facilitated positional release

2 Assume C4 FSLRL appropriate techniques to include HVLA muscle energy

articulatory Still counterstrainindirect myofascial release facilitated positional release2 Assume T2-4 NSLRR appropriate techniques to include HVLA muscle energy articulatory Still counterstrainindirect myofascial release facilitated positional release

_

4 Autonomic technique for altered vagal tone OA decompression _

  • Performed
  • Performed
  • Performed
  • Performed
  • Performed
  • Performed
Page 5: Performed · Web viewAAA, kidney stones, groin hernia, ileus, kidney/bladder CA, testicular torsion, kidney/bladder infection, appendicitis, C D Osteopathic Diagnosis: assess where

b Right unilateral sacral flexion

c Left on right sacral torsion

d Left unilateral sacral extension

e Left on left sacral torsion

16 You examine T7 and find the left transverse process to be deeper than the right The findings become symmetrical in flexion and are unchanged in extension Which of the following is the correct diagnosis

a T7 flexed sidebent left rotated left

b T7 flexed sidebent right rotated right

c T7 extended sidebent left rotated left

d T7 extended sidebent right rotated left

e T7 flexed sidebent left rotated right

17 Which of the following is true regarding lumbar diagnosis

a Rotation and sidebending occur to opposite sides in flexion dysfunctions

b If the right transverse process is deeper the vertebra is rotated to the left

c Rotation and sidebending occur to the same side in neutral dysfunctions

d A lumbar dysfunction that becomes symmetrical in both flexion and extension

is an extension dysfunction

e Only Fryettersquos Principle 1 applies to the lumbar spine

18 Which of the following is true regarding cervical mechanicsdiagnosis

a Motion testing of the AA is performed by translation

b C2-C7 only follow Fryettersquos Principle 1

c The OA is the only cervical segment that must be tested in flexion extension and neutral

d Right translation is equal to left sidebending at C4

e Left translation is equal to left rotation at C6

19 Which of the following is true regarding sacral dysfunctions

a A right unilateral sacral flexion will have a positive seated flexion test on the left

b A left on left sacral torsion will have a deep right sulcus

c A right on left sacral torsion will have a deep right sacral sulcus

d A left unilateral seated flexion will have a significantly higher ILA on the left

e A bilateral sacral flexion will have bilaterally shallow sulci

20 Which of the following is true regarding innominate dysfunctions

a Innominate flares are named for the side on which the ASIS is closer to midline

b The standing flexion test is negative with an innominate shear

c The ASIS compression test will be positive on the side opposite the positive standing flexion test

d A positive seated flexion test is expected on the right with a right innominate posterior

e The right ASIS is inferior with a right innominate anterior

21 You evaluate the 3rd ribs bilaterally and find the following

Right 3rd rib more prominent anteriorlyBoth sides rise during inhalationOnly the left side falls during exhalationWhich of the following is the correct diagnosis

a Right 3rd rib inhalation dysfunction

b Left 3rd rib inhalation dysfunction

c Right 3rd rib inferior dysfunction

d Left 3rd rib exhalation dysfunction

e Left 3rd rib superior dysfunction

22 Which of the following is paired correctly

a Fryettersquos Principle 1 ndash Typically involves a single segment

b AA dysfunctions ndash Assessed with rotation

c Ribs 11 amp 12 ndash Predominantly superiorinferior motion

d Left oblique sacral axis ndash Positive left seated flexion test

e Right innominate posterior ndash Positive left standing flexion test

23 Which of the following dysfunctions will have a left positive standing flexion test and

a negative AP compression test

a Left innominate posterior

b Right superior innominate shear

c Left innominate outflare

d Right innominate inflare

e Left inferior innominate shear

24 You examine L1 and find the right transverse process to be more shallow than the left The findings are unchanged in both flexion and extension Which of the following is the correct diagnosis

a L1 flexed sidebent left rotated left

b L1 neutral sidebent right rotated right

c L1 neutral sidebent left rotated left

d L1 extended sidebent right rotated left

e L1 neutral sidebent left rotated right

25 Which of the following is true regarding a right superior innominate shear

a Negative standing flexion test

b Negative AP compression test

c ASIS and PSIS both superior on the right

d Right ASIS closer to midline

e Positive standing flexion test on the left

MODULE 2

1 Which of the following is true regarding muscle energy treatment

a Contraindicated in patients with osteopenia

b Primary treatment for patients in the ICU

c Utilizes patient force with equal physician counterforce X

d Patient effort is held for 90 seconds

e A-P curve must be flattened out during treatment

2 A patient is diagnosed with a right anterior innominate dysfunction Which of the

following is the correct description for muscle energy treatment of this dysfunction

a Right hip extended ndash patient flexes hip against resistance

b Right hip flexed ndash patient extends hip against resistance X

c Right hip extended ndash patient extends hip against resistance

d Right hip externally rotated ndash patient internally rotates hip against resistance

e Right hip abducted ndash patient adducts hip against resistance

3 Which of the following is true regarding HVLA thrust technique

a Indirect technique

b Useful in patients with joint hypermobility

c Thrust applied after backing away from barrier for 1-2 seconds

d Lumbar HVLA is contraindicated in patients with Down Syndrome

e Restrictive barrier must be engaged in all planes of motion X

4 Which of the following is true regarding Counterstrain treatment

a Restrictive barrier must be engaged in all planes of motion

b Treatment position should decrease tenderness by at least 70 X

c Treatment position is held for 3-4 seconds then released

d Patient assists in positioning for treatment

e Patient must slowly bring himself to neutral after treatment

5 Which of the following is correctly matched

a Counterstrain treatment of hip flexor ndash patientrsquos hip passively flexed X

b FPR treatment of shoulder external rotator ndash patientrsquos shoulder internally rotated

c HVLA treatment of C3 ERRSR ndash C3 placed into extension right rotation and right sidebending

d Muscle energy treatment of a bilateral sacral flexion ndash pressure on sacral base

e HVLA treatment of right unilateral sacral extension ndash superior thrust on right ILA

6 The heart receives sympathetic innervation from which of the following levels

a OA

b T1-T5 X

c T5-T9

d T10-L2

e S2-S4

7 Which of the following is true regarding FPR treatment

a Restrictive barrier must be engaged in all planes of motion

b Treatment position should decrease tenderness by at least 70

c Treatment position is held for 3-4 seconds then released X

d A-P curve is accentuated during treatment

e Facilitating force is an isometric contraction of the involved muscles

8 Which of the following is true about autonomic control of the lungs

a Sympathetic stimulation results in bronchoconstriction

b The lower lungs receive parasympathetic innervation from S2-S4

c Sympathetic innervation arises from the T5-T9 levels

d Increased thick secretions are a result of increased sympathetic activity X

e Sacral rocking will result in increased thin watery secretions

9 Which of the following techniques would be most useful in improving upper GI

motility in a patient with ileus

a Sacral rocking

b Rib raising for 30 seconds

c OA decompression

d Paraspinal inhibition for 120 seconds X

e Sacral inhibition

10 You assess C4 translation and find translation to be easier to the left The findings

become symmetrical in flexion but not in extension Which of the following is the

correct position of C4 for a HVLA thrust

a Flexed rotated right sidebent left

b Extended rotated left sidebent left X

c Neutral rotated right sidebent left

d Flexed rotated right sidebent right

e Extended rotated right sidebent right

11 You examine L3 and find the right transverse process to be deeper than the left The

findings are unchanged in flexion and become symmetrical in extension Which of

the following is the correct position of L3 for FPR treatment

a Flexed sidebent left rotated left

b Flexed sidebent right rotated right

c Extended sidebent left rotated left X

d Extended sidebent right rotated left

e Flexed sidebent left rotated right

12 Which of the following is true regarding autonomic innervation to the GU system

a Sympathetic stimulation increases ureteral peristalsis

b Sympathetic stimulation relaxes the bladder wall X

c Parasympathetic stimulation activates the bladder sphincter

d Parasympathetic stimulation activates the trigone

e Sympathetic stimulation increases GFR

13 Which of the following is a result of parasympathetic stimulation

a Peripheral vasodilation

b Ejaculation

c Increased salivation X

d Uterine contraction

e Ureterospasm

14 The gall bladder receives sympathetic innervation from which of the following

levels

a OA

b T1-T5

c T5-T9 X

d T10-L2

e S2-S4

15 You are attempting to treat a right 7th rib exhalation dysfunction with counterstrain

Which of the following is correct regarding the treatment

a Shaft of right 7th rib positioned closer to shaft of right 8th rib X

b Position should decrease tenderness by 50

c Position is held for 30 seconds

d Angle of right 7th rib tractioned inferiorly

e Positioning involves left sidebending and extension

16 Which of the following is correctly matched

a Muscle energy ndash indirect treatment

b HVLA ndash articulation positioned in free motion

c Counterstrain ndash starts with a pain scale X

d FPR ndash active direct treatment

e Muscle energy ndash useful for painful muscle groups

17 Which of the following would be most likely to inhibit uterine contractions in a

patient in premature labor

a Sacral rocking X

b OA decompression

c Paraspinal inhibition gt90 seconds at T5-T9

d Rib raising lt60 seconds at T10-L2

e Sacral inhibition

18 You assess the innominates and find the following

Positive standing flexion test on the left

Right ASIS inferior compared to left

Right PSIS inferior compared to left

ASIS to midline distance equal bilaterally

Which of the following correctly describes a treatment for this dysfunction

a Muscle energy ndash left hip flexed patient extends against resistance

b HVLA thrust ndash thrust directed inferiorly on left innominate X

c FPR ndash right innominate pushed inferiorly for 3-4 seconds

d Muscle energy ndash right hip pushed superiorly patient pushes inferiorly

e Counterstrain ndash left innominate held inferiorly for 90 seconds

19 The pancreas receives parasympathetic innervation from which of the following

levels

a OA X

b T1-T5

c T5-T9

d T10-L2

e S2-S4

20 Which of the following would most likely directly decrease gastric secretions

a Sacral rocking

b OA decompression

c Paraspinal inhibition gt90 seconds at T10-L2

d Rib raising lt60 seconds at T5-T9 X

e Sacral inhibition

21 Which of the following statements is true regarding osteopathic treatment techniques

a Counterstrain is contraindicated for painful muscle groups

b Osteoporosis is a contraindication for FPR

c Muscle energy should not be used on a person in the CCU X

d HVLA is useful in patients with active Rheumatoid Arthritis

e The restrictive barrier is engaged when using indirect techniques

22 You examine T7 and find the left transverse process to be deeper than the right The

findings become symmetrical in flexion and are unchanged in extension Which of

the following is the correct starting position for a muscle energy treatment

a T7 flexed sidebent left rotated left

b T7 flexed sidebent right rotated right

c T7 extended sidebent left rotated left X

d T7 extended sidebent right rotated left

e T7 flexed sidebent left rotated right

23 The sigmoid colon receives parasympathetic innervation from which of the following

levels

a OA

b T1-T5

c T5-T9

d T10-L2

e S2-S4 X

24 You assess OA translation and find translation to be easier to the right The findings

persist in both flexion and extension Which of the following is the correct position

for FPR treatment

a Flexed rotated right sidebent left

b Neutral rotated left sidebent right

c Neutral rotated right sidebent left X

d Neutral rotated left sidebent left

e Extended rotated right sidebent right

25 Which of the following would most likely directly increase lacrimation

a Sacral rocking

b OA decompression X

c Paraspinal inhibition gt90 seconds at T5-T9

d Rib raising lt60 seconds at T1-T4 X

e Sacral inhibition

MODULE 3 ndash Joanne Smith

1 Top 5 ddx peptic ulcer GERD costochondritis Pancreastitis MI angina pectoris2 E no contraindications3 A OA4 E strep throat

dysfunctions Performed Omitted

1 Examine the cervical region for somatic dysfunctionA OA ndash leftright translation1 neutral 10486351048635

2 flexion 104863510486353 extension 104863510486352 Examine the thoracic spine for dysfunction (T5-T9)A palpate transverse processes for rotation1 neutral 104863510486352 flexion 104863510486353 extension 10486351048635B palpate soft tissue for TART findings1 trapezius 104863510486352 rhomboids 104863510486353 levator scapulae 104863510486354 T5 ndash T9 Paraspinal muscles 10486351048635Osteopathic Treatment You must treat a minimum of 2 dysfunctions you found during your examination1 Assume OA NSLRR appropriate techniques to include HVLA muscle energy articulatory Still counterstrainindirect myofascial release facilitated positional release

10486351048635

2 Assume T7 FRLSL appropriate techniques to include HVLA muscle energy articulatory Still counterstrainindirect myofascial release facilitated positional release

10486351048635

3 Assume acute bilateral spasm of upper thoracic (T5-T9) paraspinal muscles appropriate techniques to include soft tissue counterstrainindirect facilitated positional release

10486351048635

4 Autonomic technique for altered vagal tone OA decompression 104863510486355 Autonomic technique for hypersympathetic tone in T5-T9 paraspinal muscles rib raising or paraspinal inhibition for gt 90 seconds

10486351048635

MODULE 4- Ernesto Rodriguez

PRETEST

1 Back pain from Colon cancer Lumbar strain Osteomyelitits Diverticulitis Back pain osteoarthritis Cauda equina syndrome

2 D3 E

Osteopathic Diagnosis assess where you would expect to find somatic dysfunctions

Performed Omitted

1 Examine the lumbar region (L1-L5) for somatic dysfunction

A palpate transverse processes for rotation

1 neutral _

2 flexion _

3 extension _

B palpate soft tissue for TART (Tenderness Asymmetry Restrictions Tissue texture change) findings

1 quadratus lumborum muscles _

2 Piriformis muscles _

3 Paraspinal muscles _

2 Examine the sacrum for somatic dysfunction (requires at least one motion test)

A depth at 4 corners of sacrum _

B ILA levelness _

C seated flexion test _

D motion at 4 corners of sacrum _

E motion in backward bending test _

F respiratory motion of sacrum assess at 4 corners _

3 Examine the pelvis for somatic dysfunction

A standing flexion test or AP compression test _

B ASIS heights _

C PSIS heights _

Osteopathic Treatment You must treat a minimum of 2 dysfunctions you found during your examination

1 Assume L5 FRLSL _

2 Assume left on left sacral torsion _

3 Assume left innominate anterior _

4 Assume spasmtender point in the left piriformis muscle appropriate techniques to include counterstrainindirect myofascial release

-

MODULE 5 ndash Connie Jones

Pretest

1 Bradycardia acute pulmonary edemaCHF pneumonia acute MI Acute pulmonary edema CHF2 A3 A4 A5 For the video this is what was done and not done

OA - nf NO E --gtNslrrno AA dx doneno cervical dx doneT1-T5 - nfe --gt t2 f sl rlparapsinals TART --gt L upper thoracic paraspinalno levator scapula no trap no rhomboids dx tx -

T2 HLVAOA decompr ndash for vagal tonerib raisin for sympth

MODULE 6 ndash Gretchen White

1 Atrial fibrillation hyperthyroidism and- thyrotoxicosis thyroid cancer hyperthryroid 2 a3 a4 a5

Osteopathic Diagnosis assess where you would expect to find somatic dysfunctions

Performed Omitted

1 Examine the cervical region for somatic dysfunction

A OA ndash leftright translation

1 neutral X

2 flexion X

6 3 extension X

7 B AA ndash rotation X

8 2 Examine the upper thoracic spine for dysfunction (T1-T5)

A palpate transverse processes for rotation

1 neutral X

2 flexion X

3 extension X

B palpate soft tissue for TART findings

1 trapezius X

2 rhomboids X

3 levator scapulae X

4 T1-T5 paraspinal muscles X

Osteopathic Treatment You must treat a minimum of 2 dysfunctions you found during your examination

1 Assume OA NSLRR appropriate techniques to include counterstrainindirect myofascial release facilitated positional release

X

2 Assume T2 FSLRL appropriate techniques to include counterstrainindirect myofascial release facilitated positional release

X

3 Assume acute spasm of left upper thoracic paraspinal muscles appropriate techniques to include counterstrainindirect myofascial release and facilitated positional release

X

4 Autonomic technique for altered vagal tone OA decompression X

5 Autonomic technique for hypersympathetic tone rib raising or paraspinal inhibition for gt 90 seconds

X

MODULE 7 ndash Emmitt Brown

1 AAA kidney stones groin hernia ileus kidneybladder CA testicular torsion kidneybladder infection appendicitis

2 C3 D

Osteopathic Diagnosis assess where you would expect to find somatic dysfunctions

Performed Omitted

1 Examine the cervical region for somatic dysfunctionA OA ndash leftright translation1 neutral X2 flexion X3 extension XB AA ndash rotation must flex at least 45deg X2 Examine the thoracic spine for dysfunction (T10-L2)A palpate transverse processes for rotation1 neutral X2 flexion X3 extension XB palpate soft tissue for TART findings4 T10 ndash L2 Paraspinal muscles X3 Examine the sacrum for somatic dysfunction (requires at least one motion test)A depth at 4 corners of sacrum XB ILA levelness XC seated flexion test XD motion at 4 corners of sacrum XE motion in backward bending test XF respiratory motion of sacrum assess at 4 corners X4 Examine the pelvis for somatic dysfunctionA standing flexion test or AP compression test XB ASIS heights XC PSIS heights XOsteopathic Treatment You must treat a minimum of 2 dysfunctions you found during your examination1 Assume OA NSLRR appropriate techniques to include HVLA muscle energy articulatory Still counterstrainindirect myofascial release facilitated positional release

X _

2 Assume T10 NRRSL appropriate techniques to include HVLA muscle energy articulatory Still counterstrainindirect myofascial release facilitated positional release

X

3 Assume acute bilateral spasm of thoracolumbar (T10-L2) paraspinal muscles RgtL appropriate techniques to include soft tissue counterstrainindirect facilitated positional release

X

4 Assume left on right sacral torsion appropriate techniques to include muscle energy counterstrainindirect myofascial release and facilitated positional release (No HVLA)

X

5 Autonomic technique for hypersympathetic tone in T10-L2 paraspinal muscles rib raising or paraspinal inhibition for gt 90 seconds

X

MODULE 8 ndash Peter Parker1 Kidney stone Psoas syndrome spondylolysis piriformis syndrome lumbar compression

fracture appendicitis pelvicabdominal CA2 C3 B

Osteopathic Diagnosis assess where you would expect to find somatic dysfunctions

Performed Omitted

1 Examine the thoracolumbar region (T10-L5) for somatic dysfunctionA palpate transverse processes for rotation1 neutral _2 flexion _3 extension _B palpate soft tissue for TART (Tenderness Asymmetry Restrictions Tissue texture change) findings1 Psoas muscles _2 Piriformis muscles _3 Paraspinal muscles _2 Examine the sacrum for somatic dysfunction (requires at least one motion test)A depth at 4 corners of sacrum _B ILA levelness _C seated flexion test _D motion at 4 corners of sacrum _E motion in backward bending test _F respiratory motion of sacrum assess at 4 corners _3 Examine the pelvis for somatic dysfunctionA standing flexion test or AP compression test _B ASIS heights _C PSIS heights _Osteopathic Treatment You must treat a minimum of 2 dysfunctions you found during your examination1 Assume L1 FRRSR appropriate techniques to include muscle energy counterstrain indirect myofascial release facilitated positional release (No HVLA)

_

2 Assume left on right sacral torsion appropriate techniques to include muscle energy counterstrainindirect myofascial release and facilitated positional release (No HVLA)

_

3 Assume spasmtender point in the right psoas muscle appropriate techniques to include counterstrainindirect myofascial release FPR ME

_

4 Assume spasmtender point in the left piriformis muscle appropriate techniques to include counterstrainindirect myofascial release FPR ME

_

MODULE 9 ndash Ingrid Bergman

1 Lumbar strain Dysmenorrhea PID lumbar strain ovarian cyst sacral somatic dysfunction2 D3 E

Osteopathic Diagnosis assess where you would expect to find somatic dysfunctions

Performed Omitted

1 Examine the lumbar region (L1-L5) for somatic dysfunctionA palpate transverse processes for rotation1 neutral _2 flexion _3 extension _B palpate soft tissue for TART (Tenderness Asymmetry Restrictions Tissue texture change) findings1 quadratus lumborum muscles _2 Piriformis muscles _3 Paraspinal muscles _2 Examine the sacrum for somatic dysfunction (requires at least one motion test)A depth at 4 corners of sacrum _B ILA levelness _C seated flexion test _D motion at 4 corners of sacrum _E motion in backward bending test _F respiratory motion of sacrum assess at 4 corners _3 Examine the pelvis for somatic dysfunctionA standing flexion test or AP compression test _B ASIS heights _C PSIS heights _Osteopathic Treatment You must treat a minimum of 2 dysfunctions you found during your examination1 Assume L3 FRLSL2 Assume left on right sacral torsion _3 Assume left superior innominate shear _

4 Autonomic technique for altered parasympathetic tone

Module 10 ndash Minnie Driver

Pretest

1 Pneumonia dissecting aortic aneurysm thoracic spine fracture MI pleurisy2 B3 E

Osteopathic Diagnosis assess where you would expect to find somatic dysfunctions

Performed Omitted

1 Examine the cervical region for somatic dysfunctionA OA ndash leftright translation1 neutral 104863510486352 flexion 104863510486353 extension 10486351048635B AA ndash rotation must flex at least 45deg 10486351048635C C2-C7 - translation1 neutral 104863510486352 flexion 104863510486353 extension 104863510486352 Examine the thoracic spine for dysfunction (T5-T9)A palpate transverse processes for rotation1 neutral 104863510486352 flexion 104863510486353 extension 10486351048635B palpate soft tissue for TART findings1 trapezius 104863510486352 rhomboids 104863510486353 levator scapulae 104863510486354 T5 ndash T9 Paraspinal muscles 104863510486353 Examine the ribs for somatic dysfunctionA Pump handle ribs (ribs 2-6) at or near sternal junction1 static 104863510486352 motion 10486351048635B Bucket handle ribs (ribs 6-10) between anterior and posterior axillary lines1 static 104863510486352 motion 10486351048635Osteopathic Treatment You must treat a minimum of 2 dysfunctions you found during your examination1 Assume T5 ESLRL 104863510486352 Assume acute bilateral spasm of upper thoracic (T4-T8) paraspinal 10486351048635

muscles3 Assume (R) Rib 2 pump handle inhalation dysfunction 104863510486354 Autonomic technique for altered vagal tone OA decompression 10486351048635

MODULE 11 ndash Jeff Weaver

1 Cervical radiculopathy thoracic outlet syndrome cubital tunnel syndrome carpal tunnel syndrome angina pectoris ulnar groove entrapment Guillian Barre

2 A3 B4 C

Osteopathic Diagnosis assess where you would expect to find somatic dysfunctions

Performed Omitted

1 Examine the cervical region for somatic dysfunctionA C2-C7 - translation1 neutral 104863510486352 flexion 104863510486353 extension 10486351048635B palpate soft tissue for TART (Tenderness Asymmetry Restrictions Tissue texture change) findings1 cervical paraspinal muscles 104863510486352 sternocleidomastoids 104863510486353 scalenes 104863510486352 Examine the thoracic spine for somatic dysfunctionA palpate transverse processes for rotation1 neutral 104863510486352 flexion 104863510486353 extension 10486351048635B palpate soft tissue for TART findings1 trapezius 104863510486352 pectoral musculature 104863510486353 T1-T4 paraspinal muscle 104863510486353 Examine the 1st rib for somatic dysfunction1 static 104863510486352 dynamic 10486351048635Osteopathic Treatment You must treat a minimum of 2 dysfunctions you found during your examination1 Assume hypertonic left pectoral muscle Appropriate techniques to include ME MFR counterstrain

10486351048635

2 Assume hypertonic left scalene muscle Appropriate techniques to include MFR counterstrain ME

10486351048635

3 Assume T4 NRLSR appropriate techniques to include HVLA muscle energy articulatory Still counterstrainindirect myofascial release facilitated positional release

10486351048635

4 Assume left sided thoracic paraspinal hypertonicity T1 ndash T4 appropriate techniques to include soft tissue deep pressure counterstrainindirect facilitated positional release

10486351048635

5 Assume left 1 st rib inhalation somatic dysfunction appropriate techniques to include HVLA ME Stillrsquos articulatory

10486351048635

MODULE 12 ndash Scarlett O-Hara

1 Osteoarthritis piriformis syndrome spinal stenosis lumbar radiculopathy DVT2 C3 D

Osteopathic Diagnosis assess where you would expect to find somatic dysfunctions

Performed Omitted

1 Examine the lumbar region (L1-L5) for somatic dysfunctionA palpate transverse processes for rotation1 neutral _2 flexion _3 extension _B palpate soft tissue for TART (Tenderness Asymmetry Restrictions Tissue texture change) findings1 Psoas muscles _2 Piriformis muscles _3 Paraspinal muscles _2 Examine the sacrum for somatic dysfunction (requires at least one motion test)A depth at 4 corners of sacrum _B ILA levelness _C seated flexion test _D motion at 4 corners of sacrum _E motion in backward bending test _F respiratory motion of sacrum assess at 4 corners _3 Examine the pelvis for somatic dysfunctionA standing flexion test or AP compression test _B ASIS heights _C PSIS heights _Osteopathic Treatment You must treat a minimum of 2 dysfunctions you found during your examination1 Assume left on right sacral torsion appropriate techniques to include muscle energy counterstrainindirect myofascial release and facilitated positional release (No HVLA)2 Assume left innominate anterior appropriate techniques to include muscle energy counterstrainindirect myofascial release and facilitated positional release (No HVLA)

_

3 Assume spasmtender point in the left piriformis muscle appropriate techniques to _

include counterstrainindirect myofascial release

MODULE 13 Clark Kent1 Asthma atypical pneumonia bronchitis upper respiratory infection pneumothorax2 B3 A

Osteopathic Diagnosis assess where you would expect to find somatic dysfunctions

Performed Omitted

1 Examine the cervical region for somatic dysfunctionA OA ndash leftright translation1 neutral _2 flexion _3 extension _B AA ndash rotation must flex at least 45deg _C C2-C7 - translation1 neutral 2 flexion _3 extension 2 Examine the thoracic spine for dysfunction (T1-T4)A palpate transverse processes for rotation1 neutral _2 flexion _3 extension _B palpate soft tissue for TART findings1 trapezius X2 rhomboids X3 levator scapulae X4 T1 ndash T4 Paraspinal muscles X3 Examine the ribs for somatic dysfunctionA Pump handle ribs (ribs 2-6) at or near sternal junction1 static X2 motion XB Bucket handle ribs (ribs 6-10) between anterior and posterior axillary lines1 static X2 motion XOsteopathic Treatment You must treat a minimum of 2 dysfunctions you found during your examination1 Assume OA NSLRR appropriate techniques to include HVLA muscle energy articulatory Still counterstrainindirect myofascial release facilitated positional release

_

2 Assume T2-4 NSLRR appropriate techniques to include HVLA muscle energy articulatory Still counterstrainindirect myofascial release facilitated positional release

_

3 Assume (L) ribs 2-4 exhalation pump handle appropriate techniques to include HVLA muscle energy articulatory Still counterstrainindirect myofascial release facilitated positional release

4 Autonomic technique for altered vagal tone OA decompression _5 Autonomic technique for hypersympathetic tone in T1-T5 paraspinal muscles rib raising or paraspinal inhibition for gt 90 seconds

X

MODULE 14 Bruce Wayne

1 Depression thyroid disorder migraine headache tension tension headache migraine2 C3 A

Osteopathic Diagnosis assess where you would expect to find somatic dysfunctions

Performed Omitted

1 Examine the cervical region for somatic dysfunctionA OA ndash leftright translation1 neutral 2 flexion 3 extension B AA ndash rotation must flex at least 45deg C C2-C7 - translation1 neutral 2 flexion 3 extension _2 Examine the thoracic spine for dysfunction (T1-T4)A palpate transverse processes for rotation1 neutral X2 flexion X3 extension XB palpate soft tissue for TART findings1 trapezius X2 T1 ndash T4 Paraspinal muscles XOsteopathic Treatment You must treat a minimum of 2 dysfunctions you found during your examination1 Assume OA NSRRL appropriate techniques to include HVLA muscle energy articulatory Still counterstrainindirect myofascial release facilitated positional release

2 Assume C4 FSLRL appropriate techniques to include HVLA muscle energy

articulatory Still counterstrainindirect myofascial release facilitated positional release2 Assume T2-4 NSLRR appropriate techniques to include HVLA muscle energy articulatory Still counterstrainindirect myofascial release facilitated positional release

_

4 Autonomic technique for altered vagal tone OA decompression _

  • Performed
  • Performed
  • Performed
  • Performed
  • Performed
  • Performed
Page 6: Performed · Web viewAAA, kidney stones, groin hernia, ileus, kidney/bladder CA, testicular torsion, kidney/bladder infection, appendicitis, C D Osteopathic Diagnosis: assess where

a A right unilateral sacral flexion will have a positive seated flexion test on the left

b A left on left sacral torsion will have a deep right sulcus

c A right on left sacral torsion will have a deep right sacral sulcus

d A left unilateral seated flexion will have a significantly higher ILA on the left

e A bilateral sacral flexion will have bilaterally shallow sulci

20 Which of the following is true regarding innominate dysfunctions

a Innominate flares are named for the side on which the ASIS is closer to midline

b The standing flexion test is negative with an innominate shear

c The ASIS compression test will be positive on the side opposite the positive standing flexion test

d A positive seated flexion test is expected on the right with a right innominate posterior

e The right ASIS is inferior with a right innominate anterior

21 You evaluate the 3rd ribs bilaterally and find the following

Right 3rd rib more prominent anteriorlyBoth sides rise during inhalationOnly the left side falls during exhalationWhich of the following is the correct diagnosis

a Right 3rd rib inhalation dysfunction

b Left 3rd rib inhalation dysfunction

c Right 3rd rib inferior dysfunction

d Left 3rd rib exhalation dysfunction

e Left 3rd rib superior dysfunction

22 Which of the following is paired correctly

a Fryettersquos Principle 1 ndash Typically involves a single segment

b AA dysfunctions ndash Assessed with rotation

c Ribs 11 amp 12 ndash Predominantly superiorinferior motion

d Left oblique sacral axis ndash Positive left seated flexion test

e Right innominate posterior ndash Positive left standing flexion test

23 Which of the following dysfunctions will have a left positive standing flexion test and

a negative AP compression test

a Left innominate posterior

b Right superior innominate shear

c Left innominate outflare

d Right innominate inflare

e Left inferior innominate shear

24 You examine L1 and find the right transverse process to be more shallow than the left The findings are unchanged in both flexion and extension Which of the following is the correct diagnosis

a L1 flexed sidebent left rotated left

b L1 neutral sidebent right rotated right

c L1 neutral sidebent left rotated left

d L1 extended sidebent right rotated left

e L1 neutral sidebent left rotated right

25 Which of the following is true regarding a right superior innominate shear

a Negative standing flexion test

b Negative AP compression test

c ASIS and PSIS both superior on the right

d Right ASIS closer to midline

e Positive standing flexion test on the left

MODULE 2

1 Which of the following is true regarding muscle energy treatment

a Contraindicated in patients with osteopenia

b Primary treatment for patients in the ICU

c Utilizes patient force with equal physician counterforce X

d Patient effort is held for 90 seconds

e A-P curve must be flattened out during treatment

2 A patient is diagnosed with a right anterior innominate dysfunction Which of the

following is the correct description for muscle energy treatment of this dysfunction

a Right hip extended ndash patient flexes hip against resistance

b Right hip flexed ndash patient extends hip against resistance X

c Right hip extended ndash patient extends hip against resistance

d Right hip externally rotated ndash patient internally rotates hip against resistance

e Right hip abducted ndash patient adducts hip against resistance

3 Which of the following is true regarding HVLA thrust technique

a Indirect technique

b Useful in patients with joint hypermobility

c Thrust applied after backing away from barrier for 1-2 seconds

d Lumbar HVLA is contraindicated in patients with Down Syndrome

e Restrictive barrier must be engaged in all planes of motion X

4 Which of the following is true regarding Counterstrain treatment

a Restrictive barrier must be engaged in all planes of motion

b Treatment position should decrease tenderness by at least 70 X

c Treatment position is held for 3-4 seconds then released

d Patient assists in positioning for treatment

e Patient must slowly bring himself to neutral after treatment

5 Which of the following is correctly matched

a Counterstrain treatment of hip flexor ndash patientrsquos hip passively flexed X

b FPR treatment of shoulder external rotator ndash patientrsquos shoulder internally rotated

c HVLA treatment of C3 ERRSR ndash C3 placed into extension right rotation and right sidebending

d Muscle energy treatment of a bilateral sacral flexion ndash pressure on sacral base

e HVLA treatment of right unilateral sacral extension ndash superior thrust on right ILA

6 The heart receives sympathetic innervation from which of the following levels

a OA

b T1-T5 X

c T5-T9

d T10-L2

e S2-S4

7 Which of the following is true regarding FPR treatment

a Restrictive barrier must be engaged in all planes of motion

b Treatment position should decrease tenderness by at least 70

c Treatment position is held for 3-4 seconds then released X

d A-P curve is accentuated during treatment

e Facilitating force is an isometric contraction of the involved muscles

8 Which of the following is true about autonomic control of the lungs

a Sympathetic stimulation results in bronchoconstriction

b The lower lungs receive parasympathetic innervation from S2-S4

c Sympathetic innervation arises from the T5-T9 levels

d Increased thick secretions are a result of increased sympathetic activity X

e Sacral rocking will result in increased thin watery secretions

9 Which of the following techniques would be most useful in improving upper GI

motility in a patient with ileus

a Sacral rocking

b Rib raising for 30 seconds

c OA decompression

d Paraspinal inhibition for 120 seconds X

e Sacral inhibition

10 You assess C4 translation and find translation to be easier to the left The findings

become symmetrical in flexion but not in extension Which of the following is the

correct position of C4 for a HVLA thrust

a Flexed rotated right sidebent left

b Extended rotated left sidebent left X

c Neutral rotated right sidebent left

d Flexed rotated right sidebent right

e Extended rotated right sidebent right

11 You examine L3 and find the right transverse process to be deeper than the left The

findings are unchanged in flexion and become symmetrical in extension Which of

the following is the correct position of L3 for FPR treatment

a Flexed sidebent left rotated left

b Flexed sidebent right rotated right

c Extended sidebent left rotated left X

d Extended sidebent right rotated left

e Flexed sidebent left rotated right

12 Which of the following is true regarding autonomic innervation to the GU system

a Sympathetic stimulation increases ureteral peristalsis

b Sympathetic stimulation relaxes the bladder wall X

c Parasympathetic stimulation activates the bladder sphincter

d Parasympathetic stimulation activates the trigone

e Sympathetic stimulation increases GFR

13 Which of the following is a result of parasympathetic stimulation

a Peripheral vasodilation

b Ejaculation

c Increased salivation X

d Uterine contraction

e Ureterospasm

14 The gall bladder receives sympathetic innervation from which of the following

levels

a OA

b T1-T5

c T5-T9 X

d T10-L2

e S2-S4

15 You are attempting to treat a right 7th rib exhalation dysfunction with counterstrain

Which of the following is correct regarding the treatment

a Shaft of right 7th rib positioned closer to shaft of right 8th rib X

b Position should decrease tenderness by 50

c Position is held for 30 seconds

d Angle of right 7th rib tractioned inferiorly

e Positioning involves left sidebending and extension

16 Which of the following is correctly matched

a Muscle energy ndash indirect treatment

b HVLA ndash articulation positioned in free motion

c Counterstrain ndash starts with a pain scale X

d FPR ndash active direct treatment

e Muscle energy ndash useful for painful muscle groups

17 Which of the following would be most likely to inhibit uterine contractions in a

patient in premature labor

a Sacral rocking X

b OA decompression

c Paraspinal inhibition gt90 seconds at T5-T9

d Rib raising lt60 seconds at T10-L2

e Sacral inhibition

18 You assess the innominates and find the following

Positive standing flexion test on the left

Right ASIS inferior compared to left

Right PSIS inferior compared to left

ASIS to midline distance equal bilaterally

Which of the following correctly describes a treatment for this dysfunction

a Muscle energy ndash left hip flexed patient extends against resistance

b HVLA thrust ndash thrust directed inferiorly on left innominate X

c FPR ndash right innominate pushed inferiorly for 3-4 seconds

d Muscle energy ndash right hip pushed superiorly patient pushes inferiorly

e Counterstrain ndash left innominate held inferiorly for 90 seconds

19 The pancreas receives parasympathetic innervation from which of the following

levels

a OA X

b T1-T5

c T5-T9

d T10-L2

e S2-S4

20 Which of the following would most likely directly decrease gastric secretions

a Sacral rocking

b OA decompression

c Paraspinal inhibition gt90 seconds at T10-L2

d Rib raising lt60 seconds at T5-T9 X

e Sacral inhibition

21 Which of the following statements is true regarding osteopathic treatment techniques

a Counterstrain is contraindicated for painful muscle groups

b Osteoporosis is a contraindication for FPR

c Muscle energy should not be used on a person in the CCU X

d HVLA is useful in patients with active Rheumatoid Arthritis

e The restrictive barrier is engaged when using indirect techniques

22 You examine T7 and find the left transverse process to be deeper than the right The

findings become symmetrical in flexion and are unchanged in extension Which of

the following is the correct starting position for a muscle energy treatment

a T7 flexed sidebent left rotated left

b T7 flexed sidebent right rotated right

c T7 extended sidebent left rotated left X

d T7 extended sidebent right rotated left

e T7 flexed sidebent left rotated right

23 The sigmoid colon receives parasympathetic innervation from which of the following

levels

a OA

b T1-T5

c T5-T9

d T10-L2

e S2-S4 X

24 You assess OA translation and find translation to be easier to the right The findings

persist in both flexion and extension Which of the following is the correct position

for FPR treatment

a Flexed rotated right sidebent left

b Neutral rotated left sidebent right

c Neutral rotated right sidebent left X

d Neutral rotated left sidebent left

e Extended rotated right sidebent right

25 Which of the following would most likely directly increase lacrimation

a Sacral rocking

b OA decompression X

c Paraspinal inhibition gt90 seconds at T5-T9

d Rib raising lt60 seconds at T1-T4 X

e Sacral inhibition

MODULE 3 ndash Joanne Smith

1 Top 5 ddx peptic ulcer GERD costochondritis Pancreastitis MI angina pectoris2 E no contraindications3 A OA4 E strep throat

dysfunctions Performed Omitted

1 Examine the cervical region for somatic dysfunctionA OA ndash leftright translation1 neutral 10486351048635

2 flexion 104863510486353 extension 104863510486352 Examine the thoracic spine for dysfunction (T5-T9)A palpate transverse processes for rotation1 neutral 104863510486352 flexion 104863510486353 extension 10486351048635B palpate soft tissue for TART findings1 trapezius 104863510486352 rhomboids 104863510486353 levator scapulae 104863510486354 T5 ndash T9 Paraspinal muscles 10486351048635Osteopathic Treatment You must treat a minimum of 2 dysfunctions you found during your examination1 Assume OA NSLRR appropriate techniques to include HVLA muscle energy articulatory Still counterstrainindirect myofascial release facilitated positional release

10486351048635

2 Assume T7 FRLSL appropriate techniques to include HVLA muscle energy articulatory Still counterstrainindirect myofascial release facilitated positional release

10486351048635

3 Assume acute bilateral spasm of upper thoracic (T5-T9) paraspinal muscles appropriate techniques to include soft tissue counterstrainindirect facilitated positional release

10486351048635

4 Autonomic technique for altered vagal tone OA decompression 104863510486355 Autonomic technique for hypersympathetic tone in T5-T9 paraspinal muscles rib raising or paraspinal inhibition for gt 90 seconds

10486351048635

MODULE 4- Ernesto Rodriguez

PRETEST

1 Back pain from Colon cancer Lumbar strain Osteomyelitits Diverticulitis Back pain osteoarthritis Cauda equina syndrome

2 D3 E

Osteopathic Diagnosis assess where you would expect to find somatic dysfunctions

Performed Omitted

1 Examine the lumbar region (L1-L5) for somatic dysfunction

A palpate transverse processes for rotation

1 neutral _

2 flexion _

3 extension _

B palpate soft tissue for TART (Tenderness Asymmetry Restrictions Tissue texture change) findings

1 quadratus lumborum muscles _

2 Piriformis muscles _

3 Paraspinal muscles _

2 Examine the sacrum for somatic dysfunction (requires at least one motion test)

A depth at 4 corners of sacrum _

B ILA levelness _

C seated flexion test _

D motion at 4 corners of sacrum _

E motion in backward bending test _

F respiratory motion of sacrum assess at 4 corners _

3 Examine the pelvis for somatic dysfunction

A standing flexion test or AP compression test _

B ASIS heights _

C PSIS heights _

Osteopathic Treatment You must treat a minimum of 2 dysfunctions you found during your examination

1 Assume L5 FRLSL _

2 Assume left on left sacral torsion _

3 Assume left innominate anterior _

4 Assume spasmtender point in the left piriformis muscle appropriate techniques to include counterstrainindirect myofascial release

-

MODULE 5 ndash Connie Jones

Pretest

1 Bradycardia acute pulmonary edemaCHF pneumonia acute MI Acute pulmonary edema CHF2 A3 A4 A5 For the video this is what was done and not done

OA - nf NO E --gtNslrrno AA dx doneno cervical dx doneT1-T5 - nfe --gt t2 f sl rlparapsinals TART --gt L upper thoracic paraspinalno levator scapula no trap no rhomboids dx tx -

T2 HLVAOA decompr ndash for vagal tonerib raisin for sympth

MODULE 6 ndash Gretchen White

1 Atrial fibrillation hyperthyroidism and- thyrotoxicosis thyroid cancer hyperthryroid 2 a3 a4 a5

Osteopathic Diagnosis assess where you would expect to find somatic dysfunctions

Performed Omitted

1 Examine the cervical region for somatic dysfunction

A OA ndash leftright translation

1 neutral X

2 flexion X

6 3 extension X

7 B AA ndash rotation X

8 2 Examine the upper thoracic spine for dysfunction (T1-T5)

A palpate transverse processes for rotation

1 neutral X

2 flexion X

3 extension X

B palpate soft tissue for TART findings

1 trapezius X

2 rhomboids X

3 levator scapulae X

4 T1-T5 paraspinal muscles X

Osteopathic Treatment You must treat a minimum of 2 dysfunctions you found during your examination

1 Assume OA NSLRR appropriate techniques to include counterstrainindirect myofascial release facilitated positional release

X

2 Assume T2 FSLRL appropriate techniques to include counterstrainindirect myofascial release facilitated positional release

X

3 Assume acute spasm of left upper thoracic paraspinal muscles appropriate techniques to include counterstrainindirect myofascial release and facilitated positional release

X

4 Autonomic technique for altered vagal tone OA decompression X

5 Autonomic technique for hypersympathetic tone rib raising or paraspinal inhibition for gt 90 seconds

X

MODULE 7 ndash Emmitt Brown

1 AAA kidney stones groin hernia ileus kidneybladder CA testicular torsion kidneybladder infection appendicitis

2 C3 D

Osteopathic Diagnosis assess where you would expect to find somatic dysfunctions

Performed Omitted

1 Examine the cervical region for somatic dysfunctionA OA ndash leftright translation1 neutral X2 flexion X3 extension XB AA ndash rotation must flex at least 45deg X2 Examine the thoracic spine for dysfunction (T10-L2)A palpate transverse processes for rotation1 neutral X2 flexion X3 extension XB palpate soft tissue for TART findings4 T10 ndash L2 Paraspinal muscles X3 Examine the sacrum for somatic dysfunction (requires at least one motion test)A depth at 4 corners of sacrum XB ILA levelness XC seated flexion test XD motion at 4 corners of sacrum XE motion in backward bending test XF respiratory motion of sacrum assess at 4 corners X4 Examine the pelvis for somatic dysfunctionA standing flexion test or AP compression test XB ASIS heights XC PSIS heights XOsteopathic Treatment You must treat a minimum of 2 dysfunctions you found during your examination1 Assume OA NSLRR appropriate techniques to include HVLA muscle energy articulatory Still counterstrainindirect myofascial release facilitated positional release

X _

2 Assume T10 NRRSL appropriate techniques to include HVLA muscle energy articulatory Still counterstrainindirect myofascial release facilitated positional release

X

3 Assume acute bilateral spasm of thoracolumbar (T10-L2) paraspinal muscles RgtL appropriate techniques to include soft tissue counterstrainindirect facilitated positional release

X

4 Assume left on right sacral torsion appropriate techniques to include muscle energy counterstrainindirect myofascial release and facilitated positional release (No HVLA)

X

5 Autonomic technique for hypersympathetic tone in T10-L2 paraspinal muscles rib raising or paraspinal inhibition for gt 90 seconds

X

MODULE 8 ndash Peter Parker1 Kidney stone Psoas syndrome spondylolysis piriformis syndrome lumbar compression

fracture appendicitis pelvicabdominal CA2 C3 B

Osteopathic Diagnosis assess where you would expect to find somatic dysfunctions

Performed Omitted

1 Examine the thoracolumbar region (T10-L5) for somatic dysfunctionA palpate transverse processes for rotation1 neutral _2 flexion _3 extension _B palpate soft tissue for TART (Tenderness Asymmetry Restrictions Tissue texture change) findings1 Psoas muscles _2 Piriformis muscles _3 Paraspinal muscles _2 Examine the sacrum for somatic dysfunction (requires at least one motion test)A depth at 4 corners of sacrum _B ILA levelness _C seated flexion test _D motion at 4 corners of sacrum _E motion in backward bending test _F respiratory motion of sacrum assess at 4 corners _3 Examine the pelvis for somatic dysfunctionA standing flexion test or AP compression test _B ASIS heights _C PSIS heights _Osteopathic Treatment You must treat a minimum of 2 dysfunctions you found during your examination1 Assume L1 FRRSR appropriate techniques to include muscle energy counterstrain indirect myofascial release facilitated positional release (No HVLA)

_

2 Assume left on right sacral torsion appropriate techniques to include muscle energy counterstrainindirect myofascial release and facilitated positional release (No HVLA)

_

3 Assume spasmtender point in the right psoas muscle appropriate techniques to include counterstrainindirect myofascial release FPR ME

_

4 Assume spasmtender point in the left piriformis muscle appropriate techniques to include counterstrainindirect myofascial release FPR ME

_

MODULE 9 ndash Ingrid Bergman

1 Lumbar strain Dysmenorrhea PID lumbar strain ovarian cyst sacral somatic dysfunction2 D3 E

Osteopathic Diagnosis assess where you would expect to find somatic dysfunctions

Performed Omitted

1 Examine the lumbar region (L1-L5) for somatic dysfunctionA palpate transverse processes for rotation1 neutral _2 flexion _3 extension _B palpate soft tissue for TART (Tenderness Asymmetry Restrictions Tissue texture change) findings1 quadratus lumborum muscles _2 Piriformis muscles _3 Paraspinal muscles _2 Examine the sacrum for somatic dysfunction (requires at least one motion test)A depth at 4 corners of sacrum _B ILA levelness _C seated flexion test _D motion at 4 corners of sacrum _E motion in backward bending test _F respiratory motion of sacrum assess at 4 corners _3 Examine the pelvis for somatic dysfunctionA standing flexion test or AP compression test _B ASIS heights _C PSIS heights _Osteopathic Treatment You must treat a minimum of 2 dysfunctions you found during your examination1 Assume L3 FRLSL2 Assume left on right sacral torsion _3 Assume left superior innominate shear _

4 Autonomic technique for altered parasympathetic tone

Module 10 ndash Minnie Driver

Pretest

1 Pneumonia dissecting aortic aneurysm thoracic spine fracture MI pleurisy2 B3 E

Osteopathic Diagnosis assess where you would expect to find somatic dysfunctions

Performed Omitted

1 Examine the cervical region for somatic dysfunctionA OA ndash leftright translation1 neutral 104863510486352 flexion 104863510486353 extension 10486351048635B AA ndash rotation must flex at least 45deg 10486351048635C C2-C7 - translation1 neutral 104863510486352 flexion 104863510486353 extension 104863510486352 Examine the thoracic spine for dysfunction (T5-T9)A palpate transverse processes for rotation1 neutral 104863510486352 flexion 104863510486353 extension 10486351048635B palpate soft tissue for TART findings1 trapezius 104863510486352 rhomboids 104863510486353 levator scapulae 104863510486354 T5 ndash T9 Paraspinal muscles 104863510486353 Examine the ribs for somatic dysfunctionA Pump handle ribs (ribs 2-6) at or near sternal junction1 static 104863510486352 motion 10486351048635B Bucket handle ribs (ribs 6-10) between anterior and posterior axillary lines1 static 104863510486352 motion 10486351048635Osteopathic Treatment You must treat a minimum of 2 dysfunctions you found during your examination1 Assume T5 ESLRL 104863510486352 Assume acute bilateral spasm of upper thoracic (T4-T8) paraspinal 10486351048635

muscles3 Assume (R) Rib 2 pump handle inhalation dysfunction 104863510486354 Autonomic technique for altered vagal tone OA decompression 10486351048635

MODULE 11 ndash Jeff Weaver

1 Cervical radiculopathy thoracic outlet syndrome cubital tunnel syndrome carpal tunnel syndrome angina pectoris ulnar groove entrapment Guillian Barre

2 A3 B4 C

Osteopathic Diagnosis assess where you would expect to find somatic dysfunctions

Performed Omitted

1 Examine the cervical region for somatic dysfunctionA C2-C7 - translation1 neutral 104863510486352 flexion 104863510486353 extension 10486351048635B palpate soft tissue for TART (Tenderness Asymmetry Restrictions Tissue texture change) findings1 cervical paraspinal muscles 104863510486352 sternocleidomastoids 104863510486353 scalenes 104863510486352 Examine the thoracic spine for somatic dysfunctionA palpate transverse processes for rotation1 neutral 104863510486352 flexion 104863510486353 extension 10486351048635B palpate soft tissue for TART findings1 trapezius 104863510486352 pectoral musculature 104863510486353 T1-T4 paraspinal muscle 104863510486353 Examine the 1st rib for somatic dysfunction1 static 104863510486352 dynamic 10486351048635Osteopathic Treatment You must treat a minimum of 2 dysfunctions you found during your examination1 Assume hypertonic left pectoral muscle Appropriate techniques to include ME MFR counterstrain

10486351048635

2 Assume hypertonic left scalene muscle Appropriate techniques to include MFR counterstrain ME

10486351048635

3 Assume T4 NRLSR appropriate techniques to include HVLA muscle energy articulatory Still counterstrainindirect myofascial release facilitated positional release

10486351048635

4 Assume left sided thoracic paraspinal hypertonicity T1 ndash T4 appropriate techniques to include soft tissue deep pressure counterstrainindirect facilitated positional release

10486351048635

5 Assume left 1 st rib inhalation somatic dysfunction appropriate techniques to include HVLA ME Stillrsquos articulatory

10486351048635

MODULE 12 ndash Scarlett O-Hara

1 Osteoarthritis piriformis syndrome spinal stenosis lumbar radiculopathy DVT2 C3 D

Osteopathic Diagnosis assess where you would expect to find somatic dysfunctions

Performed Omitted

1 Examine the lumbar region (L1-L5) for somatic dysfunctionA palpate transverse processes for rotation1 neutral _2 flexion _3 extension _B palpate soft tissue for TART (Tenderness Asymmetry Restrictions Tissue texture change) findings1 Psoas muscles _2 Piriformis muscles _3 Paraspinal muscles _2 Examine the sacrum for somatic dysfunction (requires at least one motion test)A depth at 4 corners of sacrum _B ILA levelness _C seated flexion test _D motion at 4 corners of sacrum _E motion in backward bending test _F respiratory motion of sacrum assess at 4 corners _3 Examine the pelvis for somatic dysfunctionA standing flexion test or AP compression test _B ASIS heights _C PSIS heights _Osteopathic Treatment You must treat a minimum of 2 dysfunctions you found during your examination1 Assume left on right sacral torsion appropriate techniques to include muscle energy counterstrainindirect myofascial release and facilitated positional release (No HVLA)2 Assume left innominate anterior appropriate techniques to include muscle energy counterstrainindirect myofascial release and facilitated positional release (No HVLA)

_

3 Assume spasmtender point in the left piriformis muscle appropriate techniques to _

include counterstrainindirect myofascial release

MODULE 13 Clark Kent1 Asthma atypical pneumonia bronchitis upper respiratory infection pneumothorax2 B3 A

Osteopathic Diagnosis assess where you would expect to find somatic dysfunctions

Performed Omitted

1 Examine the cervical region for somatic dysfunctionA OA ndash leftright translation1 neutral _2 flexion _3 extension _B AA ndash rotation must flex at least 45deg _C C2-C7 - translation1 neutral 2 flexion _3 extension 2 Examine the thoracic spine for dysfunction (T1-T4)A palpate transverse processes for rotation1 neutral _2 flexion _3 extension _B palpate soft tissue for TART findings1 trapezius X2 rhomboids X3 levator scapulae X4 T1 ndash T4 Paraspinal muscles X3 Examine the ribs for somatic dysfunctionA Pump handle ribs (ribs 2-6) at or near sternal junction1 static X2 motion XB Bucket handle ribs (ribs 6-10) between anterior and posterior axillary lines1 static X2 motion XOsteopathic Treatment You must treat a minimum of 2 dysfunctions you found during your examination1 Assume OA NSLRR appropriate techniques to include HVLA muscle energy articulatory Still counterstrainindirect myofascial release facilitated positional release

_

2 Assume T2-4 NSLRR appropriate techniques to include HVLA muscle energy articulatory Still counterstrainindirect myofascial release facilitated positional release

_

3 Assume (L) ribs 2-4 exhalation pump handle appropriate techniques to include HVLA muscle energy articulatory Still counterstrainindirect myofascial release facilitated positional release

4 Autonomic technique for altered vagal tone OA decompression _5 Autonomic technique for hypersympathetic tone in T1-T5 paraspinal muscles rib raising or paraspinal inhibition for gt 90 seconds

X

MODULE 14 Bruce Wayne

1 Depression thyroid disorder migraine headache tension tension headache migraine2 C3 A

Osteopathic Diagnosis assess where you would expect to find somatic dysfunctions

Performed Omitted

1 Examine the cervical region for somatic dysfunctionA OA ndash leftright translation1 neutral 2 flexion 3 extension B AA ndash rotation must flex at least 45deg C C2-C7 - translation1 neutral 2 flexion 3 extension _2 Examine the thoracic spine for dysfunction (T1-T4)A palpate transverse processes for rotation1 neutral X2 flexion X3 extension XB palpate soft tissue for TART findings1 trapezius X2 T1 ndash T4 Paraspinal muscles XOsteopathic Treatment You must treat a minimum of 2 dysfunctions you found during your examination1 Assume OA NSRRL appropriate techniques to include HVLA muscle energy articulatory Still counterstrainindirect myofascial release facilitated positional release

2 Assume C4 FSLRL appropriate techniques to include HVLA muscle energy

articulatory Still counterstrainindirect myofascial release facilitated positional release2 Assume T2-4 NSLRR appropriate techniques to include HVLA muscle energy articulatory Still counterstrainindirect myofascial release facilitated positional release

_

4 Autonomic technique for altered vagal tone OA decompression _

  • Performed
  • Performed
  • Performed
  • Performed
  • Performed
  • Performed
Page 7: Performed · Web viewAAA, kidney stones, groin hernia, ileus, kidney/bladder CA, testicular torsion, kidney/bladder infection, appendicitis, C D Osteopathic Diagnosis: assess where

23 Which of the following dysfunctions will have a left positive standing flexion test and

a negative AP compression test

a Left innominate posterior

b Right superior innominate shear

c Left innominate outflare

d Right innominate inflare

e Left inferior innominate shear

24 You examine L1 and find the right transverse process to be more shallow than the left The findings are unchanged in both flexion and extension Which of the following is the correct diagnosis

a L1 flexed sidebent left rotated left

b L1 neutral sidebent right rotated right

c L1 neutral sidebent left rotated left

d L1 extended sidebent right rotated left

e L1 neutral sidebent left rotated right

25 Which of the following is true regarding a right superior innominate shear

a Negative standing flexion test

b Negative AP compression test

c ASIS and PSIS both superior on the right

d Right ASIS closer to midline

e Positive standing flexion test on the left

MODULE 2

1 Which of the following is true regarding muscle energy treatment

a Contraindicated in patients with osteopenia

b Primary treatment for patients in the ICU

c Utilizes patient force with equal physician counterforce X

d Patient effort is held for 90 seconds

e A-P curve must be flattened out during treatment

2 A patient is diagnosed with a right anterior innominate dysfunction Which of the

following is the correct description for muscle energy treatment of this dysfunction

a Right hip extended ndash patient flexes hip against resistance

b Right hip flexed ndash patient extends hip against resistance X

c Right hip extended ndash patient extends hip against resistance

d Right hip externally rotated ndash patient internally rotates hip against resistance

e Right hip abducted ndash patient adducts hip against resistance

3 Which of the following is true regarding HVLA thrust technique

a Indirect technique

b Useful in patients with joint hypermobility

c Thrust applied after backing away from barrier for 1-2 seconds

d Lumbar HVLA is contraindicated in patients with Down Syndrome

e Restrictive barrier must be engaged in all planes of motion X

4 Which of the following is true regarding Counterstrain treatment

a Restrictive barrier must be engaged in all planes of motion

b Treatment position should decrease tenderness by at least 70 X

c Treatment position is held for 3-4 seconds then released

d Patient assists in positioning for treatment

e Patient must slowly bring himself to neutral after treatment

5 Which of the following is correctly matched

a Counterstrain treatment of hip flexor ndash patientrsquos hip passively flexed X

b FPR treatment of shoulder external rotator ndash patientrsquos shoulder internally rotated

c HVLA treatment of C3 ERRSR ndash C3 placed into extension right rotation and right sidebending

d Muscle energy treatment of a bilateral sacral flexion ndash pressure on sacral base

e HVLA treatment of right unilateral sacral extension ndash superior thrust on right ILA

6 The heart receives sympathetic innervation from which of the following levels

a OA

b T1-T5 X

c T5-T9

d T10-L2

e S2-S4

7 Which of the following is true regarding FPR treatment

a Restrictive barrier must be engaged in all planes of motion

b Treatment position should decrease tenderness by at least 70

c Treatment position is held for 3-4 seconds then released X

d A-P curve is accentuated during treatment

e Facilitating force is an isometric contraction of the involved muscles

8 Which of the following is true about autonomic control of the lungs

a Sympathetic stimulation results in bronchoconstriction

b The lower lungs receive parasympathetic innervation from S2-S4

c Sympathetic innervation arises from the T5-T9 levels

d Increased thick secretions are a result of increased sympathetic activity X

e Sacral rocking will result in increased thin watery secretions

9 Which of the following techniques would be most useful in improving upper GI

motility in a patient with ileus

a Sacral rocking

b Rib raising for 30 seconds

c OA decompression

d Paraspinal inhibition for 120 seconds X

e Sacral inhibition

10 You assess C4 translation and find translation to be easier to the left The findings

become symmetrical in flexion but not in extension Which of the following is the

correct position of C4 for a HVLA thrust

a Flexed rotated right sidebent left

b Extended rotated left sidebent left X

c Neutral rotated right sidebent left

d Flexed rotated right sidebent right

e Extended rotated right sidebent right

11 You examine L3 and find the right transverse process to be deeper than the left The

findings are unchanged in flexion and become symmetrical in extension Which of

the following is the correct position of L3 for FPR treatment

a Flexed sidebent left rotated left

b Flexed sidebent right rotated right

c Extended sidebent left rotated left X

d Extended sidebent right rotated left

e Flexed sidebent left rotated right

12 Which of the following is true regarding autonomic innervation to the GU system

a Sympathetic stimulation increases ureteral peristalsis

b Sympathetic stimulation relaxes the bladder wall X

c Parasympathetic stimulation activates the bladder sphincter

d Parasympathetic stimulation activates the trigone

e Sympathetic stimulation increases GFR

13 Which of the following is a result of parasympathetic stimulation

a Peripheral vasodilation

b Ejaculation

c Increased salivation X

d Uterine contraction

e Ureterospasm

14 The gall bladder receives sympathetic innervation from which of the following

levels

a OA

b T1-T5

c T5-T9 X

d T10-L2

e S2-S4

15 You are attempting to treat a right 7th rib exhalation dysfunction with counterstrain

Which of the following is correct regarding the treatment

a Shaft of right 7th rib positioned closer to shaft of right 8th rib X

b Position should decrease tenderness by 50

c Position is held for 30 seconds

d Angle of right 7th rib tractioned inferiorly

e Positioning involves left sidebending and extension

16 Which of the following is correctly matched

a Muscle energy ndash indirect treatment

b HVLA ndash articulation positioned in free motion

c Counterstrain ndash starts with a pain scale X

d FPR ndash active direct treatment

e Muscle energy ndash useful for painful muscle groups

17 Which of the following would be most likely to inhibit uterine contractions in a

patient in premature labor

a Sacral rocking X

b OA decompression

c Paraspinal inhibition gt90 seconds at T5-T9

d Rib raising lt60 seconds at T10-L2

e Sacral inhibition

18 You assess the innominates and find the following

Positive standing flexion test on the left

Right ASIS inferior compared to left

Right PSIS inferior compared to left

ASIS to midline distance equal bilaterally

Which of the following correctly describes a treatment for this dysfunction

a Muscle energy ndash left hip flexed patient extends against resistance

b HVLA thrust ndash thrust directed inferiorly on left innominate X

c FPR ndash right innominate pushed inferiorly for 3-4 seconds

d Muscle energy ndash right hip pushed superiorly patient pushes inferiorly

e Counterstrain ndash left innominate held inferiorly for 90 seconds

19 The pancreas receives parasympathetic innervation from which of the following

levels

a OA X

b T1-T5

c T5-T9

d T10-L2

e S2-S4

20 Which of the following would most likely directly decrease gastric secretions

a Sacral rocking

b OA decompression

c Paraspinal inhibition gt90 seconds at T10-L2

d Rib raising lt60 seconds at T5-T9 X

e Sacral inhibition

21 Which of the following statements is true regarding osteopathic treatment techniques

a Counterstrain is contraindicated for painful muscle groups

b Osteoporosis is a contraindication for FPR

c Muscle energy should not be used on a person in the CCU X

d HVLA is useful in patients with active Rheumatoid Arthritis

e The restrictive barrier is engaged when using indirect techniques

22 You examine T7 and find the left transverse process to be deeper than the right The

findings become symmetrical in flexion and are unchanged in extension Which of

the following is the correct starting position for a muscle energy treatment

a T7 flexed sidebent left rotated left

b T7 flexed sidebent right rotated right

c T7 extended sidebent left rotated left X

d T7 extended sidebent right rotated left

e T7 flexed sidebent left rotated right

23 The sigmoid colon receives parasympathetic innervation from which of the following

levels

a OA

b T1-T5

c T5-T9

d T10-L2

e S2-S4 X

24 You assess OA translation and find translation to be easier to the right The findings

persist in both flexion and extension Which of the following is the correct position

for FPR treatment

a Flexed rotated right sidebent left

b Neutral rotated left sidebent right

c Neutral rotated right sidebent left X

d Neutral rotated left sidebent left

e Extended rotated right sidebent right

25 Which of the following would most likely directly increase lacrimation

a Sacral rocking

b OA decompression X

c Paraspinal inhibition gt90 seconds at T5-T9

d Rib raising lt60 seconds at T1-T4 X

e Sacral inhibition

MODULE 3 ndash Joanne Smith

1 Top 5 ddx peptic ulcer GERD costochondritis Pancreastitis MI angina pectoris2 E no contraindications3 A OA4 E strep throat

dysfunctions Performed Omitted

1 Examine the cervical region for somatic dysfunctionA OA ndash leftright translation1 neutral 10486351048635

2 flexion 104863510486353 extension 104863510486352 Examine the thoracic spine for dysfunction (T5-T9)A palpate transverse processes for rotation1 neutral 104863510486352 flexion 104863510486353 extension 10486351048635B palpate soft tissue for TART findings1 trapezius 104863510486352 rhomboids 104863510486353 levator scapulae 104863510486354 T5 ndash T9 Paraspinal muscles 10486351048635Osteopathic Treatment You must treat a minimum of 2 dysfunctions you found during your examination1 Assume OA NSLRR appropriate techniques to include HVLA muscle energy articulatory Still counterstrainindirect myofascial release facilitated positional release

10486351048635

2 Assume T7 FRLSL appropriate techniques to include HVLA muscle energy articulatory Still counterstrainindirect myofascial release facilitated positional release

10486351048635

3 Assume acute bilateral spasm of upper thoracic (T5-T9) paraspinal muscles appropriate techniques to include soft tissue counterstrainindirect facilitated positional release

10486351048635

4 Autonomic technique for altered vagal tone OA decompression 104863510486355 Autonomic technique for hypersympathetic tone in T5-T9 paraspinal muscles rib raising or paraspinal inhibition for gt 90 seconds

10486351048635

MODULE 4- Ernesto Rodriguez

PRETEST

1 Back pain from Colon cancer Lumbar strain Osteomyelitits Diverticulitis Back pain osteoarthritis Cauda equina syndrome

2 D3 E

Osteopathic Diagnosis assess where you would expect to find somatic dysfunctions

Performed Omitted

1 Examine the lumbar region (L1-L5) for somatic dysfunction

A palpate transverse processes for rotation

1 neutral _

2 flexion _

3 extension _

B palpate soft tissue for TART (Tenderness Asymmetry Restrictions Tissue texture change) findings

1 quadratus lumborum muscles _

2 Piriformis muscles _

3 Paraspinal muscles _

2 Examine the sacrum for somatic dysfunction (requires at least one motion test)

A depth at 4 corners of sacrum _

B ILA levelness _

C seated flexion test _

D motion at 4 corners of sacrum _

E motion in backward bending test _

F respiratory motion of sacrum assess at 4 corners _

3 Examine the pelvis for somatic dysfunction

A standing flexion test or AP compression test _

B ASIS heights _

C PSIS heights _

Osteopathic Treatment You must treat a minimum of 2 dysfunctions you found during your examination

1 Assume L5 FRLSL _

2 Assume left on left sacral torsion _

3 Assume left innominate anterior _

4 Assume spasmtender point in the left piriformis muscle appropriate techniques to include counterstrainindirect myofascial release

-

MODULE 5 ndash Connie Jones

Pretest

1 Bradycardia acute pulmonary edemaCHF pneumonia acute MI Acute pulmonary edema CHF2 A3 A4 A5 For the video this is what was done and not done

OA - nf NO E --gtNslrrno AA dx doneno cervical dx doneT1-T5 - nfe --gt t2 f sl rlparapsinals TART --gt L upper thoracic paraspinalno levator scapula no trap no rhomboids dx tx -

T2 HLVAOA decompr ndash for vagal tonerib raisin for sympth

MODULE 6 ndash Gretchen White

1 Atrial fibrillation hyperthyroidism and- thyrotoxicosis thyroid cancer hyperthryroid 2 a3 a4 a5

Osteopathic Diagnosis assess where you would expect to find somatic dysfunctions

Performed Omitted

1 Examine the cervical region for somatic dysfunction

A OA ndash leftright translation

1 neutral X

2 flexion X

6 3 extension X

7 B AA ndash rotation X

8 2 Examine the upper thoracic spine for dysfunction (T1-T5)

A palpate transverse processes for rotation

1 neutral X

2 flexion X

3 extension X

B palpate soft tissue for TART findings

1 trapezius X

2 rhomboids X

3 levator scapulae X

4 T1-T5 paraspinal muscles X

Osteopathic Treatment You must treat a minimum of 2 dysfunctions you found during your examination

1 Assume OA NSLRR appropriate techniques to include counterstrainindirect myofascial release facilitated positional release

X

2 Assume T2 FSLRL appropriate techniques to include counterstrainindirect myofascial release facilitated positional release

X

3 Assume acute spasm of left upper thoracic paraspinal muscles appropriate techniques to include counterstrainindirect myofascial release and facilitated positional release

X

4 Autonomic technique for altered vagal tone OA decompression X

5 Autonomic technique for hypersympathetic tone rib raising or paraspinal inhibition for gt 90 seconds

X

MODULE 7 ndash Emmitt Brown

1 AAA kidney stones groin hernia ileus kidneybladder CA testicular torsion kidneybladder infection appendicitis

2 C3 D

Osteopathic Diagnosis assess where you would expect to find somatic dysfunctions

Performed Omitted

1 Examine the cervical region for somatic dysfunctionA OA ndash leftright translation1 neutral X2 flexion X3 extension XB AA ndash rotation must flex at least 45deg X2 Examine the thoracic spine for dysfunction (T10-L2)A palpate transverse processes for rotation1 neutral X2 flexion X3 extension XB palpate soft tissue for TART findings4 T10 ndash L2 Paraspinal muscles X3 Examine the sacrum for somatic dysfunction (requires at least one motion test)A depth at 4 corners of sacrum XB ILA levelness XC seated flexion test XD motion at 4 corners of sacrum XE motion in backward bending test XF respiratory motion of sacrum assess at 4 corners X4 Examine the pelvis for somatic dysfunctionA standing flexion test or AP compression test XB ASIS heights XC PSIS heights XOsteopathic Treatment You must treat a minimum of 2 dysfunctions you found during your examination1 Assume OA NSLRR appropriate techniques to include HVLA muscle energy articulatory Still counterstrainindirect myofascial release facilitated positional release

X _

2 Assume T10 NRRSL appropriate techniques to include HVLA muscle energy articulatory Still counterstrainindirect myofascial release facilitated positional release

X

3 Assume acute bilateral spasm of thoracolumbar (T10-L2) paraspinal muscles RgtL appropriate techniques to include soft tissue counterstrainindirect facilitated positional release

X

4 Assume left on right sacral torsion appropriate techniques to include muscle energy counterstrainindirect myofascial release and facilitated positional release (No HVLA)

X

5 Autonomic technique for hypersympathetic tone in T10-L2 paraspinal muscles rib raising or paraspinal inhibition for gt 90 seconds

X

MODULE 8 ndash Peter Parker1 Kidney stone Psoas syndrome spondylolysis piriformis syndrome lumbar compression

fracture appendicitis pelvicabdominal CA2 C3 B

Osteopathic Diagnosis assess where you would expect to find somatic dysfunctions

Performed Omitted

1 Examine the thoracolumbar region (T10-L5) for somatic dysfunctionA palpate transverse processes for rotation1 neutral _2 flexion _3 extension _B palpate soft tissue for TART (Tenderness Asymmetry Restrictions Tissue texture change) findings1 Psoas muscles _2 Piriformis muscles _3 Paraspinal muscles _2 Examine the sacrum for somatic dysfunction (requires at least one motion test)A depth at 4 corners of sacrum _B ILA levelness _C seated flexion test _D motion at 4 corners of sacrum _E motion in backward bending test _F respiratory motion of sacrum assess at 4 corners _3 Examine the pelvis for somatic dysfunctionA standing flexion test or AP compression test _B ASIS heights _C PSIS heights _Osteopathic Treatment You must treat a minimum of 2 dysfunctions you found during your examination1 Assume L1 FRRSR appropriate techniques to include muscle energy counterstrain indirect myofascial release facilitated positional release (No HVLA)

_

2 Assume left on right sacral torsion appropriate techniques to include muscle energy counterstrainindirect myofascial release and facilitated positional release (No HVLA)

_

3 Assume spasmtender point in the right psoas muscle appropriate techniques to include counterstrainindirect myofascial release FPR ME

_

4 Assume spasmtender point in the left piriformis muscle appropriate techniques to include counterstrainindirect myofascial release FPR ME

_

MODULE 9 ndash Ingrid Bergman

1 Lumbar strain Dysmenorrhea PID lumbar strain ovarian cyst sacral somatic dysfunction2 D3 E

Osteopathic Diagnosis assess where you would expect to find somatic dysfunctions

Performed Omitted

1 Examine the lumbar region (L1-L5) for somatic dysfunctionA palpate transverse processes for rotation1 neutral _2 flexion _3 extension _B palpate soft tissue for TART (Tenderness Asymmetry Restrictions Tissue texture change) findings1 quadratus lumborum muscles _2 Piriformis muscles _3 Paraspinal muscles _2 Examine the sacrum for somatic dysfunction (requires at least one motion test)A depth at 4 corners of sacrum _B ILA levelness _C seated flexion test _D motion at 4 corners of sacrum _E motion in backward bending test _F respiratory motion of sacrum assess at 4 corners _3 Examine the pelvis for somatic dysfunctionA standing flexion test or AP compression test _B ASIS heights _C PSIS heights _Osteopathic Treatment You must treat a minimum of 2 dysfunctions you found during your examination1 Assume L3 FRLSL2 Assume left on right sacral torsion _3 Assume left superior innominate shear _

4 Autonomic technique for altered parasympathetic tone

Module 10 ndash Minnie Driver

Pretest

1 Pneumonia dissecting aortic aneurysm thoracic spine fracture MI pleurisy2 B3 E

Osteopathic Diagnosis assess where you would expect to find somatic dysfunctions

Performed Omitted

1 Examine the cervical region for somatic dysfunctionA OA ndash leftright translation1 neutral 104863510486352 flexion 104863510486353 extension 10486351048635B AA ndash rotation must flex at least 45deg 10486351048635C C2-C7 - translation1 neutral 104863510486352 flexion 104863510486353 extension 104863510486352 Examine the thoracic spine for dysfunction (T5-T9)A palpate transverse processes for rotation1 neutral 104863510486352 flexion 104863510486353 extension 10486351048635B palpate soft tissue for TART findings1 trapezius 104863510486352 rhomboids 104863510486353 levator scapulae 104863510486354 T5 ndash T9 Paraspinal muscles 104863510486353 Examine the ribs for somatic dysfunctionA Pump handle ribs (ribs 2-6) at or near sternal junction1 static 104863510486352 motion 10486351048635B Bucket handle ribs (ribs 6-10) between anterior and posterior axillary lines1 static 104863510486352 motion 10486351048635Osteopathic Treatment You must treat a minimum of 2 dysfunctions you found during your examination1 Assume T5 ESLRL 104863510486352 Assume acute bilateral spasm of upper thoracic (T4-T8) paraspinal 10486351048635

muscles3 Assume (R) Rib 2 pump handle inhalation dysfunction 104863510486354 Autonomic technique for altered vagal tone OA decompression 10486351048635

MODULE 11 ndash Jeff Weaver

1 Cervical radiculopathy thoracic outlet syndrome cubital tunnel syndrome carpal tunnel syndrome angina pectoris ulnar groove entrapment Guillian Barre

2 A3 B4 C

Osteopathic Diagnosis assess where you would expect to find somatic dysfunctions

Performed Omitted

1 Examine the cervical region for somatic dysfunctionA C2-C7 - translation1 neutral 104863510486352 flexion 104863510486353 extension 10486351048635B palpate soft tissue for TART (Tenderness Asymmetry Restrictions Tissue texture change) findings1 cervical paraspinal muscles 104863510486352 sternocleidomastoids 104863510486353 scalenes 104863510486352 Examine the thoracic spine for somatic dysfunctionA palpate transverse processes for rotation1 neutral 104863510486352 flexion 104863510486353 extension 10486351048635B palpate soft tissue for TART findings1 trapezius 104863510486352 pectoral musculature 104863510486353 T1-T4 paraspinal muscle 104863510486353 Examine the 1st rib for somatic dysfunction1 static 104863510486352 dynamic 10486351048635Osteopathic Treatment You must treat a minimum of 2 dysfunctions you found during your examination1 Assume hypertonic left pectoral muscle Appropriate techniques to include ME MFR counterstrain

10486351048635

2 Assume hypertonic left scalene muscle Appropriate techniques to include MFR counterstrain ME

10486351048635

3 Assume T4 NRLSR appropriate techniques to include HVLA muscle energy articulatory Still counterstrainindirect myofascial release facilitated positional release

10486351048635

4 Assume left sided thoracic paraspinal hypertonicity T1 ndash T4 appropriate techniques to include soft tissue deep pressure counterstrainindirect facilitated positional release

10486351048635

5 Assume left 1 st rib inhalation somatic dysfunction appropriate techniques to include HVLA ME Stillrsquos articulatory

10486351048635

MODULE 12 ndash Scarlett O-Hara

1 Osteoarthritis piriformis syndrome spinal stenosis lumbar radiculopathy DVT2 C3 D

Osteopathic Diagnosis assess where you would expect to find somatic dysfunctions

Performed Omitted

1 Examine the lumbar region (L1-L5) for somatic dysfunctionA palpate transverse processes for rotation1 neutral _2 flexion _3 extension _B palpate soft tissue for TART (Tenderness Asymmetry Restrictions Tissue texture change) findings1 Psoas muscles _2 Piriformis muscles _3 Paraspinal muscles _2 Examine the sacrum for somatic dysfunction (requires at least one motion test)A depth at 4 corners of sacrum _B ILA levelness _C seated flexion test _D motion at 4 corners of sacrum _E motion in backward bending test _F respiratory motion of sacrum assess at 4 corners _3 Examine the pelvis for somatic dysfunctionA standing flexion test or AP compression test _B ASIS heights _C PSIS heights _Osteopathic Treatment You must treat a minimum of 2 dysfunctions you found during your examination1 Assume left on right sacral torsion appropriate techniques to include muscle energy counterstrainindirect myofascial release and facilitated positional release (No HVLA)2 Assume left innominate anterior appropriate techniques to include muscle energy counterstrainindirect myofascial release and facilitated positional release (No HVLA)

_

3 Assume spasmtender point in the left piriformis muscle appropriate techniques to _

include counterstrainindirect myofascial release

MODULE 13 Clark Kent1 Asthma atypical pneumonia bronchitis upper respiratory infection pneumothorax2 B3 A

Osteopathic Diagnosis assess where you would expect to find somatic dysfunctions

Performed Omitted

1 Examine the cervical region for somatic dysfunctionA OA ndash leftright translation1 neutral _2 flexion _3 extension _B AA ndash rotation must flex at least 45deg _C C2-C7 - translation1 neutral 2 flexion _3 extension 2 Examine the thoracic spine for dysfunction (T1-T4)A palpate transverse processes for rotation1 neutral _2 flexion _3 extension _B palpate soft tissue for TART findings1 trapezius X2 rhomboids X3 levator scapulae X4 T1 ndash T4 Paraspinal muscles X3 Examine the ribs for somatic dysfunctionA Pump handle ribs (ribs 2-6) at or near sternal junction1 static X2 motion XB Bucket handle ribs (ribs 6-10) between anterior and posterior axillary lines1 static X2 motion XOsteopathic Treatment You must treat a minimum of 2 dysfunctions you found during your examination1 Assume OA NSLRR appropriate techniques to include HVLA muscle energy articulatory Still counterstrainindirect myofascial release facilitated positional release

_

2 Assume T2-4 NSLRR appropriate techniques to include HVLA muscle energy articulatory Still counterstrainindirect myofascial release facilitated positional release

_

3 Assume (L) ribs 2-4 exhalation pump handle appropriate techniques to include HVLA muscle energy articulatory Still counterstrainindirect myofascial release facilitated positional release

4 Autonomic technique for altered vagal tone OA decompression _5 Autonomic technique for hypersympathetic tone in T1-T5 paraspinal muscles rib raising or paraspinal inhibition for gt 90 seconds

X

MODULE 14 Bruce Wayne

1 Depression thyroid disorder migraine headache tension tension headache migraine2 C3 A

Osteopathic Diagnosis assess where you would expect to find somatic dysfunctions

Performed Omitted

1 Examine the cervical region for somatic dysfunctionA OA ndash leftright translation1 neutral 2 flexion 3 extension B AA ndash rotation must flex at least 45deg C C2-C7 - translation1 neutral 2 flexion 3 extension _2 Examine the thoracic spine for dysfunction (T1-T4)A palpate transverse processes for rotation1 neutral X2 flexion X3 extension XB palpate soft tissue for TART findings1 trapezius X2 T1 ndash T4 Paraspinal muscles XOsteopathic Treatment You must treat a minimum of 2 dysfunctions you found during your examination1 Assume OA NSRRL appropriate techniques to include HVLA muscle energy articulatory Still counterstrainindirect myofascial release facilitated positional release

2 Assume C4 FSLRL appropriate techniques to include HVLA muscle energy

articulatory Still counterstrainindirect myofascial release facilitated positional release2 Assume T2-4 NSLRR appropriate techniques to include HVLA muscle energy articulatory Still counterstrainindirect myofascial release facilitated positional release

_

4 Autonomic technique for altered vagal tone OA decompression _

  • Performed
  • Performed
  • Performed
  • Performed
  • Performed
  • Performed
Page 8: Performed · Web viewAAA, kidney stones, groin hernia, ileus, kidney/bladder CA, testicular torsion, kidney/bladder infection, appendicitis, C D Osteopathic Diagnosis: assess where

d Patient effort is held for 90 seconds

e A-P curve must be flattened out during treatment

2 A patient is diagnosed with a right anterior innominate dysfunction Which of the

following is the correct description for muscle energy treatment of this dysfunction

a Right hip extended ndash patient flexes hip against resistance

b Right hip flexed ndash patient extends hip against resistance X

c Right hip extended ndash patient extends hip against resistance

d Right hip externally rotated ndash patient internally rotates hip against resistance

e Right hip abducted ndash patient adducts hip against resistance

3 Which of the following is true regarding HVLA thrust technique

a Indirect technique

b Useful in patients with joint hypermobility

c Thrust applied after backing away from barrier for 1-2 seconds

d Lumbar HVLA is contraindicated in patients with Down Syndrome

e Restrictive barrier must be engaged in all planes of motion X

4 Which of the following is true regarding Counterstrain treatment

a Restrictive barrier must be engaged in all planes of motion

b Treatment position should decrease tenderness by at least 70 X

c Treatment position is held for 3-4 seconds then released

d Patient assists in positioning for treatment

e Patient must slowly bring himself to neutral after treatment

5 Which of the following is correctly matched

a Counterstrain treatment of hip flexor ndash patientrsquos hip passively flexed X

b FPR treatment of shoulder external rotator ndash patientrsquos shoulder internally rotated

c HVLA treatment of C3 ERRSR ndash C3 placed into extension right rotation and right sidebending

d Muscle energy treatment of a bilateral sacral flexion ndash pressure on sacral base

e HVLA treatment of right unilateral sacral extension ndash superior thrust on right ILA

6 The heart receives sympathetic innervation from which of the following levels

a OA

b T1-T5 X

c T5-T9

d T10-L2

e S2-S4

7 Which of the following is true regarding FPR treatment

a Restrictive barrier must be engaged in all planes of motion

b Treatment position should decrease tenderness by at least 70

c Treatment position is held for 3-4 seconds then released X

d A-P curve is accentuated during treatment

e Facilitating force is an isometric contraction of the involved muscles

8 Which of the following is true about autonomic control of the lungs

a Sympathetic stimulation results in bronchoconstriction

b The lower lungs receive parasympathetic innervation from S2-S4

c Sympathetic innervation arises from the T5-T9 levels

d Increased thick secretions are a result of increased sympathetic activity X

e Sacral rocking will result in increased thin watery secretions

9 Which of the following techniques would be most useful in improving upper GI

motility in a patient with ileus

a Sacral rocking

b Rib raising for 30 seconds

c OA decompression

d Paraspinal inhibition for 120 seconds X

e Sacral inhibition

10 You assess C4 translation and find translation to be easier to the left The findings

become symmetrical in flexion but not in extension Which of the following is the

correct position of C4 for a HVLA thrust

a Flexed rotated right sidebent left

b Extended rotated left sidebent left X

c Neutral rotated right sidebent left

d Flexed rotated right sidebent right

e Extended rotated right sidebent right

11 You examine L3 and find the right transverse process to be deeper than the left The

findings are unchanged in flexion and become symmetrical in extension Which of

the following is the correct position of L3 for FPR treatment

a Flexed sidebent left rotated left

b Flexed sidebent right rotated right

c Extended sidebent left rotated left X

d Extended sidebent right rotated left

e Flexed sidebent left rotated right

12 Which of the following is true regarding autonomic innervation to the GU system

a Sympathetic stimulation increases ureteral peristalsis

b Sympathetic stimulation relaxes the bladder wall X

c Parasympathetic stimulation activates the bladder sphincter

d Parasympathetic stimulation activates the trigone

e Sympathetic stimulation increases GFR

13 Which of the following is a result of parasympathetic stimulation

a Peripheral vasodilation

b Ejaculation

c Increased salivation X

d Uterine contraction

e Ureterospasm

14 The gall bladder receives sympathetic innervation from which of the following

levels

a OA

b T1-T5

c T5-T9 X

d T10-L2

e S2-S4

15 You are attempting to treat a right 7th rib exhalation dysfunction with counterstrain

Which of the following is correct regarding the treatment

a Shaft of right 7th rib positioned closer to shaft of right 8th rib X

b Position should decrease tenderness by 50

c Position is held for 30 seconds

d Angle of right 7th rib tractioned inferiorly

e Positioning involves left sidebending and extension

16 Which of the following is correctly matched

a Muscle energy ndash indirect treatment

b HVLA ndash articulation positioned in free motion

c Counterstrain ndash starts with a pain scale X

d FPR ndash active direct treatment

e Muscle energy ndash useful for painful muscle groups

17 Which of the following would be most likely to inhibit uterine contractions in a

patient in premature labor

a Sacral rocking X

b OA decompression

c Paraspinal inhibition gt90 seconds at T5-T9

d Rib raising lt60 seconds at T10-L2

e Sacral inhibition

18 You assess the innominates and find the following

Positive standing flexion test on the left

Right ASIS inferior compared to left

Right PSIS inferior compared to left

ASIS to midline distance equal bilaterally

Which of the following correctly describes a treatment for this dysfunction

a Muscle energy ndash left hip flexed patient extends against resistance

b HVLA thrust ndash thrust directed inferiorly on left innominate X

c FPR ndash right innominate pushed inferiorly for 3-4 seconds

d Muscle energy ndash right hip pushed superiorly patient pushes inferiorly

e Counterstrain ndash left innominate held inferiorly for 90 seconds

19 The pancreas receives parasympathetic innervation from which of the following

levels

a OA X

b T1-T5

c T5-T9

d T10-L2

e S2-S4

20 Which of the following would most likely directly decrease gastric secretions

a Sacral rocking

b OA decompression

c Paraspinal inhibition gt90 seconds at T10-L2

d Rib raising lt60 seconds at T5-T9 X

e Sacral inhibition

21 Which of the following statements is true regarding osteopathic treatment techniques

a Counterstrain is contraindicated for painful muscle groups

b Osteoporosis is a contraindication for FPR

c Muscle energy should not be used on a person in the CCU X

d HVLA is useful in patients with active Rheumatoid Arthritis

e The restrictive barrier is engaged when using indirect techniques

22 You examine T7 and find the left transverse process to be deeper than the right The

findings become symmetrical in flexion and are unchanged in extension Which of

the following is the correct starting position for a muscle energy treatment

a T7 flexed sidebent left rotated left

b T7 flexed sidebent right rotated right

c T7 extended sidebent left rotated left X

d T7 extended sidebent right rotated left

e T7 flexed sidebent left rotated right

23 The sigmoid colon receives parasympathetic innervation from which of the following

levels

a OA

b T1-T5

c T5-T9

d T10-L2

e S2-S4 X

24 You assess OA translation and find translation to be easier to the right The findings

persist in both flexion and extension Which of the following is the correct position

for FPR treatment

a Flexed rotated right sidebent left

b Neutral rotated left sidebent right

c Neutral rotated right sidebent left X

d Neutral rotated left sidebent left

e Extended rotated right sidebent right

25 Which of the following would most likely directly increase lacrimation

a Sacral rocking

b OA decompression X

c Paraspinal inhibition gt90 seconds at T5-T9

d Rib raising lt60 seconds at T1-T4 X

e Sacral inhibition

MODULE 3 ndash Joanne Smith

1 Top 5 ddx peptic ulcer GERD costochondritis Pancreastitis MI angina pectoris2 E no contraindications3 A OA4 E strep throat

dysfunctions Performed Omitted

1 Examine the cervical region for somatic dysfunctionA OA ndash leftright translation1 neutral 10486351048635

2 flexion 104863510486353 extension 104863510486352 Examine the thoracic spine for dysfunction (T5-T9)A palpate transverse processes for rotation1 neutral 104863510486352 flexion 104863510486353 extension 10486351048635B palpate soft tissue for TART findings1 trapezius 104863510486352 rhomboids 104863510486353 levator scapulae 104863510486354 T5 ndash T9 Paraspinal muscles 10486351048635Osteopathic Treatment You must treat a minimum of 2 dysfunctions you found during your examination1 Assume OA NSLRR appropriate techniques to include HVLA muscle energy articulatory Still counterstrainindirect myofascial release facilitated positional release

10486351048635

2 Assume T7 FRLSL appropriate techniques to include HVLA muscle energy articulatory Still counterstrainindirect myofascial release facilitated positional release

10486351048635

3 Assume acute bilateral spasm of upper thoracic (T5-T9) paraspinal muscles appropriate techniques to include soft tissue counterstrainindirect facilitated positional release

10486351048635

4 Autonomic technique for altered vagal tone OA decompression 104863510486355 Autonomic technique for hypersympathetic tone in T5-T9 paraspinal muscles rib raising or paraspinal inhibition for gt 90 seconds

10486351048635

MODULE 4- Ernesto Rodriguez

PRETEST

1 Back pain from Colon cancer Lumbar strain Osteomyelitits Diverticulitis Back pain osteoarthritis Cauda equina syndrome

2 D3 E

Osteopathic Diagnosis assess where you would expect to find somatic dysfunctions

Performed Omitted

1 Examine the lumbar region (L1-L5) for somatic dysfunction

A palpate transverse processes for rotation

1 neutral _

2 flexion _

3 extension _

B palpate soft tissue for TART (Tenderness Asymmetry Restrictions Tissue texture change) findings

1 quadratus lumborum muscles _

2 Piriformis muscles _

3 Paraspinal muscles _

2 Examine the sacrum for somatic dysfunction (requires at least one motion test)

A depth at 4 corners of sacrum _

B ILA levelness _

C seated flexion test _

D motion at 4 corners of sacrum _

E motion in backward bending test _

F respiratory motion of sacrum assess at 4 corners _

3 Examine the pelvis for somatic dysfunction

A standing flexion test or AP compression test _

B ASIS heights _

C PSIS heights _

Osteopathic Treatment You must treat a minimum of 2 dysfunctions you found during your examination

1 Assume L5 FRLSL _

2 Assume left on left sacral torsion _

3 Assume left innominate anterior _

4 Assume spasmtender point in the left piriformis muscle appropriate techniques to include counterstrainindirect myofascial release

-

MODULE 5 ndash Connie Jones

Pretest

1 Bradycardia acute pulmonary edemaCHF pneumonia acute MI Acute pulmonary edema CHF2 A3 A4 A5 For the video this is what was done and not done

OA - nf NO E --gtNslrrno AA dx doneno cervical dx doneT1-T5 - nfe --gt t2 f sl rlparapsinals TART --gt L upper thoracic paraspinalno levator scapula no trap no rhomboids dx tx -

T2 HLVAOA decompr ndash for vagal tonerib raisin for sympth

MODULE 6 ndash Gretchen White

1 Atrial fibrillation hyperthyroidism and- thyrotoxicosis thyroid cancer hyperthryroid 2 a3 a4 a5

Osteopathic Diagnosis assess where you would expect to find somatic dysfunctions

Performed Omitted

1 Examine the cervical region for somatic dysfunction

A OA ndash leftright translation

1 neutral X

2 flexion X

6 3 extension X

7 B AA ndash rotation X

8 2 Examine the upper thoracic spine for dysfunction (T1-T5)

A palpate transverse processes for rotation

1 neutral X

2 flexion X

3 extension X

B palpate soft tissue for TART findings

1 trapezius X

2 rhomboids X

3 levator scapulae X

4 T1-T5 paraspinal muscles X

Osteopathic Treatment You must treat a minimum of 2 dysfunctions you found during your examination

1 Assume OA NSLRR appropriate techniques to include counterstrainindirect myofascial release facilitated positional release

X

2 Assume T2 FSLRL appropriate techniques to include counterstrainindirect myofascial release facilitated positional release

X

3 Assume acute spasm of left upper thoracic paraspinal muscles appropriate techniques to include counterstrainindirect myofascial release and facilitated positional release

X

4 Autonomic technique for altered vagal tone OA decompression X

5 Autonomic technique for hypersympathetic tone rib raising or paraspinal inhibition for gt 90 seconds

X

MODULE 7 ndash Emmitt Brown

1 AAA kidney stones groin hernia ileus kidneybladder CA testicular torsion kidneybladder infection appendicitis

2 C3 D

Osteopathic Diagnosis assess where you would expect to find somatic dysfunctions

Performed Omitted

1 Examine the cervical region for somatic dysfunctionA OA ndash leftright translation1 neutral X2 flexion X3 extension XB AA ndash rotation must flex at least 45deg X2 Examine the thoracic spine for dysfunction (T10-L2)A palpate transverse processes for rotation1 neutral X2 flexion X3 extension XB palpate soft tissue for TART findings4 T10 ndash L2 Paraspinal muscles X3 Examine the sacrum for somatic dysfunction (requires at least one motion test)A depth at 4 corners of sacrum XB ILA levelness XC seated flexion test XD motion at 4 corners of sacrum XE motion in backward bending test XF respiratory motion of sacrum assess at 4 corners X4 Examine the pelvis for somatic dysfunctionA standing flexion test or AP compression test XB ASIS heights XC PSIS heights XOsteopathic Treatment You must treat a minimum of 2 dysfunctions you found during your examination1 Assume OA NSLRR appropriate techniques to include HVLA muscle energy articulatory Still counterstrainindirect myofascial release facilitated positional release

X _

2 Assume T10 NRRSL appropriate techniques to include HVLA muscle energy articulatory Still counterstrainindirect myofascial release facilitated positional release

X

3 Assume acute bilateral spasm of thoracolumbar (T10-L2) paraspinal muscles RgtL appropriate techniques to include soft tissue counterstrainindirect facilitated positional release

X

4 Assume left on right sacral torsion appropriate techniques to include muscle energy counterstrainindirect myofascial release and facilitated positional release (No HVLA)

X

5 Autonomic technique for hypersympathetic tone in T10-L2 paraspinal muscles rib raising or paraspinal inhibition for gt 90 seconds

X

MODULE 8 ndash Peter Parker1 Kidney stone Psoas syndrome spondylolysis piriformis syndrome lumbar compression

fracture appendicitis pelvicabdominal CA2 C3 B

Osteopathic Diagnosis assess where you would expect to find somatic dysfunctions

Performed Omitted

1 Examine the thoracolumbar region (T10-L5) for somatic dysfunctionA palpate transverse processes for rotation1 neutral _2 flexion _3 extension _B palpate soft tissue for TART (Tenderness Asymmetry Restrictions Tissue texture change) findings1 Psoas muscles _2 Piriformis muscles _3 Paraspinal muscles _2 Examine the sacrum for somatic dysfunction (requires at least one motion test)A depth at 4 corners of sacrum _B ILA levelness _C seated flexion test _D motion at 4 corners of sacrum _E motion in backward bending test _F respiratory motion of sacrum assess at 4 corners _3 Examine the pelvis for somatic dysfunctionA standing flexion test or AP compression test _B ASIS heights _C PSIS heights _Osteopathic Treatment You must treat a minimum of 2 dysfunctions you found during your examination1 Assume L1 FRRSR appropriate techniques to include muscle energy counterstrain indirect myofascial release facilitated positional release (No HVLA)

_

2 Assume left on right sacral torsion appropriate techniques to include muscle energy counterstrainindirect myofascial release and facilitated positional release (No HVLA)

_

3 Assume spasmtender point in the right psoas muscle appropriate techniques to include counterstrainindirect myofascial release FPR ME

_

4 Assume spasmtender point in the left piriformis muscle appropriate techniques to include counterstrainindirect myofascial release FPR ME

_

MODULE 9 ndash Ingrid Bergman

1 Lumbar strain Dysmenorrhea PID lumbar strain ovarian cyst sacral somatic dysfunction2 D3 E

Osteopathic Diagnosis assess where you would expect to find somatic dysfunctions

Performed Omitted

1 Examine the lumbar region (L1-L5) for somatic dysfunctionA palpate transverse processes for rotation1 neutral _2 flexion _3 extension _B palpate soft tissue for TART (Tenderness Asymmetry Restrictions Tissue texture change) findings1 quadratus lumborum muscles _2 Piriformis muscles _3 Paraspinal muscles _2 Examine the sacrum for somatic dysfunction (requires at least one motion test)A depth at 4 corners of sacrum _B ILA levelness _C seated flexion test _D motion at 4 corners of sacrum _E motion in backward bending test _F respiratory motion of sacrum assess at 4 corners _3 Examine the pelvis for somatic dysfunctionA standing flexion test or AP compression test _B ASIS heights _C PSIS heights _Osteopathic Treatment You must treat a minimum of 2 dysfunctions you found during your examination1 Assume L3 FRLSL2 Assume left on right sacral torsion _3 Assume left superior innominate shear _

4 Autonomic technique for altered parasympathetic tone

Module 10 ndash Minnie Driver

Pretest

1 Pneumonia dissecting aortic aneurysm thoracic spine fracture MI pleurisy2 B3 E

Osteopathic Diagnosis assess where you would expect to find somatic dysfunctions

Performed Omitted

1 Examine the cervical region for somatic dysfunctionA OA ndash leftright translation1 neutral 104863510486352 flexion 104863510486353 extension 10486351048635B AA ndash rotation must flex at least 45deg 10486351048635C C2-C7 - translation1 neutral 104863510486352 flexion 104863510486353 extension 104863510486352 Examine the thoracic spine for dysfunction (T5-T9)A palpate transverse processes for rotation1 neutral 104863510486352 flexion 104863510486353 extension 10486351048635B palpate soft tissue for TART findings1 trapezius 104863510486352 rhomboids 104863510486353 levator scapulae 104863510486354 T5 ndash T9 Paraspinal muscles 104863510486353 Examine the ribs for somatic dysfunctionA Pump handle ribs (ribs 2-6) at or near sternal junction1 static 104863510486352 motion 10486351048635B Bucket handle ribs (ribs 6-10) between anterior and posterior axillary lines1 static 104863510486352 motion 10486351048635Osteopathic Treatment You must treat a minimum of 2 dysfunctions you found during your examination1 Assume T5 ESLRL 104863510486352 Assume acute bilateral spasm of upper thoracic (T4-T8) paraspinal 10486351048635

muscles3 Assume (R) Rib 2 pump handle inhalation dysfunction 104863510486354 Autonomic technique for altered vagal tone OA decompression 10486351048635

MODULE 11 ndash Jeff Weaver

1 Cervical radiculopathy thoracic outlet syndrome cubital tunnel syndrome carpal tunnel syndrome angina pectoris ulnar groove entrapment Guillian Barre

2 A3 B4 C

Osteopathic Diagnosis assess where you would expect to find somatic dysfunctions

Performed Omitted

1 Examine the cervical region for somatic dysfunctionA C2-C7 - translation1 neutral 104863510486352 flexion 104863510486353 extension 10486351048635B palpate soft tissue for TART (Tenderness Asymmetry Restrictions Tissue texture change) findings1 cervical paraspinal muscles 104863510486352 sternocleidomastoids 104863510486353 scalenes 104863510486352 Examine the thoracic spine for somatic dysfunctionA palpate transverse processes for rotation1 neutral 104863510486352 flexion 104863510486353 extension 10486351048635B palpate soft tissue for TART findings1 trapezius 104863510486352 pectoral musculature 104863510486353 T1-T4 paraspinal muscle 104863510486353 Examine the 1st rib for somatic dysfunction1 static 104863510486352 dynamic 10486351048635Osteopathic Treatment You must treat a minimum of 2 dysfunctions you found during your examination1 Assume hypertonic left pectoral muscle Appropriate techniques to include ME MFR counterstrain

10486351048635

2 Assume hypertonic left scalene muscle Appropriate techniques to include MFR counterstrain ME

10486351048635

3 Assume T4 NRLSR appropriate techniques to include HVLA muscle energy articulatory Still counterstrainindirect myofascial release facilitated positional release

10486351048635

4 Assume left sided thoracic paraspinal hypertonicity T1 ndash T4 appropriate techniques to include soft tissue deep pressure counterstrainindirect facilitated positional release

10486351048635

5 Assume left 1 st rib inhalation somatic dysfunction appropriate techniques to include HVLA ME Stillrsquos articulatory

10486351048635

MODULE 12 ndash Scarlett O-Hara

1 Osteoarthritis piriformis syndrome spinal stenosis lumbar radiculopathy DVT2 C3 D

Osteopathic Diagnosis assess where you would expect to find somatic dysfunctions

Performed Omitted

1 Examine the lumbar region (L1-L5) for somatic dysfunctionA palpate transverse processes for rotation1 neutral _2 flexion _3 extension _B palpate soft tissue for TART (Tenderness Asymmetry Restrictions Tissue texture change) findings1 Psoas muscles _2 Piriformis muscles _3 Paraspinal muscles _2 Examine the sacrum for somatic dysfunction (requires at least one motion test)A depth at 4 corners of sacrum _B ILA levelness _C seated flexion test _D motion at 4 corners of sacrum _E motion in backward bending test _F respiratory motion of sacrum assess at 4 corners _3 Examine the pelvis for somatic dysfunctionA standing flexion test or AP compression test _B ASIS heights _C PSIS heights _Osteopathic Treatment You must treat a minimum of 2 dysfunctions you found during your examination1 Assume left on right sacral torsion appropriate techniques to include muscle energy counterstrainindirect myofascial release and facilitated positional release (No HVLA)2 Assume left innominate anterior appropriate techniques to include muscle energy counterstrainindirect myofascial release and facilitated positional release (No HVLA)

_

3 Assume spasmtender point in the left piriformis muscle appropriate techniques to _

include counterstrainindirect myofascial release

MODULE 13 Clark Kent1 Asthma atypical pneumonia bronchitis upper respiratory infection pneumothorax2 B3 A

Osteopathic Diagnosis assess where you would expect to find somatic dysfunctions

Performed Omitted

1 Examine the cervical region for somatic dysfunctionA OA ndash leftright translation1 neutral _2 flexion _3 extension _B AA ndash rotation must flex at least 45deg _C C2-C7 - translation1 neutral 2 flexion _3 extension 2 Examine the thoracic spine for dysfunction (T1-T4)A palpate transverse processes for rotation1 neutral _2 flexion _3 extension _B palpate soft tissue for TART findings1 trapezius X2 rhomboids X3 levator scapulae X4 T1 ndash T4 Paraspinal muscles X3 Examine the ribs for somatic dysfunctionA Pump handle ribs (ribs 2-6) at or near sternal junction1 static X2 motion XB Bucket handle ribs (ribs 6-10) between anterior and posterior axillary lines1 static X2 motion XOsteopathic Treatment You must treat a minimum of 2 dysfunctions you found during your examination1 Assume OA NSLRR appropriate techniques to include HVLA muscle energy articulatory Still counterstrainindirect myofascial release facilitated positional release

_

2 Assume T2-4 NSLRR appropriate techniques to include HVLA muscle energy articulatory Still counterstrainindirect myofascial release facilitated positional release

_

3 Assume (L) ribs 2-4 exhalation pump handle appropriate techniques to include HVLA muscle energy articulatory Still counterstrainindirect myofascial release facilitated positional release

4 Autonomic technique for altered vagal tone OA decompression _5 Autonomic technique for hypersympathetic tone in T1-T5 paraspinal muscles rib raising or paraspinal inhibition for gt 90 seconds

X

MODULE 14 Bruce Wayne

1 Depression thyroid disorder migraine headache tension tension headache migraine2 C3 A

Osteopathic Diagnosis assess where you would expect to find somatic dysfunctions

Performed Omitted

1 Examine the cervical region for somatic dysfunctionA OA ndash leftright translation1 neutral 2 flexion 3 extension B AA ndash rotation must flex at least 45deg C C2-C7 - translation1 neutral 2 flexion 3 extension _2 Examine the thoracic spine for dysfunction (T1-T4)A palpate transverse processes for rotation1 neutral X2 flexion X3 extension XB palpate soft tissue for TART findings1 trapezius X2 T1 ndash T4 Paraspinal muscles XOsteopathic Treatment You must treat a minimum of 2 dysfunctions you found during your examination1 Assume OA NSRRL appropriate techniques to include HVLA muscle energy articulatory Still counterstrainindirect myofascial release facilitated positional release

2 Assume C4 FSLRL appropriate techniques to include HVLA muscle energy

articulatory Still counterstrainindirect myofascial release facilitated positional release2 Assume T2-4 NSLRR appropriate techniques to include HVLA muscle energy articulatory Still counterstrainindirect myofascial release facilitated positional release

_

4 Autonomic technique for altered vagal tone OA decompression _

  • Performed
  • Performed
  • Performed
  • Performed
  • Performed
  • Performed
Page 9: Performed · Web viewAAA, kidney stones, groin hernia, ileus, kidney/bladder CA, testicular torsion, kidney/bladder infection, appendicitis, C D Osteopathic Diagnosis: assess where

d Muscle energy treatment of a bilateral sacral flexion ndash pressure on sacral base

e HVLA treatment of right unilateral sacral extension ndash superior thrust on right ILA

6 The heart receives sympathetic innervation from which of the following levels

a OA

b T1-T5 X

c T5-T9

d T10-L2

e S2-S4

7 Which of the following is true regarding FPR treatment

a Restrictive barrier must be engaged in all planes of motion

b Treatment position should decrease tenderness by at least 70

c Treatment position is held for 3-4 seconds then released X

d A-P curve is accentuated during treatment

e Facilitating force is an isometric contraction of the involved muscles

8 Which of the following is true about autonomic control of the lungs

a Sympathetic stimulation results in bronchoconstriction

b The lower lungs receive parasympathetic innervation from S2-S4

c Sympathetic innervation arises from the T5-T9 levels

d Increased thick secretions are a result of increased sympathetic activity X

e Sacral rocking will result in increased thin watery secretions

9 Which of the following techniques would be most useful in improving upper GI

motility in a patient with ileus

a Sacral rocking

b Rib raising for 30 seconds

c OA decompression

d Paraspinal inhibition for 120 seconds X

e Sacral inhibition

10 You assess C4 translation and find translation to be easier to the left The findings

become symmetrical in flexion but not in extension Which of the following is the

correct position of C4 for a HVLA thrust

a Flexed rotated right sidebent left

b Extended rotated left sidebent left X

c Neutral rotated right sidebent left

d Flexed rotated right sidebent right

e Extended rotated right sidebent right

11 You examine L3 and find the right transverse process to be deeper than the left The

findings are unchanged in flexion and become symmetrical in extension Which of

the following is the correct position of L3 for FPR treatment

a Flexed sidebent left rotated left

b Flexed sidebent right rotated right

c Extended sidebent left rotated left X

d Extended sidebent right rotated left

e Flexed sidebent left rotated right

12 Which of the following is true regarding autonomic innervation to the GU system

a Sympathetic stimulation increases ureteral peristalsis

b Sympathetic stimulation relaxes the bladder wall X

c Parasympathetic stimulation activates the bladder sphincter

d Parasympathetic stimulation activates the trigone

e Sympathetic stimulation increases GFR

13 Which of the following is a result of parasympathetic stimulation

a Peripheral vasodilation

b Ejaculation

c Increased salivation X

d Uterine contraction

e Ureterospasm

14 The gall bladder receives sympathetic innervation from which of the following

levels

a OA

b T1-T5

c T5-T9 X

d T10-L2

e S2-S4

15 You are attempting to treat a right 7th rib exhalation dysfunction with counterstrain

Which of the following is correct regarding the treatment

a Shaft of right 7th rib positioned closer to shaft of right 8th rib X

b Position should decrease tenderness by 50

c Position is held for 30 seconds

d Angle of right 7th rib tractioned inferiorly

e Positioning involves left sidebending and extension

16 Which of the following is correctly matched

a Muscle energy ndash indirect treatment

b HVLA ndash articulation positioned in free motion

c Counterstrain ndash starts with a pain scale X

d FPR ndash active direct treatment

e Muscle energy ndash useful for painful muscle groups

17 Which of the following would be most likely to inhibit uterine contractions in a

patient in premature labor

a Sacral rocking X

b OA decompression

c Paraspinal inhibition gt90 seconds at T5-T9

d Rib raising lt60 seconds at T10-L2

e Sacral inhibition

18 You assess the innominates and find the following

Positive standing flexion test on the left

Right ASIS inferior compared to left

Right PSIS inferior compared to left

ASIS to midline distance equal bilaterally

Which of the following correctly describes a treatment for this dysfunction

a Muscle energy ndash left hip flexed patient extends against resistance

b HVLA thrust ndash thrust directed inferiorly on left innominate X

c FPR ndash right innominate pushed inferiorly for 3-4 seconds

d Muscle energy ndash right hip pushed superiorly patient pushes inferiorly

e Counterstrain ndash left innominate held inferiorly for 90 seconds

19 The pancreas receives parasympathetic innervation from which of the following

levels

a OA X

b T1-T5

c T5-T9

d T10-L2

e S2-S4

20 Which of the following would most likely directly decrease gastric secretions

a Sacral rocking

b OA decompression

c Paraspinal inhibition gt90 seconds at T10-L2

d Rib raising lt60 seconds at T5-T9 X

e Sacral inhibition

21 Which of the following statements is true regarding osteopathic treatment techniques

a Counterstrain is contraindicated for painful muscle groups

b Osteoporosis is a contraindication for FPR

c Muscle energy should not be used on a person in the CCU X

d HVLA is useful in patients with active Rheumatoid Arthritis

e The restrictive barrier is engaged when using indirect techniques

22 You examine T7 and find the left transverse process to be deeper than the right The

findings become symmetrical in flexion and are unchanged in extension Which of

the following is the correct starting position for a muscle energy treatment

a T7 flexed sidebent left rotated left

b T7 flexed sidebent right rotated right

c T7 extended sidebent left rotated left X

d T7 extended sidebent right rotated left

e T7 flexed sidebent left rotated right

23 The sigmoid colon receives parasympathetic innervation from which of the following

levels

a OA

b T1-T5

c T5-T9

d T10-L2

e S2-S4 X

24 You assess OA translation and find translation to be easier to the right The findings

persist in both flexion and extension Which of the following is the correct position

for FPR treatment

a Flexed rotated right sidebent left

b Neutral rotated left sidebent right

c Neutral rotated right sidebent left X

d Neutral rotated left sidebent left

e Extended rotated right sidebent right

25 Which of the following would most likely directly increase lacrimation

a Sacral rocking

b OA decompression X

c Paraspinal inhibition gt90 seconds at T5-T9

d Rib raising lt60 seconds at T1-T4 X

e Sacral inhibition

MODULE 3 ndash Joanne Smith

1 Top 5 ddx peptic ulcer GERD costochondritis Pancreastitis MI angina pectoris2 E no contraindications3 A OA4 E strep throat

dysfunctions Performed Omitted

1 Examine the cervical region for somatic dysfunctionA OA ndash leftright translation1 neutral 10486351048635

2 flexion 104863510486353 extension 104863510486352 Examine the thoracic spine for dysfunction (T5-T9)A palpate transverse processes for rotation1 neutral 104863510486352 flexion 104863510486353 extension 10486351048635B palpate soft tissue for TART findings1 trapezius 104863510486352 rhomboids 104863510486353 levator scapulae 104863510486354 T5 ndash T9 Paraspinal muscles 10486351048635Osteopathic Treatment You must treat a minimum of 2 dysfunctions you found during your examination1 Assume OA NSLRR appropriate techniques to include HVLA muscle energy articulatory Still counterstrainindirect myofascial release facilitated positional release

10486351048635

2 Assume T7 FRLSL appropriate techniques to include HVLA muscle energy articulatory Still counterstrainindirect myofascial release facilitated positional release

10486351048635

3 Assume acute bilateral spasm of upper thoracic (T5-T9) paraspinal muscles appropriate techniques to include soft tissue counterstrainindirect facilitated positional release

10486351048635

4 Autonomic technique for altered vagal tone OA decompression 104863510486355 Autonomic technique for hypersympathetic tone in T5-T9 paraspinal muscles rib raising or paraspinal inhibition for gt 90 seconds

10486351048635

MODULE 4- Ernesto Rodriguez

PRETEST

1 Back pain from Colon cancer Lumbar strain Osteomyelitits Diverticulitis Back pain osteoarthritis Cauda equina syndrome

2 D3 E

Osteopathic Diagnosis assess where you would expect to find somatic dysfunctions

Performed Omitted

1 Examine the lumbar region (L1-L5) for somatic dysfunction

A palpate transverse processes for rotation

1 neutral _

2 flexion _

3 extension _

B palpate soft tissue for TART (Tenderness Asymmetry Restrictions Tissue texture change) findings

1 quadratus lumborum muscles _

2 Piriformis muscles _

3 Paraspinal muscles _

2 Examine the sacrum for somatic dysfunction (requires at least one motion test)

A depth at 4 corners of sacrum _

B ILA levelness _

C seated flexion test _

D motion at 4 corners of sacrum _

E motion in backward bending test _

F respiratory motion of sacrum assess at 4 corners _

3 Examine the pelvis for somatic dysfunction

A standing flexion test or AP compression test _

B ASIS heights _

C PSIS heights _

Osteopathic Treatment You must treat a minimum of 2 dysfunctions you found during your examination

1 Assume L5 FRLSL _

2 Assume left on left sacral torsion _

3 Assume left innominate anterior _

4 Assume spasmtender point in the left piriformis muscle appropriate techniques to include counterstrainindirect myofascial release

-

MODULE 5 ndash Connie Jones

Pretest

1 Bradycardia acute pulmonary edemaCHF pneumonia acute MI Acute pulmonary edema CHF2 A3 A4 A5 For the video this is what was done and not done

OA - nf NO E --gtNslrrno AA dx doneno cervical dx doneT1-T5 - nfe --gt t2 f sl rlparapsinals TART --gt L upper thoracic paraspinalno levator scapula no trap no rhomboids dx tx -

T2 HLVAOA decompr ndash for vagal tonerib raisin for sympth

MODULE 6 ndash Gretchen White

1 Atrial fibrillation hyperthyroidism and- thyrotoxicosis thyroid cancer hyperthryroid 2 a3 a4 a5

Osteopathic Diagnosis assess where you would expect to find somatic dysfunctions

Performed Omitted

1 Examine the cervical region for somatic dysfunction

A OA ndash leftright translation

1 neutral X

2 flexion X

6 3 extension X

7 B AA ndash rotation X

8 2 Examine the upper thoracic spine for dysfunction (T1-T5)

A palpate transverse processes for rotation

1 neutral X

2 flexion X

3 extension X

B palpate soft tissue for TART findings

1 trapezius X

2 rhomboids X

3 levator scapulae X

4 T1-T5 paraspinal muscles X

Osteopathic Treatment You must treat a minimum of 2 dysfunctions you found during your examination

1 Assume OA NSLRR appropriate techniques to include counterstrainindirect myofascial release facilitated positional release

X

2 Assume T2 FSLRL appropriate techniques to include counterstrainindirect myofascial release facilitated positional release

X

3 Assume acute spasm of left upper thoracic paraspinal muscles appropriate techniques to include counterstrainindirect myofascial release and facilitated positional release

X

4 Autonomic technique for altered vagal tone OA decompression X

5 Autonomic technique for hypersympathetic tone rib raising or paraspinal inhibition for gt 90 seconds

X

MODULE 7 ndash Emmitt Brown

1 AAA kidney stones groin hernia ileus kidneybladder CA testicular torsion kidneybladder infection appendicitis

2 C3 D

Osteopathic Diagnosis assess where you would expect to find somatic dysfunctions

Performed Omitted

1 Examine the cervical region for somatic dysfunctionA OA ndash leftright translation1 neutral X2 flexion X3 extension XB AA ndash rotation must flex at least 45deg X2 Examine the thoracic spine for dysfunction (T10-L2)A palpate transverse processes for rotation1 neutral X2 flexion X3 extension XB palpate soft tissue for TART findings4 T10 ndash L2 Paraspinal muscles X3 Examine the sacrum for somatic dysfunction (requires at least one motion test)A depth at 4 corners of sacrum XB ILA levelness XC seated flexion test XD motion at 4 corners of sacrum XE motion in backward bending test XF respiratory motion of sacrum assess at 4 corners X4 Examine the pelvis for somatic dysfunctionA standing flexion test or AP compression test XB ASIS heights XC PSIS heights XOsteopathic Treatment You must treat a minimum of 2 dysfunctions you found during your examination1 Assume OA NSLRR appropriate techniques to include HVLA muscle energy articulatory Still counterstrainindirect myofascial release facilitated positional release

X _

2 Assume T10 NRRSL appropriate techniques to include HVLA muscle energy articulatory Still counterstrainindirect myofascial release facilitated positional release

X

3 Assume acute bilateral spasm of thoracolumbar (T10-L2) paraspinal muscles RgtL appropriate techniques to include soft tissue counterstrainindirect facilitated positional release

X

4 Assume left on right sacral torsion appropriate techniques to include muscle energy counterstrainindirect myofascial release and facilitated positional release (No HVLA)

X

5 Autonomic technique for hypersympathetic tone in T10-L2 paraspinal muscles rib raising or paraspinal inhibition for gt 90 seconds

X

MODULE 8 ndash Peter Parker1 Kidney stone Psoas syndrome spondylolysis piriformis syndrome lumbar compression

fracture appendicitis pelvicabdominal CA2 C3 B

Osteopathic Diagnosis assess where you would expect to find somatic dysfunctions

Performed Omitted

1 Examine the thoracolumbar region (T10-L5) for somatic dysfunctionA palpate transverse processes for rotation1 neutral _2 flexion _3 extension _B palpate soft tissue for TART (Tenderness Asymmetry Restrictions Tissue texture change) findings1 Psoas muscles _2 Piriformis muscles _3 Paraspinal muscles _2 Examine the sacrum for somatic dysfunction (requires at least one motion test)A depth at 4 corners of sacrum _B ILA levelness _C seated flexion test _D motion at 4 corners of sacrum _E motion in backward bending test _F respiratory motion of sacrum assess at 4 corners _3 Examine the pelvis for somatic dysfunctionA standing flexion test or AP compression test _B ASIS heights _C PSIS heights _Osteopathic Treatment You must treat a minimum of 2 dysfunctions you found during your examination1 Assume L1 FRRSR appropriate techniques to include muscle energy counterstrain indirect myofascial release facilitated positional release (No HVLA)

_

2 Assume left on right sacral torsion appropriate techniques to include muscle energy counterstrainindirect myofascial release and facilitated positional release (No HVLA)

_

3 Assume spasmtender point in the right psoas muscle appropriate techniques to include counterstrainindirect myofascial release FPR ME

_

4 Assume spasmtender point in the left piriformis muscle appropriate techniques to include counterstrainindirect myofascial release FPR ME

_

MODULE 9 ndash Ingrid Bergman

1 Lumbar strain Dysmenorrhea PID lumbar strain ovarian cyst sacral somatic dysfunction2 D3 E

Osteopathic Diagnosis assess where you would expect to find somatic dysfunctions

Performed Omitted

1 Examine the lumbar region (L1-L5) for somatic dysfunctionA palpate transverse processes for rotation1 neutral _2 flexion _3 extension _B palpate soft tissue for TART (Tenderness Asymmetry Restrictions Tissue texture change) findings1 quadratus lumborum muscles _2 Piriformis muscles _3 Paraspinal muscles _2 Examine the sacrum for somatic dysfunction (requires at least one motion test)A depth at 4 corners of sacrum _B ILA levelness _C seated flexion test _D motion at 4 corners of sacrum _E motion in backward bending test _F respiratory motion of sacrum assess at 4 corners _3 Examine the pelvis for somatic dysfunctionA standing flexion test or AP compression test _B ASIS heights _C PSIS heights _Osteopathic Treatment You must treat a minimum of 2 dysfunctions you found during your examination1 Assume L3 FRLSL2 Assume left on right sacral torsion _3 Assume left superior innominate shear _

4 Autonomic technique for altered parasympathetic tone

Module 10 ndash Minnie Driver

Pretest

1 Pneumonia dissecting aortic aneurysm thoracic spine fracture MI pleurisy2 B3 E

Osteopathic Diagnosis assess where you would expect to find somatic dysfunctions

Performed Omitted

1 Examine the cervical region for somatic dysfunctionA OA ndash leftright translation1 neutral 104863510486352 flexion 104863510486353 extension 10486351048635B AA ndash rotation must flex at least 45deg 10486351048635C C2-C7 - translation1 neutral 104863510486352 flexion 104863510486353 extension 104863510486352 Examine the thoracic spine for dysfunction (T5-T9)A palpate transverse processes for rotation1 neutral 104863510486352 flexion 104863510486353 extension 10486351048635B palpate soft tissue for TART findings1 trapezius 104863510486352 rhomboids 104863510486353 levator scapulae 104863510486354 T5 ndash T9 Paraspinal muscles 104863510486353 Examine the ribs for somatic dysfunctionA Pump handle ribs (ribs 2-6) at or near sternal junction1 static 104863510486352 motion 10486351048635B Bucket handle ribs (ribs 6-10) between anterior and posterior axillary lines1 static 104863510486352 motion 10486351048635Osteopathic Treatment You must treat a minimum of 2 dysfunctions you found during your examination1 Assume T5 ESLRL 104863510486352 Assume acute bilateral spasm of upper thoracic (T4-T8) paraspinal 10486351048635

muscles3 Assume (R) Rib 2 pump handle inhalation dysfunction 104863510486354 Autonomic technique for altered vagal tone OA decompression 10486351048635

MODULE 11 ndash Jeff Weaver

1 Cervical radiculopathy thoracic outlet syndrome cubital tunnel syndrome carpal tunnel syndrome angina pectoris ulnar groove entrapment Guillian Barre

2 A3 B4 C

Osteopathic Diagnosis assess where you would expect to find somatic dysfunctions

Performed Omitted

1 Examine the cervical region for somatic dysfunctionA C2-C7 - translation1 neutral 104863510486352 flexion 104863510486353 extension 10486351048635B palpate soft tissue for TART (Tenderness Asymmetry Restrictions Tissue texture change) findings1 cervical paraspinal muscles 104863510486352 sternocleidomastoids 104863510486353 scalenes 104863510486352 Examine the thoracic spine for somatic dysfunctionA palpate transverse processes for rotation1 neutral 104863510486352 flexion 104863510486353 extension 10486351048635B palpate soft tissue for TART findings1 trapezius 104863510486352 pectoral musculature 104863510486353 T1-T4 paraspinal muscle 104863510486353 Examine the 1st rib for somatic dysfunction1 static 104863510486352 dynamic 10486351048635Osteopathic Treatment You must treat a minimum of 2 dysfunctions you found during your examination1 Assume hypertonic left pectoral muscle Appropriate techniques to include ME MFR counterstrain

10486351048635

2 Assume hypertonic left scalene muscle Appropriate techniques to include MFR counterstrain ME

10486351048635

3 Assume T4 NRLSR appropriate techniques to include HVLA muscle energy articulatory Still counterstrainindirect myofascial release facilitated positional release

10486351048635

4 Assume left sided thoracic paraspinal hypertonicity T1 ndash T4 appropriate techniques to include soft tissue deep pressure counterstrainindirect facilitated positional release

10486351048635

5 Assume left 1 st rib inhalation somatic dysfunction appropriate techniques to include HVLA ME Stillrsquos articulatory

10486351048635

MODULE 12 ndash Scarlett O-Hara

1 Osteoarthritis piriformis syndrome spinal stenosis lumbar radiculopathy DVT2 C3 D

Osteopathic Diagnosis assess where you would expect to find somatic dysfunctions

Performed Omitted

1 Examine the lumbar region (L1-L5) for somatic dysfunctionA palpate transverse processes for rotation1 neutral _2 flexion _3 extension _B palpate soft tissue for TART (Tenderness Asymmetry Restrictions Tissue texture change) findings1 Psoas muscles _2 Piriformis muscles _3 Paraspinal muscles _2 Examine the sacrum for somatic dysfunction (requires at least one motion test)A depth at 4 corners of sacrum _B ILA levelness _C seated flexion test _D motion at 4 corners of sacrum _E motion in backward bending test _F respiratory motion of sacrum assess at 4 corners _3 Examine the pelvis for somatic dysfunctionA standing flexion test or AP compression test _B ASIS heights _C PSIS heights _Osteopathic Treatment You must treat a minimum of 2 dysfunctions you found during your examination1 Assume left on right sacral torsion appropriate techniques to include muscle energy counterstrainindirect myofascial release and facilitated positional release (No HVLA)2 Assume left innominate anterior appropriate techniques to include muscle energy counterstrainindirect myofascial release and facilitated positional release (No HVLA)

_

3 Assume spasmtender point in the left piriformis muscle appropriate techniques to _

include counterstrainindirect myofascial release

MODULE 13 Clark Kent1 Asthma atypical pneumonia bronchitis upper respiratory infection pneumothorax2 B3 A

Osteopathic Diagnosis assess where you would expect to find somatic dysfunctions

Performed Omitted

1 Examine the cervical region for somatic dysfunctionA OA ndash leftright translation1 neutral _2 flexion _3 extension _B AA ndash rotation must flex at least 45deg _C C2-C7 - translation1 neutral 2 flexion _3 extension 2 Examine the thoracic spine for dysfunction (T1-T4)A palpate transverse processes for rotation1 neutral _2 flexion _3 extension _B palpate soft tissue for TART findings1 trapezius X2 rhomboids X3 levator scapulae X4 T1 ndash T4 Paraspinal muscles X3 Examine the ribs for somatic dysfunctionA Pump handle ribs (ribs 2-6) at or near sternal junction1 static X2 motion XB Bucket handle ribs (ribs 6-10) between anterior and posterior axillary lines1 static X2 motion XOsteopathic Treatment You must treat a minimum of 2 dysfunctions you found during your examination1 Assume OA NSLRR appropriate techniques to include HVLA muscle energy articulatory Still counterstrainindirect myofascial release facilitated positional release

_

2 Assume T2-4 NSLRR appropriate techniques to include HVLA muscle energy articulatory Still counterstrainindirect myofascial release facilitated positional release

_

3 Assume (L) ribs 2-4 exhalation pump handle appropriate techniques to include HVLA muscle energy articulatory Still counterstrainindirect myofascial release facilitated positional release

4 Autonomic technique for altered vagal tone OA decompression _5 Autonomic technique for hypersympathetic tone in T1-T5 paraspinal muscles rib raising or paraspinal inhibition for gt 90 seconds

X

MODULE 14 Bruce Wayne

1 Depression thyroid disorder migraine headache tension tension headache migraine2 C3 A

Osteopathic Diagnosis assess where you would expect to find somatic dysfunctions

Performed Omitted

1 Examine the cervical region for somatic dysfunctionA OA ndash leftright translation1 neutral 2 flexion 3 extension B AA ndash rotation must flex at least 45deg C C2-C7 - translation1 neutral 2 flexion 3 extension _2 Examine the thoracic spine for dysfunction (T1-T4)A palpate transverse processes for rotation1 neutral X2 flexion X3 extension XB palpate soft tissue for TART findings1 trapezius X2 T1 ndash T4 Paraspinal muscles XOsteopathic Treatment You must treat a minimum of 2 dysfunctions you found during your examination1 Assume OA NSRRL appropriate techniques to include HVLA muscle energy articulatory Still counterstrainindirect myofascial release facilitated positional release

2 Assume C4 FSLRL appropriate techniques to include HVLA muscle energy

articulatory Still counterstrainindirect myofascial release facilitated positional release2 Assume T2-4 NSLRR appropriate techniques to include HVLA muscle energy articulatory Still counterstrainindirect myofascial release facilitated positional release

_

4 Autonomic technique for altered vagal tone OA decompression _

  • Performed
  • Performed
  • Performed
  • Performed
  • Performed
  • Performed
Page 10: Performed · Web viewAAA, kidney stones, groin hernia, ileus, kidney/bladder CA, testicular torsion, kidney/bladder infection, appendicitis, C D Osteopathic Diagnosis: assess where

d Paraspinal inhibition for 120 seconds X

e Sacral inhibition

10 You assess C4 translation and find translation to be easier to the left The findings

become symmetrical in flexion but not in extension Which of the following is the

correct position of C4 for a HVLA thrust

a Flexed rotated right sidebent left

b Extended rotated left sidebent left X

c Neutral rotated right sidebent left

d Flexed rotated right sidebent right

e Extended rotated right sidebent right

11 You examine L3 and find the right transverse process to be deeper than the left The

findings are unchanged in flexion and become symmetrical in extension Which of

the following is the correct position of L3 for FPR treatment

a Flexed sidebent left rotated left

b Flexed sidebent right rotated right

c Extended sidebent left rotated left X

d Extended sidebent right rotated left

e Flexed sidebent left rotated right

12 Which of the following is true regarding autonomic innervation to the GU system

a Sympathetic stimulation increases ureteral peristalsis

b Sympathetic stimulation relaxes the bladder wall X

c Parasympathetic stimulation activates the bladder sphincter

d Parasympathetic stimulation activates the trigone

e Sympathetic stimulation increases GFR

13 Which of the following is a result of parasympathetic stimulation

a Peripheral vasodilation

b Ejaculation

c Increased salivation X

d Uterine contraction

e Ureterospasm

14 The gall bladder receives sympathetic innervation from which of the following

levels

a OA

b T1-T5

c T5-T9 X

d T10-L2

e S2-S4

15 You are attempting to treat a right 7th rib exhalation dysfunction with counterstrain

Which of the following is correct regarding the treatment

a Shaft of right 7th rib positioned closer to shaft of right 8th rib X

b Position should decrease tenderness by 50

c Position is held for 30 seconds

d Angle of right 7th rib tractioned inferiorly

e Positioning involves left sidebending and extension

16 Which of the following is correctly matched

a Muscle energy ndash indirect treatment

b HVLA ndash articulation positioned in free motion

c Counterstrain ndash starts with a pain scale X

d FPR ndash active direct treatment

e Muscle energy ndash useful for painful muscle groups

17 Which of the following would be most likely to inhibit uterine contractions in a

patient in premature labor

a Sacral rocking X

b OA decompression

c Paraspinal inhibition gt90 seconds at T5-T9

d Rib raising lt60 seconds at T10-L2

e Sacral inhibition

18 You assess the innominates and find the following

Positive standing flexion test on the left

Right ASIS inferior compared to left

Right PSIS inferior compared to left

ASIS to midline distance equal bilaterally

Which of the following correctly describes a treatment for this dysfunction

a Muscle energy ndash left hip flexed patient extends against resistance

b HVLA thrust ndash thrust directed inferiorly on left innominate X

c FPR ndash right innominate pushed inferiorly for 3-4 seconds

d Muscle energy ndash right hip pushed superiorly patient pushes inferiorly

e Counterstrain ndash left innominate held inferiorly for 90 seconds

19 The pancreas receives parasympathetic innervation from which of the following

levels

a OA X

b T1-T5

c T5-T9

d T10-L2

e S2-S4

20 Which of the following would most likely directly decrease gastric secretions

a Sacral rocking

b OA decompression

c Paraspinal inhibition gt90 seconds at T10-L2

d Rib raising lt60 seconds at T5-T9 X

e Sacral inhibition

21 Which of the following statements is true regarding osteopathic treatment techniques

a Counterstrain is contraindicated for painful muscle groups

b Osteoporosis is a contraindication for FPR

c Muscle energy should not be used on a person in the CCU X

d HVLA is useful in patients with active Rheumatoid Arthritis

e The restrictive barrier is engaged when using indirect techniques

22 You examine T7 and find the left transverse process to be deeper than the right The

findings become symmetrical in flexion and are unchanged in extension Which of

the following is the correct starting position for a muscle energy treatment

a T7 flexed sidebent left rotated left

b T7 flexed sidebent right rotated right

c T7 extended sidebent left rotated left X

d T7 extended sidebent right rotated left

e T7 flexed sidebent left rotated right

23 The sigmoid colon receives parasympathetic innervation from which of the following

levels

a OA

b T1-T5

c T5-T9

d T10-L2

e S2-S4 X

24 You assess OA translation and find translation to be easier to the right The findings

persist in both flexion and extension Which of the following is the correct position

for FPR treatment

a Flexed rotated right sidebent left

b Neutral rotated left sidebent right

c Neutral rotated right sidebent left X

d Neutral rotated left sidebent left

e Extended rotated right sidebent right

25 Which of the following would most likely directly increase lacrimation

a Sacral rocking

b OA decompression X

c Paraspinal inhibition gt90 seconds at T5-T9

d Rib raising lt60 seconds at T1-T4 X

e Sacral inhibition

MODULE 3 ndash Joanne Smith

1 Top 5 ddx peptic ulcer GERD costochondritis Pancreastitis MI angina pectoris2 E no contraindications3 A OA4 E strep throat

dysfunctions Performed Omitted

1 Examine the cervical region for somatic dysfunctionA OA ndash leftright translation1 neutral 10486351048635

2 flexion 104863510486353 extension 104863510486352 Examine the thoracic spine for dysfunction (T5-T9)A palpate transverse processes for rotation1 neutral 104863510486352 flexion 104863510486353 extension 10486351048635B palpate soft tissue for TART findings1 trapezius 104863510486352 rhomboids 104863510486353 levator scapulae 104863510486354 T5 ndash T9 Paraspinal muscles 10486351048635Osteopathic Treatment You must treat a minimum of 2 dysfunctions you found during your examination1 Assume OA NSLRR appropriate techniques to include HVLA muscle energy articulatory Still counterstrainindirect myofascial release facilitated positional release

10486351048635

2 Assume T7 FRLSL appropriate techniques to include HVLA muscle energy articulatory Still counterstrainindirect myofascial release facilitated positional release

10486351048635

3 Assume acute bilateral spasm of upper thoracic (T5-T9) paraspinal muscles appropriate techniques to include soft tissue counterstrainindirect facilitated positional release

10486351048635

4 Autonomic technique for altered vagal tone OA decompression 104863510486355 Autonomic technique for hypersympathetic tone in T5-T9 paraspinal muscles rib raising or paraspinal inhibition for gt 90 seconds

10486351048635

MODULE 4- Ernesto Rodriguez

PRETEST

1 Back pain from Colon cancer Lumbar strain Osteomyelitits Diverticulitis Back pain osteoarthritis Cauda equina syndrome

2 D3 E

Osteopathic Diagnosis assess where you would expect to find somatic dysfunctions

Performed Omitted

1 Examine the lumbar region (L1-L5) for somatic dysfunction

A palpate transverse processes for rotation

1 neutral _

2 flexion _

3 extension _

B palpate soft tissue for TART (Tenderness Asymmetry Restrictions Tissue texture change) findings

1 quadratus lumborum muscles _

2 Piriformis muscles _

3 Paraspinal muscles _

2 Examine the sacrum for somatic dysfunction (requires at least one motion test)

A depth at 4 corners of sacrum _

B ILA levelness _

C seated flexion test _

D motion at 4 corners of sacrum _

E motion in backward bending test _

F respiratory motion of sacrum assess at 4 corners _

3 Examine the pelvis for somatic dysfunction

A standing flexion test or AP compression test _

B ASIS heights _

C PSIS heights _

Osteopathic Treatment You must treat a minimum of 2 dysfunctions you found during your examination

1 Assume L5 FRLSL _

2 Assume left on left sacral torsion _

3 Assume left innominate anterior _

4 Assume spasmtender point in the left piriformis muscle appropriate techniques to include counterstrainindirect myofascial release

-

MODULE 5 ndash Connie Jones

Pretest

1 Bradycardia acute pulmonary edemaCHF pneumonia acute MI Acute pulmonary edema CHF2 A3 A4 A5 For the video this is what was done and not done

OA - nf NO E --gtNslrrno AA dx doneno cervical dx doneT1-T5 - nfe --gt t2 f sl rlparapsinals TART --gt L upper thoracic paraspinalno levator scapula no trap no rhomboids dx tx -

T2 HLVAOA decompr ndash for vagal tonerib raisin for sympth

MODULE 6 ndash Gretchen White

1 Atrial fibrillation hyperthyroidism and- thyrotoxicosis thyroid cancer hyperthryroid 2 a3 a4 a5

Osteopathic Diagnosis assess where you would expect to find somatic dysfunctions

Performed Omitted

1 Examine the cervical region for somatic dysfunction

A OA ndash leftright translation

1 neutral X

2 flexion X

6 3 extension X

7 B AA ndash rotation X

8 2 Examine the upper thoracic spine for dysfunction (T1-T5)

A palpate transverse processes for rotation

1 neutral X

2 flexion X

3 extension X

B palpate soft tissue for TART findings

1 trapezius X

2 rhomboids X

3 levator scapulae X

4 T1-T5 paraspinal muscles X

Osteopathic Treatment You must treat a minimum of 2 dysfunctions you found during your examination

1 Assume OA NSLRR appropriate techniques to include counterstrainindirect myofascial release facilitated positional release

X

2 Assume T2 FSLRL appropriate techniques to include counterstrainindirect myofascial release facilitated positional release

X

3 Assume acute spasm of left upper thoracic paraspinal muscles appropriate techniques to include counterstrainindirect myofascial release and facilitated positional release

X

4 Autonomic technique for altered vagal tone OA decompression X

5 Autonomic technique for hypersympathetic tone rib raising or paraspinal inhibition for gt 90 seconds

X

MODULE 7 ndash Emmitt Brown

1 AAA kidney stones groin hernia ileus kidneybladder CA testicular torsion kidneybladder infection appendicitis

2 C3 D

Osteopathic Diagnosis assess where you would expect to find somatic dysfunctions

Performed Omitted

1 Examine the cervical region for somatic dysfunctionA OA ndash leftright translation1 neutral X2 flexion X3 extension XB AA ndash rotation must flex at least 45deg X2 Examine the thoracic spine for dysfunction (T10-L2)A palpate transverse processes for rotation1 neutral X2 flexion X3 extension XB palpate soft tissue for TART findings4 T10 ndash L2 Paraspinal muscles X3 Examine the sacrum for somatic dysfunction (requires at least one motion test)A depth at 4 corners of sacrum XB ILA levelness XC seated flexion test XD motion at 4 corners of sacrum XE motion in backward bending test XF respiratory motion of sacrum assess at 4 corners X4 Examine the pelvis for somatic dysfunctionA standing flexion test or AP compression test XB ASIS heights XC PSIS heights XOsteopathic Treatment You must treat a minimum of 2 dysfunctions you found during your examination1 Assume OA NSLRR appropriate techniques to include HVLA muscle energy articulatory Still counterstrainindirect myofascial release facilitated positional release

X _

2 Assume T10 NRRSL appropriate techniques to include HVLA muscle energy articulatory Still counterstrainindirect myofascial release facilitated positional release

X

3 Assume acute bilateral spasm of thoracolumbar (T10-L2) paraspinal muscles RgtL appropriate techniques to include soft tissue counterstrainindirect facilitated positional release

X

4 Assume left on right sacral torsion appropriate techniques to include muscle energy counterstrainindirect myofascial release and facilitated positional release (No HVLA)

X

5 Autonomic technique for hypersympathetic tone in T10-L2 paraspinal muscles rib raising or paraspinal inhibition for gt 90 seconds

X

MODULE 8 ndash Peter Parker1 Kidney stone Psoas syndrome spondylolysis piriformis syndrome lumbar compression

fracture appendicitis pelvicabdominal CA2 C3 B

Osteopathic Diagnosis assess where you would expect to find somatic dysfunctions

Performed Omitted

1 Examine the thoracolumbar region (T10-L5) for somatic dysfunctionA palpate transverse processes for rotation1 neutral _2 flexion _3 extension _B palpate soft tissue for TART (Tenderness Asymmetry Restrictions Tissue texture change) findings1 Psoas muscles _2 Piriformis muscles _3 Paraspinal muscles _2 Examine the sacrum for somatic dysfunction (requires at least one motion test)A depth at 4 corners of sacrum _B ILA levelness _C seated flexion test _D motion at 4 corners of sacrum _E motion in backward bending test _F respiratory motion of sacrum assess at 4 corners _3 Examine the pelvis for somatic dysfunctionA standing flexion test or AP compression test _B ASIS heights _C PSIS heights _Osteopathic Treatment You must treat a minimum of 2 dysfunctions you found during your examination1 Assume L1 FRRSR appropriate techniques to include muscle energy counterstrain indirect myofascial release facilitated positional release (No HVLA)

_

2 Assume left on right sacral torsion appropriate techniques to include muscle energy counterstrainindirect myofascial release and facilitated positional release (No HVLA)

_

3 Assume spasmtender point in the right psoas muscle appropriate techniques to include counterstrainindirect myofascial release FPR ME

_

4 Assume spasmtender point in the left piriformis muscle appropriate techniques to include counterstrainindirect myofascial release FPR ME

_

MODULE 9 ndash Ingrid Bergman

1 Lumbar strain Dysmenorrhea PID lumbar strain ovarian cyst sacral somatic dysfunction2 D3 E

Osteopathic Diagnosis assess where you would expect to find somatic dysfunctions

Performed Omitted

1 Examine the lumbar region (L1-L5) for somatic dysfunctionA palpate transverse processes for rotation1 neutral _2 flexion _3 extension _B palpate soft tissue for TART (Tenderness Asymmetry Restrictions Tissue texture change) findings1 quadratus lumborum muscles _2 Piriformis muscles _3 Paraspinal muscles _2 Examine the sacrum for somatic dysfunction (requires at least one motion test)A depth at 4 corners of sacrum _B ILA levelness _C seated flexion test _D motion at 4 corners of sacrum _E motion in backward bending test _F respiratory motion of sacrum assess at 4 corners _3 Examine the pelvis for somatic dysfunctionA standing flexion test or AP compression test _B ASIS heights _C PSIS heights _Osteopathic Treatment You must treat a minimum of 2 dysfunctions you found during your examination1 Assume L3 FRLSL2 Assume left on right sacral torsion _3 Assume left superior innominate shear _

4 Autonomic technique for altered parasympathetic tone

Module 10 ndash Minnie Driver

Pretest

1 Pneumonia dissecting aortic aneurysm thoracic spine fracture MI pleurisy2 B3 E

Osteopathic Diagnosis assess where you would expect to find somatic dysfunctions

Performed Omitted

1 Examine the cervical region for somatic dysfunctionA OA ndash leftright translation1 neutral 104863510486352 flexion 104863510486353 extension 10486351048635B AA ndash rotation must flex at least 45deg 10486351048635C C2-C7 - translation1 neutral 104863510486352 flexion 104863510486353 extension 104863510486352 Examine the thoracic spine for dysfunction (T5-T9)A palpate transverse processes for rotation1 neutral 104863510486352 flexion 104863510486353 extension 10486351048635B palpate soft tissue for TART findings1 trapezius 104863510486352 rhomboids 104863510486353 levator scapulae 104863510486354 T5 ndash T9 Paraspinal muscles 104863510486353 Examine the ribs for somatic dysfunctionA Pump handle ribs (ribs 2-6) at or near sternal junction1 static 104863510486352 motion 10486351048635B Bucket handle ribs (ribs 6-10) between anterior and posterior axillary lines1 static 104863510486352 motion 10486351048635Osteopathic Treatment You must treat a minimum of 2 dysfunctions you found during your examination1 Assume T5 ESLRL 104863510486352 Assume acute bilateral spasm of upper thoracic (T4-T8) paraspinal 10486351048635

muscles3 Assume (R) Rib 2 pump handle inhalation dysfunction 104863510486354 Autonomic technique for altered vagal tone OA decompression 10486351048635

MODULE 11 ndash Jeff Weaver

1 Cervical radiculopathy thoracic outlet syndrome cubital tunnel syndrome carpal tunnel syndrome angina pectoris ulnar groove entrapment Guillian Barre

2 A3 B4 C

Osteopathic Diagnosis assess where you would expect to find somatic dysfunctions

Performed Omitted

1 Examine the cervical region for somatic dysfunctionA C2-C7 - translation1 neutral 104863510486352 flexion 104863510486353 extension 10486351048635B palpate soft tissue for TART (Tenderness Asymmetry Restrictions Tissue texture change) findings1 cervical paraspinal muscles 104863510486352 sternocleidomastoids 104863510486353 scalenes 104863510486352 Examine the thoracic spine for somatic dysfunctionA palpate transverse processes for rotation1 neutral 104863510486352 flexion 104863510486353 extension 10486351048635B palpate soft tissue for TART findings1 trapezius 104863510486352 pectoral musculature 104863510486353 T1-T4 paraspinal muscle 104863510486353 Examine the 1st rib for somatic dysfunction1 static 104863510486352 dynamic 10486351048635Osteopathic Treatment You must treat a minimum of 2 dysfunctions you found during your examination1 Assume hypertonic left pectoral muscle Appropriate techniques to include ME MFR counterstrain

10486351048635

2 Assume hypertonic left scalene muscle Appropriate techniques to include MFR counterstrain ME

10486351048635

3 Assume T4 NRLSR appropriate techniques to include HVLA muscle energy articulatory Still counterstrainindirect myofascial release facilitated positional release

10486351048635

4 Assume left sided thoracic paraspinal hypertonicity T1 ndash T4 appropriate techniques to include soft tissue deep pressure counterstrainindirect facilitated positional release

10486351048635

5 Assume left 1 st rib inhalation somatic dysfunction appropriate techniques to include HVLA ME Stillrsquos articulatory

10486351048635

MODULE 12 ndash Scarlett O-Hara

1 Osteoarthritis piriformis syndrome spinal stenosis lumbar radiculopathy DVT2 C3 D

Osteopathic Diagnosis assess where you would expect to find somatic dysfunctions

Performed Omitted

1 Examine the lumbar region (L1-L5) for somatic dysfunctionA palpate transverse processes for rotation1 neutral _2 flexion _3 extension _B palpate soft tissue for TART (Tenderness Asymmetry Restrictions Tissue texture change) findings1 Psoas muscles _2 Piriformis muscles _3 Paraspinal muscles _2 Examine the sacrum for somatic dysfunction (requires at least one motion test)A depth at 4 corners of sacrum _B ILA levelness _C seated flexion test _D motion at 4 corners of sacrum _E motion in backward bending test _F respiratory motion of sacrum assess at 4 corners _3 Examine the pelvis for somatic dysfunctionA standing flexion test or AP compression test _B ASIS heights _C PSIS heights _Osteopathic Treatment You must treat a minimum of 2 dysfunctions you found during your examination1 Assume left on right sacral torsion appropriate techniques to include muscle energy counterstrainindirect myofascial release and facilitated positional release (No HVLA)2 Assume left innominate anterior appropriate techniques to include muscle energy counterstrainindirect myofascial release and facilitated positional release (No HVLA)

_

3 Assume spasmtender point in the left piriformis muscle appropriate techniques to _

include counterstrainindirect myofascial release

MODULE 13 Clark Kent1 Asthma atypical pneumonia bronchitis upper respiratory infection pneumothorax2 B3 A

Osteopathic Diagnosis assess where you would expect to find somatic dysfunctions

Performed Omitted

1 Examine the cervical region for somatic dysfunctionA OA ndash leftright translation1 neutral _2 flexion _3 extension _B AA ndash rotation must flex at least 45deg _C C2-C7 - translation1 neutral 2 flexion _3 extension 2 Examine the thoracic spine for dysfunction (T1-T4)A palpate transverse processes for rotation1 neutral _2 flexion _3 extension _B palpate soft tissue for TART findings1 trapezius X2 rhomboids X3 levator scapulae X4 T1 ndash T4 Paraspinal muscles X3 Examine the ribs for somatic dysfunctionA Pump handle ribs (ribs 2-6) at or near sternal junction1 static X2 motion XB Bucket handle ribs (ribs 6-10) between anterior and posterior axillary lines1 static X2 motion XOsteopathic Treatment You must treat a minimum of 2 dysfunctions you found during your examination1 Assume OA NSLRR appropriate techniques to include HVLA muscle energy articulatory Still counterstrainindirect myofascial release facilitated positional release

_

2 Assume T2-4 NSLRR appropriate techniques to include HVLA muscle energy articulatory Still counterstrainindirect myofascial release facilitated positional release

_

3 Assume (L) ribs 2-4 exhalation pump handle appropriate techniques to include HVLA muscle energy articulatory Still counterstrainindirect myofascial release facilitated positional release

4 Autonomic technique for altered vagal tone OA decompression _5 Autonomic technique for hypersympathetic tone in T1-T5 paraspinal muscles rib raising or paraspinal inhibition for gt 90 seconds

X

MODULE 14 Bruce Wayne

1 Depression thyroid disorder migraine headache tension tension headache migraine2 C3 A

Osteopathic Diagnosis assess where you would expect to find somatic dysfunctions

Performed Omitted

1 Examine the cervical region for somatic dysfunctionA OA ndash leftright translation1 neutral 2 flexion 3 extension B AA ndash rotation must flex at least 45deg C C2-C7 - translation1 neutral 2 flexion 3 extension _2 Examine the thoracic spine for dysfunction (T1-T4)A palpate transverse processes for rotation1 neutral X2 flexion X3 extension XB palpate soft tissue for TART findings1 trapezius X2 T1 ndash T4 Paraspinal muscles XOsteopathic Treatment You must treat a minimum of 2 dysfunctions you found during your examination1 Assume OA NSRRL appropriate techniques to include HVLA muscle energy articulatory Still counterstrainindirect myofascial release facilitated positional release

2 Assume C4 FSLRL appropriate techniques to include HVLA muscle energy

articulatory Still counterstrainindirect myofascial release facilitated positional release2 Assume T2-4 NSLRR appropriate techniques to include HVLA muscle energy articulatory Still counterstrainindirect myofascial release facilitated positional release

_

4 Autonomic technique for altered vagal tone OA decompression _

  • Performed
  • Performed
  • Performed
  • Performed
  • Performed
  • Performed
Page 11: Performed · Web viewAAA, kidney stones, groin hernia, ileus, kidney/bladder CA, testicular torsion, kidney/bladder infection, appendicitis, C D Osteopathic Diagnosis: assess where

a Peripheral vasodilation

b Ejaculation

c Increased salivation X

d Uterine contraction

e Ureterospasm

14 The gall bladder receives sympathetic innervation from which of the following

levels

a OA

b T1-T5

c T5-T9 X

d T10-L2

e S2-S4

15 You are attempting to treat a right 7th rib exhalation dysfunction with counterstrain

Which of the following is correct regarding the treatment

a Shaft of right 7th rib positioned closer to shaft of right 8th rib X

b Position should decrease tenderness by 50

c Position is held for 30 seconds

d Angle of right 7th rib tractioned inferiorly

e Positioning involves left sidebending and extension

16 Which of the following is correctly matched

a Muscle energy ndash indirect treatment

b HVLA ndash articulation positioned in free motion

c Counterstrain ndash starts with a pain scale X

d FPR ndash active direct treatment

e Muscle energy ndash useful for painful muscle groups

17 Which of the following would be most likely to inhibit uterine contractions in a

patient in premature labor

a Sacral rocking X

b OA decompression

c Paraspinal inhibition gt90 seconds at T5-T9

d Rib raising lt60 seconds at T10-L2

e Sacral inhibition

18 You assess the innominates and find the following

Positive standing flexion test on the left

Right ASIS inferior compared to left

Right PSIS inferior compared to left

ASIS to midline distance equal bilaterally

Which of the following correctly describes a treatment for this dysfunction

a Muscle energy ndash left hip flexed patient extends against resistance

b HVLA thrust ndash thrust directed inferiorly on left innominate X

c FPR ndash right innominate pushed inferiorly for 3-4 seconds

d Muscle energy ndash right hip pushed superiorly patient pushes inferiorly

e Counterstrain ndash left innominate held inferiorly for 90 seconds

19 The pancreas receives parasympathetic innervation from which of the following

levels

a OA X

b T1-T5

c T5-T9

d T10-L2

e S2-S4

20 Which of the following would most likely directly decrease gastric secretions

a Sacral rocking

b OA decompression

c Paraspinal inhibition gt90 seconds at T10-L2

d Rib raising lt60 seconds at T5-T9 X

e Sacral inhibition

21 Which of the following statements is true regarding osteopathic treatment techniques

a Counterstrain is contraindicated for painful muscle groups

b Osteoporosis is a contraindication for FPR

c Muscle energy should not be used on a person in the CCU X

d HVLA is useful in patients with active Rheumatoid Arthritis

e The restrictive barrier is engaged when using indirect techniques

22 You examine T7 and find the left transverse process to be deeper than the right The

findings become symmetrical in flexion and are unchanged in extension Which of

the following is the correct starting position for a muscle energy treatment

a T7 flexed sidebent left rotated left

b T7 flexed sidebent right rotated right

c T7 extended sidebent left rotated left X

d T7 extended sidebent right rotated left

e T7 flexed sidebent left rotated right

23 The sigmoid colon receives parasympathetic innervation from which of the following

levels

a OA

b T1-T5

c T5-T9

d T10-L2

e S2-S4 X

24 You assess OA translation and find translation to be easier to the right The findings

persist in both flexion and extension Which of the following is the correct position

for FPR treatment

a Flexed rotated right sidebent left

b Neutral rotated left sidebent right

c Neutral rotated right sidebent left X

d Neutral rotated left sidebent left

e Extended rotated right sidebent right

25 Which of the following would most likely directly increase lacrimation

a Sacral rocking

b OA decompression X

c Paraspinal inhibition gt90 seconds at T5-T9

d Rib raising lt60 seconds at T1-T4 X

e Sacral inhibition

MODULE 3 ndash Joanne Smith

1 Top 5 ddx peptic ulcer GERD costochondritis Pancreastitis MI angina pectoris2 E no contraindications3 A OA4 E strep throat

dysfunctions Performed Omitted

1 Examine the cervical region for somatic dysfunctionA OA ndash leftright translation1 neutral 10486351048635

2 flexion 104863510486353 extension 104863510486352 Examine the thoracic spine for dysfunction (T5-T9)A palpate transverse processes for rotation1 neutral 104863510486352 flexion 104863510486353 extension 10486351048635B palpate soft tissue for TART findings1 trapezius 104863510486352 rhomboids 104863510486353 levator scapulae 104863510486354 T5 ndash T9 Paraspinal muscles 10486351048635Osteopathic Treatment You must treat a minimum of 2 dysfunctions you found during your examination1 Assume OA NSLRR appropriate techniques to include HVLA muscle energy articulatory Still counterstrainindirect myofascial release facilitated positional release

10486351048635

2 Assume T7 FRLSL appropriate techniques to include HVLA muscle energy articulatory Still counterstrainindirect myofascial release facilitated positional release

10486351048635

3 Assume acute bilateral spasm of upper thoracic (T5-T9) paraspinal muscles appropriate techniques to include soft tissue counterstrainindirect facilitated positional release

10486351048635

4 Autonomic technique for altered vagal tone OA decompression 104863510486355 Autonomic technique for hypersympathetic tone in T5-T9 paraspinal muscles rib raising or paraspinal inhibition for gt 90 seconds

10486351048635

MODULE 4- Ernesto Rodriguez

PRETEST

1 Back pain from Colon cancer Lumbar strain Osteomyelitits Diverticulitis Back pain osteoarthritis Cauda equina syndrome

2 D3 E

Osteopathic Diagnosis assess where you would expect to find somatic dysfunctions

Performed Omitted

1 Examine the lumbar region (L1-L5) for somatic dysfunction

A palpate transverse processes for rotation

1 neutral _

2 flexion _

3 extension _

B palpate soft tissue for TART (Tenderness Asymmetry Restrictions Tissue texture change) findings

1 quadratus lumborum muscles _

2 Piriformis muscles _

3 Paraspinal muscles _

2 Examine the sacrum for somatic dysfunction (requires at least one motion test)

A depth at 4 corners of sacrum _

B ILA levelness _

C seated flexion test _

D motion at 4 corners of sacrum _

E motion in backward bending test _

F respiratory motion of sacrum assess at 4 corners _

3 Examine the pelvis for somatic dysfunction

A standing flexion test or AP compression test _

B ASIS heights _

C PSIS heights _

Osteopathic Treatment You must treat a minimum of 2 dysfunctions you found during your examination

1 Assume L5 FRLSL _

2 Assume left on left sacral torsion _

3 Assume left innominate anterior _

4 Assume spasmtender point in the left piriformis muscle appropriate techniques to include counterstrainindirect myofascial release

-

MODULE 5 ndash Connie Jones

Pretest

1 Bradycardia acute pulmonary edemaCHF pneumonia acute MI Acute pulmonary edema CHF2 A3 A4 A5 For the video this is what was done and not done

OA - nf NO E --gtNslrrno AA dx doneno cervical dx doneT1-T5 - nfe --gt t2 f sl rlparapsinals TART --gt L upper thoracic paraspinalno levator scapula no trap no rhomboids dx tx -

T2 HLVAOA decompr ndash for vagal tonerib raisin for sympth

MODULE 6 ndash Gretchen White

1 Atrial fibrillation hyperthyroidism and- thyrotoxicosis thyroid cancer hyperthryroid 2 a3 a4 a5

Osteopathic Diagnosis assess where you would expect to find somatic dysfunctions

Performed Omitted

1 Examine the cervical region for somatic dysfunction

A OA ndash leftright translation

1 neutral X

2 flexion X

6 3 extension X

7 B AA ndash rotation X

8 2 Examine the upper thoracic spine for dysfunction (T1-T5)

A palpate transverse processes for rotation

1 neutral X

2 flexion X

3 extension X

B palpate soft tissue for TART findings

1 trapezius X

2 rhomboids X

3 levator scapulae X

4 T1-T5 paraspinal muscles X

Osteopathic Treatment You must treat a minimum of 2 dysfunctions you found during your examination

1 Assume OA NSLRR appropriate techniques to include counterstrainindirect myofascial release facilitated positional release

X

2 Assume T2 FSLRL appropriate techniques to include counterstrainindirect myofascial release facilitated positional release

X

3 Assume acute spasm of left upper thoracic paraspinal muscles appropriate techniques to include counterstrainindirect myofascial release and facilitated positional release

X

4 Autonomic technique for altered vagal tone OA decompression X

5 Autonomic technique for hypersympathetic tone rib raising or paraspinal inhibition for gt 90 seconds

X

MODULE 7 ndash Emmitt Brown

1 AAA kidney stones groin hernia ileus kidneybladder CA testicular torsion kidneybladder infection appendicitis

2 C3 D

Osteopathic Diagnosis assess where you would expect to find somatic dysfunctions

Performed Omitted

1 Examine the cervical region for somatic dysfunctionA OA ndash leftright translation1 neutral X2 flexion X3 extension XB AA ndash rotation must flex at least 45deg X2 Examine the thoracic spine for dysfunction (T10-L2)A palpate transverse processes for rotation1 neutral X2 flexion X3 extension XB palpate soft tissue for TART findings4 T10 ndash L2 Paraspinal muscles X3 Examine the sacrum for somatic dysfunction (requires at least one motion test)A depth at 4 corners of sacrum XB ILA levelness XC seated flexion test XD motion at 4 corners of sacrum XE motion in backward bending test XF respiratory motion of sacrum assess at 4 corners X4 Examine the pelvis for somatic dysfunctionA standing flexion test or AP compression test XB ASIS heights XC PSIS heights XOsteopathic Treatment You must treat a minimum of 2 dysfunctions you found during your examination1 Assume OA NSLRR appropriate techniques to include HVLA muscle energy articulatory Still counterstrainindirect myofascial release facilitated positional release

X _

2 Assume T10 NRRSL appropriate techniques to include HVLA muscle energy articulatory Still counterstrainindirect myofascial release facilitated positional release

X

3 Assume acute bilateral spasm of thoracolumbar (T10-L2) paraspinal muscles RgtL appropriate techniques to include soft tissue counterstrainindirect facilitated positional release

X

4 Assume left on right sacral torsion appropriate techniques to include muscle energy counterstrainindirect myofascial release and facilitated positional release (No HVLA)

X

5 Autonomic technique for hypersympathetic tone in T10-L2 paraspinal muscles rib raising or paraspinal inhibition for gt 90 seconds

X

MODULE 8 ndash Peter Parker1 Kidney stone Psoas syndrome spondylolysis piriformis syndrome lumbar compression

fracture appendicitis pelvicabdominal CA2 C3 B

Osteopathic Diagnosis assess where you would expect to find somatic dysfunctions

Performed Omitted

1 Examine the thoracolumbar region (T10-L5) for somatic dysfunctionA palpate transverse processes for rotation1 neutral _2 flexion _3 extension _B palpate soft tissue for TART (Tenderness Asymmetry Restrictions Tissue texture change) findings1 Psoas muscles _2 Piriformis muscles _3 Paraspinal muscles _2 Examine the sacrum for somatic dysfunction (requires at least one motion test)A depth at 4 corners of sacrum _B ILA levelness _C seated flexion test _D motion at 4 corners of sacrum _E motion in backward bending test _F respiratory motion of sacrum assess at 4 corners _3 Examine the pelvis for somatic dysfunctionA standing flexion test or AP compression test _B ASIS heights _C PSIS heights _Osteopathic Treatment You must treat a minimum of 2 dysfunctions you found during your examination1 Assume L1 FRRSR appropriate techniques to include muscle energy counterstrain indirect myofascial release facilitated positional release (No HVLA)

_

2 Assume left on right sacral torsion appropriate techniques to include muscle energy counterstrainindirect myofascial release and facilitated positional release (No HVLA)

_

3 Assume spasmtender point in the right psoas muscle appropriate techniques to include counterstrainindirect myofascial release FPR ME

_

4 Assume spasmtender point in the left piriformis muscle appropriate techniques to include counterstrainindirect myofascial release FPR ME

_

MODULE 9 ndash Ingrid Bergman

1 Lumbar strain Dysmenorrhea PID lumbar strain ovarian cyst sacral somatic dysfunction2 D3 E

Osteopathic Diagnosis assess where you would expect to find somatic dysfunctions

Performed Omitted

1 Examine the lumbar region (L1-L5) for somatic dysfunctionA palpate transverse processes for rotation1 neutral _2 flexion _3 extension _B palpate soft tissue for TART (Tenderness Asymmetry Restrictions Tissue texture change) findings1 quadratus lumborum muscles _2 Piriformis muscles _3 Paraspinal muscles _2 Examine the sacrum for somatic dysfunction (requires at least one motion test)A depth at 4 corners of sacrum _B ILA levelness _C seated flexion test _D motion at 4 corners of sacrum _E motion in backward bending test _F respiratory motion of sacrum assess at 4 corners _3 Examine the pelvis for somatic dysfunctionA standing flexion test or AP compression test _B ASIS heights _C PSIS heights _Osteopathic Treatment You must treat a minimum of 2 dysfunctions you found during your examination1 Assume L3 FRLSL2 Assume left on right sacral torsion _3 Assume left superior innominate shear _

4 Autonomic technique for altered parasympathetic tone

Module 10 ndash Minnie Driver

Pretest

1 Pneumonia dissecting aortic aneurysm thoracic spine fracture MI pleurisy2 B3 E

Osteopathic Diagnosis assess where you would expect to find somatic dysfunctions

Performed Omitted

1 Examine the cervical region for somatic dysfunctionA OA ndash leftright translation1 neutral 104863510486352 flexion 104863510486353 extension 10486351048635B AA ndash rotation must flex at least 45deg 10486351048635C C2-C7 - translation1 neutral 104863510486352 flexion 104863510486353 extension 104863510486352 Examine the thoracic spine for dysfunction (T5-T9)A palpate transverse processes for rotation1 neutral 104863510486352 flexion 104863510486353 extension 10486351048635B palpate soft tissue for TART findings1 trapezius 104863510486352 rhomboids 104863510486353 levator scapulae 104863510486354 T5 ndash T9 Paraspinal muscles 104863510486353 Examine the ribs for somatic dysfunctionA Pump handle ribs (ribs 2-6) at or near sternal junction1 static 104863510486352 motion 10486351048635B Bucket handle ribs (ribs 6-10) between anterior and posterior axillary lines1 static 104863510486352 motion 10486351048635Osteopathic Treatment You must treat a minimum of 2 dysfunctions you found during your examination1 Assume T5 ESLRL 104863510486352 Assume acute bilateral spasm of upper thoracic (T4-T8) paraspinal 10486351048635

muscles3 Assume (R) Rib 2 pump handle inhalation dysfunction 104863510486354 Autonomic technique for altered vagal tone OA decompression 10486351048635

MODULE 11 ndash Jeff Weaver

1 Cervical radiculopathy thoracic outlet syndrome cubital tunnel syndrome carpal tunnel syndrome angina pectoris ulnar groove entrapment Guillian Barre

2 A3 B4 C

Osteopathic Diagnosis assess where you would expect to find somatic dysfunctions

Performed Omitted

1 Examine the cervical region for somatic dysfunctionA C2-C7 - translation1 neutral 104863510486352 flexion 104863510486353 extension 10486351048635B palpate soft tissue for TART (Tenderness Asymmetry Restrictions Tissue texture change) findings1 cervical paraspinal muscles 104863510486352 sternocleidomastoids 104863510486353 scalenes 104863510486352 Examine the thoracic spine for somatic dysfunctionA palpate transverse processes for rotation1 neutral 104863510486352 flexion 104863510486353 extension 10486351048635B palpate soft tissue for TART findings1 trapezius 104863510486352 pectoral musculature 104863510486353 T1-T4 paraspinal muscle 104863510486353 Examine the 1st rib for somatic dysfunction1 static 104863510486352 dynamic 10486351048635Osteopathic Treatment You must treat a minimum of 2 dysfunctions you found during your examination1 Assume hypertonic left pectoral muscle Appropriate techniques to include ME MFR counterstrain

10486351048635

2 Assume hypertonic left scalene muscle Appropriate techniques to include MFR counterstrain ME

10486351048635

3 Assume T4 NRLSR appropriate techniques to include HVLA muscle energy articulatory Still counterstrainindirect myofascial release facilitated positional release

10486351048635

4 Assume left sided thoracic paraspinal hypertonicity T1 ndash T4 appropriate techniques to include soft tissue deep pressure counterstrainindirect facilitated positional release

10486351048635

5 Assume left 1 st rib inhalation somatic dysfunction appropriate techniques to include HVLA ME Stillrsquos articulatory

10486351048635

MODULE 12 ndash Scarlett O-Hara

1 Osteoarthritis piriformis syndrome spinal stenosis lumbar radiculopathy DVT2 C3 D

Osteopathic Diagnosis assess where you would expect to find somatic dysfunctions

Performed Omitted

1 Examine the lumbar region (L1-L5) for somatic dysfunctionA palpate transverse processes for rotation1 neutral _2 flexion _3 extension _B palpate soft tissue for TART (Tenderness Asymmetry Restrictions Tissue texture change) findings1 Psoas muscles _2 Piriformis muscles _3 Paraspinal muscles _2 Examine the sacrum for somatic dysfunction (requires at least one motion test)A depth at 4 corners of sacrum _B ILA levelness _C seated flexion test _D motion at 4 corners of sacrum _E motion in backward bending test _F respiratory motion of sacrum assess at 4 corners _3 Examine the pelvis for somatic dysfunctionA standing flexion test or AP compression test _B ASIS heights _C PSIS heights _Osteopathic Treatment You must treat a minimum of 2 dysfunctions you found during your examination1 Assume left on right sacral torsion appropriate techniques to include muscle energy counterstrainindirect myofascial release and facilitated positional release (No HVLA)2 Assume left innominate anterior appropriate techniques to include muscle energy counterstrainindirect myofascial release and facilitated positional release (No HVLA)

_

3 Assume spasmtender point in the left piriformis muscle appropriate techniques to _

include counterstrainindirect myofascial release

MODULE 13 Clark Kent1 Asthma atypical pneumonia bronchitis upper respiratory infection pneumothorax2 B3 A

Osteopathic Diagnosis assess where you would expect to find somatic dysfunctions

Performed Omitted

1 Examine the cervical region for somatic dysfunctionA OA ndash leftright translation1 neutral _2 flexion _3 extension _B AA ndash rotation must flex at least 45deg _C C2-C7 - translation1 neutral 2 flexion _3 extension 2 Examine the thoracic spine for dysfunction (T1-T4)A palpate transverse processes for rotation1 neutral _2 flexion _3 extension _B palpate soft tissue for TART findings1 trapezius X2 rhomboids X3 levator scapulae X4 T1 ndash T4 Paraspinal muscles X3 Examine the ribs for somatic dysfunctionA Pump handle ribs (ribs 2-6) at or near sternal junction1 static X2 motion XB Bucket handle ribs (ribs 6-10) between anterior and posterior axillary lines1 static X2 motion XOsteopathic Treatment You must treat a minimum of 2 dysfunctions you found during your examination1 Assume OA NSLRR appropriate techniques to include HVLA muscle energy articulatory Still counterstrainindirect myofascial release facilitated positional release

_

2 Assume T2-4 NSLRR appropriate techniques to include HVLA muscle energy articulatory Still counterstrainindirect myofascial release facilitated positional release

_

3 Assume (L) ribs 2-4 exhalation pump handle appropriate techniques to include HVLA muscle energy articulatory Still counterstrainindirect myofascial release facilitated positional release

4 Autonomic technique for altered vagal tone OA decompression _5 Autonomic technique for hypersympathetic tone in T1-T5 paraspinal muscles rib raising or paraspinal inhibition for gt 90 seconds

X

MODULE 14 Bruce Wayne

1 Depression thyroid disorder migraine headache tension tension headache migraine2 C3 A

Osteopathic Diagnosis assess where you would expect to find somatic dysfunctions

Performed Omitted

1 Examine the cervical region for somatic dysfunctionA OA ndash leftright translation1 neutral 2 flexion 3 extension B AA ndash rotation must flex at least 45deg C C2-C7 - translation1 neutral 2 flexion 3 extension _2 Examine the thoracic spine for dysfunction (T1-T4)A palpate transverse processes for rotation1 neutral X2 flexion X3 extension XB palpate soft tissue for TART findings1 trapezius X2 T1 ndash T4 Paraspinal muscles XOsteopathic Treatment You must treat a minimum of 2 dysfunctions you found during your examination1 Assume OA NSRRL appropriate techniques to include HVLA muscle energy articulatory Still counterstrainindirect myofascial release facilitated positional release

2 Assume C4 FSLRL appropriate techniques to include HVLA muscle energy

articulatory Still counterstrainindirect myofascial release facilitated positional release2 Assume T2-4 NSLRR appropriate techniques to include HVLA muscle energy articulatory Still counterstrainindirect myofascial release facilitated positional release

_

4 Autonomic technique for altered vagal tone OA decompression _

  • Performed
  • Performed
  • Performed
  • Performed
  • Performed
  • Performed
Page 12: Performed · Web viewAAA, kidney stones, groin hernia, ileus, kidney/bladder CA, testicular torsion, kidney/bladder infection, appendicitis, C D Osteopathic Diagnosis: assess where

17 Which of the following would be most likely to inhibit uterine contractions in a

patient in premature labor

a Sacral rocking X

b OA decompression

c Paraspinal inhibition gt90 seconds at T5-T9

d Rib raising lt60 seconds at T10-L2

e Sacral inhibition

18 You assess the innominates and find the following

Positive standing flexion test on the left

Right ASIS inferior compared to left

Right PSIS inferior compared to left

ASIS to midline distance equal bilaterally

Which of the following correctly describes a treatment for this dysfunction

a Muscle energy ndash left hip flexed patient extends against resistance

b HVLA thrust ndash thrust directed inferiorly on left innominate X

c FPR ndash right innominate pushed inferiorly for 3-4 seconds

d Muscle energy ndash right hip pushed superiorly patient pushes inferiorly

e Counterstrain ndash left innominate held inferiorly for 90 seconds

19 The pancreas receives parasympathetic innervation from which of the following

levels

a OA X

b T1-T5

c T5-T9

d T10-L2

e S2-S4

20 Which of the following would most likely directly decrease gastric secretions

a Sacral rocking

b OA decompression

c Paraspinal inhibition gt90 seconds at T10-L2

d Rib raising lt60 seconds at T5-T9 X

e Sacral inhibition

21 Which of the following statements is true regarding osteopathic treatment techniques

a Counterstrain is contraindicated for painful muscle groups

b Osteoporosis is a contraindication for FPR

c Muscle energy should not be used on a person in the CCU X

d HVLA is useful in patients with active Rheumatoid Arthritis

e The restrictive barrier is engaged when using indirect techniques

22 You examine T7 and find the left transverse process to be deeper than the right The

findings become symmetrical in flexion and are unchanged in extension Which of

the following is the correct starting position for a muscle energy treatment

a T7 flexed sidebent left rotated left

b T7 flexed sidebent right rotated right

c T7 extended sidebent left rotated left X

d T7 extended sidebent right rotated left

e T7 flexed sidebent left rotated right

23 The sigmoid colon receives parasympathetic innervation from which of the following

levels

a OA

b T1-T5

c T5-T9

d T10-L2

e S2-S4 X

24 You assess OA translation and find translation to be easier to the right The findings

persist in both flexion and extension Which of the following is the correct position

for FPR treatment

a Flexed rotated right sidebent left

b Neutral rotated left sidebent right

c Neutral rotated right sidebent left X

d Neutral rotated left sidebent left

e Extended rotated right sidebent right

25 Which of the following would most likely directly increase lacrimation

a Sacral rocking

b OA decompression X

c Paraspinal inhibition gt90 seconds at T5-T9

d Rib raising lt60 seconds at T1-T4 X

e Sacral inhibition

MODULE 3 ndash Joanne Smith

1 Top 5 ddx peptic ulcer GERD costochondritis Pancreastitis MI angina pectoris2 E no contraindications3 A OA4 E strep throat

dysfunctions Performed Omitted

1 Examine the cervical region for somatic dysfunctionA OA ndash leftright translation1 neutral 10486351048635

2 flexion 104863510486353 extension 104863510486352 Examine the thoracic spine for dysfunction (T5-T9)A palpate transverse processes for rotation1 neutral 104863510486352 flexion 104863510486353 extension 10486351048635B palpate soft tissue for TART findings1 trapezius 104863510486352 rhomboids 104863510486353 levator scapulae 104863510486354 T5 ndash T9 Paraspinal muscles 10486351048635Osteopathic Treatment You must treat a minimum of 2 dysfunctions you found during your examination1 Assume OA NSLRR appropriate techniques to include HVLA muscle energy articulatory Still counterstrainindirect myofascial release facilitated positional release

10486351048635

2 Assume T7 FRLSL appropriate techniques to include HVLA muscle energy articulatory Still counterstrainindirect myofascial release facilitated positional release

10486351048635

3 Assume acute bilateral spasm of upper thoracic (T5-T9) paraspinal muscles appropriate techniques to include soft tissue counterstrainindirect facilitated positional release

10486351048635

4 Autonomic technique for altered vagal tone OA decompression 104863510486355 Autonomic technique for hypersympathetic tone in T5-T9 paraspinal muscles rib raising or paraspinal inhibition for gt 90 seconds

10486351048635

MODULE 4- Ernesto Rodriguez

PRETEST

1 Back pain from Colon cancer Lumbar strain Osteomyelitits Diverticulitis Back pain osteoarthritis Cauda equina syndrome

2 D3 E

Osteopathic Diagnosis assess where you would expect to find somatic dysfunctions

Performed Omitted

1 Examine the lumbar region (L1-L5) for somatic dysfunction

A palpate transverse processes for rotation

1 neutral _

2 flexion _

3 extension _

B palpate soft tissue for TART (Tenderness Asymmetry Restrictions Tissue texture change) findings

1 quadratus lumborum muscles _

2 Piriformis muscles _

3 Paraspinal muscles _

2 Examine the sacrum for somatic dysfunction (requires at least one motion test)

A depth at 4 corners of sacrum _

B ILA levelness _

C seated flexion test _

D motion at 4 corners of sacrum _

E motion in backward bending test _

F respiratory motion of sacrum assess at 4 corners _

3 Examine the pelvis for somatic dysfunction

A standing flexion test or AP compression test _

B ASIS heights _

C PSIS heights _

Osteopathic Treatment You must treat a minimum of 2 dysfunctions you found during your examination

1 Assume L5 FRLSL _

2 Assume left on left sacral torsion _

3 Assume left innominate anterior _

4 Assume spasmtender point in the left piriformis muscle appropriate techniques to include counterstrainindirect myofascial release

-

MODULE 5 ndash Connie Jones

Pretest

1 Bradycardia acute pulmonary edemaCHF pneumonia acute MI Acute pulmonary edema CHF2 A3 A4 A5 For the video this is what was done and not done

OA - nf NO E --gtNslrrno AA dx doneno cervical dx doneT1-T5 - nfe --gt t2 f sl rlparapsinals TART --gt L upper thoracic paraspinalno levator scapula no trap no rhomboids dx tx -

T2 HLVAOA decompr ndash for vagal tonerib raisin for sympth

MODULE 6 ndash Gretchen White

1 Atrial fibrillation hyperthyroidism and- thyrotoxicosis thyroid cancer hyperthryroid 2 a3 a4 a5

Osteopathic Diagnosis assess where you would expect to find somatic dysfunctions

Performed Omitted

1 Examine the cervical region for somatic dysfunction

A OA ndash leftright translation

1 neutral X

2 flexion X

6 3 extension X

7 B AA ndash rotation X

8 2 Examine the upper thoracic spine for dysfunction (T1-T5)

A palpate transverse processes for rotation

1 neutral X

2 flexion X

3 extension X

B palpate soft tissue for TART findings

1 trapezius X

2 rhomboids X

3 levator scapulae X

4 T1-T5 paraspinal muscles X

Osteopathic Treatment You must treat a minimum of 2 dysfunctions you found during your examination

1 Assume OA NSLRR appropriate techniques to include counterstrainindirect myofascial release facilitated positional release

X

2 Assume T2 FSLRL appropriate techniques to include counterstrainindirect myofascial release facilitated positional release

X

3 Assume acute spasm of left upper thoracic paraspinal muscles appropriate techniques to include counterstrainindirect myofascial release and facilitated positional release

X

4 Autonomic technique for altered vagal tone OA decompression X

5 Autonomic technique for hypersympathetic tone rib raising or paraspinal inhibition for gt 90 seconds

X

MODULE 7 ndash Emmitt Brown

1 AAA kidney stones groin hernia ileus kidneybladder CA testicular torsion kidneybladder infection appendicitis

2 C3 D

Osteopathic Diagnosis assess where you would expect to find somatic dysfunctions

Performed Omitted

1 Examine the cervical region for somatic dysfunctionA OA ndash leftright translation1 neutral X2 flexion X3 extension XB AA ndash rotation must flex at least 45deg X2 Examine the thoracic spine for dysfunction (T10-L2)A palpate transverse processes for rotation1 neutral X2 flexion X3 extension XB palpate soft tissue for TART findings4 T10 ndash L2 Paraspinal muscles X3 Examine the sacrum for somatic dysfunction (requires at least one motion test)A depth at 4 corners of sacrum XB ILA levelness XC seated flexion test XD motion at 4 corners of sacrum XE motion in backward bending test XF respiratory motion of sacrum assess at 4 corners X4 Examine the pelvis for somatic dysfunctionA standing flexion test or AP compression test XB ASIS heights XC PSIS heights XOsteopathic Treatment You must treat a minimum of 2 dysfunctions you found during your examination1 Assume OA NSLRR appropriate techniques to include HVLA muscle energy articulatory Still counterstrainindirect myofascial release facilitated positional release

X _

2 Assume T10 NRRSL appropriate techniques to include HVLA muscle energy articulatory Still counterstrainindirect myofascial release facilitated positional release

X

3 Assume acute bilateral spasm of thoracolumbar (T10-L2) paraspinal muscles RgtL appropriate techniques to include soft tissue counterstrainindirect facilitated positional release

X

4 Assume left on right sacral torsion appropriate techniques to include muscle energy counterstrainindirect myofascial release and facilitated positional release (No HVLA)

X

5 Autonomic technique for hypersympathetic tone in T10-L2 paraspinal muscles rib raising or paraspinal inhibition for gt 90 seconds

X

MODULE 8 ndash Peter Parker1 Kidney stone Psoas syndrome spondylolysis piriformis syndrome lumbar compression

fracture appendicitis pelvicabdominal CA2 C3 B

Osteopathic Diagnosis assess where you would expect to find somatic dysfunctions

Performed Omitted

1 Examine the thoracolumbar region (T10-L5) for somatic dysfunctionA palpate transverse processes for rotation1 neutral _2 flexion _3 extension _B palpate soft tissue for TART (Tenderness Asymmetry Restrictions Tissue texture change) findings1 Psoas muscles _2 Piriformis muscles _3 Paraspinal muscles _2 Examine the sacrum for somatic dysfunction (requires at least one motion test)A depth at 4 corners of sacrum _B ILA levelness _C seated flexion test _D motion at 4 corners of sacrum _E motion in backward bending test _F respiratory motion of sacrum assess at 4 corners _3 Examine the pelvis for somatic dysfunctionA standing flexion test or AP compression test _B ASIS heights _C PSIS heights _Osteopathic Treatment You must treat a minimum of 2 dysfunctions you found during your examination1 Assume L1 FRRSR appropriate techniques to include muscle energy counterstrain indirect myofascial release facilitated positional release (No HVLA)

_

2 Assume left on right sacral torsion appropriate techniques to include muscle energy counterstrainindirect myofascial release and facilitated positional release (No HVLA)

_

3 Assume spasmtender point in the right psoas muscle appropriate techniques to include counterstrainindirect myofascial release FPR ME

_

4 Assume spasmtender point in the left piriformis muscle appropriate techniques to include counterstrainindirect myofascial release FPR ME

_

MODULE 9 ndash Ingrid Bergman

1 Lumbar strain Dysmenorrhea PID lumbar strain ovarian cyst sacral somatic dysfunction2 D3 E

Osteopathic Diagnosis assess where you would expect to find somatic dysfunctions

Performed Omitted

1 Examine the lumbar region (L1-L5) for somatic dysfunctionA palpate transverse processes for rotation1 neutral _2 flexion _3 extension _B palpate soft tissue for TART (Tenderness Asymmetry Restrictions Tissue texture change) findings1 quadratus lumborum muscles _2 Piriformis muscles _3 Paraspinal muscles _2 Examine the sacrum for somatic dysfunction (requires at least one motion test)A depth at 4 corners of sacrum _B ILA levelness _C seated flexion test _D motion at 4 corners of sacrum _E motion in backward bending test _F respiratory motion of sacrum assess at 4 corners _3 Examine the pelvis for somatic dysfunctionA standing flexion test or AP compression test _B ASIS heights _C PSIS heights _Osteopathic Treatment You must treat a minimum of 2 dysfunctions you found during your examination1 Assume L3 FRLSL2 Assume left on right sacral torsion _3 Assume left superior innominate shear _

4 Autonomic technique for altered parasympathetic tone

Module 10 ndash Minnie Driver

Pretest

1 Pneumonia dissecting aortic aneurysm thoracic spine fracture MI pleurisy2 B3 E

Osteopathic Diagnosis assess where you would expect to find somatic dysfunctions

Performed Omitted

1 Examine the cervical region for somatic dysfunctionA OA ndash leftright translation1 neutral 104863510486352 flexion 104863510486353 extension 10486351048635B AA ndash rotation must flex at least 45deg 10486351048635C C2-C7 - translation1 neutral 104863510486352 flexion 104863510486353 extension 104863510486352 Examine the thoracic spine for dysfunction (T5-T9)A palpate transverse processes for rotation1 neutral 104863510486352 flexion 104863510486353 extension 10486351048635B palpate soft tissue for TART findings1 trapezius 104863510486352 rhomboids 104863510486353 levator scapulae 104863510486354 T5 ndash T9 Paraspinal muscles 104863510486353 Examine the ribs for somatic dysfunctionA Pump handle ribs (ribs 2-6) at or near sternal junction1 static 104863510486352 motion 10486351048635B Bucket handle ribs (ribs 6-10) between anterior and posterior axillary lines1 static 104863510486352 motion 10486351048635Osteopathic Treatment You must treat a minimum of 2 dysfunctions you found during your examination1 Assume T5 ESLRL 104863510486352 Assume acute bilateral spasm of upper thoracic (T4-T8) paraspinal 10486351048635

muscles3 Assume (R) Rib 2 pump handle inhalation dysfunction 104863510486354 Autonomic technique for altered vagal tone OA decompression 10486351048635

MODULE 11 ndash Jeff Weaver

1 Cervical radiculopathy thoracic outlet syndrome cubital tunnel syndrome carpal tunnel syndrome angina pectoris ulnar groove entrapment Guillian Barre

2 A3 B4 C

Osteopathic Diagnosis assess where you would expect to find somatic dysfunctions

Performed Omitted

1 Examine the cervical region for somatic dysfunctionA C2-C7 - translation1 neutral 104863510486352 flexion 104863510486353 extension 10486351048635B palpate soft tissue for TART (Tenderness Asymmetry Restrictions Tissue texture change) findings1 cervical paraspinal muscles 104863510486352 sternocleidomastoids 104863510486353 scalenes 104863510486352 Examine the thoracic spine for somatic dysfunctionA palpate transverse processes for rotation1 neutral 104863510486352 flexion 104863510486353 extension 10486351048635B palpate soft tissue for TART findings1 trapezius 104863510486352 pectoral musculature 104863510486353 T1-T4 paraspinal muscle 104863510486353 Examine the 1st rib for somatic dysfunction1 static 104863510486352 dynamic 10486351048635Osteopathic Treatment You must treat a minimum of 2 dysfunctions you found during your examination1 Assume hypertonic left pectoral muscle Appropriate techniques to include ME MFR counterstrain

10486351048635

2 Assume hypertonic left scalene muscle Appropriate techniques to include MFR counterstrain ME

10486351048635

3 Assume T4 NRLSR appropriate techniques to include HVLA muscle energy articulatory Still counterstrainindirect myofascial release facilitated positional release

10486351048635

4 Assume left sided thoracic paraspinal hypertonicity T1 ndash T4 appropriate techniques to include soft tissue deep pressure counterstrainindirect facilitated positional release

10486351048635

5 Assume left 1 st rib inhalation somatic dysfunction appropriate techniques to include HVLA ME Stillrsquos articulatory

10486351048635

MODULE 12 ndash Scarlett O-Hara

1 Osteoarthritis piriformis syndrome spinal stenosis lumbar radiculopathy DVT2 C3 D

Osteopathic Diagnosis assess where you would expect to find somatic dysfunctions

Performed Omitted

1 Examine the lumbar region (L1-L5) for somatic dysfunctionA palpate transverse processes for rotation1 neutral _2 flexion _3 extension _B palpate soft tissue for TART (Tenderness Asymmetry Restrictions Tissue texture change) findings1 Psoas muscles _2 Piriformis muscles _3 Paraspinal muscles _2 Examine the sacrum for somatic dysfunction (requires at least one motion test)A depth at 4 corners of sacrum _B ILA levelness _C seated flexion test _D motion at 4 corners of sacrum _E motion in backward bending test _F respiratory motion of sacrum assess at 4 corners _3 Examine the pelvis for somatic dysfunctionA standing flexion test or AP compression test _B ASIS heights _C PSIS heights _Osteopathic Treatment You must treat a minimum of 2 dysfunctions you found during your examination1 Assume left on right sacral torsion appropriate techniques to include muscle energy counterstrainindirect myofascial release and facilitated positional release (No HVLA)2 Assume left innominate anterior appropriate techniques to include muscle energy counterstrainindirect myofascial release and facilitated positional release (No HVLA)

_

3 Assume spasmtender point in the left piriformis muscle appropriate techniques to _

include counterstrainindirect myofascial release

MODULE 13 Clark Kent1 Asthma atypical pneumonia bronchitis upper respiratory infection pneumothorax2 B3 A

Osteopathic Diagnosis assess where you would expect to find somatic dysfunctions

Performed Omitted

1 Examine the cervical region for somatic dysfunctionA OA ndash leftright translation1 neutral _2 flexion _3 extension _B AA ndash rotation must flex at least 45deg _C C2-C7 - translation1 neutral 2 flexion _3 extension 2 Examine the thoracic spine for dysfunction (T1-T4)A palpate transverse processes for rotation1 neutral _2 flexion _3 extension _B palpate soft tissue for TART findings1 trapezius X2 rhomboids X3 levator scapulae X4 T1 ndash T4 Paraspinal muscles X3 Examine the ribs for somatic dysfunctionA Pump handle ribs (ribs 2-6) at or near sternal junction1 static X2 motion XB Bucket handle ribs (ribs 6-10) between anterior and posterior axillary lines1 static X2 motion XOsteopathic Treatment You must treat a minimum of 2 dysfunctions you found during your examination1 Assume OA NSLRR appropriate techniques to include HVLA muscle energy articulatory Still counterstrainindirect myofascial release facilitated positional release

_

2 Assume T2-4 NSLRR appropriate techniques to include HVLA muscle energy articulatory Still counterstrainindirect myofascial release facilitated positional release

_

3 Assume (L) ribs 2-4 exhalation pump handle appropriate techniques to include HVLA muscle energy articulatory Still counterstrainindirect myofascial release facilitated positional release

4 Autonomic technique for altered vagal tone OA decompression _5 Autonomic technique for hypersympathetic tone in T1-T5 paraspinal muscles rib raising or paraspinal inhibition for gt 90 seconds

X

MODULE 14 Bruce Wayne

1 Depression thyroid disorder migraine headache tension tension headache migraine2 C3 A

Osteopathic Diagnosis assess where you would expect to find somatic dysfunctions

Performed Omitted

1 Examine the cervical region for somatic dysfunctionA OA ndash leftright translation1 neutral 2 flexion 3 extension B AA ndash rotation must flex at least 45deg C C2-C7 - translation1 neutral 2 flexion 3 extension _2 Examine the thoracic spine for dysfunction (T1-T4)A palpate transverse processes for rotation1 neutral X2 flexion X3 extension XB palpate soft tissue for TART findings1 trapezius X2 T1 ndash T4 Paraspinal muscles XOsteopathic Treatment You must treat a minimum of 2 dysfunctions you found during your examination1 Assume OA NSRRL appropriate techniques to include HVLA muscle energy articulatory Still counterstrainindirect myofascial release facilitated positional release

2 Assume C4 FSLRL appropriate techniques to include HVLA muscle energy

articulatory Still counterstrainindirect myofascial release facilitated positional release2 Assume T2-4 NSLRR appropriate techniques to include HVLA muscle energy articulatory Still counterstrainindirect myofascial release facilitated positional release

_

4 Autonomic technique for altered vagal tone OA decompression _

  • Performed
  • Performed
  • Performed
  • Performed
  • Performed
  • Performed
Page 13: Performed · Web viewAAA, kidney stones, groin hernia, ileus, kidney/bladder CA, testicular torsion, kidney/bladder infection, appendicitis, C D Osteopathic Diagnosis: assess where

20 Which of the following would most likely directly decrease gastric secretions

a Sacral rocking

b OA decompression

c Paraspinal inhibition gt90 seconds at T10-L2

d Rib raising lt60 seconds at T5-T9 X

e Sacral inhibition

21 Which of the following statements is true regarding osteopathic treatment techniques

a Counterstrain is contraindicated for painful muscle groups

b Osteoporosis is a contraindication for FPR

c Muscle energy should not be used on a person in the CCU X

d HVLA is useful in patients with active Rheumatoid Arthritis

e The restrictive barrier is engaged when using indirect techniques

22 You examine T7 and find the left transverse process to be deeper than the right The

findings become symmetrical in flexion and are unchanged in extension Which of

the following is the correct starting position for a muscle energy treatment

a T7 flexed sidebent left rotated left

b T7 flexed sidebent right rotated right

c T7 extended sidebent left rotated left X

d T7 extended sidebent right rotated left

e T7 flexed sidebent left rotated right

23 The sigmoid colon receives parasympathetic innervation from which of the following

levels

a OA

b T1-T5

c T5-T9

d T10-L2

e S2-S4 X

24 You assess OA translation and find translation to be easier to the right The findings

persist in both flexion and extension Which of the following is the correct position

for FPR treatment

a Flexed rotated right sidebent left

b Neutral rotated left sidebent right

c Neutral rotated right sidebent left X

d Neutral rotated left sidebent left

e Extended rotated right sidebent right

25 Which of the following would most likely directly increase lacrimation

a Sacral rocking

b OA decompression X

c Paraspinal inhibition gt90 seconds at T5-T9

d Rib raising lt60 seconds at T1-T4 X

e Sacral inhibition

MODULE 3 ndash Joanne Smith

1 Top 5 ddx peptic ulcer GERD costochondritis Pancreastitis MI angina pectoris2 E no contraindications3 A OA4 E strep throat

dysfunctions Performed Omitted

1 Examine the cervical region for somatic dysfunctionA OA ndash leftright translation1 neutral 10486351048635

2 flexion 104863510486353 extension 104863510486352 Examine the thoracic spine for dysfunction (T5-T9)A palpate transverse processes for rotation1 neutral 104863510486352 flexion 104863510486353 extension 10486351048635B palpate soft tissue for TART findings1 trapezius 104863510486352 rhomboids 104863510486353 levator scapulae 104863510486354 T5 ndash T9 Paraspinal muscles 10486351048635Osteopathic Treatment You must treat a minimum of 2 dysfunctions you found during your examination1 Assume OA NSLRR appropriate techniques to include HVLA muscle energy articulatory Still counterstrainindirect myofascial release facilitated positional release

10486351048635

2 Assume T7 FRLSL appropriate techniques to include HVLA muscle energy articulatory Still counterstrainindirect myofascial release facilitated positional release

10486351048635

3 Assume acute bilateral spasm of upper thoracic (T5-T9) paraspinal muscles appropriate techniques to include soft tissue counterstrainindirect facilitated positional release

10486351048635

4 Autonomic technique for altered vagal tone OA decompression 104863510486355 Autonomic technique for hypersympathetic tone in T5-T9 paraspinal muscles rib raising or paraspinal inhibition for gt 90 seconds

10486351048635

MODULE 4- Ernesto Rodriguez

PRETEST

1 Back pain from Colon cancer Lumbar strain Osteomyelitits Diverticulitis Back pain osteoarthritis Cauda equina syndrome

2 D3 E

Osteopathic Diagnosis assess where you would expect to find somatic dysfunctions

Performed Omitted

1 Examine the lumbar region (L1-L5) for somatic dysfunction

A palpate transverse processes for rotation

1 neutral _

2 flexion _

3 extension _

B palpate soft tissue for TART (Tenderness Asymmetry Restrictions Tissue texture change) findings

1 quadratus lumborum muscles _

2 Piriformis muscles _

3 Paraspinal muscles _

2 Examine the sacrum for somatic dysfunction (requires at least one motion test)

A depth at 4 corners of sacrum _

B ILA levelness _

C seated flexion test _

D motion at 4 corners of sacrum _

E motion in backward bending test _

F respiratory motion of sacrum assess at 4 corners _

3 Examine the pelvis for somatic dysfunction

A standing flexion test or AP compression test _

B ASIS heights _

C PSIS heights _

Osteopathic Treatment You must treat a minimum of 2 dysfunctions you found during your examination

1 Assume L5 FRLSL _

2 Assume left on left sacral torsion _

3 Assume left innominate anterior _

4 Assume spasmtender point in the left piriformis muscle appropriate techniques to include counterstrainindirect myofascial release

-

MODULE 5 ndash Connie Jones

Pretest

1 Bradycardia acute pulmonary edemaCHF pneumonia acute MI Acute pulmonary edema CHF2 A3 A4 A5 For the video this is what was done and not done

OA - nf NO E --gtNslrrno AA dx doneno cervical dx doneT1-T5 - nfe --gt t2 f sl rlparapsinals TART --gt L upper thoracic paraspinalno levator scapula no trap no rhomboids dx tx -

T2 HLVAOA decompr ndash for vagal tonerib raisin for sympth

MODULE 6 ndash Gretchen White

1 Atrial fibrillation hyperthyroidism and- thyrotoxicosis thyroid cancer hyperthryroid 2 a3 a4 a5

Osteopathic Diagnosis assess where you would expect to find somatic dysfunctions

Performed Omitted

1 Examine the cervical region for somatic dysfunction

A OA ndash leftright translation

1 neutral X

2 flexion X

6 3 extension X

7 B AA ndash rotation X

8 2 Examine the upper thoracic spine for dysfunction (T1-T5)

A palpate transverse processes for rotation

1 neutral X

2 flexion X

3 extension X

B palpate soft tissue for TART findings

1 trapezius X

2 rhomboids X

3 levator scapulae X

4 T1-T5 paraspinal muscles X

Osteopathic Treatment You must treat a minimum of 2 dysfunctions you found during your examination

1 Assume OA NSLRR appropriate techniques to include counterstrainindirect myofascial release facilitated positional release

X

2 Assume T2 FSLRL appropriate techniques to include counterstrainindirect myofascial release facilitated positional release

X

3 Assume acute spasm of left upper thoracic paraspinal muscles appropriate techniques to include counterstrainindirect myofascial release and facilitated positional release

X

4 Autonomic technique for altered vagal tone OA decompression X

5 Autonomic technique for hypersympathetic tone rib raising or paraspinal inhibition for gt 90 seconds

X

MODULE 7 ndash Emmitt Brown

1 AAA kidney stones groin hernia ileus kidneybladder CA testicular torsion kidneybladder infection appendicitis

2 C3 D

Osteopathic Diagnosis assess where you would expect to find somatic dysfunctions

Performed Omitted

1 Examine the cervical region for somatic dysfunctionA OA ndash leftright translation1 neutral X2 flexion X3 extension XB AA ndash rotation must flex at least 45deg X2 Examine the thoracic spine for dysfunction (T10-L2)A palpate transverse processes for rotation1 neutral X2 flexion X3 extension XB palpate soft tissue for TART findings4 T10 ndash L2 Paraspinal muscles X3 Examine the sacrum for somatic dysfunction (requires at least one motion test)A depth at 4 corners of sacrum XB ILA levelness XC seated flexion test XD motion at 4 corners of sacrum XE motion in backward bending test XF respiratory motion of sacrum assess at 4 corners X4 Examine the pelvis for somatic dysfunctionA standing flexion test or AP compression test XB ASIS heights XC PSIS heights XOsteopathic Treatment You must treat a minimum of 2 dysfunctions you found during your examination1 Assume OA NSLRR appropriate techniques to include HVLA muscle energy articulatory Still counterstrainindirect myofascial release facilitated positional release

X _

2 Assume T10 NRRSL appropriate techniques to include HVLA muscle energy articulatory Still counterstrainindirect myofascial release facilitated positional release

X

3 Assume acute bilateral spasm of thoracolumbar (T10-L2) paraspinal muscles RgtL appropriate techniques to include soft tissue counterstrainindirect facilitated positional release

X

4 Assume left on right sacral torsion appropriate techniques to include muscle energy counterstrainindirect myofascial release and facilitated positional release (No HVLA)

X

5 Autonomic technique for hypersympathetic tone in T10-L2 paraspinal muscles rib raising or paraspinal inhibition for gt 90 seconds

X

MODULE 8 ndash Peter Parker1 Kidney stone Psoas syndrome spondylolysis piriformis syndrome lumbar compression

fracture appendicitis pelvicabdominal CA2 C3 B

Osteopathic Diagnosis assess where you would expect to find somatic dysfunctions

Performed Omitted

1 Examine the thoracolumbar region (T10-L5) for somatic dysfunctionA palpate transverse processes for rotation1 neutral _2 flexion _3 extension _B palpate soft tissue for TART (Tenderness Asymmetry Restrictions Tissue texture change) findings1 Psoas muscles _2 Piriformis muscles _3 Paraspinal muscles _2 Examine the sacrum for somatic dysfunction (requires at least one motion test)A depth at 4 corners of sacrum _B ILA levelness _C seated flexion test _D motion at 4 corners of sacrum _E motion in backward bending test _F respiratory motion of sacrum assess at 4 corners _3 Examine the pelvis for somatic dysfunctionA standing flexion test or AP compression test _B ASIS heights _C PSIS heights _Osteopathic Treatment You must treat a minimum of 2 dysfunctions you found during your examination1 Assume L1 FRRSR appropriate techniques to include muscle energy counterstrain indirect myofascial release facilitated positional release (No HVLA)

_

2 Assume left on right sacral torsion appropriate techniques to include muscle energy counterstrainindirect myofascial release and facilitated positional release (No HVLA)

_

3 Assume spasmtender point in the right psoas muscle appropriate techniques to include counterstrainindirect myofascial release FPR ME

_

4 Assume spasmtender point in the left piriformis muscle appropriate techniques to include counterstrainindirect myofascial release FPR ME

_

MODULE 9 ndash Ingrid Bergman

1 Lumbar strain Dysmenorrhea PID lumbar strain ovarian cyst sacral somatic dysfunction2 D3 E

Osteopathic Diagnosis assess where you would expect to find somatic dysfunctions

Performed Omitted

1 Examine the lumbar region (L1-L5) for somatic dysfunctionA palpate transverse processes for rotation1 neutral _2 flexion _3 extension _B palpate soft tissue for TART (Tenderness Asymmetry Restrictions Tissue texture change) findings1 quadratus lumborum muscles _2 Piriformis muscles _3 Paraspinal muscles _2 Examine the sacrum for somatic dysfunction (requires at least one motion test)A depth at 4 corners of sacrum _B ILA levelness _C seated flexion test _D motion at 4 corners of sacrum _E motion in backward bending test _F respiratory motion of sacrum assess at 4 corners _3 Examine the pelvis for somatic dysfunctionA standing flexion test or AP compression test _B ASIS heights _C PSIS heights _Osteopathic Treatment You must treat a minimum of 2 dysfunctions you found during your examination1 Assume L3 FRLSL2 Assume left on right sacral torsion _3 Assume left superior innominate shear _

4 Autonomic technique for altered parasympathetic tone

Module 10 ndash Minnie Driver

Pretest

1 Pneumonia dissecting aortic aneurysm thoracic spine fracture MI pleurisy2 B3 E

Osteopathic Diagnosis assess where you would expect to find somatic dysfunctions

Performed Omitted

1 Examine the cervical region for somatic dysfunctionA OA ndash leftright translation1 neutral 104863510486352 flexion 104863510486353 extension 10486351048635B AA ndash rotation must flex at least 45deg 10486351048635C C2-C7 - translation1 neutral 104863510486352 flexion 104863510486353 extension 104863510486352 Examine the thoracic spine for dysfunction (T5-T9)A palpate transverse processes for rotation1 neutral 104863510486352 flexion 104863510486353 extension 10486351048635B palpate soft tissue for TART findings1 trapezius 104863510486352 rhomboids 104863510486353 levator scapulae 104863510486354 T5 ndash T9 Paraspinal muscles 104863510486353 Examine the ribs for somatic dysfunctionA Pump handle ribs (ribs 2-6) at or near sternal junction1 static 104863510486352 motion 10486351048635B Bucket handle ribs (ribs 6-10) between anterior and posterior axillary lines1 static 104863510486352 motion 10486351048635Osteopathic Treatment You must treat a minimum of 2 dysfunctions you found during your examination1 Assume T5 ESLRL 104863510486352 Assume acute bilateral spasm of upper thoracic (T4-T8) paraspinal 10486351048635

muscles3 Assume (R) Rib 2 pump handle inhalation dysfunction 104863510486354 Autonomic technique for altered vagal tone OA decompression 10486351048635

MODULE 11 ndash Jeff Weaver

1 Cervical radiculopathy thoracic outlet syndrome cubital tunnel syndrome carpal tunnel syndrome angina pectoris ulnar groove entrapment Guillian Barre

2 A3 B4 C

Osteopathic Diagnosis assess where you would expect to find somatic dysfunctions

Performed Omitted

1 Examine the cervical region for somatic dysfunctionA C2-C7 - translation1 neutral 104863510486352 flexion 104863510486353 extension 10486351048635B palpate soft tissue for TART (Tenderness Asymmetry Restrictions Tissue texture change) findings1 cervical paraspinal muscles 104863510486352 sternocleidomastoids 104863510486353 scalenes 104863510486352 Examine the thoracic spine for somatic dysfunctionA palpate transverse processes for rotation1 neutral 104863510486352 flexion 104863510486353 extension 10486351048635B palpate soft tissue for TART findings1 trapezius 104863510486352 pectoral musculature 104863510486353 T1-T4 paraspinal muscle 104863510486353 Examine the 1st rib for somatic dysfunction1 static 104863510486352 dynamic 10486351048635Osteopathic Treatment You must treat a minimum of 2 dysfunctions you found during your examination1 Assume hypertonic left pectoral muscle Appropriate techniques to include ME MFR counterstrain

10486351048635

2 Assume hypertonic left scalene muscle Appropriate techniques to include MFR counterstrain ME

10486351048635

3 Assume T4 NRLSR appropriate techniques to include HVLA muscle energy articulatory Still counterstrainindirect myofascial release facilitated positional release

10486351048635

4 Assume left sided thoracic paraspinal hypertonicity T1 ndash T4 appropriate techniques to include soft tissue deep pressure counterstrainindirect facilitated positional release

10486351048635

5 Assume left 1 st rib inhalation somatic dysfunction appropriate techniques to include HVLA ME Stillrsquos articulatory

10486351048635

MODULE 12 ndash Scarlett O-Hara

1 Osteoarthritis piriformis syndrome spinal stenosis lumbar radiculopathy DVT2 C3 D

Osteopathic Diagnosis assess where you would expect to find somatic dysfunctions

Performed Omitted

1 Examine the lumbar region (L1-L5) for somatic dysfunctionA palpate transverse processes for rotation1 neutral _2 flexion _3 extension _B palpate soft tissue for TART (Tenderness Asymmetry Restrictions Tissue texture change) findings1 Psoas muscles _2 Piriformis muscles _3 Paraspinal muscles _2 Examine the sacrum for somatic dysfunction (requires at least one motion test)A depth at 4 corners of sacrum _B ILA levelness _C seated flexion test _D motion at 4 corners of sacrum _E motion in backward bending test _F respiratory motion of sacrum assess at 4 corners _3 Examine the pelvis for somatic dysfunctionA standing flexion test or AP compression test _B ASIS heights _C PSIS heights _Osteopathic Treatment You must treat a minimum of 2 dysfunctions you found during your examination1 Assume left on right sacral torsion appropriate techniques to include muscle energy counterstrainindirect myofascial release and facilitated positional release (No HVLA)2 Assume left innominate anterior appropriate techniques to include muscle energy counterstrainindirect myofascial release and facilitated positional release (No HVLA)

_

3 Assume spasmtender point in the left piriformis muscle appropriate techniques to _

include counterstrainindirect myofascial release

MODULE 13 Clark Kent1 Asthma atypical pneumonia bronchitis upper respiratory infection pneumothorax2 B3 A

Osteopathic Diagnosis assess where you would expect to find somatic dysfunctions

Performed Omitted

1 Examine the cervical region for somatic dysfunctionA OA ndash leftright translation1 neutral _2 flexion _3 extension _B AA ndash rotation must flex at least 45deg _C C2-C7 - translation1 neutral 2 flexion _3 extension 2 Examine the thoracic spine for dysfunction (T1-T4)A palpate transverse processes for rotation1 neutral _2 flexion _3 extension _B palpate soft tissue for TART findings1 trapezius X2 rhomboids X3 levator scapulae X4 T1 ndash T4 Paraspinal muscles X3 Examine the ribs for somatic dysfunctionA Pump handle ribs (ribs 2-6) at or near sternal junction1 static X2 motion XB Bucket handle ribs (ribs 6-10) between anterior and posterior axillary lines1 static X2 motion XOsteopathic Treatment You must treat a minimum of 2 dysfunctions you found during your examination1 Assume OA NSLRR appropriate techniques to include HVLA muscle energy articulatory Still counterstrainindirect myofascial release facilitated positional release

_

2 Assume T2-4 NSLRR appropriate techniques to include HVLA muscle energy articulatory Still counterstrainindirect myofascial release facilitated positional release

_

3 Assume (L) ribs 2-4 exhalation pump handle appropriate techniques to include HVLA muscle energy articulatory Still counterstrainindirect myofascial release facilitated positional release

4 Autonomic technique for altered vagal tone OA decompression _5 Autonomic technique for hypersympathetic tone in T1-T5 paraspinal muscles rib raising or paraspinal inhibition for gt 90 seconds

X

MODULE 14 Bruce Wayne

1 Depression thyroid disorder migraine headache tension tension headache migraine2 C3 A

Osteopathic Diagnosis assess where you would expect to find somatic dysfunctions

Performed Omitted

1 Examine the cervical region for somatic dysfunctionA OA ndash leftright translation1 neutral 2 flexion 3 extension B AA ndash rotation must flex at least 45deg C C2-C7 - translation1 neutral 2 flexion 3 extension _2 Examine the thoracic spine for dysfunction (T1-T4)A palpate transverse processes for rotation1 neutral X2 flexion X3 extension XB palpate soft tissue for TART findings1 trapezius X2 T1 ndash T4 Paraspinal muscles XOsteopathic Treatment You must treat a minimum of 2 dysfunctions you found during your examination1 Assume OA NSRRL appropriate techniques to include HVLA muscle energy articulatory Still counterstrainindirect myofascial release facilitated positional release

2 Assume C4 FSLRL appropriate techniques to include HVLA muscle energy

articulatory Still counterstrainindirect myofascial release facilitated positional release2 Assume T2-4 NSLRR appropriate techniques to include HVLA muscle energy articulatory Still counterstrainindirect myofascial release facilitated positional release

_

4 Autonomic technique for altered vagal tone OA decompression _

  • Performed
  • Performed
  • Performed
  • Performed
  • Performed
  • Performed
Page 14: Performed · Web viewAAA, kidney stones, groin hernia, ileus, kidney/bladder CA, testicular torsion, kidney/bladder infection, appendicitis, C D Osteopathic Diagnosis: assess where

d T10-L2

e S2-S4 X

24 You assess OA translation and find translation to be easier to the right The findings

persist in both flexion and extension Which of the following is the correct position

for FPR treatment

a Flexed rotated right sidebent left

b Neutral rotated left sidebent right

c Neutral rotated right sidebent left X

d Neutral rotated left sidebent left

e Extended rotated right sidebent right

25 Which of the following would most likely directly increase lacrimation

a Sacral rocking

b OA decompression X

c Paraspinal inhibition gt90 seconds at T5-T9

d Rib raising lt60 seconds at T1-T4 X

e Sacral inhibition

MODULE 3 ndash Joanne Smith

1 Top 5 ddx peptic ulcer GERD costochondritis Pancreastitis MI angina pectoris2 E no contraindications3 A OA4 E strep throat

dysfunctions Performed Omitted

1 Examine the cervical region for somatic dysfunctionA OA ndash leftright translation1 neutral 10486351048635

2 flexion 104863510486353 extension 104863510486352 Examine the thoracic spine for dysfunction (T5-T9)A palpate transverse processes for rotation1 neutral 104863510486352 flexion 104863510486353 extension 10486351048635B palpate soft tissue for TART findings1 trapezius 104863510486352 rhomboids 104863510486353 levator scapulae 104863510486354 T5 ndash T9 Paraspinal muscles 10486351048635Osteopathic Treatment You must treat a minimum of 2 dysfunctions you found during your examination1 Assume OA NSLRR appropriate techniques to include HVLA muscle energy articulatory Still counterstrainindirect myofascial release facilitated positional release

10486351048635

2 Assume T7 FRLSL appropriate techniques to include HVLA muscle energy articulatory Still counterstrainindirect myofascial release facilitated positional release

10486351048635

3 Assume acute bilateral spasm of upper thoracic (T5-T9) paraspinal muscles appropriate techniques to include soft tissue counterstrainindirect facilitated positional release

10486351048635

4 Autonomic technique for altered vagal tone OA decompression 104863510486355 Autonomic technique for hypersympathetic tone in T5-T9 paraspinal muscles rib raising or paraspinal inhibition for gt 90 seconds

10486351048635

MODULE 4- Ernesto Rodriguez

PRETEST

1 Back pain from Colon cancer Lumbar strain Osteomyelitits Diverticulitis Back pain osteoarthritis Cauda equina syndrome

2 D3 E

Osteopathic Diagnosis assess where you would expect to find somatic dysfunctions

Performed Omitted

1 Examine the lumbar region (L1-L5) for somatic dysfunction

A palpate transverse processes for rotation

1 neutral _

2 flexion _

3 extension _

B palpate soft tissue for TART (Tenderness Asymmetry Restrictions Tissue texture change) findings

1 quadratus lumborum muscles _

2 Piriformis muscles _

3 Paraspinal muscles _

2 Examine the sacrum for somatic dysfunction (requires at least one motion test)

A depth at 4 corners of sacrum _

B ILA levelness _

C seated flexion test _

D motion at 4 corners of sacrum _

E motion in backward bending test _

F respiratory motion of sacrum assess at 4 corners _

3 Examine the pelvis for somatic dysfunction

A standing flexion test or AP compression test _

B ASIS heights _

C PSIS heights _

Osteopathic Treatment You must treat a minimum of 2 dysfunctions you found during your examination

1 Assume L5 FRLSL _

2 Assume left on left sacral torsion _

3 Assume left innominate anterior _

4 Assume spasmtender point in the left piriformis muscle appropriate techniques to include counterstrainindirect myofascial release

-

MODULE 5 ndash Connie Jones

Pretest

1 Bradycardia acute pulmonary edemaCHF pneumonia acute MI Acute pulmonary edema CHF2 A3 A4 A5 For the video this is what was done and not done

OA - nf NO E --gtNslrrno AA dx doneno cervical dx doneT1-T5 - nfe --gt t2 f sl rlparapsinals TART --gt L upper thoracic paraspinalno levator scapula no trap no rhomboids dx tx -

T2 HLVAOA decompr ndash for vagal tonerib raisin for sympth

MODULE 6 ndash Gretchen White

1 Atrial fibrillation hyperthyroidism and- thyrotoxicosis thyroid cancer hyperthryroid 2 a3 a4 a5

Osteopathic Diagnosis assess where you would expect to find somatic dysfunctions

Performed Omitted

1 Examine the cervical region for somatic dysfunction

A OA ndash leftright translation

1 neutral X

2 flexion X

6 3 extension X

7 B AA ndash rotation X

8 2 Examine the upper thoracic spine for dysfunction (T1-T5)

A palpate transverse processes for rotation

1 neutral X

2 flexion X

3 extension X

B palpate soft tissue for TART findings

1 trapezius X

2 rhomboids X

3 levator scapulae X

4 T1-T5 paraspinal muscles X

Osteopathic Treatment You must treat a minimum of 2 dysfunctions you found during your examination

1 Assume OA NSLRR appropriate techniques to include counterstrainindirect myofascial release facilitated positional release

X

2 Assume T2 FSLRL appropriate techniques to include counterstrainindirect myofascial release facilitated positional release

X

3 Assume acute spasm of left upper thoracic paraspinal muscles appropriate techniques to include counterstrainindirect myofascial release and facilitated positional release

X

4 Autonomic technique for altered vagal tone OA decompression X

5 Autonomic technique for hypersympathetic tone rib raising or paraspinal inhibition for gt 90 seconds

X

MODULE 7 ndash Emmitt Brown

1 AAA kidney stones groin hernia ileus kidneybladder CA testicular torsion kidneybladder infection appendicitis

2 C3 D

Osteopathic Diagnosis assess where you would expect to find somatic dysfunctions

Performed Omitted

1 Examine the cervical region for somatic dysfunctionA OA ndash leftright translation1 neutral X2 flexion X3 extension XB AA ndash rotation must flex at least 45deg X2 Examine the thoracic spine for dysfunction (T10-L2)A palpate transverse processes for rotation1 neutral X2 flexion X3 extension XB palpate soft tissue for TART findings4 T10 ndash L2 Paraspinal muscles X3 Examine the sacrum for somatic dysfunction (requires at least one motion test)A depth at 4 corners of sacrum XB ILA levelness XC seated flexion test XD motion at 4 corners of sacrum XE motion in backward bending test XF respiratory motion of sacrum assess at 4 corners X4 Examine the pelvis for somatic dysfunctionA standing flexion test or AP compression test XB ASIS heights XC PSIS heights XOsteopathic Treatment You must treat a minimum of 2 dysfunctions you found during your examination1 Assume OA NSLRR appropriate techniques to include HVLA muscle energy articulatory Still counterstrainindirect myofascial release facilitated positional release

X _

2 Assume T10 NRRSL appropriate techniques to include HVLA muscle energy articulatory Still counterstrainindirect myofascial release facilitated positional release

X

3 Assume acute bilateral spasm of thoracolumbar (T10-L2) paraspinal muscles RgtL appropriate techniques to include soft tissue counterstrainindirect facilitated positional release

X

4 Assume left on right sacral torsion appropriate techniques to include muscle energy counterstrainindirect myofascial release and facilitated positional release (No HVLA)

X

5 Autonomic technique for hypersympathetic tone in T10-L2 paraspinal muscles rib raising or paraspinal inhibition for gt 90 seconds

X

MODULE 8 ndash Peter Parker1 Kidney stone Psoas syndrome spondylolysis piriformis syndrome lumbar compression

fracture appendicitis pelvicabdominal CA2 C3 B

Osteopathic Diagnosis assess where you would expect to find somatic dysfunctions

Performed Omitted

1 Examine the thoracolumbar region (T10-L5) for somatic dysfunctionA palpate transverse processes for rotation1 neutral _2 flexion _3 extension _B palpate soft tissue for TART (Tenderness Asymmetry Restrictions Tissue texture change) findings1 Psoas muscles _2 Piriformis muscles _3 Paraspinal muscles _2 Examine the sacrum for somatic dysfunction (requires at least one motion test)A depth at 4 corners of sacrum _B ILA levelness _C seated flexion test _D motion at 4 corners of sacrum _E motion in backward bending test _F respiratory motion of sacrum assess at 4 corners _3 Examine the pelvis for somatic dysfunctionA standing flexion test or AP compression test _B ASIS heights _C PSIS heights _Osteopathic Treatment You must treat a minimum of 2 dysfunctions you found during your examination1 Assume L1 FRRSR appropriate techniques to include muscle energy counterstrain indirect myofascial release facilitated positional release (No HVLA)

_

2 Assume left on right sacral torsion appropriate techniques to include muscle energy counterstrainindirect myofascial release and facilitated positional release (No HVLA)

_

3 Assume spasmtender point in the right psoas muscle appropriate techniques to include counterstrainindirect myofascial release FPR ME

_

4 Assume spasmtender point in the left piriformis muscle appropriate techniques to include counterstrainindirect myofascial release FPR ME

_

MODULE 9 ndash Ingrid Bergman

1 Lumbar strain Dysmenorrhea PID lumbar strain ovarian cyst sacral somatic dysfunction2 D3 E

Osteopathic Diagnosis assess where you would expect to find somatic dysfunctions

Performed Omitted

1 Examine the lumbar region (L1-L5) for somatic dysfunctionA palpate transverse processes for rotation1 neutral _2 flexion _3 extension _B palpate soft tissue for TART (Tenderness Asymmetry Restrictions Tissue texture change) findings1 quadratus lumborum muscles _2 Piriformis muscles _3 Paraspinal muscles _2 Examine the sacrum for somatic dysfunction (requires at least one motion test)A depth at 4 corners of sacrum _B ILA levelness _C seated flexion test _D motion at 4 corners of sacrum _E motion in backward bending test _F respiratory motion of sacrum assess at 4 corners _3 Examine the pelvis for somatic dysfunctionA standing flexion test or AP compression test _B ASIS heights _C PSIS heights _Osteopathic Treatment You must treat a minimum of 2 dysfunctions you found during your examination1 Assume L3 FRLSL2 Assume left on right sacral torsion _3 Assume left superior innominate shear _

4 Autonomic technique for altered parasympathetic tone

Module 10 ndash Minnie Driver

Pretest

1 Pneumonia dissecting aortic aneurysm thoracic spine fracture MI pleurisy2 B3 E

Osteopathic Diagnosis assess where you would expect to find somatic dysfunctions

Performed Omitted

1 Examine the cervical region for somatic dysfunctionA OA ndash leftright translation1 neutral 104863510486352 flexion 104863510486353 extension 10486351048635B AA ndash rotation must flex at least 45deg 10486351048635C C2-C7 - translation1 neutral 104863510486352 flexion 104863510486353 extension 104863510486352 Examine the thoracic spine for dysfunction (T5-T9)A palpate transverse processes for rotation1 neutral 104863510486352 flexion 104863510486353 extension 10486351048635B palpate soft tissue for TART findings1 trapezius 104863510486352 rhomboids 104863510486353 levator scapulae 104863510486354 T5 ndash T9 Paraspinal muscles 104863510486353 Examine the ribs for somatic dysfunctionA Pump handle ribs (ribs 2-6) at or near sternal junction1 static 104863510486352 motion 10486351048635B Bucket handle ribs (ribs 6-10) between anterior and posterior axillary lines1 static 104863510486352 motion 10486351048635Osteopathic Treatment You must treat a minimum of 2 dysfunctions you found during your examination1 Assume T5 ESLRL 104863510486352 Assume acute bilateral spasm of upper thoracic (T4-T8) paraspinal 10486351048635

muscles3 Assume (R) Rib 2 pump handle inhalation dysfunction 104863510486354 Autonomic technique for altered vagal tone OA decompression 10486351048635

MODULE 11 ndash Jeff Weaver

1 Cervical radiculopathy thoracic outlet syndrome cubital tunnel syndrome carpal tunnel syndrome angina pectoris ulnar groove entrapment Guillian Barre

2 A3 B4 C

Osteopathic Diagnosis assess where you would expect to find somatic dysfunctions

Performed Omitted

1 Examine the cervical region for somatic dysfunctionA C2-C7 - translation1 neutral 104863510486352 flexion 104863510486353 extension 10486351048635B palpate soft tissue for TART (Tenderness Asymmetry Restrictions Tissue texture change) findings1 cervical paraspinal muscles 104863510486352 sternocleidomastoids 104863510486353 scalenes 104863510486352 Examine the thoracic spine for somatic dysfunctionA palpate transverse processes for rotation1 neutral 104863510486352 flexion 104863510486353 extension 10486351048635B palpate soft tissue for TART findings1 trapezius 104863510486352 pectoral musculature 104863510486353 T1-T4 paraspinal muscle 104863510486353 Examine the 1st rib for somatic dysfunction1 static 104863510486352 dynamic 10486351048635Osteopathic Treatment You must treat a minimum of 2 dysfunctions you found during your examination1 Assume hypertonic left pectoral muscle Appropriate techniques to include ME MFR counterstrain

10486351048635

2 Assume hypertonic left scalene muscle Appropriate techniques to include MFR counterstrain ME

10486351048635

3 Assume T4 NRLSR appropriate techniques to include HVLA muscle energy articulatory Still counterstrainindirect myofascial release facilitated positional release

10486351048635

4 Assume left sided thoracic paraspinal hypertonicity T1 ndash T4 appropriate techniques to include soft tissue deep pressure counterstrainindirect facilitated positional release

10486351048635

5 Assume left 1 st rib inhalation somatic dysfunction appropriate techniques to include HVLA ME Stillrsquos articulatory

10486351048635

MODULE 12 ndash Scarlett O-Hara

1 Osteoarthritis piriformis syndrome spinal stenosis lumbar radiculopathy DVT2 C3 D

Osteopathic Diagnosis assess where you would expect to find somatic dysfunctions

Performed Omitted

1 Examine the lumbar region (L1-L5) for somatic dysfunctionA palpate transverse processes for rotation1 neutral _2 flexion _3 extension _B palpate soft tissue for TART (Tenderness Asymmetry Restrictions Tissue texture change) findings1 Psoas muscles _2 Piriformis muscles _3 Paraspinal muscles _2 Examine the sacrum for somatic dysfunction (requires at least one motion test)A depth at 4 corners of sacrum _B ILA levelness _C seated flexion test _D motion at 4 corners of sacrum _E motion in backward bending test _F respiratory motion of sacrum assess at 4 corners _3 Examine the pelvis for somatic dysfunctionA standing flexion test or AP compression test _B ASIS heights _C PSIS heights _Osteopathic Treatment You must treat a minimum of 2 dysfunctions you found during your examination1 Assume left on right sacral torsion appropriate techniques to include muscle energy counterstrainindirect myofascial release and facilitated positional release (No HVLA)2 Assume left innominate anterior appropriate techniques to include muscle energy counterstrainindirect myofascial release and facilitated positional release (No HVLA)

_

3 Assume spasmtender point in the left piriformis muscle appropriate techniques to _

include counterstrainindirect myofascial release

MODULE 13 Clark Kent1 Asthma atypical pneumonia bronchitis upper respiratory infection pneumothorax2 B3 A

Osteopathic Diagnosis assess where you would expect to find somatic dysfunctions

Performed Omitted

1 Examine the cervical region for somatic dysfunctionA OA ndash leftright translation1 neutral _2 flexion _3 extension _B AA ndash rotation must flex at least 45deg _C C2-C7 - translation1 neutral 2 flexion _3 extension 2 Examine the thoracic spine for dysfunction (T1-T4)A palpate transverse processes for rotation1 neutral _2 flexion _3 extension _B palpate soft tissue for TART findings1 trapezius X2 rhomboids X3 levator scapulae X4 T1 ndash T4 Paraspinal muscles X3 Examine the ribs for somatic dysfunctionA Pump handle ribs (ribs 2-6) at or near sternal junction1 static X2 motion XB Bucket handle ribs (ribs 6-10) between anterior and posterior axillary lines1 static X2 motion XOsteopathic Treatment You must treat a minimum of 2 dysfunctions you found during your examination1 Assume OA NSLRR appropriate techniques to include HVLA muscle energy articulatory Still counterstrainindirect myofascial release facilitated positional release

_

2 Assume T2-4 NSLRR appropriate techniques to include HVLA muscle energy articulatory Still counterstrainindirect myofascial release facilitated positional release

_

3 Assume (L) ribs 2-4 exhalation pump handle appropriate techniques to include HVLA muscle energy articulatory Still counterstrainindirect myofascial release facilitated positional release

4 Autonomic technique for altered vagal tone OA decompression _5 Autonomic technique for hypersympathetic tone in T1-T5 paraspinal muscles rib raising or paraspinal inhibition for gt 90 seconds

X

MODULE 14 Bruce Wayne

1 Depression thyroid disorder migraine headache tension tension headache migraine2 C3 A

Osteopathic Diagnosis assess where you would expect to find somatic dysfunctions

Performed Omitted

1 Examine the cervical region for somatic dysfunctionA OA ndash leftright translation1 neutral 2 flexion 3 extension B AA ndash rotation must flex at least 45deg C C2-C7 - translation1 neutral 2 flexion 3 extension _2 Examine the thoracic spine for dysfunction (T1-T4)A palpate transverse processes for rotation1 neutral X2 flexion X3 extension XB palpate soft tissue for TART findings1 trapezius X2 T1 ndash T4 Paraspinal muscles XOsteopathic Treatment You must treat a minimum of 2 dysfunctions you found during your examination1 Assume OA NSRRL appropriate techniques to include HVLA muscle energy articulatory Still counterstrainindirect myofascial release facilitated positional release

2 Assume C4 FSLRL appropriate techniques to include HVLA muscle energy

articulatory Still counterstrainindirect myofascial release facilitated positional release2 Assume T2-4 NSLRR appropriate techniques to include HVLA muscle energy articulatory Still counterstrainindirect myofascial release facilitated positional release

_

4 Autonomic technique for altered vagal tone OA decompression _

  • Performed
  • Performed
  • Performed
  • Performed
  • Performed
  • Performed
Page 15: Performed · Web viewAAA, kidney stones, groin hernia, ileus, kidney/bladder CA, testicular torsion, kidney/bladder infection, appendicitis, C D Osteopathic Diagnosis: assess where

2 flexion 104863510486353 extension 104863510486352 Examine the thoracic spine for dysfunction (T5-T9)A palpate transverse processes for rotation1 neutral 104863510486352 flexion 104863510486353 extension 10486351048635B palpate soft tissue for TART findings1 trapezius 104863510486352 rhomboids 104863510486353 levator scapulae 104863510486354 T5 ndash T9 Paraspinal muscles 10486351048635Osteopathic Treatment You must treat a minimum of 2 dysfunctions you found during your examination1 Assume OA NSLRR appropriate techniques to include HVLA muscle energy articulatory Still counterstrainindirect myofascial release facilitated positional release

10486351048635

2 Assume T7 FRLSL appropriate techniques to include HVLA muscle energy articulatory Still counterstrainindirect myofascial release facilitated positional release

10486351048635

3 Assume acute bilateral spasm of upper thoracic (T5-T9) paraspinal muscles appropriate techniques to include soft tissue counterstrainindirect facilitated positional release

10486351048635

4 Autonomic technique for altered vagal tone OA decompression 104863510486355 Autonomic technique for hypersympathetic tone in T5-T9 paraspinal muscles rib raising or paraspinal inhibition for gt 90 seconds

10486351048635

MODULE 4- Ernesto Rodriguez

PRETEST

1 Back pain from Colon cancer Lumbar strain Osteomyelitits Diverticulitis Back pain osteoarthritis Cauda equina syndrome

2 D3 E

Osteopathic Diagnosis assess where you would expect to find somatic dysfunctions

Performed Omitted

1 Examine the lumbar region (L1-L5) for somatic dysfunction

A palpate transverse processes for rotation

1 neutral _

2 flexion _

3 extension _

B palpate soft tissue for TART (Tenderness Asymmetry Restrictions Tissue texture change) findings

1 quadratus lumborum muscles _

2 Piriformis muscles _

3 Paraspinal muscles _

2 Examine the sacrum for somatic dysfunction (requires at least one motion test)

A depth at 4 corners of sacrum _

B ILA levelness _

C seated flexion test _

D motion at 4 corners of sacrum _

E motion in backward bending test _

F respiratory motion of sacrum assess at 4 corners _

3 Examine the pelvis for somatic dysfunction

A standing flexion test or AP compression test _

B ASIS heights _

C PSIS heights _

Osteopathic Treatment You must treat a minimum of 2 dysfunctions you found during your examination

1 Assume L5 FRLSL _

2 Assume left on left sacral torsion _

3 Assume left innominate anterior _

4 Assume spasmtender point in the left piriformis muscle appropriate techniques to include counterstrainindirect myofascial release

-

MODULE 5 ndash Connie Jones

Pretest

1 Bradycardia acute pulmonary edemaCHF pneumonia acute MI Acute pulmonary edema CHF2 A3 A4 A5 For the video this is what was done and not done

OA - nf NO E --gtNslrrno AA dx doneno cervical dx doneT1-T5 - nfe --gt t2 f sl rlparapsinals TART --gt L upper thoracic paraspinalno levator scapula no trap no rhomboids dx tx -

T2 HLVAOA decompr ndash for vagal tonerib raisin for sympth

MODULE 6 ndash Gretchen White

1 Atrial fibrillation hyperthyroidism and- thyrotoxicosis thyroid cancer hyperthryroid 2 a3 a4 a5

Osteopathic Diagnosis assess where you would expect to find somatic dysfunctions

Performed Omitted

1 Examine the cervical region for somatic dysfunction

A OA ndash leftright translation

1 neutral X

2 flexion X

6 3 extension X

7 B AA ndash rotation X

8 2 Examine the upper thoracic spine for dysfunction (T1-T5)

A palpate transverse processes for rotation

1 neutral X

2 flexion X

3 extension X

B palpate soft tissue for TART findings

1 trapezius X

2 rhomboids X

3 levator scapulae X

4 T1-T5 paraspinal muscles X

Osteopathic Treatment You must treat a minimum of 2 dysfunctions you found during your examination

1 Assume OA NSLRR appropriate techniques to include counterstrainindirect myofascial release facilitated positional release

X

2 Assume T2 FSLRL appropriate techniques to include counterstrainindirect myofascial release facilitated positional release

X

3 Assume acute spasm of left upper thoracic paraspinal muscles appropriate techniques to include counterstrainindirect myofascial release and facilitated positional release

X

4 Autonomic technique for altered vagal tone OA decompression X

5 Autonomic technique for hypersympathetic tone rib raising or paraspinal inhibition for gt 90 seconds

X

MODULE 7 ndash Emmitt Brown

1 AAA kidney stones groin hernia ileus kidneybladder CA testicular torsion kidneybladder infection appendicitis

2 C3 D

Osteopathic Diagnosis assess where you would expect to find somatic dysfunctions

Performed Omitted

1 Examine the cervical region for somatic dysfunctionA OA ndash leftright translation1 neutral X2 flexion X3 extension XB AA ndash rotation must flex at least 45deg X2 Examine the thoracic spine for dysfunction (T10-L2)A palpate transverse processes for rotation1 neutral X2 flexion X3 extension XB palpate soft tissue for TART findings4 T10 ndash L2 Paraspinal muscles X3 Examine the sacrum for somatic dysfunction (requires at least one motion test)A depth at 4 corners of sacrum XB ILA levelness XC seated flexion test XD motion at 4 corners of sacrum XE motion in backward bending test XF respiratory motion of sacrum assess at 4 corners X4 Examine the pelvis for somatic dysfunctionA standing flexion test or AP compression test XB ASIS heights XC PSIS heights XOsteopathic Treatment You must treat a minimum of 2 dysfunctions you found during your examination1 Assume OA NSLRR appropriate techniques to include HVLA muscle energy articulatory Still counterstrainindirect myofascial release facilitated positional release

X _

2 Assume T10 NRRSL appropriate techniques to include HVLA muscle energy articulatory Still counterstrainindirect myofascial release facilitated positional release

X

3 Assume acute bilateral spasm of thoracolumbar (T10-L2) paraspinal muscles RgtL appropriate techniques to include soft tissue counterstrainindirect facilitated positional release

X

4 Assume left on right sacral torsion appropriate techniques to include muscle energy counterstrainindirect myofascial release and facilitated positional release (No HVLA)

X

5 Autonomic technique for hypersympathetic tone in T10-L2 paraspinal muscles rib raising or paraspinal inhibition for gt 90 seconds

X

MODULE 8 ndash Peter Parker1 Kidney stone Psoas syndrome spondylolysis piriformis syndrome lumbar compression

fracture appendicitis pelvicabdominal CA2 C3 B

Osteopathic Diagnosis assess where you would expect to find somatic dysfunctions

Performed Omitted

1 Examine the thoracolumbar region (T10-L5) for somatic dysfunctionA palpate transverse processes for rotation1 neutral _2 flexion _3 extension _B palpate soft tissue for TART (Tenderness Asymmetry Restrictions Tissue texture change) findings1 Psoas muscles _2 Piriformis muscles _3 Paraspinal muscles _2 Examine the sacrum for somatic dysfunction (requires at least one motion test)A depth at 4 corners of sacrum _B ILA levelness _C seated flexion test _D motion at 4 corners of sacrum _E motion in backward bending test _F respiratory motion of sacrum assess at 4 corners _3 Examine the pelvis for somatic dysfunctionA standing flexion test or AP compression test _B ASIS heights _C PSIS heights _Osteopathic Treatment You must treat a minimum of 2 dysfunctions you found during your examination1 Assume L1 FRRSR appropriate techniques to include muscle energy counterstrain indirect myofascial release facilitated positional release (No HVLA)

_

2 Assume left on right sacral torsion appropriate techniques to include muscle energy counterstrainindirect myofascial release and facilitated positional release (No HVLA)

_

3 Assume spasmtender point in the right psoas muscle appropriate techniques to include counterstrainindirect myofascial release FPR ME

_

4 Assume spasmtender point in the left piriformis muscle appropriate techniques to include counterstrainindirect myofascial release FPR ME

_

MODULE 9 ndash Ingrid Bergman

1 Lumbar strain Dysmenorrhea PID lumbar strain ovarian cyst sacral somatic dysfunction2 D3 E

Osteopathic Diagnosis assess where you would expect to find somatic dysfunctions

Performed Omitted

1 Examine the lumbar region (L1-L5) for somatic dysfunctionA palpate transverse processes for rotation1 neutral _2 flexion _3 extension _B palpate soft tissue for TART (Tenderness Asymmetry Restrictions Tissue texture change) findings1 quadratus lumborum muscles _2 Piriformis muscles _3 Paraspinal muscles _2 Examine the sacrum for somatic dysfunction (requires at least one motion test)A depth at 4 corners of sacrum _B ILA levelness _C seated flexion test _D motion at 4 corners of sacrum _E motion in backward bending test _F respiratory motion of sacrum assess at 4 corners _3 Examine the pelvis for somatic dysfunctionA standing flexion test or AP compression test _B ASIS heights _C PSIS heights _Osteopathic Treatment You must treat a minimum of 2 dysfunctions you found during your examination1 Assume L3 FRLSL2 Assume left on right sacral torsion _3 Assume left superior innominate shear _

4 Autonomic technique for altered parasympathetic tone

Module 10 ndash Minnie Driver

Pretest

1 Pneumonia dissecting aortic aneurysm thoracic spine fracture MI pleurisy2 B3 E

Osteopathic Diagnosis assess where you would expect to find somatic dysfunctions

Performed Omitted

1 Examine the cervical region for somatic dysfunctionA OA ndash leftright translation1 neutral 104863510486352 flexion 104863510486353 extension 10486351048635B AA ndash rotation must flex at least 45deg 10486351048635C C2-C7 - translation1 neutral 104863510486352 flexion 104863510486353 extension 104863510486352 Examine the thoracic spine for dysfunction (T5-T9)A palpate transverse processes for rotation1 neutral 104863510486352 flexion 104863510486353 extension 10486351048635B palpate soft tissue for TART findings1 trapezius 104863510486352 rhomboids 104863510486353 levator scapulae 104863510486354 T5 ndash T9 Paraspinal muscles 104863510486353 Examine the ribs for somatic dysfunctionA Pump handle ribs (ribs 2-6) at or near sternal junction1 static 104863510486352 motion 10486351048635B Bucket handle ribs (ribs 6-10) between anterior and posterior axillary lines1 static 104863510486352 motion 10486351048635Osteopathic Treatment You must treat a minimum of 2 dysfunctions you found during your examination1 Assume T5 ESLRL 104863510486352 Assume acute bilateral spasm of upper thoracic (T4-T8) paraspinal 10486351048635

muscles3 Assume (R) Rib 2 pump handle inhalation dysfunction 104863510486354 Autonomic technique for altered vagal tone OA decompression 10486351048635

MODULE 11 ndash Jeff Weaver

1 Cervical radiculopathy thoracic outlet syndrome cubital tunnel syndrome carpal tunnel syndrome angina pectoris ulnar groove entrapment Guillian Barre

2 A3 B4 C

Osteopathic Diagnosis assess where you would expect to find somatic dysfunctions

Performed Omitted

1 Examine the cervical region for somatic dysfunctionA C2-C7 - translation1 neutral 104863510486352 flexion 104863510486353 extension 10486351048635B palpate soft tissue for TART (Tenderness Asymmetry Restrictions Tissue texture change) findings1 cervical paraspinal muscles 104863510486352 sternocleidomastoids 104863510486353 scalenes 104863510486352 Examine the thoracic spine for somatic dysfunctionA palpate transverse processes for rotation1 neutral 104863510486352 flexion 104863510486353 extension 10486351048635B palpate soft tissue for TART findings1 trapezius 104863510486352 pectoral musculature 104863510486353 T1-T4 paraspinal muscle 104863510486353 Examine the 1st rib for somatic dysfunction1 static 104863510486352 dynamic 10486351048635Osteopathic Treatment You must treat a minimum of 2 dysfunctions you found during your examination1 Assume hypertonic left pectoral muscle Appropriate techniques to include ME MFR counterstrain

10486351048635

2 Assume hypertonic left scalene muscle Appropriate techniques to include MFR counterstrain ME

10486351048635

3 Assume T4 NRLSR appropriate techniques to include HVLA muscle energy articulatory Still counterstrainindirect myofascial release facilitated positional release

10486351048635

4 Assume left sided thoracic paraspinal hypertonicity T1 ndash T4 appropriate techniques to include soft tissue deep pressure counterstrainindirect facilitated positional release

10486351048635

5 Assume left 1 st rib inhalation somatic dysfunction appropriate techniques to include HVLA ME Stillrsquos articulatory

10486351048635

MODULE 12 ndash Scarlett O-Hara

1 Osteoarthritis piriformis syndrome spinal stenosis lumbar radiculopathy DVT2 C3 D

Osteopathic Diagnosis assess where you would expect to find somatic dysfunctions

Performed Omitted

1 Examine the lumbar region (L1-L5) for somatic dysfunctionA palpate transverse processes for rotation1 neutral _2 flexion _3 extension _B palpate soft tissue for TART (Tenderness Asymmetry Restrictions Tissue texture change) findings1 Psoas muscles _2 Piriformis muscles _3 Paraspinal muscles _2 Examine the sacrum for somatic dysfunction (requires at least one motion test)A depth at 4 corners of sacrum _B ILA levelness _C seated flexion test _D motion at 4 corners of sacrum _E motion in backward bending test _F respiratory motion of sacrum assess at 4 corners _3 Examine the pelvis for somatic dysfunctionA standing flexion test or AP compression test _B ASIS heights _C PSIS heights _Osteopathic Treatment You must treat a minimum of 2 dysfunctions you found during your examination1 Assume left on right sacral torsion appropriate techniques to include muscle energy counterstrainindirect myofascial release and facilitated positional release (No HVLA)2 Assume left innominate anterior appropriate techniques to include muscle energy counterstrainindirect myofascial release and facilitated positional release (No HVLA)

_

3 Assume spasmtender point in the left piriformis muscle appropriate techniques to _

include counterstrainindirect myofascial release

MODULE 13 Clark Kent1 Asthma atypical pneumonia bronchitis upper respiratory infection pneumothorax2 B3 A

Osteopathic Diagnosis assess where you would expect to find somatic dysfunctions

Performed Omitted

1 Examine the cervical region for somatic dysfunctionA OA ndash leftright translation1 neutral _2 flexion _3 extension _B AA ndash rotation must flex at least 45deg _C C2-C7 - translation1 neutral 2 flexion _3 extension 2 Examine the thoracic spine for dysfunction (T1-T4)A palpate transverse processes for rotation1 neutral _2 flexion _3 extension _B palpate soft tissue for TART findings1 trapezius X2 rhomboids X3 levator scapulae X4 T1 ndash T4 Paraspinal muscles X3 Examine the ribs for somatic dysfunctionA Pump handle ribs (ribs 2-6) at or near sternal junction1 static X2 motion XB Bucket handle ribs (ribs 6-10) between anterior and posterior axillary lines1 static X2 motion XOsteopathic Treatment You must treat a minimum of 2 dysfunctions you found during your examination1 Assume OA NSLRR appropriate techniques to include HVLA muscle energy articulatory Still counterstrainindirect myofascial release facilitated positional release

_

2 Assume T2-4 NSLRR appropriate techniques to include HVLA muscle energy articulatory Still counterstrainindirect myofascial release facilitated positional release

_

3 Assume (L) ribs 2-4 exhalation pump handle appropriate techniques to include HVLA muscle energy articulatory Still counterstrainindirect myofascial release facilitated positional release

4 Autonomic technique for altered vagal tone OA decompression _5 Autonomic technique for hypersympathetic tone in T1-T5 paraspinal muscles rib raising or paraspinal inhibition for gt 90 seconds

X

MODULE 14 Bruce Wayne

1 Depression thyroid disorder migraine headache tension tension headache migraine2 C3 A

Osteopathic Diagnosis assess where you would expect to find somatic dysfunctions

Performed Omitted

1 Examine the cervical region for somatic dysfunctionA OA ndash leftright translation1 neutral 2 flexion 3 extension B AA ndash rotation must flex at least 45deg C C2-C7 - translation1 neutral 2 flexion 3 extension _2 Examine the thoracic spine for dysfunction (T1-T4)A palpate transverse processes for rotation1 neutral X2 flexion X3 extension XB palpate soft tissue for TART findings1 trapezius X2 T1 ndash T4 Paraspinal muscles XOsteopathic Treatment You must treat a minimum of 2 dysfunctions you found during your examination1 Assume OA NSRRL appropriate techniques to include HVLA muscle energy articulatory Still counterstrainindirect myofascial release facilitated positional release

2 Assume C4 FSLRL appropriate techniques to include HVLA muscle energy

articulatory Still counterstrainindirect myofascial release facilitated positional release2 Assume T2-4 NSLRR appropriate techniques to include HVLA muscle energy articulatory Still counterstrainindirect myofascial release facilitated positional release

_

4 Autonomic technique for altered vagal tone OA decompression _

  • Performed
  • Performed
  • Performed
  • Performed
  • Performed
  • Performed
Page 16: Performed · Web viewAAA, kidney stones, groin hernia, ileus, kidney/bladder CA, testicular torsion, kidney/bladder infection, appendicitis, C D Osteopathic Diagnosis: assess where

A palpate transverse processes for rotation

1 neutral _

2 flexion _

3 extension _

B palpate soft tissue for TART (Tenderness Asymmetry Restrictions Tissue texture change) findings

1 quadratus lumborum muscles _

2 Piriformis muscles _

3 Paraspinal muscles _

2 Examine the sacrum for somatic dysfunction (requires at least one motion test)

A depth at 4 corners of sacrum _

B ILA levelness _

C seated flexion test _

D motion at 4 corners of sacrum _

E motion in backward bending test _

F respiratory motion of sacrum assess at 4 corners _

3 Examine the pelvis for somatic dysfunction

A standing flexion test or AP compression test _

B ASIS heights _

C PSIS heights _

Osteopathic Treatment You must treat a minimum of 2 dysfunctions you found during your examination

1 Assume L5 FRLSL _

2 Assume left on left sacral torsion _

3 Assume left innominate anterior _

4 Assume spasmtender point in the left piriformis muscle appropriate techniques to include counterstrainindirect myofascial release

-

MODULE 5 ndash Connie Jones

Pretest

1 Bradycardia acute pulmonary edemaCHF pneumonia acute MI Acute pulmonary edema CHF2 A3 A4 A5 For the video this is what was done and not done

OA - nf NO E --gtNslrrno AA dx doneno cervical dx doneT1-T5 - nfe --gt t2 f sl rlparapsinals TART --gt L upper thoracic paraspinalno levator scapula no trap no rhomboids dx tx -

T2 HLVAOA decompr ndash for vagal tonerib raisin for sympth

MODULE 6 ndash Gretchen White

1 Atrial fibrillation hyperthyroidism and- thyrotoxicosis thyroid cancer hyperthryroid 2 a3 a4 a5

Osteopathic Diagnosis assess where you would expect to find somatic dysfunctions

Performed Omitted

1 Examine the cervical region for somatic dysfunction

A OA ndash leftright translation

1 neutral X

2 flexion X

6 3 extension X

7 B AA ndash rotation X

8 2 Examine the upper thoracic spine for dysfunction (T1-T5)

A palpate transverse processes for rotation

1 neutral X

2 flexion X

3 extension X

B palpate soft tissue for TART findings

1 trapezius X

2 rhomboids X

3 levator scapulae X

4 T1-T5 paraspinal muscles X

Osteopathic Treatment You must treat a minimum of 2 dysfunctions you found during your examination

1 Assume OA NSLRR appropriate techniques to include counterstrainindirect myofascial release facilitated positional release

X

2 Assume T2 FSLRL appropriate techniques to include counterstrainindirect myofascial release facilitated positional release

X

3 Assume acute spasm of left upper thoracic paraspinal muscles appropriate techniques to include counterstrainindirect myofascial release and facilitated positional release

X

4 Autonomic technique for altered vagal tone OA decompression X

5 Autonomic technique for hypersympathetic tone rib raising or paraspinal inhibition for gt 90 seconds

X

MODULE 7 ndash Emmitt Brown

1 AAA kidney stones groin hernia ileus kidneybladder CA testicular torsion kidneybladder infection appendicitis

2 C3 D

Osteopathic Diagnosis assess where you would expect to find somatic dysfunctions

Performed Omitted

1 Examine the cervical region for somatic dysfunctionA OA ndash leftright translation1 neutral X2 flexion X3 extension XB AA ndash rotation must flex at least 45deg X2 Examine the thoracic spine for dysfunction (T10-L2)A palpate transverse processes for rotation1 neutral X2 flexion X3 extension XB palpate soft tissue for TART findings4 T10 ndash L2 Paraspinal muscles X3 Examine the sacrum for somatic dysfunction (requires at least one motion test)A depth at 4 corners of sacrum XB ILA levelness XC seated flexion test XD motion at 4 corners of sacrum XE motion in backward bending test XF respiratory motion of sacrum assess at 4 corners X4 Examine the pelvis for somatic dysfunctionA standing flexion test or AP compression test XB ASIS heights XC PSIS heights XOsteopathic Treatment You must treat a minimum of 2 dysfunctions you found during your examination1 Assume OA NSLRR appropriate techniques to include HVLA muscle energy articulatory Still counterstrainindirect myofascial release facilitated positional release

X _

2 Assume T10 NRRSL appropriate techniques to include HVLA muscle energy articulatory Still counterstrainindirect myofascial release facilitated positional release

X

3 Assume acute bilateral spasm of thoracolumbar (T10-L2) paraspinal muscles RgtL appropriate techniques to include soft tissue counterstrainindirect facilitated positional release

X

4 Assume left on right sacral torsion appropriate techniques to include muscle energy counterstrainindirect myofascial release and facilitated positional release (No HVLA)

X

5 Autonomic technique for hypersympathetic tone in T10-L2 paraspinal muscles rib raising or paraspinal inhibition for gt 90 seconds

X

MODULE 8 ndash Peter Parker1 Kidney stone Psoas syndrome spondylolysis piriformis syndrome lumbar compression

fracture appendicitis pelvicabdominal CA2 C3 B

Osteopathic Diagnosis assess where you would expect to find somatic dysfunctions

Performed Omitted

1 Examine the thoracolumbar region (T10-L5) for somatic dysfunctionA palpate transverse processes for rotation1 neutral _2 flexion _3 extension _B palpate soft tissue for TART (Tenderness Asymmetry Restrictions Tissue texture change) findings1 Psoas muscles _2 Piriformis muscles _3 Paraspinal muscles _2 Examine the sacrum for somatic dysfunction (requires at least one motion test)A depth at 4 corners of sacrum _B ILA levelness _C seated flexion test _D motion at 4 corners of sacrum _E motion in backward bending test _F respiratory motion of sacrum assess at 4 corners _3 Examine the pelvis for somatic dysfunctionA standing flexion test or AP compression test _B ASIS heights _C PSIS heights _Osteopathic Treatment You must treat a minimum of 2 dysfunctions you found during your examination1 Assume L1 FRRSR appropriate techniques to include muscle energy counterstrain indirect myofascial release facilitated positional release (No HVLA)

_

2 Assume left on right sacral torsion appropriate techniques to include muscle energy counterstrainindirect myofascial release and facilitated positional release (No HVLA)

_

3 Assume spasmtender point in the right psoas muscle appropriate techniques to include counterstrainindirect myofascial release FPR ME

_

4 Assume spasmtender point in the left piriformis muscle appropriate techniques to include counterstrainindirect myofascial release FPR ME

_

MODULE 9 ndash Ingrid Bergman

1 Lumbar strain Dysmenorrhea PID lumbar strain ovarian cyst sacral somatic dysfunction2 D3 E

Osteopathic Diagnosis assess where you would expect to find somatic dysfunctions

Performed Omitted

1 Examine the lumbar region (L1-L5) for somatic dysfunctionA palpate transverse processes for rotation1 neutral _2 flexion _3 extension _B palpate soft tissue for TART (Tenderness Asymmetry Restrictions Tissue texture change) findings1 quadratus lumborum muscles _2 Piriformis muscles _3 Paraspinal muscles _2 Examine the sacrum for somatic dysfunction (requires at least one motion test)A depth at 4 corners of sacrum _B ILA levelness _C seated flexion test _D motion at 4 corners of sacrum _E motion in backward bending test _F respiratory motion of sacrum assess at 4 corners _3 Examine the pelvis for somatic dysfunctionA standing flexion test or AP compression test _B ASIS heights _C PSIS heights _Osteopathic Treatment You must treat a minimum of 2 dysfunctions you found during your examination1 Assume L3 FRLSL2 Assume left on right sacral torsion _3 Assume left superior innominate shear _

4 Autonomic technique for altered parasympathetic tone

Module 10 ndash Minnie Driver

Pretest

1 Pneumonia dissecting aortic aneurysm thoracic spine fracture MI pleurisy2 B3 E

Osteopathic Diagnosis assess where you would expect to find somatic dysfunctions

Performed Omitted

1 Examine the cervical region for somatic dysfunctionA OA ndash leftright translation1 neutral 104863510486352 flexion 104863510486353 extension 10486351048635B AA ndash rotation must flex at least 45deg 10486351048635C C2-C7 - translation1 neutral 104863510486352 flexion 104863510486353 extension 104863510486352 Examine the thoracic spine for dysfunction (T5-T9)A palpate transverse processes for rotation1 neutral 104863510486352 flexion 104863510486353 extension 10486351048635B palpate soft tissue for TART findings1 trapezius 104863510486352 rhomboids 104863510486353 levator scapulae 104863510486354 T5 ndash T9 Paraspinal muscles 104863510486353 Examine the ribs for somatic dysfunctionA Pump handle ribs (ribs 2-6) at or near sternal junction1 static 104863510486352 motion 10486351048635B Bucket handle ribs (ribs 6-10) between anterior and posterior axillary lines1 static 104863510486352 motion 10486351048635Osteopathic Treatment You must treat a minimum of 2 dysfunctions you found during your examination1 Assume T5 ESLRL 104863510486352 Assume acute bilateral spasm of upper thoracic (T4-T8) paraspinal 10486351048635

muscles3 Assume (R) Rib 2 pump handle inhalation dysfunction 104863510486354 Autonomic technique for altered vagal tone OA decompression 10486351048635

MODULE 11 ndash Jeff Weaver

1 Cervical radiculopathy thoracic outlet syndrome cubital tunnel syndrome carpal tunnel syndrome angina pectoris ulnar groove entrapment Guillian Barre

2 A3 B4 C

Osteopathic Diagnosis assess where you would expect to find somatic dysfunctions

Performed Omitted

1 Examine the cervical region for somatic dysfunctionA C2-C7 - translation1 neutral 104863510486352 flexion 104863510486353 extension 10486351048635B palpate soft tissue for TART (Tenderness Asymmetry Restrictions Tissue texture change) findings1 cervical paraspinal muscles 104863510486352 sternocleidomastoids 104863510486353 scalenes 104863510486352 Examine the thoracic spine for somatic dysfunctionA palpate transverse processes for rotation1 neutral 104863510486352 flexion 104863510486353 extension 10486351048635B palpate soft tissue for TART findings1 trapezius 104863510486352 pectoral musculature 104863510486353 T1-T4 paraspinal muscle 104863510486353 Examine the 1st rib for somatic dysfunction1 static 104863510486352 dynamic 10486351048635Osteopathic Treatment You must treat a minimum of 2 dysfunctions you found during your examination1 Assume hypertonic left pectoral muscle Appropriate techniques to include ME MFR counterstrain

10486351048635

2 Assume hypertonic left scalene muscle Appropriate techniques to include MFR counterstrain ME

10486351048635

3 Assume T4 NRLSR appropriate techniques to include HVLA muscle energy articulatory Still counterstrainindirect myofascial release facilitated positional release

10486351048635

4 Assume left sided thoracic paraspinal hypertonicity T1 ndash T4 appropriate techniques to include soft tissue deep pressure counterstrainindirect facilitated positional release

10486351048635

5 Assume left 1 st rib inhalation somatic dysfunction appropriate techniques to include HVLA ME Stillrsquos articulatory

10486351048635

MODULE 12 ndash Scarlett O-Hara

1 Osteoarthritis piriformis syndrome spinal stenosis lumbar radiculopathy DVT2 C3 D

Osteopathic Diagnosis assess where you would expect to find somatic dysfunctions

Performed Omitted

1 Examine the lumbar region (L1-L5) for somatic dysfunctionA palpate transverse processes for rotation1 neutral _2 flexion _3 extension _B palpate soft tissue for TART (Tenderness Asymmetry Restrictions Tissue texture change) findings1 Psoas muscles _2 Piriformis muscles _3 Paraspinal muscles _2 Examine the sacrum for somatic dysfunction (requires at least one motion test)A depth at 4 corners of sacrum _B ILA levelness _C seated flexion test _D motion at 4 corners of sacrum _E motion in backward bending test _F respiratory motion of sacrum assess at 4 corners _3 Examine the pelvis for somatic dysfunctionA standing flexion test or AP compression test _B ASIS heights _C PSIS heights _Osteopathic Treatment You must treat a minimum of 2 dysfunctions you found during your examination1 Assume left on right sacral torsion appropriate techniques to include muscle energy counterstrainindirect myofascial release and facilitated positional release (No HVLA)2 Assume left innominate anterior appropriate techniques to include muscle energy counterstrainindirect myofascial release and facilitated positional release (No HVLA)

_

3 Assume spasmtender point in the left piriformis muscle appropriate techniques to _

include counterstrainindirect myofascial release

MODULE 13 Clark Kent1 Asthma atypical pneumonia bronchitis upper respiratory infection pneumothorax2 B3 A

Osteopathic Diagnosis assess where you would expect to find somatic dysfunctions

Performed Omitted

1 Examine the cervical region for somatic dysfunctionA OA ndash leftright translation1 neutral _2 flexion _3 extension _B AA ndash rotation must flex at least 45deg _C C2-C7 - translation1 neutral 2 flexion _3 extension 2 Examine the thoracic spine for dysfunction (T1-T4)A palpate transverse processes for rotation1 neutral _2 flexion _3 extension _B palpate soft tissue for TART findings1 trapezius X2 rhomboids X3 levator scapulae X4 T1 ndash T4 Paraspinal muscles X3 Examine the ribs for somatic dysfunctionA Pump handle ribs (ribs 2-6) at or near sternal junction1 static X2 motion XB Bucket handle ribs (ribs 6-10) between anterior and posterior axillary lines1 static X2 motion XOsteopathic Treatment You must treat a minimum of 2 dysfunctions you found during your examination1 Assume OA NSLRR appropriate techniques to include HVLA muscle energy articulatory Still counterstrainindirect myofascial release facilitated positional release

_

2 Assume T2-4 NSLRR appropriate techniques to include HVLA muscle energy articulatory Still counterstrainindirect myofascial release facilitated positional release

_

3 Assume (L) ribs 2-4 exhalation pump handle appropriate techniques to include HVLA muscle energy articulatory Still counterstrainindirect myofascial release facilitated positional release

4 Autonomic technique for altered vagal tone OA decompression _5 Autonomic technique for hypersympathetic tone in T1-T5 paraspinal muscles rib raising or paraspinal inhibition for gt 90 seconds

X

MODULE 14 Bruce Wayne

1 Depression thyroid disorder migraine headache tension tension headache migraine2 C3 A

Osteopathic Diagnosis assess where you would expect to find somatic dysfunctions

Performed Omitted

1 Examine the cervical region for somatic dysfunctionA OA ndash leftright translation1 neutral 2 flexion 3 extension B AA ndash rotation must flex at least 45deg C C2-C7 - translation1 neutral 2 flexion 3 extension _2 Examine the thoracic spine for dysfunction (T1-T4)A palpate transverse processes for rotation1 neutral X2 flexion X3 extension XB palpate soft tissue for TART findings1 trapezius X2 T1 ndash T4 Paraspinal muscles XOsteopathic Treatment You must treat a minimum of 2 dysfunctions you found during your examination1 Assume OA NSRRL appropriate techniques to include HVLA muscle energy articulatory Still counterstrainindirect myofascial release facilitated positional release

2 Assume C4 FSLRL appropriate techniques to include HVLA muscle energy

articulatory Still counterstrainindirect myofascial release facilitated positional release2 Assume T2-4 NSLRR appropriate techniques to include HVLA muscle energy articulatory Still counterstrainindirect myofascial release facilitated positional release

_

4 Autonomic technique for altered vagal tone OA decompression _

  • Performed
  • Performed
  • Performed
  • Performed
  • Performed
  • Performed
Page 17: Performed · Web viewAAA, kidney stones, groin hernia, ileus, kidney/bladder CA, testicular torsion, kidney/bladder infection, appendicitis, C D Osteopathic Diagnosis: assess where

4 Assume spasmtender point in the left piriformis muscle appropriate techniques to include counterstrainindirect myofascial release

-

MODULE 5 ndash Connie Jones

Pretest

1 Bradycardia acute pulmonary edemaCHF pneumonia acute MI Acute pulmonary edema CHF2 A3 A4 A5 For the video this is what was done and not done

OA - nf NO E --gtNslrrno AA dx doneno cervical dx doneT1-T5 - nfe --gt t2 f sl rlparapsinals TART --gt L upper thoracic paraspinalno levator scapula no trap no rhomboids dx tx -

T2 HLVAOA decompr ndash for vagal tonerib raisin for sympth

MODULE 6 ndash Gretchen White

1 Atrial fibrillation hyperthyroidism and- thyrotoxicosis thyroid cancer hyperthryroid 2 a3 a4 a5

Osteopathic Diagnosis assess where you would expect to find somatic dysfunctions

Performed Omitted

1 Examine the cervical region for somatic dysfunction

A OA ndash leftright translation

1 neutral X

2 flexion X

6 3 extension X

7 B AA ndash rotation X

8 2 Examine the upper thoracic spine for dysfunction (T1-T5)

A palpate transverse processes for rotation

1 neutral X

2 flexion X

3 extension X

B palpate soft tissue for TART findings

1 trapezius X

2 rhomboids X

3 levator scapulae X

4 T1-T5 paraspinal muscles X

Osteopathic Treatment You must treat a minimum of 2 dysfunctions you found during your examination

1 Assume OA NSLRR appropriate techniques to include counterstrainindirect myofascial release facilitated positional release

X

2 Assume T2 FSLRL appropriate techniques to include counterstrainindirect myofascial release facilitated positional release

X

3 Assume acute spasm of left upper thoracic paraspinal muscles appropriate techniques to include counterstrainindirect myofascial release and facilitated positional release

X

4 Autonomic technique for altered vagal tone OA decompression X

5 Autonomic technique for hypersympathetic tone rib raising or paraspinal inhibition for gt 90 seconds

X

MODULE 7 ndash Emmitt Brown

1 AAA kidney stones groin hernia ileus kidneybladder CA testicular torsion kidneybladder infection appendicitis

2 C3 D

Osteopathic Diagnosis assess where you would expect to find somatic dysfunctions

Performed Omitted

1 Examine the cervical region for somatic dysfunctionA OA ndash leftright translation1 neutral X2 flexion X3 extension XB AA ndash rotation must flex at least 45deg X2 Examine the thoracic spine for dysfunction (T10-L2)A palpate transverse processes for rotation1 neutral X2 flexion X3 extension XB palpate soft tissue for TART findings4 T10 ndash L2 Paraspinal muscles X3 Examine the sacrum for somatic dysfunction (requires at least one motion test)A depth at 4 corners of sacrum XB ILA levelness XC seated flexion test XD motion at 4 corners of sacrum XE motion in backward bending test XF respiratory motion of sacrum assess at 4 corners X4 Examine the pelvis for somatic dysfunctionA standing flexion test or AP compression test XB ASIS heights XC PSIS heights XOsteopathic Treatment You must treat a minimum of 2 dysfunctions you found during your examination1 Assume OA NSLRR appropriate techniques to include HVLA muscle energy articulatory Still counterstrainindirect myofascial release facilitated positional release

X _

2 Assume T10 NRRSL appropriate techniques to include HVLA muscle energy articulatory Still counterstrainindirect myofascial release facilitated positional release

X

3 Assume acute bilateral spasm of thoracolumbar (T10-L2) paraspinal muscles RgtL appropriate techniques to include soft tissue counterstrainindirect facilitated positional release

X

4 Assume left on right sacral torsion appropriate techniques to include muscle energy counterstrainindirect myofascial release and facilitated positional release (No HVLA)

X

5 Autonomic technique for hypersympathetic tone in T10-L2 paraspinal muscles rib raising or paraspinal inhibition for gt 90 seconds

X

MODULE 8 ndash Peter Parker1 Kidney stone Psoas syndrome spondylolysis piriformis syndrome lumbar compression

fracture appendicitis pelvicabdominal CA2 C3 B

Osteopathic Diagnosis assess where you would expect to find somatic dysfunctions

Performed Omitted

1 Examine the thoracolumbar region (T10-L5) for somatic dysfunctionA palpate transverse processes for rotation1 neutral _2 flexion _3 extension _B palpate soft tissue for TART (Tenderness Asymmetry Restrictions Tissue texture change) findings1 Psoas muscles _2 Piriformis muscles _3 Paraspinal muscles _2 Examine the sacrum for somatic dysfunction (requires at least one motion test)A depth at 4 corners of sacrum _B ILA levelness _C seated flexion test _D motion at 4 corners of sacrum _E motion in backward bending test _F respiratory motion of sacrum assess at 4 corners _3 Examine the pelvis for somatic dysfunctionA standing flexion test or AP compression test _B ASIS heights _C PSIS heights _Osteopathic Treatment You must treat a minimum of 2 dysfunctions you found during your examination1 Assume L1 FRRSR appropriate techniques to include muscle energy counterstrain indirect myofascial release facilitated positional release (No HVLA)

_

2 Assume left on right sacral torsion appropriate techniques to include muscle energy counterstrainindirect myofascial release and facilitated positional release (No HVLA)

_

3 Assume spasmtender point in the right psoas muscle appropriate techniques to include counterstrainindirect myofascial release FPR ME

_

4 Assume spasmtender point in the left piriformis muscle appropriate techniques to include counterstrainindirect myofascial release FPR ME

_

MODULE 9 ndash Ingrid Bergman

1 Lumbar strain Dysmenorrhea PID lumbar strain ovarian cyst sacral somatic dysfunction2 D3 E

Osteopathic Diagnosis assess where you would expect to find somatic dysfunctions

Performed Omitted

1 Examine the lumbar region (L1-L5) for somatic dysfunctionA palpate transverse processes for rotation1 neutral _2 flexion _3 extension _B palpate soft tissue for TART (Tenderness Asymmetry Restrictions Tissue texture change) findings1 quadratus lumborum muscles _2 Piriformis muscles _3 Paraspinal muscles _2 Examine the sacrum for somatic dysfunction (requires at least one motion test)A depth at 4 corners of sacrum _B ILA levelness _C seated flexion test _D motion at 4 corners of sacrum _E motion in backward bending test _F respiratory motion of sacrum assess at 4 corners _3 Examine the pelvis for somatic dysfunctionA standing flexion test or AP compression test _B ASIS heights _C PSIS heights _Osteopathic Treatment You must treat a minimum of 2 dysfunctions you found during your examination1 Assume L3 FRLSL2 Assume left on right sacral torsion _3 Assume left superior innominate shear _

4 Autonomic technique for altered parasympathetic tone

Module 10 ndash Minnie Driver

Pretest

1 Pneumonia dissecting aortic aneurysm thoracic spine fracture MI pleurisy2 B3 E

Osteopathic Diagnosis assess where you would expect to find somatic dysfunctions

Performed Omitted

1 Examine the cervical region for somatic dysfunctionA OA ndash leftright translation1 neutral 104863510486352 flexion 104863510486353 extension 10486351048635B AA ndash rotation must flex at least 45deg 10486351048635C C2-C7 - translation1 neutral 104863510486352 flexion 104863510486353 extension 104863510486352 Examine the thoracic spine for dysfunction (T5-T9)A palpate transverse processes for rotation1 neutral 104863510486352 flexion 104863510486353 extension 10486351048635B palpate soft tissue for TART findings1 trapezius 104863510486352 rhomboids 104863510486353 levator scapulae 104863510486354 T5 ndash T9 Paraspinal muscles 104863510486353 Examine the ribs for somatic dysfunctionA Pump handle ribs (ribs 2-6) at or near sternal junction1 static 104863510486352 motion 10486351048635B Bucket handle ribs (ribs 6-10) between anterior and posterior axillary lines1 static 104863510486352 motion 10486351048635Osteopathic Treatment You must treat a minimum of 2 dysfunctions you found during your examination1 Assume T5 ESLRL 104863510486352 Assume acute bilateral spasm of upper thoracic (T4-T8) paraspinal 10486351048635

muscles3 Assume (R) Rib 2 pump handle inhalation dysfunction 104863510486354 Autonomic technique for altered vagal tone OA decompression 10486351048635

MODULE 11 ndash Jeff Weaver

1 Cervical radiculopathy thoracic outlet syndrome cubital tunnel syndrome carpal tunnel syndrome angina pectoris ulnar groove entrapment Guillian Barre

2 A3 B4 C

Osteopathic Diagnosis assess where you would expect to find somatic dysfunctions

Performed Omitted

1 Examine the cervical region for somatic dysfunctionA C2-C7 - translation1 neutral 104863510486352 flexion 104863510486353 extension 10486351048635B palpate soft tissue for TART (Tenderness Asymmetry Restrictions Tissue texture change) findings1 cervical paraspinal muscles 104863510486352 sternocleidomastoids 104863510486353 scalenes 104863510486352 Examine the thoracic spine for somatic dysfunctionA palpate transverse processes for rotation1 neutral 104863510486352 flexion 104863510486353 extension 10486351048635B palpate soft tissue for TART findings1 trapezius 104863510486352 pectoral musculature 104863510486353 T1-T4 paraspinal muscle 104863510486353 Examine the 1st rib for somatic dysfunction1 static 104863510486352 dynamic 10486351048635Osteopathic Treatment You must treat a minimum of 2 dysfunctions you found during your examination1 Assume hypertonic left pectoral muscle Appropriate techniques to include ME MFR counterstrain

10486351048635

2 Assume hypertonic left scalene muscle Appropriate techniques to include MFR counterstrain ME

10486351048635

3 Assume T4 NRLSR appropriate techniques to include HVLA muscle energy articulatory Still counterstrainindirect myofascial release facilitated positional release

10486351048635

4 Assume left sided thoracic paraspinal hypertonicity T1 ndash T4 appropriate techniques to include soft tissue deep pressure counterstrainindirect facilitated positional release

10486351048635

5 Assume left 1 st rib inhalation somatic dysfunction appropriate techniques to include HVLA ME Stillrsquos articulatory

10486351048635

MODULE 12 ndash Scarlett O-Hara

1 Osteoarthritis piriformis syndrome spinal stenosis lumbar radiculopathy DVT2 C3 D

Osteopathic Diagnosis assess where you would expect to find somatic dysfunctions

Performed Omitted

1 Examine the lumbar region (L1-L5) for somatic dysfunctionA palpate transverse processes for rotation1 neutral _2 flexion _3 extension _B palpate soft tissue for TART (Tenderness Asymmetry Restrictions Tissue texture change) findings1 Psoas muscles _2 Piriformis muscles _3 Paraspinal muscles _2 Examine the sacrum for somatic dysfunction (requires at least one motion test)A depth at 4 corners of sacrum _B ILA levelness _C seated flexion test _D motion at 4 corners of sacrum _E motion in backward bending test _F respiratory motion of sacrum assess at 4 corners _3 Examine the pelvis for somatic dysfunctionA standing flexion test or AP compression test _B ASIS heights _C PSIS heights _Osteopathic Treatment You must treat a minimum of 2 dysfunctions you found during your examination1 Assume left on right sacral torsion appropriate techniques to include muscle energy counterstrainindirect myofascial release and facilitated positional release (No HVLA)2 Assume left innominate anterior appropriate techniques to include muscle energy counterstrainindirect myofascial release and facilitated positional release (No HVLA)

_

3 Assume spasmtender point in the left piriformis muscle appropriate techniques to _

include counterstrainindirect myofascial release

MODULE 13 Clark Kent1 Asthma atypical pneumonia bronchitis upper respiratory infection pneumothorax2 B3 A

Osteopathic Diagnosis assess where you would expect to find somatic dysfunctions

Performed Omitted

1 Examine the cervical region for somatic dysfunctionA OA ndash leftright translation1 neutral _2 flexion _3 extension _B AA ndash rotation must flex at least 45deg _C C2-C7 - translation1 neutral 2 flexion _3 extension 2 Examine the thoracic spine for dysfunction (T1-T4)A palpate transverse processes for rotation1 neutral _2 flexion _3 extension _B palpate soft tissue for TART findings1 trapezius X2 rhomboids X3 levator scapulae X4 T1 ndash T4 Paraspinal muscles X3 Examine the ribs for somatic dysfunctionA Pump handle ribs (ribs 2-6) at or near sternal junction1 static X2 motion XB Bucket handle ribs (ribs 6-10) between anterior and posterior axillary lines1 static X2 motion XOsteopathic Treatment You must treat a minimum of 2 dysfunctions you found during your examination1 Assume OA NSLRR appropriate techniques to include HVLA muscle energy articulatory Still counterstrainindirect myofascial release facilitated positional release

_

2 Assume T2-4 NSLRR appropriate techniques to include HVLA muscle energy articulatory Still counterstrainindirect myofascial release facilitated positional release

_

3 Assume (L) ribs 2-4 exhalation pump handle appropriate techniques to include HVLA muscle energy articulatory Still counterstrainindirect myofascial release facilitated positional release

4 Autonomic technique for altered vagal tone OA decompression _5 Autonomic technique for hypersympathetic tone in T1-T5 paraspinal muscles rib raising or paraspinal inhibition for gt 90 seconds

X

MODULE 14 Bruce Wayne

1 Depression thyroid disorder migraine headache tension tension headache migraine2 C3 A

Osteopathic Diagnosis assess where you would expect to find somatic dysfunctions

Performed Omitted

1 Examine the cervical region for somatic dysfunctionA OA ndash leftright translation1 neutral 2 flexion 3 extension B AA ndash rotation must flex at least 45deg C C2-C7 - translation1 neutral 2 flexion 3 extension _2 Examine the thoracic spine for dysfunction (T1-T4)A palpate transverse processes for rotation1 neutral X2 flexion X3 extension XB palpate soft tissue for TART findings1 trapezius X2 T1 ndash T4 Paraspinal muscles XOsteopathic Treatment You must treat a minimum of 2 dysfunctions you found during your examination1 Assume OA NSRRL appropriate techniques to include HVLA muscle energy articulatory Still counterstrainindirect myofascial release facilitated positional release

2 Assume C4 FSLRL appropriate techniques to include HVLA muscle energy

articulatory Still counterstrainindirect myofascial release facilitated positional release2 Assume T2-4 NSLRR appropriate techniques to include HVLA muscle energy articulatory Still counterstrainindirect myofascial release facilitated positional release

_

4 Autonomic technique for altered vagal tone OA decompression _

  • Performed
  • Performed
  • Performed
  • Performed
  • Performed
  • Performed
Page 18: Performed · Web viewAAA, kidney stones, groin hernia, ileus, kidney/bladder CA, testicular torsion, kidney/bladder infection, appendicitis, C D Osteopathic Diagnosis: assess where

7 B AA ndash rotation X

8 2 Examine the upper thoracic spine for dysfunction (T1-T5)

A palpate transverse processes for rotation

1 neutral X

2 flexion X

3 extension X

B palpate soft tissue for TART findings

1 trapezius X

2 rhomboids X

3 levator scapulae X

4 T1-T5 paraspinal muscles X

Osteopathic Treatment You must treat a minimum of 2 dysfunctions you found during your examination

1 Assume OA NSLRR appropriate techniques to include counterstrainindirect myofascial release facilitated positional release

X

2 Assume T2 FSLRL appropriate techniques to include counterstrainindirect myofascial release facilitated positional release

X

3 Assume acute spasm of left upper thoracic paraspinal muscles appropriate techniques to include counterstrainindirect myofascial release and facilitated positional release

X

4 Autonomic technique for altered vagal tone OA decompression X

5 Autonomic technique for hypersympathetic tone rib raising or paraspinal inhibition for gt 90 seconds

X

MODULE 7 ndash Emmitt Brown

1 AAA kidney stones groin hernia ileus kidneybladder CA testicular torsion kidneybladder infection appendicitis

2 C3 D

Osteopathic Diagnosis assess where you would expect to find somatic dysfunctions

Performed Omitted

1 Examine the cervical region for somatic dysfunctionA OA ndash leftright translation1 neutral X2 flexion X3 extension XB AA ndash rotation must flex at least 45deg X2 Examine the thoracic spine for dysfunction (T10-L2)A palpate transverse processes for rotation1 neutral X2 flexion X3 extension XB palpate soft tissue for TART findings4 T10 ndash L2 Paraspinal muscles X3 Examine the sacrum for somatic dysfunction (requires at least one motion test)A depth at 4 corners of sacrum XB ILA levelness XC seated flexion test XD motion at 4 corners of sacrum XE motion in backward bending test XF respiratory motion of sacrum assess at 4 corners X4 Examine the pelvis for somatic dysfunctionA standing flexion test or AP compression test XB ASIS heights XC PSIS heights XOsteopathic Treatment You must treat a minimum of 2 dysfunctions you found during your examination1 Assume OA NSLRR appropriate techniques to include HVLA muscle energy articulatory Still counterstrainindirect myofascial release facilitated positional release

X _

2 Assume T10 NRRSL appropriate techniques to include HVLA muscle energy articulatory Still counterstrainindirect myofascial release facilitated positional release

X

3 Assume acute bilateral spasm of thoracolumbar (T10-L2) paraspinal muscles RgtL appropriate techniques to include soft tissue counterstrainindirect facilitated positional release

X

4 Assume left on right sacral torsion appropriate techniques to include muscle energy counterstrainindirect myofascial release and facilitated positional release (No HVLA)

X

5 Autonomic technique for hypersympathetic tone in T10-L2 paraspinal muscles rib raising or paraspinal inhibition for gt 90 seconds

X

MODULE 8 ndash Peter Parker1 Kidney stone Psoas syndrome spondylolysis piriformis syndrome lumbar compression

fracture appendicitis pelvicabdominal CA2 C3 B

Osteopathic Diagnosis assess where you would expect to find somatic dysfunctions

Performed Omitted

1 Examine the thoracolumbar region (T10-L5) for somatic dysfunctionA palpate transverse processes for rotation1 neutral _2 flexion _3 extension _B palpate soft tissue for TART (Tenderness Asymmetry Restrictions Tissue texture change) findings1 Psoas muscles _2 Piriformis muscles _3 Paraspinal muscles _2 Examine the sacrum for somatic dysfunction (requires at least one motion test)A depth at 4 corners of sacrum _B ILA levelness _C seated flexion test _D motion at 4 corners of sacrum _E motion in backward bending test _F respiratory motion of sacrum assess at 4 corners _3 Examine the pelvis for somatic dysfunctionA standing flexion test or AP compression test _B ASIS heights _C PSIS heights _Osteopathic Treatment You must treat a minimum of 2 dysfunctions you found during your examination1 Assume L1 FRRSR appropriate techniques to include muscle energy counterstrain indirect myofascial release facilitated positional release (No HVLA)

_

2 Assume left on right sacral torsion appropriate techniques to include muscle energy counterstrainindirect myofascial release and facilitated positional release (No HVLA)

_

3 Assume spasmtender point in the right psoas muscle appropriate techniques to include counterstrainindirect myofascial release FPR ME

_

4 Assume spasmtender point in the left piriformis muscle appropriate techniques to include counterstrainindirect myofascial release FPR ME

_

MODULE 9 ndash Ingrid Bergman

1 Lumbar strain Dysmenorrhea PID lumbar strain ovarian cyst sacral somatic dysfunction2 D3 E

Osteopathic Diagnosis assess where you would expect to find somatic dysfunctions

Performed Omitted

1 Examine the lumbar region (L1-L5) for somatic dysfunctionA palpate transverse processes for rotation1 neutral _2 flexion _3 extension _B palpate soft tissue for TART (Tenderness Asymmetry Restrictions Tissue texture change) findings1 quadratus lumborum muscles _2 Piriformis muscles _3 Paraspinal muscles _2 Examine the sacrum for somatic dysfunction (requires at least one motion test)A depth at 4 corners of sacrum _B ILA levelness _C seated flexion test _D motion at 4 corners of sacrum _E motion in backward bending test _F respiratory motion of sacrum assess at 4 corners _3 Examine the pelvis for somatic dysfunctionA standing flexion test or AP compression test _B ASIS heights _C PSIS heights _Osteopathic Treatment You must treat a minimum of 2 dysfunctions you found during your examination1 Assume L3 FRLSL2 Assume left on right sacral torsion _3 Assume left superior innominate shear _

4 Autonomic technique for altered parasympathetic tone

Module 10 ndash Minnie Driver

Pretest

1 Pneumonia dissecting aortic aneurysm thoracic spine fracture MI pleurisy2 B3 E

Osteopathic Diagnosis assess where you would expect to find somatic dysfunctions

Performed Omitted

1 Examine the cervical region for somatic dysfunctionA OA ndash leftright translation1 neutral 104863510486352 flexion 104863510486353 extension 10486351048635B AA ndash rotation must flex at least 45deg 10486351048635C C2-C7 - translation1 neutral 104863510486352 flexion 104863510486353 extension 104863510486352 Examine the thoracic spine for dysfunction (T5-T9)A palpate transverse processes for rotation1 neutral 104863510486352 flexion 104863510486353 extension 10486351048635B palpate soft tissue for TART findings1 trapezius 104863510486352 rhomboids 104863510486353 levator scapulae 104863510486354 T5 ndash T9 Paraspinal muscles 104863510486353 Examine the ribs for somatic dysfunctionA Pump handle ribs (ribs 2-6) at or near sternal junction1 static 104863510486352 motion 10486351048635B Bucket handle ribs (ribs 6-10) between anterior and posterior axillary lines1 static 104863510486352 motion 10486351048635Osteopathic Treatment You must treat a minimum of 2 dysfunctions you found during your examination1 Assume T5 ESLRL 104863510486352 Assume acute bilateral spasm of upper thoracic (T4-T8) paraspinal 10486351048635

muscles3 Assume (R) Rib 2 pump handle inhalation dysfunction 104863510486354 Autonomic technique for altered vagal tone OA decompression 10486351048635

MODULE 11 ndash Jeff Weaver

1 Cervical radiculopathy thoracic outlet syndrome cubital tunnel syndrome carpal tunnel syndrome angina pectoris ulnar groove entrapment Guillian Barre

2 A3 B4 C

Osteopathic Diagnosis assess where you would expect to find somatic dysfunctions

Performed Omitted

1 Examine the cervical region for somatic dysfunctionA C2-C7 - translation1 neutral 104863510486352 flexion 104863510486353 extension 10486351048635B palpate soft tissue for TART (Tenderness Asymmetry Restrictions Tissue texture change) findings1 cervical paraspinal muscles 104863510486352 sternocleidomastoids 104863510486353 scalenes 104863510486352 Examine the thoracic spine for somatic dysfunctionA palpate transverse processes for rotation1 neutral 104863510486352 flexion 104863510486353 extension 10486351048635B palpate soft tissue for TART findings1 trapezius 104863510486352 pectoral musculature 104863510486353 T1-T4 paraspinal muscle 104863510486353 Examine the 1st rib for somatic dysfunction1 static 104863510486352 dynamic 10486351048635Osteopathic Treatment You must treat a minimum of 2 dysfunctions you found during your examination1 Assume hypertonic left pectoral muscle Appropriate techniques to include ME MFR counterstrain

10486351048635

2 Assume hypertonic left scalene muscle Appropriate techniques to include MFR counterstrain ME

10486351048635

3 Assume T4 NRLSR appropriate techniques to include HVLA muscle energy articulatory Still counterstrainindirect myofascial release facilitated positional release

10486351048635

4 Assume left sided thoracic paraspinal hypertonicity T1 ndash T4 appropriate techniques to include soft tissue deep pressure counterstrainindirect facilitated positional release

10486351048635

5 Assume left 1 st rib inhalation somatic dysfunction appropriate techniques to include HVLA ME Stillrsquos articulatory

10486351048635

MODULE 12 ndash Scarlett O-Hara

1 Osteoarthritis piriformis syndrome spinal stenosis lumbar radiculopathy DVT2 C3 D

Osteopathic Diagnosis assess where you would expect to find somatic dysfunctions

Performed Omitted

1 Examine the lumbar region (L1-L5) for somatic dysfunctionA palpate transverse processes for rotation1 neutral _2 flexion _3 extension _B palpate soft tissue for TART (Tenderness Asymmetry Restrictions Tissue texture change) findings1 Psoas muscles _2 Piriformis muscles _3 Paraspinal muscles _2 Examine the sacrum for somatic dysfunction (requires at least one motion test)A depth at 4 corners of sacrum _B ILA levelness _C seated flexion test _D motion at 4 corners of sacrum _E motion in backward bending test _F respiratory motion of sacrum assess at 4 corners _3 Examine the pelvis for somatic dysfunctionA standing flexion test or AP compression test _B ASIS heights _C PSIS heights _Osteopathic Treatment You must treat a minimum of 2 dysfunctions you found during your examination1 Assume left on right sacral torsion appropriate techniques to include muscle energy counterstrainindirect myofascial release and facilitated positional release (No HVLA)2 Assume left innominate anterior appropriate techniques to include muscle energy counterstrainindirect myofascial release and facilitated positional release (No HVLA)

_

3 Assume spasmtender point in the left piriformis muscle appropriate techniques to _

include counterstrainindirect myofascial release

MODULE 13 Clark Kent1 Asthma atypical pneumonia bronchitis upper respiratory infection pneumothorax2 B3 A

Osteopathic Diagnosis assess where you would expect to find somatic dysfunctions

Performed Omitted

1 Examine the cervical region for somatic dysfunctionA OA ndash leftright translation1 neutral _2 flexion _3 extension _B AA ndash rotation must flex at least 45deg _C C2-C7 - translation1 neutral 2 flexion _3 extension 2 Examine the thoracic spine for dysfunction (T1-T4)A palpate transverse processes for rotation1 neutral _2 flexion _3 extension _B palpate soft tissue for TART findings1 trapezius X2 rhomboids X3 levator scapulae X4 T1 ndash T4 Paraspinal muscles X3 Examine the ribs for somatic dysfunctionA Pump handle ribs (ribs 2-6) at or near sternal junction1 static X2 motion XB Bucket handle ribs (ribs 6-10) between anterior and posterior axillary lines1 static X2 motion XOsteopathic Treatment You must treat a minimum of 2 dysfunctions you found during your examination1 Assume OA NSLRR appropriate techniques to include HVLA muscle energy articulatory Still counterstrainindirect myofascial release facilitated positional release

_

2 Assume T2-4 NSLRR appropriate techniques to include HVLA muscle energy articulatory Still counterstrainindirect myofascial release facilitated positional release

_

3 Assume (L) ribs 2-4 exhalation pump handle appropriate techniques to include HVLA muscle energy articulatory Still counterstrainindirect myofascial release facilitated positional release

4 Autonomic technique for altered vagal tone OA decompression _5 Autonomic technique for hypersympathetic tone in T1-T5 paraspinal muscles rib raising or paraspinal inhibition for gt 90 seconds

X

MODULE 14 Bruce Wayne

1 Depression thyroid disorder migraine headache tension tension headache migraine2 C3 A

Osteopathic Diagnosis assess where you would expect to find somatic dysfunctions

Performed Omitted

1 Examine the cervical region for somatic dysfunctionA OA ndash leftright translation1 neutral 2 flexion 3 extension B AA ndash rotation must flex at least 45deg C C2-C7 - translation1 neutral 2 flexion 3 extension _2 Examine the thoracic spine for dysfunction (T1-T4)A palpate transverse processes for rotation1 neutral X2 flexion X3 extension XB palpate soft tissue for TART findings1 trapezius X2 T1 ndash T4 Paraspinal muscles XOsteopathic Treatment You must treat a minimum of 2 dysfunctions you found during your examination1 Assume OA NSRRL appropriate techniques to include HVLA muscle energy articulatory Still counterstrainindirect myofascial release facilitated positional release

2 Assume C4 FSLRL appropriate techniques to include HVLA muscle energy

articulatory Still counterstrainindirect myofascial release facilitated positional release2 Assume T2-4 NSLRR appropriate techniques to include HVLA muscle energy articulatory Still counterstrainindirect myofascial release facilitated positional release

_

4 Autonomic technique for altered vagal tone OA decompression _

  • Performed
  • Performed
  • Performed
  • Performed
  • Performed
  • Performed
Page 19: Performed · Web viewAAA, kidney stones, groin hernia, ileus, kidney/bladder CA, testicular torsion, kidney/bladder infection, appendicitis, C D Osteopathic Diagnosis: assess where

2 C3 D

Osteopathic Diagnosis assess where you would expect to find somatic dysfunctions

Performed Omitted

1 Examine the cervical region for somatic dysfunctionA OA ndash leftright translation1 neutral X2 flexion X3 extension XB AA ndash rotation must flex at least 45deg X2 Examine the thoracic spine for dysfunction (T10-L2)A palpate transverse processes for rotation1 neutral X2 flexion X3 extension XB palpate soft tissue for TART findings4 T10 ndash L2 Paraspinal muscles X3 Examine the sacrum for somatic dysfunction (requires at least one motion test)A depth at 4 corners of sacrum XB ILA levelness XC seated flexion test XD motion at 4 corners of sacrum XE motion in backward bending test XF respiratory motion of sacrum assess at 4 corners X4 Examine the pelvis for somatic dysfunctionA standing flexion test or AP compression test XB ASIS heights XC PSIS heights XOsteopathic Treatment You must treat a minimum of 2 dysfunctions you found during your examination1 Assume OA NSLRR appropriate techniques to include HVLA muscle energy articulatory Still counterstrainindirect myofascial release facilitated positional release

X _

2 Assume T10 NRRSL appropriate techniques to include HVLA muscle energy articulatory Still counterstrainindirect myofascial release facilitated positional release

X

3 Assume acute bilateral spasm of thoracolumbar (T10-L2) paraspinal muscles RgtL appropriate techniques to include soft tissue counterstrainindirect facilitated positional release

X

4 Assume left on right sacral torsion appropriate techniques to include muscle energy counterstrainindirect myofascial release and facilitated positional release (No HVLA)

X

5 Autonomic technique for hypersympathetic tone in T10-L2 paraspinal muscles rib raising or paraspinal inhibition for gt 90 seconds

X

MODULE 8 ndash Peter Parker1 Kidney stone Psoas syndrome spondylolysis piriformis syndrome lumbar compression

fracture appendicitis pelvicabdominal CA2 C3 B

Osteopathic Diagnosis assess where you would expect to find somatic dysfunctions

Performed Omitted

1 Examine the thoracolumbar region (T10-L5) for somatic dysfunctionA palpate transverse processes for rotation1 neutral _2 flexion _3 extension _B palpate soft tissue for TART (Tenderness Asymmetry Restrictions Tissue texture change) findings1 Psoas muscles _2 Piriformis muscles _3 Paraspinal muscles _2 Examine the sacrum for somatic dysfunction (requires at least one motion test)A depth at 4 corners of sacrum _B ILA levelness _C seated flexion test _D motion at 4 corners of sacrum _E motion in backward bending test _F respiratory motion of sacrum assess at 4 corners _3 Examine the pelvis for somatic dysfunctionA standing flexion test or AP compression test _B ASIS heights _C PSIS heights _Osteopathic Treatment You must treat a minimum of 2 dysfunctions you found during your examination1 Assume L1 FRRSR appropriate techniques to include muscle energy counterstrain indirect myofascial release facilitated positional release (No HVLA)

_

2 Assume left on right sacral torsion appropriate techniques to include muscle energy counterstrainindirect myofascial release and facilitated positional release (No HVLA)

_

3 Assume spasmtender point in the right psoas muscle appropriate techniques to include counterstrainindirect myofascial release FPR ME

_

4 Assume spasmtender point in the left piriformis muscle appropriate techniques to include counterstrainindirect myofascial release FPR ME

_

MODULE 9 ndash Ingrid Bergman

1 Lumbar strain Dysmenorrhea PID lumbar strain ovarian cyst sacral somatic dysfunction2 D3 E

Osteopathic Diagnosis assess where you would expect to find somatic dysfunctions

Performed Omitted

1 Examine the lumbar region (L1-L5) for somatic dysfunctionA palpate transverse processes for rotation1 neutral _2 flexion _3 extension _B palpate soft tissue for TART (Tenderness Asymmetry Restrictions Tissue texture change) findings1 quadratus lumborum muscles _2 Piriformis muscles _3 Paraspinal muscles _2 Examine the sacrum for somatic dysfunction (requires at least one motion test)A depth at 4 corners of sacrum _B ILA levelness _C seated flexion test _D motion at 4 corners of sacrum _E motion in backward bending test _F respiratory motion of sacrum assess at 4 corners _3 Examine the pelvis for somatic dysfunctionA standing flexion test or AP compression test _B ASIS heights _C PSIS heights _Osteopathic Treatment You must treat a minimum of 2 dysfunctions you found during your examination1 Assume L3 FRLSL2 Assume left on right sacral torsion _3 Assume left superior innominate shear _

4 Autonomic technique for altered parasympathetic tone

Module 10 ndash Minnie Driver

Pretest

1 Pneumonia dissecting aortic aneurysm thoracic spine fracture MI pleurisy2 B3 E

Osteopathic Diagnosis assess where you would expect to find somatic dysfunctions

Performed Omitted

1 Examine the cervical region for somatic dysfunctionA OA ndash leftright translation1 neutral 104863510486352 flexion 104863510486353 extension 10486351048635B AA ndash rotation must flex at least 45deg 10486351048635C C2-C7 - translation1 neutral 104863510486352 flexion 104863510486353 extension 104863510486352 Examine the thoracic spine for dysfunction (T5-T9)A palpate transverse processes for rotation1 neutral 104863510486352 flexion 104863510486353 extension 10486351048635B palpate soft tissue for TART findings1 trapezius 104863510486352 rhomboids 104863510486353 levator scapulae 104863510486354 T5 ndash T9 Paraspinal muscles 104863510486353 Examine the ribs for somatic dysfunctionA Pump handle ribs (ribs 2-6) at or near sternal junction1 static 104863510486352 motion 10486351048635B Bucket handle ribs (ribs 6-10) between anterior and posterior axillary lines1 static 104863510486352 motion 10486351048635Osteopathic Treatment You must treat a minimum of 2 dysfunctions you found during your examination1 Assume T5 ESLRL 104863510486352 Assume acute bilateral spasm of upper thoracic (T4-T8) paraspinal 10486351048635

muscles3 Assume (R) Rib 2 pump handle inhalation dysfunction 104863510486354 Autonomic technique for altered vagal tone OA decompression 10486351048635

MODULE 11 ndash Jeff Weaver

1 Cervical radiculopathy thoracic outlet syndrome cubital tunnel syndrome carpal tunnel syndrome angina pectoris ulnar groove entrapment Guillian Barre

2 A3 B4 C

Osteopathic Diagnosis assess where you would expect to find somatic dysfunctions

Performed Omitted

1 Examine the cervical region for somatic dysfunctionA C2-C7 - translation1 neutral 104863510486352 flexion 104863510486353 extension 10486351048635B palpate soft tissue for TART (Tenderness Asymmetry Restrictions Tissue texture change) findings1 cervical paraspinal muscles 104863510486352 sternocleidomastoids 104863510486353 scalenes 104863510486352 Examine the thoracic spine for somatic dysfunctionA palpate transverse processes for rotation1 neutral 104863510486352 flexion 104863510486353 extension 10486351048635B palpate soft tissue for TART findings1 trapezius 104863510486352 pectoral musculature 104863510486353 T1-T4 paraspinal muscle 104863510486353 Examine the 1st rib for somatic dysfunction1 static 104863510486352 dynamic 10486351048635Osteopathic Treatment You must treat a minimum of 2 dysfunctions you found during your examination1 Assume hypertonic left pectoral muscle Appropriate techniques to include ME MFR counterstrain

10486351048635

2 Assume hypertonic left scalene muscle Appropriate techniques to include MFR counterstrain ME

10486351048635

3 Assume T4 NRLSR appropriate techniques to include HVLA muscle energy articulatory Still counterstrainindirect myofascial release facilitated positional release

10486351048635

4 Assume left sided thoracic paraspinal hypertonicity T1 ndash T4 appropriate techniques to include soft tissue deep pressure counterstrainindirect facilitated positional release

10486351048635

5 Assume left 1 st rib inhalation somatic dysfunction appropriate techniques to include HVLA ME Stillrsquos articulatory

10486351048635

MODULE 12 ndash Scarlett O-Hara

1 Osteoarthritis piriformis syndrome spinal stenosis lumbar radiculopathy DVT2 C3 D

Osteopathic Diagnosis assess where you would expect to find somatic dysfunctions

Performed Omitted

1 Examine the lumbar region (L1-L5) for somatic dysfunctionA palpate transverse processes for rotation1 neutral _2 flexion _3 extension _B palpate soft tissue for TART (Tenderness Asymmetry Restrictions Tissue texture change) findings1 Psoas muscles _2 Piriformis muscles _3 Paraspinal muscles _2 Examine the sacrum for somatic dysfunction (requires at least one motion test)A depth at 4 corners of sacrum _B ILA levelness _C seated flexion test _D motion at 4 corners of sacrum _E motion in backward bending test _F respiratory motion of sacrum assess at 4 corners _3 Examine the pelvis for somatic dysfunctionA standing flexion test or AP compression test _B ASIS heights _C PSIS heights _Osteopathic Treatment You must treat a minimum of 2 dysfunctions you found during your examination1 Assume left on right sacral torsion appropriate techniques to include muscle energy counterstrainindirect myofascial release and facilitated positional release (No HVLA)2 Assume left innominate anterior appropriate techniques to include muscle energy counterstrainindirect myofascial release and facilitated positional release (No HVLA)

_

3 Assume spasmtender point in the left piriformis muscle appropriate techniques to _

include counterstrainindirect myofascial release

MODULE 13 Clark Kent1 Asthma atypical pneumonia bronchitis upper respiratory infection pneumothorax2 B3 A

Osteopathic Diagnosis assess where you would expect to find somatic dysfunctions

Performed Omitted

1 Examine the cervical region for somatic dysfunctionA OA ndash leftright translation1 neutral _2 flexion _3 extension _B AA ndash rotation must flex at least 45deg _C C2-C7 - translation1 neutral 2 flexion _3 extension 2 Examine the thoracic spine for dysfunction (T1-T4)A palpate transverse processes for rotation1 neutral _2 flexion _3 extension _B palpate soft tissue for TART findings1 trapezius X2 rhomboids X3 levator scapulae X4 T1 ndash T4 Paraspinal muscles X3 Examine the ribs for somatic dysfunctionA Pump handle ribs (ribs 2-6) at or near sternal junction1 static X2 motion XB Bucket handle ribs (ribs 6-10) between anterior and posterior axillary lines1 static X2 motion XOsteopathic Treatment You must treat a minimum of 2 dysfunctions you found during your examination1 Assume OA NSLRR appropriate techniques to include HVLA muscle energy articulatory Still counterstrainindirect myofascial release facilitated positional release

_

2 Assume T2-4 NSLRR appropriate techniques to include HVLA muscle energy articulatory Still counterstrainindirect myofascial release facilitated positional release

_

3 Assume (L) ribs 2-4 exhalation pump handle appropriate techniques to include HVLA muscle energy articulatory Still counterstrainindirect myofascial release facilitated positional release

4 Autonomic technique for altered vagal tone OA decompression _5 Autonomic technique for hypersympathetic tone in T1-T5 paraspinal muscles rib raising or paraspinal inhibition for gt 90 seconds

X

MODULE 14 Bruce Wayne

1 Depression thyroid disorder migraine headache tension tension headache migraine2 C3 A

Osteopathic Diagnosis assess where you would expect to find somatic dysfunctions

Performed Omitted

1 Examine the cervical region for somatic dysfunctionA OA ndash leftright translation1 neutral 2 flexion 3 extension B AA ndash rotation must flex at least 45deg C C2-C7 - translation1 neutral 2 flexion 3 extension _2 Examine the thoracic spine for dysfunction (T1-T4)A palpate transverse processes for rotation1 neutral X2 flexion X3 extension XB palpate soft tissue for TART findings1 trapezius X2 T1 ndash T4 Paraspinal muscles XOsteopathic Treatment You must treat a minimum of 2 dysfunctions you found during your examination1 Assume OA NSRRL appropriate techniques to include HVLA muscle energy articulatory Still counterstrainindirect myofascial release facilitated positional release

2 Assume C4 FSLRL appropriate techniques to include HVLA muscle energy

articulatory Still counterstrainindirect myofascial release facilitated positional release2 Assume T2-4 NSLRR appropriate techniques to include HVLA muscle energy articulatory Still counterstrainindirect myofascial release facilitated positional release

_

4 Autonomic technique for altered vagal tone OA decompression _

  • Performed
  • Performed
  • Performed
  • Performed
  • Performed
  • Performed
Page 20: Performed · Web viewAAA, kidney stones, groin hernia, ileus, kidney/bladder CA, testicular torsion, kidney/bladder infection, appendicitis, C D Osteopathic Diagnosis: assess where

4 Assume left on right sacral torsion appropriate techniques to include muscle energy counterstrainindirect myofascial release and facilitated positional release (No HVLA)

X

5 Autonomic technique for hypersympathetic tone in T10-L2 paraspinal muscles rib raising or paraspinal inhibition for gt 90 seconds

X

MODULE 8 ndash Peter Parker1 Kidney stone Psoas syndrome spondylolysis piriformis syndrome lumbar compression

fracture appendicitis pelvicabdominal CA2 C3 B

Osteopathic Diagnosis assess where you would expect to find somatic dysfunctions

Performed Omitted

1 Examine the thoracolumbar region (T10-L5) for somatic dysfunctionA palpate transverse processes for rotation1 neutral _2 flexion _3 extension _B palpate soft tissue for TART (Tenderness Asymmetry Restrictions Tissue texture change) findings1 Psoas muscles _2 Piriformis muscles _3 Paraspinal muscles _2 Examine the sacrum for somatic dysfunction (requires at least one motion test)A depth at 4 corners of sacrum _B ILA levelness _C seated flexion test _D motion at 4 corners of sacrum _E motion in backward bending test _F respiratory motion of sacrum assess at 4 corners _3 Examine the pelvis for somatic dysfunctionA standing flexion test or AP compression test _B ASIS heights _C PSIS heights _Osteopathic Treatment You must treat a minimum of 2 dysfunctions you found during your examination1 Assume L1 FRRSR appropriate techniques to include muscle energy counterstrain indirect myofascial release facilitated positional release (No HVLA)

_

2 Assume left on right sacral torsion appropriate techniques to include muscle energy counterstrainindirect myofascial release and facilitated positional release (No HVLA)

_

3 Assume spasmtender point in the right psoas muscle appropriate techniques to include counterstrainindirect myofascial release FPR ME

_

4 Assume spasmtender point in the left piriformis muscle appropriate techniques to include counterstrainindirect myofascial release FPR ME

_

MODULE 9 ndash Ingrid Bergman

1 Lumbar strain Dysmenorrhea PID lumbar strain ovarian cyst sacral somatic dysfunction2 D3 E

Osteopathic Diagnosis assess where you would expect to find somatic dysfunctions

Performed Omitted

1 Examine the lumbar region (L1-L5) for somatic dysfunctionA palpate transverse processes for rotation1 neutral _2 flexion _3 extension _B palpate soft tissue for TART (Tenderness Asymmetry Restrictions Tissue texture change) findings1 quadratus lumborum muscles _2 Piriformis muscles _3 Paraspinal muscles _2 Examine the sacrum for somatic dysfunction (requires at least one motion test)A depth at 4 corners of sacrum _B ILA levelness _C seated flexion test _D motion at 4 corners of sacrum _E motion in backward bending test _F respiratory motion of sacrum assess at 4 corners _3 Examine the pelvis for somatic dysfunctionA standing flexion test or AP compression test _B ASIS heights _C PSIS heights _Osteopathic Treatment You must treat a minimum of 2 dysfunctions you found during your examination1 Assume L3 FRLSL2 Assume left on right sacral torsion _3 Assume left superior innominate shear _

4 Autonomic technique for altered parasympathetic tone

Module 10 ndash Minnie Driver

Pretest

1 Pneumonia dissecting aortic aneurysm thoracic spine fracture MI pleurisy2 B3 E

Osteopathic Diagnosis assess where you would expect to find somatic dysfunctions

Performed Omitted

1 Examine the cervical region for somatic dysfunctionA OA ndash leftright translation1 neutral 104863510486352 flexion 104863510486353 extension 10486351048635B AA ndash rotation must flex at least 45deg 10486351048635C C2-C7 - translation1 neutral 104863510486352 flexion 104863510486353 extension 104863510486352 Examine the thoracic spine for dysfunction (T5-T9)A palpate transverse processes for rotation1 neutral 104863510486352 flexion 104863510486353 extension 10486351048635B palpate soft tissue for TART findings1 trapezius 104863510486352 rhomboids 104863510486353 levator scapulae 104863510486354 T5 ndash T9 Paraspinal muscles 104863510486353 Examine the ribs for somatic dysfunctionA Pump handle ribs (ribs 2-6) at or near sternal junction1 static 104863510486352 motion 10486351048635B Bucket handle ribs (ribs 6-10) between anterior and posterior axillary lines1 static 104863510486352 motion 10486351048635Osteopathic Treatment You must treat a minimum of 2 dysfunctions you found during your examination1 Assume T5 ESLRL 104863510486352 Assume acute bilateral spasm of upper thoracic (T4-T8) paraspinal 10486351048635

muscles3 Assume (R) Rib 2 pump handle inhalation dysfunction 104863510486354 Autonomic technique for altered vagal tone OA decompression 10486351048635

MODULE 11 ndash Jeff Weaver

1 Cervical radiculopathy thoracic outlet syndrome cubital tunnel syndrome carpal tunnel syndrome angina pectoris ulnar groove entrapment Guillian Barre

2 A3 B4 C

Osteopathic Diagnosis assess where you would expect to find somatic dysfunctions

Performed Omitted

1 Examine the cervical region for somatic dysfunctionA C2-C7 - translation1 neutral 104863510486352 flexion 104863510486353 extension 10486351048635B palpate soft tissue for TART (Tenderness Asymmetry Restrictions Tissue texture change) findings1 cervical paraspinal muscles 104863510486352 sternocleidomastoids 104863510486353 scalenes 104863510486352 Examine the thoracic spine for somatic dysfunctionA palpate transverse processes for rotation1 neutral 104863510486352 flexion 104863510486353 extension 10486351048635B palpate soft tissue for TART findings1 trapezius 104863510486352 pectoral musculature 104863510486353 T1-T4 paraspinal muscle 104863510486353 Examine the 1st rib for somatic dysfunction1 static 104863510486352 dynamic 10486351048635Osteopathic Treatment You must treat a minimum of 2 dysfunctions you found during your examination1 Assume hypertonic left pectoral muscle Appropriate techniques to include ME MFR counterstrain

10486351048635

2 Assume hypertonic left scalene muscle Appropriate techniques to include MFR counterstrain ME

10486351048635

3 Assume T4 NRLSR appropriate techniques to include HVLA muscle energy articulatory Still counterstrainindirect myofascial release facilitated positional release

10486351048635

4 Assume left sided thoracic paraspinal hypertonicity T1 ndash T4 appropriate techniques to include soft tissue deep pressure counterstrainindirect facilitated positional release

10486351048635

5 Assume left 1 st rib inhalation somatic dysfunction appropriate techniques to include HVLA ME Stillrsquos articulatory

10486351048635

MODULE 12 ndash Scarlett O-Hara

1 Osteoarthritis piriformis syndrome spinal stenosis lumbar radiculopathy DVT2 C3 D

Osteopathic Diagnosis assess where you would expect to find somatic dysfunctions

Performed Omitted

1 Examine the lumbar region (L1-L5) for somatic dysfunctionA palpate transverse processes for rotation1 neutral _2 flexion _3 extension _B palpate soft tissue for TART (Tenderness Asymmetry Restrictions Tissue texture change) findings1 Psoas muscles _2 Piriformis muscles _3 Paraspinal muscles _2 Examine the sacrum for somatic dysfunction (requires at least one motion test)A depth at 4 corners of sacrum _B ILA levelness _C seated flexion test _D motion at 4 corners of sacrum _E motion in backward bending test _F respiratory motion of sacrum assess at 4 corners _3 Examine the pelvis for somatic dysfunctionA standing flexion test or AP compression test _B ASIS heights _C PSIS heights _Osteopathic Treatment You must treat a minimum of 2 dysfunctions you found during your examination1 Assume left on right sacral torsion appropriate techniques to include muscle energy counterstrainindirect myofascial release and facilitated positional release (No HVLA)2 Assume left innominate anterior appropriate techniques to include muscle energy counterstrainindirect myofascial release and facilitated positional release (No HVLA)

_

3 Assume spasmtender point in the left piriformis muscle appropriate techniques to _

include counterstrainindirect myofascial release

MODULE 13 Clark Kent1 Asthma atypical pneumonia bronchitis upper respiratory infection pneumothorax2 B3 A

Osteopathic Diagnosis assess where you would expect to find somatic dysfunctions

Performed Omitted

1 Examine the cervical region for somatic dysfunctionA OA ndash leftright translation1 neutral _2 flexion _3 extension _B AA ndash rotation must flex at least 45deg _C C2-C7 - translation1 neutral 2 flexion _3 extension 2 Examine the thoracic spine for dysfunction (T1-T4)A palpate transverse processes for rotation1 neutral _2 flexion _3 extension _B palpate soft tissue for TART findings1 trapezius X2 rhomboids X3 levator scapulae X4 T1 ndash T4 Paraspinal muscles X3 Examine the ribs for somatic dysfunctionA Pump handle ribs (ribs 2-6) at or near sternal junction1 static X2 motion XB Bucket handle ribs (ribs 6-10) between anterior and posterior axillary lines1 static X2 motion XOsteopathic Treatment You must treat a minimum of 2 dysfunctions you found during your examination1 Assume OA NSLRR appropriate techniques to include HVLA muscle energy articulatory Still counterstrainindirect myofascial release facilitated positional release

_

2 Assume T2-4 NSLRR appropriate techniques to include HVLA muscle energy articulatory Still counterstrainindirect myofascial release facilitated positional release

_

3 Assume (L) ribs 2-4 exhalation pump handle appropriate techniques to include HVLA muscle energy articulatory Still counterstrainindirect myofascial release facilitated positional release

4 Autonomic technique for altered vagal tone OA decompression _5 Autonomic technique for hypersympathetic tone in T1-T5 paraspinal muscles rib raising or paraspinal inhibition for gt 90 seconds

X

MODULE 14 Bruce Wayne

1 Depression thyroid disorder migraine headache tension tension headache migraine2 C3 A

Osteopathic Diagnosis assess where you would expect to find somatic dysfunctions

Performed Omitted

1 Examine the cervical region for somatic dysfunctionA OA ndash leftright translation1 neutral 2 flexion 3 extension B AA ndash rotation must flex at least 45deg C C2-C7 - translation1 neutral 2 flexion 3 extension _2 Examine the thoracic spine for dysfunction (T1-T4)A palpate transverse processes for rotation1 neutral X2 flexion X3 extension XB palpate soft tissue for TART findings1 trapezius X2 T1 ndash T4 Paraspinal muscles XOsteopathic Treatment You must treat a minimum of 2 dysfunctions you found during your examination1 Assume OA NSRRL appropriate techniques to include HVLA muscle energy articulatory Still counterstrainindirect myofascial release facilitated positional release

2 Assume C4 FSLRL appropriate techniques to include HVLA muscle energy

articulatory Still counterstrainindirect myofascial release facilitated positional release2 Assume T2-4 NSLRR appropriate techniques to include HVLA muscle energy articulatory Still counterstrainindirect myofascial release facilitated positional release

_

4 Autonomic technique for altered vagal tone OA decompression _

  • Performed
  • Performed
  • Performed
  • Performed
  • Performed
  • Performed
Page 21: Performed · Web viewAAA, kidney stones, groin hernia, ileus, kidney/bladder CA, testicular torsion, kidney/bladder infection, appendicitis, C D Osteopathic Diagnosis: assess where

2 Assume left on right sacral torsion appropriate techniques to include muscle energy counterstrainindirect myofascial release and facilitated positional release (No HVLA)

_

3 Assume spasmtender point in the right psoas muscle appropriate techniques to include counterstrainindirect myofascial release FPR ME

_

4 Assume spasmtender point in the left piriformis muscle appropriate techniques to include counterstrainindirect myofascial release FPR ME

_

MODULE 9 ndash Ingrid Bergman

1 Lumbar strain Dysmenorrhea PID lumbar strain ovarian cyst sacral somatic dysfunction2 D3 E

Osteopathic Diagnosis assess where you would expect to find somatic dysfunctions

Performed Omitted

1 Examine the lumbar region (L1-L5) for somatic dysfunctionA palpate transverse processes for rotation1 neutral _2 flexion _3 extension _B palpate soft tissue for TART (Tenderness Asymmetry Restrictions Tissue texture change) findings1 quadratus lumborum muscles _2 Piriformis muscles _3 Paraspinal muscles _2 Examine the sacrum for somatic dysfunction (requires at least one motion test)A depth at 4 corners of sacrum _B ILA levelness _C seated flexion test _D motion at 4 corners of sacrum _E motion in backward bending test _F respiratory motion of sacrum assess at 4 corners _3 Examine the pelvis for somatic dysfunctionA standing flexion test or AP compression test _B ASIS heights _C PSIS heights _Osteopathic Treatment You must treat a minimum of 2 dysfunctions you found during your examination1 Assume L3 FRLSL2 Assume left on right sacral torsion _3 Assume left superior innominate shear _

4 Autonomic technique for altered parasympathetic tone

Module 10 ndash Minnie Driver

Pretest

1 Pneumonia dissecting aortic aneurysm thoracic spine fracture MI pleurisy2 B3 E

Osteopathic Diagnosis assess where you would expect to find somatic dysfunctions

Performed Omitted

1 Examine the cervical region for somatic dysfunctionA OA ndash leftright translation1 neutral 104863510486352 flexion 104863510486353 extension 10486351048635B AA ndash rotation must flex at least 45deg 10486351048635C C2-C7 - translation1 neutral 104863510486352 flexion 104863510486353 extension 104863510486352 Examine the thoracic spine for dysfunction (T5-T9)A palpate transverse processes for rotation1 neutral 104863510486352 flexion 104863510486353 extension 10486351048635B palpate soft tissue for TART findings1 trapezius 104863510486352 rhomboids 104863510486353 levator scapulae 104863510486354 T5 ndash T9 Paraspinal muscles 104863510486353 Examine the ribs for somatic dysfunctionA Pump handle ribs (ribs 2-6) at or near sternal junction1 static 104863510486352 motion 10486351048635B Bucket handle ribs (ribs 6-10) between anterior and posterior axillary lines1 static 104863510486352 motion 10486351048635Osteopathic Treatment You must treat a minimum of 2 dysfunctions you found during your examination1 Assume T5 ESLRL 104863510486352 Assume acute bilateral spasm of upper thoracic (T4-T8) paraspinal 10486351048635

muscles3 Assume (R) Rib 2 pump handle inhalation dysfunction 104863510486354 Autonomic technique for altered vagal tone OA decompression 10486351048635

MODULE 11 ndash Jeff Weaver

1 Cervical radiculopathy thoracic outlet syndrome cubital tunnel syndrome carpal tunnel syndrome angina pectoris ulnar groove entrapment Guillian Barre

2 A3 B4 C

Osteopathic Diagnosis assess where you would expect to find somatic dysfunctions

Performed Omitted

1 Examine the cervical region for somatic dysfunctionA C2-C7 - translation1 neutral 104863510486352 flexion 104863510486353 extension 10486351048635B palpate soft tissue for TART (Tenderness Asymmetry Restrictions Tissue texture change) findings1 cervical paraspinal muscles 104863510486352 sternocleidomastoids 104863510486353 scalenes 104863510486352 Examine the thoracic spine for somatic dysfunctionA palpate transverse processes for rotation1 neutral 104863510486352 flexion 104863510486353 extension 10486351048635B palpate soft tissue for TART findings1 trapezius 104863510486352 pectoral musculature 104863510486353 T1-T4 paraspinal muscle 104863510486353 Examine the 1st rib for somatic dysfunction1 static 104863510486352 dynamic 10486351048635Osteopathic Treatment You must treat a minimum of 2 dysfunctions you found during your examination1 Assume hypertonic left pectoral muscle Appropriate techniques to include ME MFR counterstrain

10486351048635

2 Assume hypertonic left scalene muscle Appropriate techniques to include MFR counterstrain ME

10486351048635

3 Assume T4 NRLSR appropriate techniques to include HVLA muscle energy articulatory Still counterstrainindirect myofascial release facilitated positional release

10486351048635

4 Assume left sided thoracic paraspinal hypertonicity T1 ndash T4 appropriate techniques to include soft tissue deep pressure counterstrainindirect facilitated positional release

10486351048635

5 Assume left 1 st rib inhalation somatic dysfunction appropriate techniques to include HVLA ME Stillrsquos articulatory

10486351048635

MODULE 12 ndash Scarlett O-Hara

1 Osteoarthritis piriformis syndrome spinal stenosis lumbar radiculopathy DVT2 C3 D

Osteopathic Diagnosis assess where you would expect to find somatic dysfunctions

Performed Omitted

1 Examine the lumbar region (L1-L5) for somatic dysfunctionA palpate transverse processes for rotation1 neutral _2 flexion _3 extension _B palpate soft tissue for TART (Tenderness Asymmetry Restrictions Tissue texture change) findings1 Psoas muscles _2 Piriformis muscles _3 Paraspinal muscles _2 Examine the sacrum for somatic dysfunction (requires at least one motion test)A depth at 4 corners of sacrum _B ILA levelness _C seated flexion test _D motion at 4 corners of sacrum _E motion in backward bending test _F respiratory motion of sacrum assess at 4 corners _3 Examine the pelvis for somatic dysfunctionA standing flexion test or AP compression test _B ASIS heights _C PSIS heights _Osteopathic Treatment You must treat a minimum of 2 dysfunctions you found during your examination1 Assume left on right sacral torsion appropriate techniques to include muscle energy counterstrainindirect myofascial release and facilitated positional release (No HVLA)2 Assume left innominate anterior appropriate techniques to include muscle energy counterstrainindirect myofascial release and facilitated positional release (No HVLA)

_

3 Assume spasmtender point in the left piriformis muscle appropriate techniques to _

include counterstrainindirect myofascial release

MODULE 13 Clark Kent1 Asthma atypical pneumonia bronchitis upper respiratory infection pneumothorax2 B3 A

Osteopathic Diagnosis assess where you would expect to find somatic dysfunctions

Performed Omitted

1 Examine the cervical region for somatic dysfunctionA OA ndash leftright translation1 neutral _2 flexion _3 extension _B AA ndash rotation must flex at least 45deg _C C2-C7 - translation1 neutral 2 flexion _3 extension 2 Examine the thoracic spine for dysfunction (T1-T4)A palpate transverse processes for rotation1 neutral _2 flexion _3 extension _B palpate soft tissue for TART findings1 trapezius X2 rhomboids X3 levator scapulae X4 T1 ndash T4 Paraspinal muscles X3 Examine the ribs for somatic dysfunctionA Pump handle ribs (ribs 2-6) at or near sternal junction1 static X2 motion XB Bucket handle ribs (ribs 6-10) between anterior and posterior axillary lines1 static X2 motion XOsteopathic Treatment You must treat a minimum of 2 dysfunctions you found during your examination1 Assume OA NSLRR appropriate techniques to include HVLA muscle energy articulatory Still counterstrainindirect myofascial release facilitated positional release

_

2 Assume T2-4 NSLRR appropriate techniques to include HVLA muscle energy articulatory Still counterstrainindirect myofascial release facilitated positional release

_

3 Assume (L) ribs 2-4 exhalation pump handle appropriate techniques to include HVLA muscle energy articulatory Still counterstrainindirect myofascial release facilitated positional release

4 Autonomic technique for altered vagal tone OA decompression _5 Autonomic technique for hypersympathetic tone in T1-T5 paraspinal muscles rib raising or paraspinal inhibition for gt 90 seconds

X

MODULE 14 Bruce Wayne

1 Depression thyroid disorder migraine headache tension tension headache migraine2 C3 A

Osteopathic Diagnosis assess where you would expect to find somatic dysfunctions

Performed Omitted

1 Examine the cervical region for somatic dysfunctionA OA ndash leftright translation1 neutral 2 flexion 3 extension B AA ndash rotation must flex at least 45deg C C2-C7 - translation1 neutral 2 flexion 3 extension _2 Examine the thoracic spine for dysfunction (T1-T4)A palpate transverse processes for rotation1 neutral X2 flexion X3 extension XB palpate soft tissue for TART findings1 trapezius X2 T1 ndash T4 Paraspinal muscles XOsteopathic Treatment You must treat a minimum of 2 dysfunctions you found during your examination1 Assume OA NSRRL appropriate techniques to include HVLA muscle energy articulatory Still counterstrainindirect myofascial release facilitated positional release

2 Assume C4 FSLRL appropriate techniques to include HVLA muscle energy

articulatory Still counterstrainindirect myofascial release facilitated positional release2 Assume T2-4 NSLRR appropriate techniques to include HVLA muscle energy articulatory Still counterstrainindirect myofascial release facilitated positional release

_

4 Autonomic technique for altered vagal tone OA decompression _

  • Performed
  • Performed
  • Performed
  • Performed
  • Performed
  • Performed
Page 22: Performed · Web viewAAA, kidney stones, groin hernia, ileus, kidney/bladder CA, testicular torsion, kidney/bladder infection, appendicitis, C D Osteopathic Diagnosis: assess where

4 Autonomic technique for altered parasympathetic tone

Module 10 ndash Minnie Driver

Pretest

1 Pneumonia dissecting aortic aneurysm thoracic spine fracture MI pleurisy2 B3 E

Osteopathic Diagnosis assess where you would expect to find somatic dysfunctions

Performed Omitted

1 Examine the cervical region for somatic dysfunctionA OA ndash leftright translation1 neutral 104863510486352 flexion 104863510486353 extension 10486351048635B AA ndash rotation must flex at least 45deg 10486351048635C C2-C7 - translation1 neutral 104863510486352 flexion 104863510486353 extension 104863510486352 Examine the thoracic spine for dysfunction (T5-T9)A palpate transverse processes for rotation1 neutral 104863510486352 flexion 104863510486353 extension 10486351048635B palpate soft tissue for TART findings1 trapezius 104863510486352 rhomboids 104863510486353 levator scapulae 104863510486354 T5 ndash T9 Paraspinal muscles 104863510486353 Examine the ribs for somatic dysfunctionA Pump handle ribs (ribs 2-6) at or near sternal junction1 static 104863510486352 motion 10486351048635B Bucket handle ribs (ribs 6-10) between anterior and posterior axillary lines1 static 104863510486352 motion 10486351048635Osteopathic Treatment You must treat a minimum of 2 dysfunctions you found during your examination1 Assume T5 ESLRL 104863510486352 Assume acute bilateral spasm of upper thoracic (T4-T8) paraspinal 10486351048635

muscles3 Assume (R) Rib 2 pump handle inhalation dysfunction 104863510486354 Autonomic technique for altered vagal tone OA decompression 10486351048635

MODULE 11 ndash Jeff Weaver

1 Cervical radiculopathy thoracic outlet syndrome cubital tunnel syndrome carpal tunnel syndrome angina pectoris ulnar groove entrapment Guillian Barre

2 A3 B4 C

Osteopathic Diagnosis assess where you would expect to find somatic dysfunctions

Performed Omitted

1 Examine the cervical region for somatic dysfunctionA C2-C7 - translation1 neutral 104863510486352 flexion 104863510486353 extension 10486351048635B palpate soft tissue for TART (Tenderness Asymmetry Restrictions Tissue texture change) findings1 cervical paraspinal muscles 104863510486352 sternocleidomastoids 104863510486353 scalenes 104863510486352 Examine the thoracic spine for somatic dysfunctionA palpate transverse processes for rotation1 neutral 104863510486352 flexion 104863510486353 extension 10486351048635B palpate soft tissue for TART findings1 trapezius 104863510486352 pectoral musculature 104863510486353 T1-T4 paraspinal muscle 104863510486353 Examine the 1st rib for somatic dysfunction1 static 104863510486352 dynamic 10486351048635Osteopathic Treatment You must treat a minimum of 2 dysfunctions you found during your examination1 Assume hypertonic left pectoral muscle Appropriate techniques to include ME MFR counterstrain

10486351048635

2 Assume hypertonic left scalene muscle Appropriate techniques to include MFR counterstrain ME

10486351048635

3 Assume T4 NRLSR appropriate techniques to include HVLA muscle energy articulatory Still counterstrainindirect myofascial release facilitated positional release

10486351048635

4 Assume left sided thoracic paraspinal hypertonicity T1 ndash T4 appropriate techniques to include soft tissue deep pressure counterstrainindirect facilitated positional release

10486351048635

5 Assume left 1 st rib inhalation somatic dysfunction appropriate techniques to include HVLA ME Stillrsquos articulatory

10486351048635

MODULE 12 ndash Scarlett O-Hara

1 Osteoarthritis piriformis syndrome spinal stenosis lumbar radiculopathy DVT2 C3 D

Osteopathic Diagnosis assess where you would expect to find somatic dysfunctions

Performed Omitted

1 Examine the lumbar region (L1-L5) for somatic dysfunctionA palpate transverse processes for rotation1 neutral _2 flexion _3 extension _B palpate soft tissue for TART (Tenderness Asymmetry Restrictions Tissue texture change) findings1 Psoas muscles _2 Piriformis muscles _3 Paraspinal muscles _2 Examine the sacrum for somatic dysfunction (requires at least one motion test)A depth at 4 corners of sacrum _B ILA levelness _C seated flexion test _D motion at 4 corners of sacrum _E motion in backward bending test _F respiratory motion of sacrum assess at 4 corners _3 Examine the pelvis for somatic dysfunctionA standing flexion test or AP compression test _B ASIS heights _C PSIS heights _Osteopathic Treatment You must treat a minimum of 2 dysfunctions you found during your examination1 Assume left on right sacral torsion appropriate techniques to include muscle energy counterstrainindirect myofascial release and facilitated positional release (No HVLA)2 Assume left innominate anterior appropriate techniques to include muscle energy counterstrainindirect myofascial release and facilitated positional release (No HVLA)

_

3 Assume spasmtender point in the left piriformis muscle appropriate techniques to _

include counterstrainindirect myofascial release

MODULE 13 Clark Kent1 Asthma atypical pneumonia bronchitis upper respiratory infection pneumothorax2 B3 A

Osteopathic Diagnosis assess where you would expect to find somatic dysfunctions

Performed Omitted

1 Examine the cervical region for somatic dysfunctionA OA ndash leftright translation1 neutral _2 flexion _3 extension _B AA ndash rotation must flex at least 45deg _C C2-C7 - translation1 neutral 2 flexion _3 extension 2 Examine the thoracic spine for dysfunction (T1-T4)A palpate transverse processes for rotation1 neutral _2 flexion _3 extension _B palpate soft tissue for TART findings1 trapezius X2 rhomboids X3 levator scapulae X4 T1 ndash T4 Paraspinal muscles X3 Examine the ribs for somatic dysfunctionA Pump handle ribs (ribs 2-6) at or near sternal junction1 static X2 motion XB Bucket handle ribs (ribs 6-10) between anterior and posterior axillary lines1 static X2 motion XOsteopathic Treatment You must treat a minimum of 2 dysfunctions you found during your examination1 Assume OA NSLRR appropriate techniques to include HVLA muscle energy articulatory Still counterstrainindirect myofascial release facilitated positional release

_

2 Assume T2-4 NSLRR appropriate techniques to include HVLA muscle energy articulatory Still counterstrainindirect myofascial release facilitated positional release

_

3 Assume (L) ribs 2-4 exhalation pump handle appropriate techniques to include HVLA muscle energy articulatory Still counterstrainindirect myofascial release facilitated positional release

4 Autonomic technique for altered vagal tone OA decompression _5 Autonomic technique for hypersympathetic tone in T1-T5 paraspinal muscles rib raising or paraspinal inhibition for gt 90 seconds

X

MODULE 14 Bruce Wayne

1 Depression thyroid disorder migraine headache tension tension headache migraine2 C3 A

Osteopathic Diagnosis assess where you would expect to find somatic dysfunctions

Performed Omitted

1 Examine the cervical region for somatic dysfunctionA OA ndash leftright translation1 neutral 2 flexion 3 extension B AA ndash rotation must flex at least 45deg C C2-C7 - translation1 neutral 2 flexion 3 extension _2 Examine the thoracic spine for dysfunction (T1-T4)A palpate transverse processes for rotation1 neutral X2 flexion X3 extension XB palpate soft tissue for TART findings1 trapezius X2 T1 ndash T4 Paraspinal muscles XOsteopathic Treatment You must treat a minimum of 2 dysfunctions you found during your examination1 Assume OA NSRRL appropriate techniques to include HVLA muscle energy articulatory Still counterstrainindirect myofascial release facilitated positional release

2 Assume C4 FSLRL appropriate techniques to include HVLA muscle energy

articulatory Still counterstrainindirect myofascial release facilitated positional release2 Assume T2-4 NSLRR appropriate techniques to include HVLA muscle energy articulatory Still counterstrainindirect myofascial release facilitated positional release

_

4 Autonomic technique for altered vagal tone OA decompression _

  • Performed
  • Performed
  • Performed
  • Performed
  • Performed
  • Performed
Page 23: Performed · Web viewAAA, kidney stones, groin hernia, ileus, kidney/bladder CA, testicular torsion, kidney/bladder infection, appendicitis, C D Osteopathic Diagnosis: assess where

muscles3 Assume (R) Rib 2 pump handle inhalation dysfunction 104863510486354 Autonomic technique for altered vagal tone OA decompression 10486351048635

MODULE 11 ndash Jeff Weaver

1 Cervical radiculopathy thoracic outlet syndrome cubital tunnel syndrome carpal tunnel syndrome angina pectoris ulnar groove entrapment Guillian Barre

2 A3 B4 C

Osteopathic Diagnosis assess where you would expect to find somatic dysfunctions

Performed Omitted

1 Examine the cervical region for somatic dysfunctionA C2-C7 - translation1 neutral 104863510486352 flexion 104863510486353 extension 10486351048635B palpate soft tissue for TART (Tenderness Asymmetry Restrictions Tissue texture change) findings1 cervical paraspinal muscles 104863510486352 sternocleidomastoids 104863510486353 scalenes 104863510486352 Examine the thoracic spine for somatic dysfunctionA palpate transverse processes for rotation1 neutral 104863510486352 flexion 104863510486353 extension 10486351048635B palpate soft tissue for TART findings1 trapezius 104863510486352 pectoral musculature 104863510486353 T1-T4 paraspinal muscle 104863510486353 Examine the 1st rib for somatic dysfunction1 static 104863510486352 dynamic 10486351048635Osteopathic Treatment You must treat a minimum of 2 dysfunctions you found during your examination1 Assume hypertonic left pectoral muscle Appropriate techniques to include ME MFR counterstrain

10486351048635

2 Assume hypertonic left scalene muscle Appropriate techniques to include MFR counterstrain ME

10486351048635

3 Assume T4 NRLSR appropriate techniques to include HVLA muscle energy articulatory Still counterstrainindirect myofascial release facilitated positional release

10486351048635

4 Assume left sided thoracic paraspinal hypertonicity T1 ndash T4 appropriate techniques to include soft tissue deep pressure counterstrainindirect facilitated positional release

10486351048635

5 Assume left 1 st rib inhalation somatic dysfunction appropriate techniques to include HVLA ME Stillrsquos articulatory

10486351048635

MODULE 12 ndash Scarlett O-Hara

1 Osteoarthritis piriformis syndrome spinal stenosis lumbar radiculopathy DVT2 C3 D

Osteopathic Diagnosis assess where you would expect to find somatic dysfunctions

Performed Omitted

1 Examine the lumbar region (L1-L5) for somatic dysfunctionA palpate transverse processes for rotation1 neutral _2 flexion _3 extension _B palpate soft tissue for TART (Tenderness Asymmetry Restrictions Tissue texture change) findings1 Psoas muscles _2 Piriformis muscles _3 Paraspinal muscles _2 Examine the sacrum for somatic dysfunction (requires at least one motion test)A depth at 4 corners of sacrum _B ILA levelness _C seated flexion test _D motion at 4 corners of sacrum _E motion in backward bending test _F respiratory motion of sacrum assess at 4 corners _3 Examine the pelvis for somatic dysfunctionA standing flexion test or AP compression test _B ASIS heights _C PSIS heights _Osteopathic Treatment You must treat a minimum of 2 dysfunctions you found during your examination1 Assume left on right sacral torsion appropriate techniques to include muscle energy counterstrainindirect myofascial release and facilitated positional release (No HVLA)2 Assume left innominate anterior appropriate techniques to include muscle energy counterstrainindirect myofascial release and facilitated positional release (No HVLA)

_

3 Assume spasmtender point in the left piriformis muscle appropriate techniques to _

include counterstrainindirect myofascial release

MODULE 13 Clark Kent1 Asthma atypical pneumonia bronchitis upper respiratory infection pneumothorax2 B3 A

Osteopathic Diagnosis assess where you would expect to find somatic dysfunctions

Performed Omitted

1 Examine the cervical region for somatic dysfunctionA OA ndash leftright translation1 neutral _2 flexion _3 extension _B AA ndash rotation must flex at least 45deg _C C2-C7 - translation1 neutral 2 flexion _3 extension 2 Examine the thoracic spine for dysfunction (T1-T4)A palpate transverse processes for rotation1 neutral _2 flexion _3 extension _B palpate soft tissue for TART findings1 trapezius X2 rhomboids X3 levator scapulae X4 T1 ndash T4 Paraspinal muscles X3 Examine the ribs for somatic dysfunctionA Pump handle ribs (ribs 2-6) at or near sternal junction1 static X2 motion XB Bucket handle ribs (ribs 6-10) between anterior and posterior axillary lines1 static X2 motion XOsteopathic Treatment You must treat a minimum of 2 dysfunctions you found during your examination1 Assume OA NSLRR appropriate techniques to include HVLA muscle energy articulatory Still counterstrainindirect myofascial release facilitated positional release

_

2 Assume T2-4 NSLRR appropriate techniques to include HVLA muscle energy articulatory Still counterstrainindirect myofascial release facilitated positional release

_

3 Assume (L) ribs 2-4 exhalation pump handle appropriate techniques to include HVLA muscle energy articulatory Still counterstrainindirect myofascial release facilitated positional release

4 Autonomic technique for altered vagal tone OA decompression _5 Autonomic technique for hypersympathetic tone in T1-T5 paraspinal muscles rib raising or paraspinal inhibition for gt 90 seconds

X

MODULE 14 Bruce Wayne

1 Depression thyroid disorder migraine headache tension tension headache migraine2 C3 A

Osteopathic Diagnosis assess where you would expect to find somatic dysfunctions

Performed Omitted

1 Examine the cervical region for somatic dysfunctionA OA ndash leftright translation1 neutral 2 flexion 3 extension B AA ndash rotation must flex at least 45deg C C2-C7 - translation1 neutral 2 flexion 3 extension _2 Examine the thoracic spine for dysfunction (T1-T4)A palpate transverse processes for rotation1 neutral X2 flexion X3 extension XB palpate soft tissue for TART findings1 trapezius X2 T1 ndash T4 Paraspinal muscles XOsteopathic Treatment You must treat a minimum of 2 dysfunctions you found during your examination1 Assume OA NSRRL appropriate techniques to include HVLA muscle energy articulatory Still counterstrainindirect myofascial release facilitated positional release

2 Assume C4 FSLRL appropriate techniques to include HVLA muscle energy

articulatory Still counterstrainindirect myofascial release facilitated positional release2 Assume T2-4 NSLRR appropriate techniques to include HVLA muscle energy articulatory Still counterstrainindirect myofascial release facilitated positional release

_

4 Autonomic technique for altered vagal tone OA decompression _

  • Performed
  • Performed
  • Performed
  • Performed
  • Performed
  • Performed
Page 24: Performed · Web viewAAA, kidney stones, groin hernia, ileus, kidney/bladder CA, testicular torsion, kidney/bladder infection, appendicitis, C D Osteopathic Diagnosis: assess where

4 Assume left sided thoracic paraspinal hypertonicity T1 ndash T4 appropriate techniques to include soft tissue deep pressure counterstrainindirect facilitated positional release

10486351048635

5 Assume left 1 st rib inhalation somatic dysfunction appropriate techniques to include HVLA ME Stillrsquos articulatory

10486351048635

MODULE 12 ndash Scarlett O-Hara

1 Osteoarthritis piriformis syndrome spinal stenosis lumbar radiculopathy DVT2 C3 D

Osteopathic Diagnosis assess where you would expect to find somatic dysfunctions

Performed Omitted

1 Examine the lumbar region (L1-L5) for somatic dysfunctionA palpate transverse processes for rotation1 neutral _2 flexion _3 extension _B palpate soft tissue for TART (Tenderness Asymmetry Restrictions Tissue texture change) findings1 Psoas muscles _2 Piriformis muscles _3 Paraspinal muscles _2 Examine the sacrum for somatic dysfunction (requires at least one motion test)A depth at 4 corners of sacrum _B ILA levelness _C seated flexion test _D motion at 4 corners of sacrum _E motion in backward bending test _F respiratory motion of sacrum assess at 4 corners _3 Examine the pelvis for somatic dysfunctionA standing flexion test or AP compression test _B ASIS heights _C PSIS heights _Osteopathic Treatment You must treat a minimum of 2 dysfunctions you found during your examination1 Assume left on right sacral torsion appropriate techniques to include muscle energy counterstrainindirect myofascial release and facilitated positional release (No HVLA)2 Assume left innominate anterior appropriate techniques to include muscle energy counterstrainindirect myofascial release and facilitated positional release (No HVLA)

_

3 Assume spasmtender point in the left piriformis muscle appropriate techniques to _

include counterstrainindirect myofascial release

MODULE 13 Clark Kent1 Asthma atypical pneumonia bronchitis upper respiratory infection pneumothorax2 B3 A

Osteopathic Diagnosis assess where you would expect to find somatic dysfunctions

Performed Omitted

1 Examine the cervical region for somatic dysfunctionA OA ndash leftright translation1 neutral _2 flexion _3 extension _B AA ndash rotation must flex at least 45deg _C C2-C7 - translation1 neutral 2 flexion _3 extension 2 Examine the thoracic spine for dysfunction (T1-T4)A palpate transverse processes for rotation1 neutral _2 flexion _3 extension _B palpate soft tissue for TART findings1 trapezius X2 rhomboids X3 levator scapulae X4 T1 ndash T4 Paraspinal muscles X3 Examine the ribs for somatic dysfunctionA Pump handle ribs (ribs 2-6) at or near sternal junction1 static X2 motion XB Bucket handle ribs (ribs 6-10) between anterior and posterior axillary lines1 static X2 motion XOsteopathic Treatment You must treat a minimum of 2 dysfunctions you found during your examination1 Assume OA NSLRR appropriate techniques to include HVLA muscle energy articulatory Still counterstrainindirect myofascial release facilitated positional release

_

2 Assume T2-4 NSLRR appropriate techniques to include HVLA muscle energy articulatory Still counterstrainindirect myofascial release facilitated positional release

_

3 Assume (L) ribs 2-4 exhalation pump handle appropriate techniques to include HVLA muscle energy articulatory Still counterstrainindirect myofascial release facilitated positional release

4 Autonomic technique for altered vagal tone OA decompression _5 Autonomic technique for hypersympathetic tone in T1-T5 paraspinal muscles rib raising or paraspinal inhibition for gt 90 seconds

X

MODULE 14 Bruce Wayne

1 Depression thyroid disorder migraine headache tension tension headache migraine2 C3 A

Osteopathic Diagnosis assess where you would expect to find somatic dysfunctions

Performed Omitted

1 Examine the cervical region for somatic dysfunctionA OA ndash leftright translation1 neutral 2 flexion 3 extension B AA ndash rotation must flex at least 45deg C C2-C7 - translation1 neutral 2 flexion 3 extension _2 Examine the thoracic spine for dysfunction (T1-T4)A palpate transverse processes for rotation1 neutral X2 flexion X3 extension XB palpate soft tissue for TART findings1 trapezius X2 T1 ndash T4 Paraspinal muscles XOsteopathic Treatment You must treat a minimum of 2 dysfunctions you found during your examination1 Assume OA NSRRL appropriate techniques to include HVLA muscle energy articulatory Still counterstrainindirect myofascial release facilitated positional release

2 Assume C4 FSLRL appropriate techniques to include HVLA muscle energy

articulatory Still counterstrainindirect myofascial release facilitated positional release2 Assume T2-4 NSLRR appropriate techniques to include HVLA muscle energy articulatory Still counterstrainindirect myofascial release facilitated positional release

_

4 Autonomic technique for altered vagal tone OA decompression _

  • Performed
  • Performed
  • Performed
  • Performed
  • Performed
  • Performed
Page 25: Performed · Web viewAAA, kidney stones, groin hernia, ileus, kidney/bladder CA, testicular torsion, kidney/bladder infection, appendicitis, C D Osteopathic Diagnosis: assess where

include counterstrainindirect myofascial release

MODULE 13 Clark Kent1 Asthma atypical pneumonia bronchitis upper respiratory infection pneumothorax2 B3 A

Osteopathic Diagnosis assess where you would expect to find somatic dysfunctions

Performed Omitted

1 Examine the cervical region for somatic dysfunctionA OA ndash leftright translation1 neutral _2 flexion _3 extension _B AA ndash rotation must flex at least 45deg _C C2-C7 - translation1 neutral 2 flexion _3 extension 2 Examine the thoracic spine for dysfunction (T1-T4)A palpate transverse processes for rotation1 neutral _2 flexion _3 extension _B palpate soft tissue for TART findings1 trapezius X2 rhomboids X3 levator scapulae X4 T1 ndash T4 Paraspinal muscles X3 Examine the ribs for somatic dysfunctionA Pump handle ribs (ribs 2-6) at or near sternal junction1 static X2 motion XB Bucket handle ribs (ribs 6-10) between anterior and posterior axillary lines1 static X2 motion XOsteopathic Treatment You must treat a minimum of 2 dysfunctions you found during your examination1 Assume OA NSLRR appropriate techniques to include HVLA muscle energy articulatory Still counterstrainindirect myofascial release facilitated positional release

_

2 Assume T2-4 NSLRR appropriate techniques to include HVLA muscle energy articulatory Still counterstrainindirect myofascial release facilitated positional release

_

3 Assume (L) ribs 2-4 exhalation pump handle appropriate techniques to include HVLA muscle energy articulatory Still counterstrainindirect myofascial release facilitated positional release

4 Autonomic technique for altered vagal tone OA decompression _5 Autonomic technique for hypersympathetic tone in T1-T5 paraspinal muscles rib raising or paraspinal inhibition for gt 90 seconds

X

MODULE 14 Bruce Wayne

1 Depression thyroid disorder migraine headache tension tension headache migraine2 C3 A

Osteopathic Diagnosis assess where you would expect to find somatic dysfunctions

Performed Omitted

1 Examine the cervical region for somatic dysfunctionA OA ndash leftright translation1 neutral 2 flexion 3 extension B AA ndash rotation must flex at least 45deg C C2-C7 - translation1 neutral 2 flexion 3 extension _2 Examine the thoracic spine for dysfunction (T1-T4)A palpate transverse processes for rotation1 neutral X2 flexion X3 extension XB palpate soft tissue for TART findings1 trapezius X2 T1 ndash T4 Paraspinal muscles XOsteopathic Treatment You must treat a minimum of 2 dysfunctions you found during your examination1 Assume OA NSRRL appropriate techniques to include HVLA muscle energy articulatory Still counterstrainindirect myofascial release facilitated positional release

2 Assume C4 FSLRL appropriate techniques to include HVLA muscle energy

articulatory Still counterstrainindirect myofascial release facilitated positional release2 Assume T2-4 NSLRR appropriate techniques to include HVLA muscle energy articulatory Still counterstrainindirect myofascial release facilitated positional release

_

4 Autonomic technique for altered vagal tone OA decompression _

  • Performed
  • Performed
  • Performed
  • Performed
  • Performed
  • Performed
Page 26: Performed · Web viewAAA, kidney stones, groin hernia, ileus, kidney/bladder CA, testicular torsion, kidney/bladder infection, appendicitis, C D Osteopathic Diagnosis: assess where

2 Assume T2-4 NSLRR appropriate techniques to include HVLA muscle energy articulatory Still counterstrainindirect myofascial release facilitated positional release

_

3 Assume (L) ribs 2-4 exhalation pump handle appropriate techniques to include HVLA muscle energy articulatory Still counterstrainindirect myofascial release facilitated positional release

4 Autonomic technique for altered vagal tone OA decompression _5 Autonomic technique for hypersympathetic tone in T1-T5 paraspinal muscles rib raising or paraspinal inhibition for gt 90 seconds

X

MODULE 14 Bruce Wayne

1 Depression thyroid disorder migraine headache tension tension headache migraine2 C3 A

Osteopathic Diagnosis assess where you would expect to find somatic dysfunctions

Performed Omitted

1 Examine the cervical region for somatic dysfunctionA OA ndash leftright translation1 neutral 2 flexion 3 extension B AA ndash rotation must flex at least 45deg C C2-C7 - translation1 neutral 2 flexion 3 extension _2 Examine the thoracic spine for dysfunction (T1-T4)A palpate transverse processes for rotation1 neutral X2 flexion X3 extension XB palpate soft tissue for TART findings1 trapezius X2 T1 ndash T4 Paraspinal muscles XOsteopathic Treatment You must treat a minimum of 2 dysfunctions you found during your examination1 Assume OA NSRRL appropriate techniques to include HVLA muscle energy articulatory Still counterstrainindirect myofascial release facilitated positional release

2 Assume C4 FSLRL appropriate techniques to include HVLA muscle energy

articulatory Still counterstrainindirect myofascial release facilitated positional release2 Assume T2-4 NSLRR appropriate techniques to include HVLA muscle energy articulatory Still counterstrainindirect myofascial release facilitated positional release

_

4 Autonomic technique for altered vagal tone OA decompression _

  • Performed
  • Performed
  • Performed
  • Performed
  • Performed
  • Performed
Page 27: Performed · Web viewAAA, kidney stones, groin hernia, ileus, kidney/bladder CA, testicular torsion, kidney/bladder infection, appendicitis, C D Osteopathic Diagnosis: assess where

articulatory Still counterstrainindirect myofascial release facilitated positional release2 Assume T2-4 NSLRR appropriate techniques to include HVLA muscle energy articulatory Still counterstrainindirect myofascial release facilitated positional release

_

4 Autonomic technique for altered vagal tone OA decompression _

  • Performed
  • Performed
  • Performed
  • Performed
  • Performed
  • Performed